facts learned from mcqs Flashcards

1
Q

Does blood for epidural blood patch spread predominantly cranially or caudally?

A

cranially, so do epidural blood patch at same level or the level below initial dural puncture

How well did you know this?
1
Not at all
2
3
4
5
Perfectly
2
Q

Are mastocytosis, anaphylaxis, ACS, lymphomas/leukemias/myeloproliferative disorders & CKD associated with raised mast cell tryptases?

A

Yes, tryptases often reduced in chronic liver disease)

How well did you know this?
1
Not at all
2
3
4
5
Perfectly
3
Q

*What’s the IM dose of carboprost?

A

250microg, repeat q15-minutely up to 8 doses (2mg)

How well did you know this?
1
Not at all
2
3
4
5
Perfectly
4
Q

:) What lead changes correspond to which coronary arteries?

A

V1, V2 = septal = proximal LAD
V3, V4 = anterior = LAD
V5, V6 = apical = distal LAD, L) circumflex or RCA
I, aVL = lateral = L) circumflex
II, III, aVF= inferior = 90% RCA, 10% L) Cx
V7-9 (reciprocal ST depressions frequently seen in V1-3) = posterior = RCA or L) Cx
Reciprocal changes: PAILS (P->A, A->I, I->L, L->S/I, S->P)

How well did you know this?
1
Not at all
2
3
4
5
Perfectly
5
Q

*What techniques may improve the speed of onset & spread of a peribulbar block?

A

hyaluronidase 5-70IU/mL (hydrolysis of hyaluronic acid, a GAG molecule forming part of the ECM) increases tissue permeability & promotes dispersion of LA
lignocaine (faster onset 5-10 min vs bupiv/ropivacaine 10-15 min)
gentle digital or compression device (eg. honan balloon limits to 30mmHg; improves spread, avoid ocular massage which can increase IOP to >400mmHg)
LA injectate volume

How well did you know this?
1
Not at all
2
3
4
5
Perfectly
6
Q

*What ASA is an ESRD patient on haemodialysis?

A

3 (severe systemic disease)

How well did you know this?
1
Not at all
2
3
4
5
Perfectly
7
Q

*definition & examples of ASAIII?

A

severe systemic disease, substantive functional limitations
one or more moderate to severe diseases
poorly controlled DM or HTN
COPD
BMI >=40
active hepatitis
ETOH dependence or abuse
implanted pacemaker
moderately reduced EF
ESRD undergoing regular scheduled haemodialysis
H/o (>3/12) post MI/CVA/TIA/CAD/stents
severe OSA
oncologic state
CF
Hx organ transplant
premature infant PCA <60wks
autism with severe limitations
difficult airway
full-term infants <6/52 of age.
Pre-eclampsia with severe features
gestational DM with complications or high insulin requirement
thrombophilic disease requiring anticoagulation in pregnancy

How well did you know this?
1
Not at all
2
3
4
5
Perfectly
8
Q

*What’s the sensory innervation to the breast? and motor?

A

Supraclavicular nerve (C3-4)- skin from clavicle to 2nd rib
Intercostobrachial (T2)- axilla & medial upper arm
segmental somatic sensory innervation from the T2-6 intercostal nerves which are from the anterior rami of these Tx spinal nerves:
-Lateral breast: lateral cutaneous branches of the T2-6 intercostal nerves (pierce SAM, anterior divisions serve lateral breast)
-Medial breast: anterior cutaneous branches of the T2-6 intercostal nerves (pierce PM near sternum)

Motor:
Long thoracic nerve C5-7 (SA)
Thoracodorsal nerve C6-8 (LD)
Lateral pectoral nerve C5-7 (PM/m)
medial pectoral nerve C8-T1 (PM)
subscapular nerve C5-6 (TM)
upper/lower subscapular nerves C5-8 (subscapularis)

How well did you know this?
1
Not at all
2
3
4
5
Perfectly
9
Q

*What’s a derived value from an ABG & from what is it calculated?

A

HCO3-; from CO2 & H+ using H-H equation

Base excess (HH & Siggaard-Anderson equation)- either base excess (the amount of alkali that must be added to the sample to return it to a normal pH at roome tem (37degC) & PaCo2 40mmHg, or standard base excess calculated for the blood with a Hb [] 50g/L (thought to better represent ECF as a whole)

pH (electrode with HCl, blood & buffer solutions), pO2 (clark electrode), pCO2 (Severinghaus electrode), Hb & some biochem are measured.

How well did you know this?
1
Not at all
2
3
4
5
Perfectly
10
Q

What are some features consistent with SIADH?

A

hypoosmolar hyponatraemia
urine osmolality >100mOsmol/kg, ie. concentrated urine despite hypotonic blood (plasma osmolality <275mosb/kg))
urine Na >20mmol/L
normal renal/hepatic/cardiac/pituitary/adrenal/thyroid
absence of hypoT/hypovolaemia/oedema/ADH-influencing drugs (vasopressin, desmopression, terlipressin. Carbamazepine, cyclophosphamide, SSRIs also promote ADH release), hyponatremia corrects with H2O restriction. Na+, inhibit ADH (demeclocycline, tolvaptan)

Causes: MADCHOPS
major OT, ADH from tumour (eg. SCLC), drugs, CNS, hormone deficiency

How well did you know this?
1
Not at all
2
3
4
5
Perfectly
11
Q

*What are some causes of SIADH?

A

MADCHOPS:
major OT
ADH from tumour
Drugs
CNS (trauma, SDH)
Hormone deficiency
Others
Pulmonary

How well did you know this?
1
Not at all
2
3
4
5
Perfectly
12
Q

*How to treat hypoosmolar hyponatremia?

A

FR, Na+, decrease ADH secretion (demeclocycline, tolvaptan)

How well did you know this?
1
Not at all
2
3
4
5
Perfectly
13
Q

*what are some risk factors for OIVI?

A

Pt:
obesity
sleep-disordered breathing
Age >65yo
female
COPD
renal/neuro/cardio disease
DM
HTN 2+ comorbidities
opioid dependence
ASA 3/4
CYP450 enzyme polymorphisms

external:
concomitant administration of sedatives (BZD, gabapentanoids, antipsychotics)
different routes of opioid administration
continuous opioid infusions
multiple prescribers
inadequate nursing Ax or responses
SR opioids

How well did you know this?
1
Not at all
2
3
4
5
Perfectly
14
Q

*What are some benefits of robot-assisted laparoscopic prostatectomy surgery in comparison with open prostatectomy?

A

Risks with robot-assisted:
Pneumoperitoneum: physiological effects, venous air embolus (CO2), subcut or mediastinal emphysema, pTx, CO2 retention, pain related to intra-abdo gas
Steep head down (well leg compartment syndrome)
Problems during surgical access, including small bowel, iliac artery, colon, iliac vein damage (bleeding may not immediately be obvious as bleeding into retroperitoneum may occur)
Mechanical failures (eg. uncontrolled movements, arcing from diathermy causing burns to surrounding tissue)
Robot less bleeding, transfusion & shorter LoS, less postop med/surg complications cf laparoscopic/open

How well did you know this?
1
Not at all
2
3
4
5
Perfectly
15
Q

*What are some complications of pneumoperitoneum?

A

decr PL, incr SVR, decr CO, venous pooling + DVT, incr BP, incr ICP, incr CO2, atelectasis, VQ mismatch, incr VD alveolar, incr PaCO2-PE’CO2, incr PaO2/FiO2, decr renal BF, incr RAAS/ADH, IAP - =<10mmHg incr VR/CO, 10-20mmHg decr CO incr SVR, >20mmHg decr MAP/CO), trendelenburg, surgical access (50% complications at this time, SB>iliac a>colon >iliac vs)

How well did you know this?
1
Not at all
2
3
4
5
Perfectly
16
Q

*What are some complications & rates with US-guided L) IJ CVC insertion?

A

arterial puncture 6.3-9.4%, CLABSI 1.4%, DVT 0.9%, PTX <0.1-0.2%, haematoma <0.1-2.2%, higher rate lymphatic injury due to anatomic location of thoracic duct

How well did you know this?
1
Not at all
2
3
4
5
Perfectly
17
Q

*What’s the most common complication of subclavian cannulation?

A

arterial puncture (3.7%)
mechanical complication 2.1% (subclavian has the highest mechanical complication rate)
then PTx 1.5%
CLABSI &DVT 0.5% (lowest infection/DVT risk w subclavian but highest rate mechanical complications)

How well did you know this?
1
Not at all
2
3
4
5
Perfectly
18
Q

*What are some complications & rates of femoral vein cannulation?

A

Arterial puncture most common w fem lines (10%) so they have overall higher complication rate
DVT 1.4%
CLABSI 1.2%

How well did you know this?
1
Not at all
2
3
4
5
Perfectly
19
Q

*What type of drug is benztropine & what are it’s uses?

A

anticholinergic (selective M1)/antihistamine agent, increases dopamine availability by blocking it’s reuptake/storage

Adjunct in Parkinsonism, Tx of EPS except tardive dyskinesia from neuroleptic drugs

Ameliorates side effects of drugs that antagonise the dopamine receptor

How well did you know this?
1
Not at all
2
3
4
5
Perfectly
20
Q

*Regarding healthcare research, the PICO framework describes what?

A

Patient/problem/population

Intervention

Comparison/control/comparator

Outcome

Used to frame and answer a clinical or health care related question

How well did you know this?
1
Not at all
2
3
4
5
Perfectly
21
Q

*What drug (& dose) is used to treat duct dependent congenital heart disease?

A

Alprostadil (synthetic PGE1), infusion to maintain ductal patency. important for both right & left heart lesions.
Most effective within 96hrs of birth (before anatomical closure), palliative therapy until surgery, 0.1microg/kg/min, see effect in 30-60 min, reduce dose to 0.01-0.02microg/kg/min
causes vasodilation of all arterioles & inhibition of platelet aggregation
side effects incl apnoea, hypotension

0.05mcg/kg/min if ductus restrictive or status unknown (the dose used for transport). max 0.1mcg/kg/min.

How well did you know this?
1
Not at all
2
3
4
5
Perfectly
22
Q

*What are some adverse effects of alprostatil?

A

apnoea (dose-dependent- have intubation equipment on-hand), fever, cutaneous flushing, bradycardia, hypoT, oedema, seizures, decreased platelet aggregation, thrombocytopenia

necrotising enterocolitis (mesenteric hypoperfusion combined cyanosis & low diastolic BP- monitor infants for abdo distension, bilious vomit, bloody stools).

cyanotic baby DDx sepsis; if not yet proven duct-dependent, start alprostadil 0.05mcg/kg/min & start BS Abx (amp. &gent) once blood cultures taken.

How well did you know this?
1
Not at all
2
3
4
5
Perfectly
23
Q

*If a pt sustains blunt chest trauma, after how much immediate blood drainage after closed thoracostomy is a thoracotomy indicate?

A

1500mL (>=20mL/kg) or 200mL/hr in the first 3hrs or 100mL/hr in the first 6hrs

How well did you know this?
1
Not at all
2
3
4
5
Perfectly
24
Q

*What factors are used to calculate the Child-Pugh score?

A

INR
Bilirubin
Ascites
Albumin
Encephalopathy

ABCDE: albumin, bilirubin, cogs, distended abdo, encephalopathy

each scored out of 3, total 15

How well did you know this?
1
Not at all
2
3
4
5
Perfectly
25
Q

*What are the points for Child-Pugh classification?

A

each of INR, Bilirubin, Ascites, Albumin & encephalopathy gets 1-3 points. 5-6= Child-Pugh A, 7-9= C-P B, 10+= C-P C

How well did you know this?
1
Not at all
2
3
4
5
Perfectly
26
Q

*What’s the abdo surgery perioperative mortality & life expectancy for Childs A, B & C?

A

A= 10%, 15-30yrs
B= 30%, eval for transplant
C= 82%, life expectancy 1-3yrs

How well did you know this?
1
Not at all
2
3
4
5
Perfectly
27
Q

*What’s the pKa of bupivacaine? how does this influence it’s onset?

A

8.1, only 17% unionised @ physiological pH, lower degree of transfer across lipophilic membranes so slower onset cf other LAs eg. lignocaine pKa 7.9, 25% unionised @ physiology pH. (Adding bicarb speeds onset)
Also, bupivacaine has large MW which slows onset but there’s little difference in MW btwn LAs.

How well did you know this?
1
Not at all
2
3
4
5
Perfectly
28
Q
  • What property does protein binding impart for LAs?
A

duration of action; highly PB (eg. bupiv 95%) = longer duration cf lower PB (lignocaine 70%)

How well did you know this?
1
Not at all
2
3
4
5
Perfectly
29
Q

*What property does lipid solubility impart to LAs?

A

potency. Bupivacaine more lipid soluble cf lignocaine so more potent.

How well did you know this?
1
Not at all
2
3
4
5
Perfectly
30
Q

*how does indocyanine green influence pulse vs cerebral oxygen tissue saturation?

A

decrease SpO2, increase SctO2 (a strong near-infrared absorber)

How well did you know this?
1
Not at all
2
3
4
5
Perfectly
31
Q

*what did CRASH-2 show that TxA did for trauma victims?

A

Reduces risk of death in bleeding trauma patients if given in the first 3hrs (all cause mortality @ 28d, death due to bleeding)
without increase in vaso-occlusive events
Did not reduce blood product use

How well did you know this?
1
Not at all
2
3
4
5
Perfectly
32
Q

What’s the dose of lignocaine in mg/kg for a bier’s block?

A

3mg/kg (eg. for a 30kg child, 18mL of 0.5% lignocaine)

How well did you know this?
1
Not at all
2
3
4
5
Perfectly
33
Q

What should the fluid prescription be for a 25kg NBM child overnight?

A

65mL/hr 0.9% saline with 5% dextrose

How well did you know this?
1
Not at all
2
3
4
5
Perfectly
34
Q

What lifestyle modifications are effective in reducing essential HTN?

A

Effective lifestyle modifications=
weight loss
dietary salt restriction (limit to 65mmol/day)
potassium supplementation (preferably by diet) unless contraindicated by CKD
DASH diet (high vegetables, fruit, low-fat dairy, nuts & low in red meat & sweets- rich in K+, Mg++, Ca++, protein & fibre, low in fat saturated fat & cholesterol)
smoking cessation
exercise (aerobic, dynamic resistance & isometric- 3-4 sessions/week of moderate-intensity aerobic exercise)
reduced ETOH

How well did you know this?
1
Not at all
2
3
4
5
Perfectly
35
Q

What’s sensory innervation of the cornea?

A

Nasociliary (from the ophthalmic (V1) branch of trigeminal nerve), meets ciliary ganglion & gives off long & short ciliary nerves supplying cornea

ie:
Trigeminal–> nasociliary (of the ophthalmic)–> long & short ciliary nerves (which are the terminal supply)

How well did you know this?
1
Not at all
2
3
4
5
Perfectly
36
Q

What are some laboratory findings with haemophilia?

A

prol APTT, normal platelets/bleeding time/PT

How well did you know this?
1
Not at all
2
3
4
5
Perfectly
37
Q

How is haemophilia A, B & C inherited? incidence & what deficiencies?

A

A is XLR, f8 def, 1/5000 male births
B is XLR, f9 def, 1/25000 male births
C is AD/AR, f11 def

How well did you know this?
1
Not at all
2
3
4
5
Perfectly
38
Q

How is vWD inherited? lab findings? And lab findings for haemophilia?

A

AD>AR
normal/reduced plt, normal or prolonged bleeding time, normal/prol APTT, normal PT
can do VWF:Ag & VWF:RCx, F8
VWF:RCx & F8 should both be 100IU/dl periop & >50IUl/dl immediately postop
vWf qual/quant defect clinically significant in 1/10,000

for haemophilia may have prol APTT

How well did you know this?
1
Not at all
2
3
4
5
Perfectly
39
Q

What are some conditions for which volatile inappropriate?

A

Increased risk MH: known MH, FHx MH, congenital myopathy, exertional rhabdomyolysis, exertional heat illness, idiopathic hyperCKaemia, carrier of RYR1 variant of unknown significance

How well did you know this?
1
Not at all
2
3
4
5
Perfectly
40
Q

What are lab findings indicating inadequate or low iron stores? what are the Hb targets & Fe replacement?

A

Serum ferritin <30ug/l

In the presence of inflammation (CRP>5mg/l) and/or transferrin saturation <20%, serum ferritin <100ug/l

Target Hb>=130g/l in both sexes

PO iron 6-8/52 preop

IV iron <6/52 preop or non-responder/not tolerating PO iron

(International Consensus Statement Ass of Anaesthetists 2017)

How well did you know this?
1
Not at all
2
3
4
5
Perfectly
41
Q

Globe perforation during eye block is more common in myopic eyes because:

A

Staphylomas (sclera outpouches) are more common in myopes (increased axial length >26mm)

Other risk factors for globe perforation: thinner eye, deep set eyes, previous scleral buckle, posterior staphyloma (staphyloma more common in myopes), retrobulbar (intraconal) or peribulbar > subtenon’s

How well did you know this?
1
Not at all
2
3
4
5
Perfectly
42
Q

What does adenosine do to pt w heart transplant?

A

enhance Brady effect

How well did you know this?
1
Not at all
2
3
4
5
Perfectly
43
Q

What do atropine & glyco do to pt w heart transplant?

A

no effect

How well did you know this?
1
Not at all
2
3
4
5
Perfectly
44
Q

What does neostigmine do to pt w heart transplant?

A

bradycardia may develop, esp if >6/12 post-transplant

How well did you know this?
1
Not at all
2
3
4
5
Perfectly
45
Q

what do B-blockers do to pt w heart transplant?

A

effective (but caution as heart reliant on circulating catecholamines to increase CO)

How well did you know this?
1
Not at all
2
3
4
5
Perfectly
46
Q

What does lignocaine do to pt w heart transplant?

A

effective

How well did you know this?
1
Not at all
2
3
4
5
Perfectly
47
Q

What does dopamine do to pt w heart transplant?

A

normal response

How well did you know this?
1
Not at all
2
3
4
5
Perfectly
48
Q

What do Adr & NAdr do to pt w heart transplant?

A

exaggerated effect as increased adrenoceptor density

How well did you know this?
1
Not at all
2
3
4
5
Perfectly
49
Q

What do metaraminol & phenylephrine do to pt w heart transplant?

A

effective but no reflex bradycardia

How well did you know this?
1
Not at all
2
3
4
5
Perfectly
50
Q

What does ephedrine do to pt w heart transplant?

A

decreased effect- indirect mechanism

How well did you know this?
1
Not at all
2
3
4
5
Perfectly
51
Q

What does dobutamine do to pt w heart transplant?

A

exaggerated effect

How well did you know this?
1
Not at all
2
3
4
5
Perfectly
52
Q

What does digoxin do to pt w heart transplant?

A

inotropic effect intact, conduction effects on AVN likely absent

How well did you know this?
1
Not at all
2
3
4
5
Perfectly
53
Q

What does isoprenaline do to pt w heart transplant?

A

effective

How well did you know this?
1
Not at all
2
3
4
5
Perfectly
54
Q

Normal A-a gradient?

A

Normal A-a gradient <15mmHg

How well did you know this?
1
Not at all
2
3
4
5
Perfectly
55
Q

What are some features NOT consistent with SIADH?

A

Low urine osmolality (<100mOsmol/kg)
High serum osmolality
Low urine sodium
Hypovolaemia
Abnormal thyroid, adrenal function

How well did you know this?
1
Not at all
2
3
4
5
Perfectly
56
Q

Which conditions may be associated with MH susceptibility?

A

myopathies with RYR1 abnormalities (or STAC3)- central core myopathy, multiminicore disease, King-Denborough syndrome, periodic paralysis, idiopathic hyperCKemia, carrier of RYR1 variant of unknown significance

conditions associated with rhabdo:
exertion rhabdoymolysis
severe statin-induced myopathy

some muscle disorders don’t confer higher risk of MH but are associated with significant rhabdo & hyperkalemia following sux or volatile eg. Duchenne or Becker muscular dystrophy (both X-linked, recessive leading to abnormal formation of dystrophin)- treat with non-triggering agents

Immediate action if peaked T waves, raised eTCO2 in pt w suspected Duchennes:
-stop volatile & give calcium
Duchenne= most common childhood musc dystrophy. X-linked recessive. 1:3500 live births. lack dystrophin (anchor muscle cells to ECM), sarcolemma becomes incr permeable, incr intracellular Ca++
progressive wasting/weakness prox muscles, fatal late adolescence from resp/cardiac failure. waddling gait, calf pseudohypertrophy by age 3-5, inh anaes rhabdo sec incr Ca++ not MH but avoid triggers

How well did you know this?
1
Not at all
2
3
4
5
Perfectly
57
Q

What ARE some high-risk transthoracic echo findings associated with aortic dissection?

A

intimal flap

SEVERE PROXIMAL AORTIC DILATATION
regurg
pericardial effusion

Type A associated features and complications:
aortic regurgitation (acute dilatation of the aortic root, aortic leaflet prolapse, dissection flap prolapse, pre-existing disease, e.g. bicuspid valve)
pericardial effusion and/or CARDIAC TAMPONADE
REGIONAL WALL MOTION ABNORMALITIES or SEVERELY IMPAIRED LVEF
intramural haematoma, aortic ulcer.

colour flow Doppler:
identifies true and false lumen
detect aortic branch occlusion/ dissection (absent flow)

TTE is considered limited inthe Dx of aortic dissection BUT recent contrast-TTE thought to have similar accuracy to TOE for Dx type A (sens 93%, spec 97%) but more limited in type B (sens 84%, spec 94%).
Still useful as availability, rapidity, additional info on cardiac status. useful for emergency room but has low negative predictive value so can’t rule out dissection (further tests if the TTE is -ve).
limited if abnormal chest wall, obesity, emphysema, mech vent.

How well did you know this?
1
Not at all
2
3
4
5
Perfectly
58
Q

Which ingested objects are high-risk & require imaging?

A

button batteries & magnets, a metal object & magnet or >1 magnet, lead objects (may lead to systemic absorption)

How well did you know this?
1
Not at all
2
3
4
5
Perfectly
59
Q

What should be offered to children >12 months old with suspected button battery ingestion?

A

honey at regular intervals

How well did you know this?
1
Not at all
2
3
4
5
Perfectly
60
Q

what size objects may become trapped in the pylorus?

A

> 2cm wide & >6cm long

How well did you know this?
1
Not at all
2
3
4
5
Perfectly
61
Q

Which metal is only variably detected on X-ray?

A

aluminium

How well did you know this?
1
Not at all
2
3
4
5
Perfectly
62
Q

Which children should NOT have honey after button battery ingestion?

A

Those aged <12 months (rare risk of botulism with honey)
Battery ingested >12hrs ago (higher risk oesophageal perforation)

How well did you know this?
1
Not at all
2
3
4
5
Perfectly
63
Q

What should children >12 months with suspected battery ingestion <12hrs ago take enroute to definitive care? then, upon hospital arrival once XR has confirmed oesophageal location of battery, what to administer?

A

Honey 10mL q10min if >12 months old & ingestion was within the last 12hrs

Sucralfate suspension (10mLs q10mins)

Battery removal by endoscopy shouldn’t be delayed- ideally within 2hrs of battery ingestion, anaesthesia can proceed regardless of PO honey intake
irrigate. ifno perf (sterile acetic acid)
if in stomach, minimal risk complications (consider removing if remains for 4 days)

How well did you know this?
1
Not at all
2
3
4
5
Perfectly
64
Q

What are some contraindications for stellate ganglion block?

A

unable to consent
local infection or neospasm
anatomical or vascular anomalies

recent MI
anti-coagulated pts or those with coagulopathy
glaucoma
severe emphysema
cardiac conduction block
pre-existing contralateral phrenic nerve palsy (may precipitate resp distress)

Indications:
head/neck pain (sympathetically mediated) eg. CRPS, facial herpes zoster, phantom UL/ischaemic pain, vascular disease (Reynaud’s, vasospasm), scleroderma, arrhythmia (eg. VF, VT) refractory to other therapy.

stellate ganglion is fusion inf Cx with 1st-2nd SNS ganglion

AEs= Horner’s, tracheal/oes injury, haemorrhage, brachial plexus/RLN/phrenic palsy, PHx, intrathecal

ant paratracheal approach, head extended, needle 2-3cm above & 2cm lat to suprasternal notch, 2-3cm above & 2cm lat to suprasternal notch

How well did you know this?
1
Not at all
2
3
4
5
Perfectly
65
Q

Is it safe to do neuraxial in a pt with MS?

A

little conclusive evidence for or against

epidural considered safer than spinal as demyelinated fibres more susceptible to toxic effects from LA- inform pt of risk of new/worsening neurologic Sx
Regional is OK as long as pt understands potential for relapse, have follow-up in place
PRISMS study 2004- slight reduction in relapse during pregnancy, significantly increase in relapse @ 3/12 postpartum regardless of anaesthetic technique or mode of delivery, no difference in relapse rates epidural vs no epidural

How well did you know this?
1
Not at all
2
3
4
5
Perfectly
66
Q

What are the heart sounds?

A

S1= closure of MV/TV
S2= closure AV/PV
S3= tightening of papillary muscles (rapid diastolic filling)
S4= atria contracting against stiffened ventricle eg. with AS, PS, PHTN, HOCM

How well did you know this?
1
Not at all
2
3
4
5
Perfectly
67
Q

What’s normal QRS axis on an ecg? L)? R)? extreme?

A

-30 to +90 deg

LAD= -30 to -90
RAD= +90 to 180
extreme is 180 to -90

How well did you know this?
1
Not at all
2
3
4
5
Perfectly
68
Q

What’s a more reliable clinical indicator of early OIVI? sedation score or decrease RR? What’s the most reliable way to detect OIVI at an early stage? Why is SpO2 unreliable?

A

sedation score. APMSE book: “best early clinical indicator is increasing sedation”

continuous CO2 monitoring is the most reliable way to detect OIVI at an early stage BUT this is limited to OT/PACU/ICU/some HDU

Pts may have reasons other than OIVI to have low SpO2 & use of supplemental O2 makes SpO2 unreliable for early detection of OIVI

How well did you know this?
1
Not at all
2
3
4
5
Perfectly
69
Q

What do we see with middle cerebral artery syndrome: non-dominant MCA? how does this contrast with Gerstmann (dominant MCA)?

A

contralateral arm weakness, sensory loss, HEMI-NEGLECT, homonymous hemianopia, apraxia, spatial disorientation

dominant: global receptive & expressive aphasia, agraphia, acalculia, finger agnosia, R)/L) disorientation,

How well did you know this?
1
Not at all
2
3
4
5
Perfectly
70
Q

What’s the purpose of the 4th bottle in the 4-chamber pleural drain?

A

protect against the consequences of suction failure/blockage- protecting the pt from pneumothorax.

How well did you know this?
1
Not at all
2
3
4
5
Perfectly
71
Q

Which anaesthetic medications are CONTRAINDICATED while breastfeeding?

A

codeine
aspirin (analgesic doses- can use low dose for anti-platelet action if strongly indicated)
parecoxib

use caution with:
tramadol (observe child for unusual drowsiness)
oxycodone (greater risk drowsiness in doses >40mg/day)

How well did you know this?
1
Not at all
2
3
4
5
Perfectly
72
Q

What’s static compliance? and dynamic?

A

change in volume/final change in pressure @ points of no gas flow (total compliance when all alveoli @ equilibrium w external environment)

= (VT) / (Pplat -PEEP)

in mL/cmH2O

dynamic is pulmonary compliance during periods of gas flow. always less than or equal to static compliance as peak insp pressure is always greater than Pplat.
Cdyn = (VT) / (PIP - PEEP)

How well did you know this?
1
Not at all
2
3
4
5
Perfectly
73
Q

What are the borders of the cubital fossa?
What is the median cubital vein & it’s surrounding structures?

A

epicondylar line= superior border. brachioradialis= lateral border. pronator teres= medial border.

Median cubital vein connects the cephalic (lateral) & basilic (medial) veins

median nerve (giving off antebrachial cutaneous nerve of the forearm) lies underneath & to the medial edge of it.
Radial nerve (giving off superficial & deep radial nerve) lies underneath & to the lateral edge of it.
MCNF is a branch from median nerve, LCNF from medradial, both may be damaged w cannulation
separated from brachial artery (which bifurcates into radial & ulnar arteries) by bicipital aponeurosis

biceps tendon lies deep to the median cubital vein & lateral to brachial artery.

ulnar nerve lies outside of it

How well did you know this?
1
Not at all
2
3
4
5
Perfectly
74
Q

How to control seizures due to LAST?

A

small incremental doses of benzodiazepine, thiopental or propofol

How well did you know this?
1
Not at all
2
3
4
5
Perfectly
75
Q

What’s ANOVA used for?

A

check if the means of or 2 more groups are significantly different from each other- normally distributed interval data (it CAN be used for 2 groups but more commonly use t-test). can use it for categorical independent variable w normally distributed interval dependent variable

How well did you know this?
1
Not at all
2
3
4
5
Perfectly
76
Q

OME of 50mg tapentadol?

A

15mg ( x 0.3)

How well did you know this?
1
Not at all
2
3
4
5
Perfectly
77
Q

What’s the gold standard test for establishing tolerance to penicillin? What test first? whats the NPV & PPV of these tests?

A

graded drug provocation test (DPT) using the index penicillin.

First skin test using SPTs and IDTs- skin tests for penicillin have a NPV approaching 100% & pts who don’t react to SPT or IDT are unlikely to have severe immediate reaction on the DPT. PPV is <50%.
In a pt who’s had a clinical reaction, a +ve skin test, with immediate readings, can identify presence of IgE sensitisation so then avoid DPTs.

Delayed readings required for Dx of non-immediate type IV hypersensitivity reactions but predictive value of these readings isn’t well established.

How well did you know this?
1
Not at all
2
3
4
5
Perfectly
78
Q

What are some limitations of skin tests?

A

reduced sensitivity over time in Dx of immediate reactions
low S&S in pts with non-severe, non-immediate & vague reactions

How well did you know this?
1
Not at all
2
3
4
5
Perfectly
79
Q

What’s an alternative to skin testing? pros/cons?

A

serum-specific IgE assay

low sens & spec but some pts can have -ve skin test, +ve serum-specific IgE & have anaphylaxis when exposed to the drug

How well did you know this?
1
Not at all
2
3
4
5
Perfectly
80
Q

What accounts for cross-reactivity between penicillins & cephalosporins? What should occur for pts who have suspected cefazolin anaphylaxis?

A

Probably mainly the R1 side chain, however cefazolin has different R1 side chain to other penicillins and cephalosporins (except ceftezole).
Cross-reactivity may also occur through IgE antibodies that recognise neodeterminants of the B-lactam ring structure.

After a diagnosis of cefazolin anaphylaxis, SPT to the culprit (only small numbers of pts with cefazolin anaphylaxis have a PST), then penicillin DPT. If negative DPT, penicillins can be prescribed.

Low ++ risk cross reactivity penecillin & 3rd gen cephalosporin

How well did you know this?
1
Not at all
2
3
4
5
Perfectly
81
Q

What are cardiovascular physiological changes seen in the first 24-48 hours & from 48-72hrs following significant burn?

A

Early hypodynamic phase (1st 24-48hrs):

increased SVR & PVR (ADH, catecholamines, haemoconcentration)
reduced intravascular volume, fluid loss from circulation & hypoperfusion
responds to fluid challenge, may need stress dose steroids IV hydrocortisone q6h
reduced CO, even before reduction in plasma volume
decreased SvO2

later hyper dynamic hyper metabolic phase (from 48-72hrs):
reduced SVR (impaired vasoconstriction)
limited response to fluid resus (may need inotropes)
increased ++ CO (2-3x)
tachycardia
massive energy expenditure
increased SvO2 (peripheral AV shunting)

can last days to weeks
if left untreated, multi-organ failure= the inevitable result

How well did you know this?
1
Not at all
2
3
4
5
Perfectly
82
Q

pathophysiology, symptoms & common causes of intracranial hypotension?

A

imbalance in production, absorption or flow of CSF leads to low ICP & sagging of the brain within the skull, . Usually postural headaches worse upright, eased by recumbency (or agg by coughing/strain, jugular venous compression) but they may be nonpostural & other neurologic symptoms can occur (or N&V, neck pain/stiffness, tinnitus, dizziness). May get more severe symptoms due to brainstem compression. Most commonly due to persistent CSF leak after lumbar puncture, may be spontaneous (eg. due to dural weakness).

How well did you know this?
1
Not at all
2
3
4
5
Perfectly
83
Q

Next step if a pt presents with headache, typically orthostatic, +/- associated symptoms, without Hx of dural puncture?

A

Brain MRI shows diffuse meningeal enhancement & “brain sagging” (SEEPS: subdural fluid, enhancement of pachymeninges, engorgement of venous structures, pituitary enlargment, sagging of brain). Next step is MRI brain with & without gadolinium & MRI spine without gadolinium, looking for CSF leak. If no evidence CSF leak but still strong clinical suspicion, could do heavily T2-weighted MR myelography without dural puncture or gadolinium (benefit as doesn’t risk gadolinium exposure, radiation or risks of intrathecal injection) or CT myelography (if noninvasive studies nondiagnostic- this involves contrast infusion via dural puncture also uses ionising radiation) or MR myelography with gadolinium injection or, if nondiagnostic or suspect CSF-venous fistula, digital subtraction myelography (useful for rapid leaks) & if still no evidence, proceed to radioisotope cisternography (intrathecal injection). If still no evidence of CSF leak, consider other causes.

How well did you know this?
1
Not at all
2
3
4
5
Perfectly
84
Q

What’s low CSF pressure?

A

<60cmH2O, it may be normal even if active CSF leak.

How well did you know this?
1
Not at all
2
3
4
5
Perfectly
85
Q

Is head CT useful for Dx spontaneous intracranial hypoT?

A

no- it’s often normal.

How well did you know this?
1
Not at all
2
3
4
5
Perfectly
86
Q

Is head CT useful for Dx spontaneous intracranial hypoT?

A

no- it’s often normal.

How well did you know this?
1
Not at all
2
3
4
5
Perfectly
87
Q

Whats normal CSF opening pressure in lat decubitus?

A

6-20cmH2O in adults or children, may be up to 25cmH2O in obesity

How well did you know this?
1
Not at all
2
3
4
5
Perfectly
88
Q

DDx for spont intracranial hypotension?

A

migraine, cervicogenic headache, Chiari I malformation (low-lying cerebellar tonsils), postural tachycardia syndrome (orthostatic intolerance), PDPH

How well did you know this?
1
Not at all
2
3
4
5
Perfectly
89
Q

Once diagnosis established, what is the Rx for spontaneous intracranial hypotension?

A

lumbar epidural blood patch effective in 64%- do with 10-20mL autologous blood, at “blind” lumbar level vs attempting the site of leak if the leak is above to SC terminus or site unknown, but if mild/moderate symptoms <2/52 (and NO Hx precipitating injury or connective tissue disease), consider conservative therapy (bedrest/minimise upright posture, oral hydration, caffeine PO 200-300mg 2-3x/day, abdo binder, analgesia). Approx 50% will require >1 EBP. If sysmptoms persist, could try location of leak & EBP there.

How well did you know this?
1
Not at all
2
3
4
5
Perfectly
90
Q

contraindications to EBP? Adverse effects?

A

pt refusal, localised infection at the site, increased ICP, coagulopathy, taking anticoagulants, active systemic infection (pt must be afebrile).
Adverse effects: back pain, radiculopathy, leg paraesthesia, fever. Rarely transient bilat paraplegia & cauda equina syndrome from arachnoiditis if high-volume EBP, may develop headache/other symptoms from intracranial HTN.

How well did you know this?
1
Not at all
2
3
4
5
Perfectly
91
Q

How to tell if an image is T1 or T2?

A

For T2, the grey-white matter will be switched (ie. grey lighter than white). Usual (grey darker than white)= T1-weighted. Fat bright on T1, fluid brighter on T2.

How well did you know this?
1
Not at all
2
3
4
5
Perfectly
92
Q

mechanism of EBP?

A

tamponade of the leak (immediate symptom improvement may occur) then fibrin deposition & scar formation (within 3/52)

How well did you know this?
1
Not at all
2
3
4
5
Perfectly
93
Q

mechanism of EBP?

A

tamponade of the leak (immediate symptom improvement may occur) then fibrin deposition & scar formation (within 3/52)

How well did you know this?
1
Not at all
2
3
4
5
Perfectly
94
Q

is efficacy of EBP lower for spont intracranial hypoT or PDPH?

A

spontaneous intracranial hypotension, up to 57% may require repeat blood patch

How well did you know this?
1
Not at all
2
3
4
5
Perfectly
95
Q

How to manage a pt with persistent symptoms of spontaneous intracranial hypoT despite EBP?

A

diagnostic evaluation to lacate site of CSF lead & guide targeted treatement of EBP at the spinal level of the leak or surgical/endovascular repair.

How well did you know this?
1
Not at all
2
3
4
5
Perfectly
96
Q

which conditions are NOT associated with raised baseline serum mast cell tryptase level?

A

liver failure
ETOH consumption

How well did you know this?
1
Not at all
2
3
4
5
Perfectly
97
Q

What’s response surface modelling used for in anaesthesia?

A

evaluating the interaction of (typically 2) drugs for a given endpoint

How well did you know this?
1
Not at all
2
3
4
5
Perfectly
98
Q

does tracheal tube size refer to internal or external diameter?

A

internal

How well did you know this?
1
Not at all
2
3
4
5
Perfectly
99
Q

to which side should I face the ett bevel for the best view?

A

left

How well did you know this?
1
Not at all
2
3
4
5
Perfectly
100
Q

to which side should I face the ett bevel for the best view?

A

left

How well did you know this?
1
Not at all
2
3
4
5
Perfectly
101
Q

What’s the reduction in renal blood flow with infrarenal aortic XC? Why?

A

up to 40%, due to incr renal vascular resistance of up to 75%

How well did you know this?
1
Not at all
2
3
4
5
Perfectly
102
Q

Why may renal & hepatic flow be relatively preserved with infrarenal aortic XC in modern anaesthesia cf splanchnic?

A

kidney has intrinsic autoregulation, liver has dual supply of blood flow (hepatic arterial buffer response), GI region may lack these mechanisms

How well did you know this?
1
Not at all
2
3
4
5
Perfectly
103
Q

What happens to descending aortic blood flow & cardiac index during infrarenal aortic XC?

A

significant decrease, likely due to increased afterload & myocardial dysfunction

How well did you know this?
1
Not at all
2
3
4
5
Perfectly
104
Q

what’s the clinical importance of splanchnic hypoperfusion (which occurs with descending aortic blood flow reductions during XC)?

A

associated with delayed postop recovery of GI function, prolonged GI intramucosal acidosis is ass’d with periop M&M

How well did you know this?
1
Not at all
2
3
4
5
Perfectly
105
Q

What’s the outer diameter of the ETT connector for the breathing system?

A

15mm

How well did you know this?
1
Not at all
2
3
4
5
Perfectly
106
Q

what are some safety features of laser ETTs?

A

may be wrapped in laser resistant metal foil, cuff may be filled with saline, which may be coloured with methylene blue dye, may have 2 cuffs

How well did you know this?
1
Not at all
2
3
4
5
Perfectly
107
Q

benefits of fenestrated tracheostomy tubes

A

allows pt to speak. Breathing around the cuff & through fenestration as well as stoma reduces airway resistance & assists weaning when spont breathing

How well did you know this?
1
Not at all
2
3
4
5
Perfectly
108
Q

another name for laryngectomy tube? Benefits?

A

Montandon tube, offers better surgical access as breathing system connected well away from surgical field

How well did you know this?
1
Not at all
2
3
4
5
Perfectly
109
Q

contraindications to methylene blue?

A

G6PD deficiency (lack of NADPH prevents methylene blue from working & leads to haemolysis)
renal impairment
methaemoglobin reductase deficiency
nitrite-induced methaemoglobinemia due to cyanide poising
hypersensitivity

Also methylene blue may precipitate serotonergic crisis in pts taking serotonergic meds (MAO inhibition)

How well did you know this?
1
Not at all
2
3
4
5
Perfectly
110
Q

what CAN methylene blue be used for?

A

methemoglobinaemia
catecholamine-refractory vasoplegia
septic shock
hepatopulmonary syndrome
antimalarial
antineoplastic
dye/stain (eg. enhance surgical visualisation of parathyroid glands, place in ETT cuff of laser tubes)
neutralises heparin
priapism
treating ifosfamide neurotoxicitysympt or asympt w&raquo_space;20% metHb (>10% if risk factors eg. anaemia or IHD)

How well did you know this?
1
Not at all
2
3
4
5
Perfectly
111
Q

Dose of methylene blue for vasoplegia?

A

1.5-2mg/kg IV over 20-60mins

How well did you know this?
1
Not at all
2
3
4
5
Perfectly
112
Q

what clinically important enzyme can methylene blue inhibit? Clinical implications?

A

monoamine oxidase with high selectivity for MAO-A, the isoform responsible for serotonin metabolism.
Doses as low as 1mg/kg may –> severe serotonin toxicity.

How well did you know this?
1
Not at all
2
3
4
5
Perfectly
113
Q

What are signs of Horner’s syndrome?

A

ptosis (& may have “reverse ptosis” elevation of lower lid)
miosis (pupillary near & light responses preserved but may be lag at dilating in dim conditions)
anhydrosis
flushing face
warm arm

How well did you know this?
1
Not at all
2
3
4
5
Perfectly
114
Q

What does a successful stellate ganglion block produce?

A

Horner’s syndrome, since it causes sympathetic blockade of the ipsilateral face & arm.

How well did you know this?
1
Not at all
2
3
4
5
Perfectly
115
Q

What are WHO IV pregnancy categories (pregnancy not recommended)?

A

pulmonary arterial HTN
severe systemic ventricular dysfunction (EF <30%)
severe MS
severe symptomatic AS
severe aortic dilatation (>50mm) in aortic disease associated with bicuspid aortic valve
vascular ehlers-danlos
Marfan with aorta dilated >45mm
native severe coarctation
complicated Fontan

How well did you know this?
1
Not at all
2
3
4
5
Perfectly
116
Q

At what fibrinogen level should replacement be started during PPH?

A

2g/L

How well did you know this?
1
Not at all
2
3
4
5
Perfectly
117
Q

What are the ANZCOR recommendations for tachycardia with a pulse if the pt stable with narrow QRS & irregular rhythm?

A

rate control with B blocker or IV digoxin. Consider amiodarone if onset <48hrs (300mg IV over 20-60mins then 900mg over 24hrs (consider reduced dose if pt on digoxin))

How well did you know this?
1
Not at all
2
3
4
5
Perfectly
118
Q

What are the ANZCOR recommendations for tachycardia with a pulse if the pt stable with narrow QRS & irregular rhythm?

A

rate control with B blocker or IV digoxin. Consider amiodarone if onset <48hrs (300mg IV over 20-60mins then 900mg over 24hrs (consider reduced dose if pt on digoxin))

How well did you know this?
1
Not at all
2
3
4
5
Perfectly
119
Q

what classes a pt with tachycardia as “unstable”?

A

reduced consciousness
SBP <90mmHg
chest pain
heart failure

rate-related symptoms uncommon @ <150bpm

How well did you know this?
1
Not at all
2
3
4
5
Perfectly
120
Q

what’s the simplest approach for rapid AF in acute setting?

A

B blocker (metoprolol 5mg IV) BUT contraindications= bronchospasm or evidence decompensated heart failure
OR
digoxin 250-500microg IV or PO(which is the drug of choice fro rate control in heart failure however has slower rate of onset- IV 5-30mins, PO >30 mins, vs B blocker 3-7mins))

How well did you know this?
1
Not at all
2
3
4
5
Perfectly
121
Q

What energy is used for synchronised cardioversion?

A

70-120J biphasic but in AF an initial larger shock (120-150J biphasic) recommended

How well did you know this?
1
Not at all
2
3
4
5
Perfectly
122
Q

risks of using amiodarone & digoxin together?

A

risk dig toxicity due to reduced digoxin clearance (risk severe bradycardia, conduction disturbances, idioventricular rhythm
risks torsades

How well did you know this?
1
Not at all
2
3
4
5
Perfectly
123
Q

What’s MINS? significance?

A

myocardial injury during the first 30 days after non cardiac surgery, of ischaemic etiology.
There is evidence of elevated cTn with at least one value above the 99th percentile upper reference limit.
Independently associated with mortality.
includes MI (symptomatic & non symptomatic) or post-op troponin elevation with no evidence of non ischaemic etiology.

How well did you know this?
1
Not at all
2
3
4
5
Perfectly
124
Q

What’s medical management for asymptomatic troponin elevation (MINS)?

A

control BP, HR
consider aspirin & statin
consider, for risk stratification/management: cardiac enzyme trends until falling, lipid panel, HbA1c, echo, functional stress test

How well did you know this?
1
Not at all
2
3
4
5
Perfectly
125
Q

What’s the medical management for type 1 NSTEMI?

A

control BP, HR (goal 50-70bpm if able to tolerate this BP & no concern for depressed LVEF), pain
consider aspirin
consider P2Y12 inhibitor (only if ischaemic benefit outweighs bleeding risk)
heparin drip
B-blocker
atorvastatin 80mg

How well did you know this?
1
Not at all
2
3
4
5
Perfectly
126
Q

What is NOT a complication from dural puncture and resultant intracranial hypotension?

A

encephalitis

How well did you know this?
1
Not at all
2
3
4
5
Perfectly
127
Q

Differential diagnosis for postpartum headache (in order of prevalence) & their Rx?

A

tension (simple analgesia, massage, physiological) & migraine (usually recurring, unilateral, may be pulsating & with nausea/photophobia/visual disturbances, may have “aura”- NSAIDs, 5-HT agonists (sumatriptan)) headache

pre-eclampsia (headache a serious premonitory sign & is present in 50% who go on to develop eclampsia) or eclampsia (hypertensive encephalopathy w headache, visual disturbance, N&V, seizures, stupor which may –> coma). analgesia & control underlying condition

PDPH

cortical vein thrombosis (headache often ass’d w focal neurology & seizures. may have a postural component- may be ass’d w PDPH (?secondary to cerebral vasodilation after CSF leak & prolonged dehydration). Dx with MRI & MR venography. Mx with anticoagulation, symptom control with focus on seizure prevention)

SAH (more common if an arteriovenous malformation, cerebral aneurysms or hypertensive encephalopathy. Acute onset intense, incapacitating unilateral headache accompanied by nausea, neck stiffness & altered consciousness). diagnose with CT, seek urgent neurosurgeon opinion.

posterior reversible leucoencephalopathy syndrome: severe, diffuse headache, may have focal neurological deficit, may be an association with pre-eclampsia (loss of cerebral autoregulation compromises BBB)- imaging shows symmetrical cerebral oedema.

space-occupying lesions (tumour, subdural haematoma): dull headache ass’d with symptoms of raised ICP (N&V). may have focal neurology & altered consciousness. Neurosurg if bleed or tumour.

cerebral infarction/ischaemia

sinusitis

meningitis (neck stiffness, photophobia, fever, +ve Kernig & Brudzinski & may have petechial rash, do CT to exclude other Dx & confirm with CSF culture, early ABx)

How well did you know this?
1
Not at all
2
3
4
5
Perfectly
128
Q

What’s the risk of accidental dural puncture with epidural?

A

1:100-1:200, 50% of these will get PDPH.

How well did you know this?
1
Not at all
2
3
4
5
Perfectly
129
Q

what causes the pain of PDPH?

A

CSF leak–> intracranial hypotension & pain is from traction on intracranial structures or compensatory cerebral vasodilation. with standing, increased hydrostatic pressure gradient augments CSF leakage.

How well did you know this?
1
Not at all
2
3
4
5
Perfectly
130
Q

symptoms associated with PDPH?

A

postural headache (worse upright, strain, cough & improves with lying down)
associated: neck stiffness, nausea, visual disturbance, photophobia, auditory symptoms
in severe cases, cranial nerve palsy of abducens (susceptible to traction when CSF volumes are low)

How well did you know this?
1
Not at all
2
3
4
5
Perfectly
131
Q

symptoms associated with PDPH? significant complications?

A

postural headache (worse upright, strain, cough & improves with lying down)
associated: neck stiffness, nausea, visual disturbance, photophobia, auditory symptoms
in severe cases, cranial nerve palsy of abducens (susceptible to traction when CSF volumes are low) may occur. CN VII may also be affected. only do EBP if other causes of cranial n palsy (haemorrhage & thrombosis) excluded.
cortical vein thrombosis or cerebral venous sinus thrombosis may be associated with PDPH (cerebral vasodilation after CSF leak, damage to cerebral venous endothelium & prolonged dehydration)
cranial-subdural haematoma: caudal shift of brain may–> rupture of the fragile subdural bridging veins
seizures

How well did you know this?
1
Not at all
2
3
4
5
Perfectly
132
Q

what effects do caffeine & 5-HT agonists (eg. sumatriptan) have on cerebral vasculature?

A

vasoconstriction

How well did you know this?
1
Not at all
2
3
4
5
Perfectly
133
Q

what’s the chance of complete headache cure from a single epidural blood patch? how many cases require a repeat blood patch?

A

50%
40%

How well did you know this?
1
Not at all
2
3
4
5
Perfectly
134
Q

How soon after accidental dural puncture should EBP be performed?

A

48-72hrs, higher failure rate if within 24hrs of dural puncture.

How well did you know this?
1
Not at all
2
3
4
5
Perfectly
135
Q

what volume of blood should be injected for an epidural blood patch?

A

up to 20mL; inject until the pt feels back or buttock or leg pain/pressure (nerve root irritation)

How well did you know this?
1
Not at all
2
3
4
5
Perfectly
136
Q

what advice should be given post EBP?

A

lie flat for…, have stool softeners, admit overnight for observation, avoid strain or too much physical exertion

How well did you know this?
1
Not at all
2
3
4
5
Perfectly
137
Q

early complications after EBP?

A

backache (which may be severe) during injection, neck or backache may persist for 24hrs even if get instantaneous relief of headache, fever, bradycardia, seizures. Late= meningitis, spinal-subdural haematoma or intrathecal haematoma, arachnoidits, cerebral venous sinus thrombosis, radicular pain, seizures, infection (either localised to the lower back or meningitis)

How well did you know this?
1
Not at all
2
3
4
5
Perfectly
138
Q

What are some signs/symptoms & examination findings in acromegaly?

A

pituitary adenoma on brain MRI or CT
visual field defects
prominent supraorbital ridge
large nose & jaw
teeth separated or lacking
abnormal glucose tolerance test, glycosuria, polyuria
hypertrophy of sebaceous & sweat glands
galactorrhea (prolactin)
cardiomegaly
HTN
spade-shaped hands & feet
arthrosis
peripheral neuropathy
sexual dysfunction

How well did you know this?
1
Not at all
2
3
4
5
Perfectly
139
Q

What are the findings of growth hormone tests for a patient with acromegaly?

A

They’d have elevated growth hormone (>0/4microg/L) and/or elevated IGF-1
during the OGTT, growth hormone will NOT be suppressed to <0.4microg/L

How well did you know this?
1
Not at all
2
3
4
5
Perfectly
140
Q

paediatric dosing for IM adrenaline?

A

150microg (0.15mL of 1mg/mL) if <6yo
300microg (0.3mL) if 6-12yo
every 5 mins prn

How well did you know this?
1
Not at all
2
3
4
5
Perfectly
141
Q

which body weight should be used for tidal volumes in ARDS?

A

predicted body weight based on gender & height; not ideal body weight.
5mL/kg predicted body weight.

How well did you know this?
1
Not at all
2
3
4
5
Perfectly
142
Q

equations for predicted body weight in males & females?

A

males:
PBW= 50 + 0.91 x (height in centimetres - 152.4) kg

females:
= 45.5+ 0.91 x (height in cm - 152.4) kg

How well did you know this?
1
Not at all
2
3
4
5
Perfectly
143
Q

Goal Pplat for ARDs?

A

<30cmH2O

How well did you know this?
1
Not at all
2
3
4
5
Perfectly
144
Q

Goal VT for ARDS?

A

6mL/kg predicted body weight in absence of severe acidosis

How well did you know this?
1
Not at all
2
3
4
5
Perfectly
145
Q

ARDs diagnostic criteria?

A

PaO2/FiO2 <=300mmHg with PEEP >=5cmH2O
within 1 wk of known clinical insult
billet opacities on chest imaging
not fully explained by cardiac failure or fluid overload

How well did you know this?
1
Not at all
2
3
4
5
Perfectly
146
Q

ARDs diagnostic criteria?

A

PaO2/FiO2 <=300mmHg with PEEP >=5cmH2O
within 1 wk of known clinical insult
billet opacities on chest imaging
not fully explained by cardiac failure or fluid overload

How well did you know this?
1
Not at all
2
3
4
5
Perfectly
147
Q

What should probably not be done for ARDs?

A

systematic recruitment maneouvers

How well did you know this?
1
Not at all
2
3
4
5
Perfectly
148
Q

What should be applied for moderate-severe ARDS (P/F <200mmHg)?

A

PEEP >12cmH2O if it improves oxygenation without haemodynamic compromise or significant decrease in lung compliance; must maintain Pplat <30cmH2O

How well did you know this?
1
Not at all
2
3
4
5
Perfectly
149
Q

What should be applied for ARDS with P/F ratio <150mmHg?

A

continuous NMB infusion (within first 48hrs of ARDS Dx ideally) & prone positioning >16hrs/day for several consecutive days

How well did you know this?
1
Not at all
2
3
4
5
Perfectly
150
Q

What to consider if P/F <80mmHg?

A

discussion of V-V ECMO (refractory hypoxaemia or where protective ventilation can’t be applied)

How well did you know this?
1
Not at all
2
3
4
5
Perfectly
151
Q

Which muscle group has the greatest resistance to action of NDMRs?

A

diaphragm- contains a high proportion of type II fibres (diaphragm= 45% type I) with relatively high density of nAChR (conferring some resistance to NMBDs), relatively large muscle fibres attenuate rapidity of onset, yet high perfusion along with the higher density of glycolytic type II fibres helps speed offset

How well did you know this?
1
Not at all
2
3
4
5
Perfectly
152
Q

Mnemonic for colour of doppler?

A

BART
blue= flow away from transducer
red= flow towards transducer

How well did you know this?
1
Not at all
2
3
4
5
Perfectly
153
Q

Do plasma components (FFP, cryo, cryodepleted plasma) have to be ABO compatible? how about RhD type?

A

yes (as they may contain donor anti-A or anti-B)
however plasma components of any RhD type can be given regardless of the RhD type of the recipient & RhD immunoglobulin isn’t required in these situations

How well did you know this?
1
Not at all
2
3
4
5
Perfectly
154
Q

Do platelets have to be ABO compatible? how about RhD type?

A

ideally- since incompatible plasma may cause haemolysis (eg. a group A pt given group O platelets- the ABO is compatible but plasma not compatible). Apheresis platelets have low-titre anti-A/B.
If the RhD is incompatible (RhD +ve platelets given to a RhD negative pt), there may be sensitisation to residual red cell antigens so prophylactic RhD Ig may be needed if RhD +ve puts given to a RhD -ve pt, esp female children or women of childbearing age.

How well did you know this?
1
Not at all
2
3
4
5
Perfectly
155
Q

What’s the first & 2nd choice for plasma components for a group A pt?

A

A (-ve or +ve)
then AB

How well did you know this?
1
Not at all
2
3
4
5
Perfectly
156
Q

What’s the equation for NNT?

A

1/ARR

How well did you know this?
1
Not at all
2
3
4
5
Perfectly
157
Q

What’s absolute risk reduction?

A

incidence in control - incidence in treatment group

How well did you know this?
1
Not at all
2
3
4
5
Perfectly
158
Q

When should the aortic balloon pump be inflated? why?

A

onset of diastole. it gives rise to a start “V” on the art waveform (then diastolic augmentation). aims to increase coronary perfusion.

How well did you know this?
1
Not at all
2
3
4
5
Perfectly
159
Q

when should aortic balloon pump be deflated? why?

A

end of diastole, reduces aortic EDP & systolic pressures which decreases afterload, cardiac work, myocardial consumption & increases CO. it’ll reduce aortic end-diastolic pressure cf unassisted.

How well did you know this?
1
Not at all
2
3
4
5
Perfectly
160
Q

what’s the problem with balloon pump being inflated before dicrotic notch?

A

diastolic augmentation encroaches on systole & may cause premature closure of AV, increase LVEDV/P, incr LV wall stress or afterload.

How well did you know this?
1
Not at all
2
3
4
5
Perfectly
161
Q

what’s the problem if IABP inflated after dicrotic notch?

A

trace shows lack of sharp V. suboptimal coronary perfusion.

How well did you know this?
1
Not at all
2
3
4
5
Perfectly
162
Q

problem of deflating IABP early?

A

sharp decrease after diastolic augmentation, making the diastolic augmentation suboptimal which reduces coronary perfusion & there may even be retrograde carotid blood flow, plus the afterload reduction may be suboptimal

How well did you know this?
1
Not at all
2
3
4
5
Perfectly
163
Q

problem with deflating IABP too late?

A

incr afterload (the assisted aortic EDP may be equal to unassisted) may impede LV ejection, there may be a prolonged rate of rise & reduced assisted systole & the augmented diastole may appear prolonged.

How well did you know this?
1
Not at all
2
3
4
5
Perfectly
164
Q

with what should the IABP deflation correspond in pts with arrhythmia?

A

While electrical triggers are better, if arrhytthmias use art pressure waveform instead of ecg. R wave, mechanical event is just prior to AV opening, just before upstroke on systolic pressure waveform (inflation is just at dicrotic notch if ecg reliable, T wave ecg (TIN))

How well did you know this?
1
Not at all
2
3
4
5
Perfectly
165
Q

What’s a sentinel event? examples?

A

subset of adverse patient safety event that is:
wholly preventable
resulted in serious harm to or death of a patient
the most serious incidents

surgery at wrong site, wrong pt or wrong surgery that led to serious harm or death.
retention of foreign object–> serious harm or death
haemolytic blood transfusion reaction from ABO incompatibility–> serious harm or death
suspected suicide of pt on acute psych unit/ward
med error–> serious harm or death
physical/mech restraint–> SHOD
d/c or release of an infant to an unauthorised person
use of incorrectly positioned oral or NGT–> serious harm or death

How well did you know this?
1
Not at all
2
3
4
5
Perfectly
166
Q

what’s malefeasance?

A

wilful & intentional act injuring a party

How well did you know this?
1
Not at all
2
3
4
5
Perfectly
167
Q

what’s an adverse event?

A

incident leading to pt harm

How well did you know this?
1
Not at all
2
3
4
5
Perfectly
168
Q

what’s medical misconduct?

A

Behavior that is professionally unethical and/or illegal, e.g., negligence, incompetence, impairment from drugs or alcohol, egregious substandard care.

How well did you know this?
1
Not at all
2
3
4
5
Perfectly
169
Q

What’s the most likely case of collapse & LL weakness immediately on standing following foam sclerotherapy?

A

thromboembolic event

How well did you know this?
1
Not at all
2
3
4
5
Perfectly
170
Q

which anticoagulant most effectively cleared by haemodialysis?

A

dabigatran

How well did you know this?
1
Not at all
2
3
4
5
Perfectly
171
Q

Which tooth most commonly damaged during laryngoscopy?

A

L) central maxillary incisor

How well did you know this?
1
Not at all
2
3
4
5
Perfectly
172
Q

What’s a benefit of cefazolin?

A

superior gram-positive antimicrobial activity compared with cephalosporins of later generations

How well did you know this?
1
Not at all
2
3
4
5
Perfectly
173
Q

which generation cephalosporin is cefazolin?

A

1st

How well did you know this?
1
Not at all
2
3
4
5
Perfectly
174
Q

What are absolute contraindications to drug provocation test?

A

drug reaction with eosinophilia & systemic symptoms (DRESS), Stevens-Johnson syndrome, toxic epidermal necrolysis & other severe cutaneous reactions, severe IHD, pregnancy

How well did you know this?
1
Not at all
2
3
4
5
Perfectly
175
Q

What’s the estimated rate of anaphylaxis to cefazolin in a pt with IgE-mediated reaction to penicillin?

A

1%

How well did you know this?
1
Not at all
2
3
4
5
Perfectly
176
Q

What are some of the early (first 24-48hrs) physiological changes with burns?

A

“hypodynamic” phase
hypoperfusion (fluid loss)
decreased intravascular volume
incr SVR & PVR (ADH, catecholamines, haemoconcentration)
reduced CO
decreased SvO2
responsive to fluid challenges

How well did you know this?
1
Not at all
2
3
4
5
Perfectly
177
Q

side effects of adenosine administration in order of likelihood?

A

facial flushing (36%)
dyspnoea (35%)
chest pain (35%)
GI discomfort
headache
AV block
ST-T changes
arrhythmias
bronchospasm rare (0.1%)

also likely have:
impending sense of doom
coronary steal

How well did you know this?
1
Not at all
2
3
4
5
Perfectly
178
Q

Overal there are higher risks of what soon after TAVI? SAVR? How about mortality & disabling stroke @ 2yrs?

A

TAVI: HIGHER rates of short-term re-intervention, AV conduction disturbances & need for a PPM, major vasc complications, paravalvular leak (AR), mod-severe HF symptoms. Does have lower rates of major bleeding & AF.

SAVR: higher gradient across valve & smaller valve area, new AF, major bleeding, AKI, 30-day transfusion requirement.

All-cause mortality & disabling stroke similar across groups (nominally but not significantly higher in the TAVI group).

How well did you know this?
1
Not at all
2
3
4
5
Perfectly
179
Q

What does power mean in research?

A

the probability of correctly finding a given, existing difference as significant. Is 1-B (type II error) & represents the sensitivity. Arbitrarily set @ 80% (ie. if a difference or effect exists, there’s 20% probability of type II error).

How well did you know this?
1
Not at all
2
3
4
5
Perfectly
180
Q

What’s a type II error?

A

the null hypothesis incorrectly accepted

How well did you know this?
1
Not at all
2
3
4
5
Perfectly
181
Q

How to increase the power of a study?

A

increase the sample size, have a larger minimum effect size or difference representing clinical importance, having a smaller standard deviation in the population (hence smaller sample size needed), p value, (?higher incidence of outcome of interest in the population)

How well did you know this?
1
Not at all
2
3
4
5
Perfectly
182
Q

4 determinants of power calculation

A

p value
standard deviation (variance) in population
difference to be detected
sample size

How well did you know this?
1
Not at all
2
3
4
5
Perfectly
183
Q

What’s the type 1 error rate usually set at?

A

What’s the type 1 error rate usually set at?

How well did you know this?
1
Not at all
2
3
4
5
Perfectly
184
Q

If there’s more than one outcome of interest, is the probability of a type 1 error (false +ve) higher or lower? How to account for this?

A

higher. Can reduce risk by lowering the significance level. Bonferroni correction lowers the threshold for significance by dividing the overall type 1 error rate by the number of comparisons or hypotheses tested.

How well did you know this?
1
Not at all
2
3
4
5
Perfectly
185
Q

Epidural filters are designed to retain particles down to a diameter of

A

0.2 micrometers (200 nanometers)

How well did you know this?
1
Not at all
2
3
4
5
Perfectly
186
Q

What’s the HR goal in AF?

A

reduce it to 80bpm, usually with rate slowing therapy eg. oral B blocker or CCB as outpt if no or mild symptoms. If pt is haemodynamically unstable after rate control, could use rhythm control.

How well did you know this?
1
Not at all
2
3
4
5
Perfectly
187
Q

which pts with new onset AF should not have immediate anticoagulation?

A

if bleeding risk exceeds benefits or CHA2DS2-Vasc score of 0 in men or 1 in women & short paroxysms of AF that self-terminate.

How well did you know this?
1
Not at all
2
3
4
5
Perfectly
188
Q

What’s the most important initial therapeutic intervention in DKA?

A

fluid replacement followed by insulin administration- use crystalloid with sodium 130-154mmol/L

How well did you know this?
1
Not at all
2
3
4
5
Perfectly
189
Q

which lobe of lung has a medial & lateral segment?

A

R) ML

How well did you know this?
1
Not at all
2
3
4
5
Perfectly
190
Q

with which medication is a pt with known sux allergy most likely to demonstrate cross-reactivity?

A

rocuronium

How well did you know this?
1
Not at all
2
3
4
5
Perfectly
191
Q

what’s a condition associated with an elevated A-a gradient that CAN’T be corrected with incr FiO2?

A

shunt, eg. atelectasis, ARDs

How well did you know this?
1
Not at all
2
3
4
5
Perfectly
192
Q

what’s the alveolar gas equation?

A

PAO2 = FiO2 x (Pb-PsvpH2O) - (PaCO2/RQ)

How well did you know this?
1
Not at all
2
3
4
5
Perfectly
193
Q

In experienced operators, what’s more sensitive for detecting PTx? CXR or lung PoCUS?

A

lung US

How well did you know this?
1
Not at all
2
3
4
5
Perfectly
194
Q

setup position for PoCUS for Dx of PTx?

A

pt supine/semi-recumbent, probe anterior chest wall @ 2nd ICS MCL, sagittal orientation, marker cephalad

How well did you know this?
1
Not at all
2
3
4
5
Perfectly
195
Q

in M mode, what would we see with normal lung movement vs PTx?

A

seashore sign (motion of lung) for normal
bardode/stratosphere sign if no lung motion (PTx)

How well did you know this?
1
Not at all
2
3
4
5
Perfectly
196
Q

What are the A lines on lung US?

A

horizontal, reverberation artefact of the pleural line, indicate dry interlobular septa, if they are predominant, it suggests PCWP <=13mmHg (90% sens, 67% spec) & suggests IVT may safely be given without concern for pulm oedema

How well did you know this?
1
Not at all
2
3
4
5
Perfectly
197
Q

what are B lines on lung US?

A

indicate alveolar/interstitial fluid or fibrosis at the lung surface (if >-3 B lines). B lines are sensitive for pulm oedema & are ABSENT IN PTx.

How well did you know this?
1
Not at all
2
3
4
5
Perfectly
198
Q

what sign on PoCUS is pathognomonic for PTx?

A

lung point- on M mode, transition btwn where there is & isn’t lung sliding

How well did you know this?
1
Not at all
2
3
4
5
Perfectly
199
Q

What’s a parachute device

A

aka ventricular partitioning device, percutaneously inserted cardiac device aimed at improving cardiac output and reducing cardiac remodeling in patients following MI.
Improves compliance, reduces LVEDP & improves ventricular filling, reduces wall stress, helps maintain SV & CO

How well did you know this?
1
Not at all
2
3
4
5
Perfectly
200
Q

The brachial plexus is formed from what?

A

anterior rami of the C5-T1 spinal nerves

How well did you know this?
1
Not at all
2
3
4
5
Perfectly
201
Q

what does the brachial plexus (ant rami of C5-T1 spinal nerves) pass between as it enters the base of the neck?

A

anterior & middle scalene muscles

How well did you know this?
1
Not at all
2
3
4
5
Perfectly
202
Q

what do the posterior rami of spinal nerves supply?

A

skin & musculature of intrinsic back mm

How well did you know this?
1
Not at all
2
3
4
5
Perfectly
203
Q

what are the 3 trunks of the brachial plexus & of what do they comprise?

A

superior (C5&6)
middle (C7)
inferior (C8, T1)

How well did you know this?
1
Not at all
2
3
4
5
Perfectly
204
Q

what’s the posterior triangle of the neck & which anatomical structures are contained within?

A

SCM, middle 1/3 of clavicle, trapezius
contains divisions of brachial plexus (ant & post)

How well did you know this?
1
Not at all
2
3
4
5
Perfectly
205
Q

what do the divisions (ant & post) of the brachial plexus form when they have left the posterior triangle & entered the axilla?

A

cords

How well did you know this?
1
Not at all
2
3
4
5
Perfectly
206
Q

what are the cords of the brachial plexus named relative to?

A

axillary artery
lateral (ant divisions of sup & middle trunk)
posterior (posterior division of sup, middle & inf trunk)
medial (ant division of inferior trunk)

How well did you know this?
1
Not at all
2
3
4
5
Perfectly
207
Q

What are the branches of the brachial plexus & their sensory/motor supply?

A

axillary (C5,6): teres minor (stabilise GHJ, ER shoulder) & deltoid (stabilise HOH & abduct GHJO, superior lat cutaneous nerve of arm (regimental badge area)
musculocutaneous (C5-7): brachialis (pure flex), biceps brachii (flexion & supination), coracobrachialis (flex & adduct shoulder & stabilise humeral head (w deltoid & triceps)), sensory lateral forearm
median (C5-T1): forearm flexors, thenar mm, 2 lateral lumbricals (index & middle fingers), sensory to lateral palm & lateral 3.5 fingers of palmar surface of hand
radial (C5-T1): triceps, wrist & finger extensors, sensory to posterior arm & forearm, posterolateral hand
Ulna (C8-T1): hand muscles aside from thenar mm & 2 lateral lumbricals, FCU & medial FDP. medial 1.5 fingers & associated palm area.

How well did you know this?
1
Not at all
2
3
4
5
Perfectly
208
Q

from what is the cervical plexus formed?

A

anterior rami of C1-4 spinal nerves

How well did you know this?
1
Not at all
2
3
4
5
Perfectly
209
Q

What’s erb’s point?

A

the “nerve point” of the neck, where the cutaneous branches of the cervical plexus exit the middle of the posterior border of SCM, utilised for cervical plexus blocks

How well did you know this?
1
Not at all
2
3
4
5
Perfectly
210
Q

What are the sensory branches of cervical plexus? (pic in “on hand”)

A

greater auricular from C2-3, sensation to external ear & skin over parotid gland
transverse cervical nerve from C2-3, curves around SCM & supplies anterior neck, anterolateral skin & upper sternum
lesser occipital nerve C2-3, posterosuperior scalp
supraclavicular: C3-4, supplies skin of supraclavicular fossa

How well did you know this?
1
Not at all
2
3
4
5
Perfectly
211
Q

From what is the lumbar plexus formed?

A

anterior rami of T12-L4

How well did you know this?
1
Not at all
2
3
4
5
Perfectly
212
Q

What are the six major peripheral nerves of the lumbar plexus?

A

iliohypogastric- T12-L1, IO & TA, posterolateral gluteal skin
Ilioinguinal- L1, IO & TA, supplies antero-medial thigh, genital branch: scrotal & root of penis, labia/mons
genitofemoral- L1-2, cremasteric, skin of scrotum, mons & labia, femoral branch: upper anterior thigh
lateral cutaneous nerve of thigh: L2,3, no motor, anterolateral thigh down to knee
obutrator: L2-4, medial thigh muscles (OE, AL, AB, AM & gracilis), sensory to medial thigh, branches to hip & knee
Femoral: L2-4, anterior thigh muscles (iliacus, pectineus, sartorius, quads femoris), sensory anterior thigh–> medial leg

How well did you know this?
1
Not at all
2
3
4
5
Perfectly
213
Q

From where does the sacral plexus originate?

A

anterior rami of L4-S4 spinal nerves (the sacral plexus S1-4 join L4-5 to form the lumbosacral trunk)

How well did you know this?
1
Not at all
2
3
4
5
Perfectly
214
Q

What are the major peripheral branches of lumbosacral plexus?

A

superior gluteal- L4-S1, glute min, med & TFL, no sensory
inf gluteal- L5-S1, glute max
post fem cutaneous: S1-3, skin posterior thigh, leg & perineum
pudendal: S2-4, perineal skeletalmuscles incl sphincters, penis & clitoris & most perineal skin (S2,3&4 keeps poo off the floor)
sciatic: L4-S2, tibial innervates muscles of posterior compartment of thigh (aside from SH BF), hamstring component of add magnus & all muscles of post compartment of leg & sole of foot. common fibular (L4-S2) for SHB, muscles of ant/lat leg & EDB. tibial (L4-S3) supplies skin of posterolat leg, lateral foot & sole of foot. common fibular supplies lateral leg, dorsum of foot (sup fibular) & btwn 1st & 2nd webspace (deep fibular)

How well did you know this?
1
Not at all
2
3
4
5
Perfectly
215
Q

diagnostic cutoffs for OSA severity in children?

A

AHI or RDI 1-4.9= mild
5-9.9= mod
>10= severe

adults mild 5-14, mod 15-30, severe >30

How well did you know this?
1
Not at all
2
3
4
5
Perfectly
216
Q

What’s the S1 heart sound? and S2?

A

S1= closing of atrioventricular valves (mitral & tricuspid)
S2= closure of semilunar (aortic & pulmonic) valves

How well did you know this?
1
Not at all
2
3
4
5
Perfectly
217
Q

What’s the normal pattern of S2 splitting with respiratory cycle?

A

A2 is heard before P2, increased split with inspiration

How well did you know this?
1
Not at all
2
3
4
5
Perfectly
218
Q

What may cause wide splitting of S2 (ie. splitting during expiration, wider with inspiration)?

A

Anything delaying R) heart emptying:
-delayed closure of the pulmonary valve
-may occur with delayed conduction down the R) bundle (R) BBB, pre-excitation LV, pacing LV, LV prem beats)
-pulmonary stenosis
-PAH

How well did you know this?
1
Not at all
2
3
4
5
Perfectly
219
Q

what may increase intensity of P2?

A

pulmonary HTN
ASD

first rule out causes of a lower intensity A2, eg. MR, AR, low diastolic arterial pressure, severe immobile aortic valve

How well did you know this?
1
Not at all
2
3
4
5
Perfectly
220
Q

what causes fixed splitting of S2 (ie. split w both insp & exp & doesn’t lengthen with insp’n)?

A

ASD, R) heart failure, pulm HTN

How well did you know this?
1
Not at all
2
3
4
5
Perfectly
221
Q

What causes paradoxical splitting?

A

delayed conduction down L) bundle branch (L) BBB, pre-excitation RV, RV pacing, prem RV beats)
aortic stenosis

How well did you know this?
1
Not at all
2
3
4
5
Perfectly
222
Q

what causes a single S2?

A

loss of either A2 or P2, eg:
severe AS or AR
congen absence of pulm valve
may be difficult to hear P2 w obesity, emphysema, pericardial fluid

How well did you know this?
1
Not at all
2
3
4
5
Perfectly
223
Q

which vascular access is most commonly used for cardiac catheterisation to obtain haemodynamic data?

A

radial- less bleeding complications
often use the antecubital vein for R) heart cath when using radio for arteriography & LV haemodynamics

How well did you know this?
1
Not at all
2
3
4
5
Perfectly
224
Q

when may the transseptal approach to accessing LA or ventricle be desirable?

A

accurate decision making in MS, where PCWP is unreliable as surrogate for LA pressure

How well did you know this?
1
Not at all
2
3
4
5
Perfectly
225
Q

name of balloon passed through fossa ovalis, under fluoroscopic guidance, to Ax MV disease, access for mitral balloon valvuloplasty & perc repair of sec MR?

A

Brockenbrough

How well did you know this?
1
Not at all
2
3
4
5
Perfectly
226
Q

How does the pressure trace of the fem artery compare with aortic root?

A

delayed & higher systolic pressure, overshoot (by <20mmHg) due to summation effect of reflected waves within arterial system

How well did you know this?
1
Not at all
2
3
4
5
Perfectly
227
Q

In which situations is the femoral trace overshoot inaccurate? what should do?

A

young pts, aortic insufficiency or evidence of PVD (which would markedly reduce the overshoot)- should measure central aortic pressure above the AV (simultaneous with LV pressure)

How well did you know this?
1
Not at all
2
3
4
5
Perfectly
228
Q

normal mean RA pressure?

A

1-8mmHg

How well did you know this?
1
Not at all
2
3
4
5
Perfectly
229
Q

normal mean RA pressure?

A

1-8mmHg

How well did you know this?
1
Not at all
2
3
4
5
Perfectly
230
Q

what does the RA A wave follow on ecg?

A

p wave

How well did you know this?
1
Not at all
2
3
4
5
Perfectly
231
Q

what does the RV systolic upstroke immediately follow on the ecg?

A

qrs

How well did you know this?
1
Not at all
2
3
4
5
Perfectly
232
Q

normal RV diastolic & peak pressures?

A

1-8 & 15-30mmHg

How well did you know this?
1
Not at all
2
3
4
5
Perfectly
233
Q

what’s the dicrotic notch or incisura on the pulm artery pressure waveform?

A

pulmonic valve closure

How well did you know this?
1
Not at all
2
3
4
5
Perfectly
234
Q

normal PA pressures?

A

15-30/4-12mmHg

How well did you know this?
1
Not at all
2
3
4
5
Perfectly
235
Q

How does LA & pulm cap wedge pressure tracings vary?

A

nearly identical but the PCW is lower & slightly delayed due to transmission of pressure backward through pulm capillaries

How well did you know this?
1
Not at all
2
3
4
5
Perfectly
236
Q

normal mean LA pressure?

A

4-12mmHg

How well did you know this?
1
Not at all
2
3
4
5
Perfectly
237
Q

what is peak LV systolic pressure coincident with on the ecg?

A

T wave

How well did you know this?
1
Not at all
2
3
4
5
Perfectly
238
Q

what is peak LV systolic pressure coincident with on the ecg?

A

T wave

How well did you know this?
1
Not at all
2
3
4
5
Perfectly
239
Q

what’s normal LVEDP?

A

4-12mmHg

How well did you know this?
1
Not at all
2
3
4
5
Perfectly
240
Q

in the normal situation, are mean PCWP & end-diastolic PA pressure equal?

A

yes

How well did you know this?
1
Not at all
2
3
4
5
Perfectly
241
Q

With what does the LVEDP correspond on the ecg? what’s it known as on the LV pressure trace?

A

R wave, “z point”, situated @ the downslope of LV “a” wave

How well did you know this?
1
Not at all
2
3
4
5
Perfectly
242
Q

How would the LV pressure tracing appear in diastolic dysfunction?

A

abnormal- continuing pressure decline over mid-diastole (rather than the rapid, then slow phases of diastolic filling followed by the “a” wave generated by atrial contraction)

How well did you know this?
1
Not at all
2
3
4
5
Perfectly
243
Q

what are some factors that may raise LVEDP (normally <12mmHg)?

A

volume overload (AR, MR, high-volume shunts)
reduced LV contractility
concentric hypertrophy (eg. HTN, valvular stenosis, restrictive or infiltrative cardiomyopathy)

How well did you know this?
1
Not at all
2
3
4
5
Perfectly
244
Q

If aortic pressure trace showed reduced diastolic pressure, likely DDx?

A

AR

How well did you know this?
1
Not at all
2
3
4
5
Perfectly
245
Q

findings on LV pressure trace of pt with HOCM?

A

elevated & abnormal LVEDP waveform, may also have elevated LVEDP in MR

How well did you know this?
1
Not at all
2
3
4
5
Perfectly
246
Q

What generally & specifically needs to be anaesthetised for successful awake intubation?

A

Nasal passages (if nasal route) or oral
gag and cough reflexes need to be suppressed effectively and laryngospasm prevented

How well did you know this?
1
Not at all
2
3
4
5
Perfectly
247
Q

Nerves that need to be topicalised prior to awake nasal fibreoptic intubation?

A

Upper airway supplied by:
TRIGEMINAL
GLOSSOPHARYNGEAL
VAGUS nerves

Sensation to the nasal cavity ALL from trigeminal nerve:
anterior ethmoidal (from ophthalmic (V1)–> nasociliary–> ant ethmoidal) provides anterior 1/3 of septum & ant nasal cavity (eg. nares)
the greater & lesser palatine & nasopalatine nerves, branches of maxillary (V2) which travel through pterygopalatine ganglion (in sphenopalatine fossa posterior to middle turbinate) supply remainder of nasal cavity (turbinates, post 2/3 nasal septum)

Glossopharyngeal (CNIX) provides sensory to most of pharynx (pharyngeal branch), post 1/3 tongue/anterior surface epiglottis/vallecula (lingual branch), fauces, tonsils (tonsilar branch), & most of the pharynx

branches of vagus (CNX)
-posterior & lateral walls of the pharynx are from pharyngeal nerve (branch of vagus- joins with pharyngeal branch of glossopharyngeal to form pharyngeal plexus)
-sup laryngeal nerve has ext branch (cricothyroid) & internal branch (sensory to skin above VCs incl base of tongue, laryngeal surface of epiglottis, aryepiglottic folds & arytenoids)
-RLN: motor to other VC muscles aside from cricothyroid & sensory below glottis

ant 2/3 tongue is from lingual nerve (from V3, mandibular, not nec for nasal approach to fibreoptic)

How well did you know this?
1
Not at all
2
3
4
5
Perfectly
248
Q

What minimum macroshock current could cause VF if pass through heart?

A

100mA (current is from surface contact)

microshock current (direct contact with myocardium): 100microamps (0.1mA)

How well did you know this?
1
Not at all
2
3
4
5
Perfectly
249
Q

How to spot a posterior MI?

A

Posterior MI is suggested by the following changes in V1-3:

Horizontal ST depression
Tall, broad R waves (>30ms)
Upright T waves
Dominant R wave (R/S ratio > 1) in V2
In patients presenting with ischaemic symptoms, horizontal ST depression in the anteroseptal leads (V1-3) should raise the suspicion of posterior MI.

How well did you know this?
1
Not at all
2
3
4
5
Perfectly
250
Q

How soon after spinal cord injury is return of reflexes generally seen?

A

1-3 days (following the initial flaccid paralysis & areflexia/neurogenic shock)

How well did you know this?
1
Not at all
2
3
4
5
Perfectly
251
Q

What’s spinal shock? what happens after that?

A

loss of reflexes below level of SCI, results in flaccid areflexia, usually along with hypotension of neurogenic shock
gradual return of reflex activity when reflex arcs below redevelop, often–> spasticity & autonomic hyperreflexia
areflexia days 0-1
initial reflex return days 1-3
early hyperreflexia days 4-28
late hyperreflexia 1-12/12

How well did you know this?
1
Not at all
2
3
4
5
Perfectly
252
Q

If giving 8.45% sodium bicarb for hyperkalaemia cardiac arrest, what dose?

A

1mmol/kg, initially given over 2-3 minutes then as guided by arterial blood gases

1mmol/mL therefore 60mL for a 60kg adult

How well did you know this?
1
Not at all
2
3
4
5
Perfectly
253
Q

what causes the coagulopathy due to intrahepatic cholestasis of pregnancy?

A

lack of fat-soluble vitamin (K) absorption (due to impaired bile acid excretion), required for manufacture of coag factors II, VII, IX & X; water-soluble vit K prescribed for obstetric cholestasis.

How well did you know this?
1
Not at all
2
3
4
5
Perfectly
254
Q

Adverse effects of mild hypothremia

A

coagulopathy
incr risk surgical wound infection
incr stay in PACU & hospital LoS
incr ventricular arrhythmias
prol DoA vecuronium & rocuronium, increases plasma [] propofol (mainly due to reduced hepatic blood flow), decreases MAC
postop shivering (which augments metabolic rate)
increased myocardial O2 consumption (HTN, incr metabolic rate)
incr pain

How well did you know this?
1
Not at all
2
3
4
5
Perfectly
255
Q

What’s the CXR level of the cavo-atrial junction?

A

2 vertebral bodies below the carina

How well did you know this?
1
Not at all
2
3
4
5
Perfectly
256
Q

Hip adduction in pt undergoing TURBT most likely due to

A

stimulation of obturator nerve

How well did you know this?
1
Not at all
2
3
4
5
Perfectly
257
Q

what rate averaged over 48hours risks propofol infusion syndrome?

A

5mg/kg/hr for >48hrs (product recomends no >4mg/kg/hr)

How well did you know this?
1
Not at all
2
3
4
5
Perfectly
258
Q

what should be monitored if concerned re: propofol infusion syndrome?

A

pH, lactate, CK after 48hrs (CK [] takes 12-24hrs to peak after onset rhabdo so earlier measurement not of benefit), limit infusion rate to lowest possible, use multimodal sedation.

How well did you know this?
1
Not at all
2
3
4
5
Perfectly
259
Q

Signs of cholinergic poisoning (eg. organophasphates, physostigmine, carbamate insecticides, mushrooms, sarin nerve gas)

A

SLUDGE, Bradycardia/bronchorrhoea/bronchospasm. Confusion, CNS depression, weakness, miosis, GI cramping, vomiting, sweating, brady or tachy

How well did you know this?
1
Not at all
2
3
4
5
Perfectly
260
Q

Rx for cholinergic toxicity

A

100% O2, atropine 0.02mg/kg boluses (for brady, hypoT, secretions). BZD for seizures/agitation, may use bicarb for acidosis, for organophosohates use pralidoxime choride for muscle weakness. Activated charcoal NOT recommended. Avoid sux since the organophosphates inhibit acetylcholinesterase so prol NMB. can use NDNMBDs but they may be less effective due to competitive inhibition @ NMJ (likely need incr doses).

How well did you know this?
1
Not at all
2
3
4
5
Perfectly
261
Q

Draw the waveforms for PCV. Pros & cons of this mode?

A

pros:
higher mean airway pressures & duration of alveolar recruitment- may be better for oxygenation & gas exchange
less risk barotrauma
better @ compensating for leak (higher initial flow)
WOB & pt comfort may be improved

cons:
harder to get consistent TV (depends on compliance) so minute ventilation variable, disadvantageous if need tight PaCo2 (eg. TBI)
may get volutrauma

How well did you know this?
1
Not at all
2
3
4
5
Perfectly
262
Q

Draw the waveforms for VCV. Pros & cons of this mode?

A

more stable MV, useful for tight control of PaCo2 (eg. TBI)

cons:
lower mean airway pressure, may not be as good for hypoxia (insp pause doesn’t really help)
poorer recruitment of lung units with poor compliance, greater risk atelectasis
constant flows may not compensate for leak

How well did you know this?
1
Not at all
2
3
4
5
Perfectly
263
Q

Draw the waveforms for PCV-VG. Pros & cons of this mode?

A

decelerating flow main difference
takes 3 breaths (VCV) to calculate dynamic compliance & then works out the pressure required to delivery desired volume
pros= fuses benefits without magnifying cons of the other modes, ie:
square pressure waveform, favours alveolar recruitment
same relatively high mean airway pressures as with PCV- good for oxygenation
low risk of barotrauma as pressure minimised for prescribed volume
guaranteed MV, preserves PaCo2 control

cons: may get variable TV with variable pt effort as each breath depends on the pressure characteristics of the preceding breath

How well did you know this?
1
Not at all
2
3
4
5
Perfectly
264
Q

pressure goal for ARDS ventilation?

A

<30cmH2O (higher RR, lower TV)

How well did you know this?
1
Not at all
2
3
4
5
Perfectly
265
Q

The ANZCA guidelines regarding pre-operative oral intake for infants under 6 months of age
having an elective procedure under anaesthesia are

A

clear fluids (no more than 3mL/kg) up to 1hr prior to anaesthesia, breastmilk up until 3 hours, formula 4 hours, all else 6 hrs prior to procedure

How well did you know this?
1
Not at all
2
3
4
5
Perfectly
266
Q

ANZCA chn >6/12 pre-op oral intake guidelines?

A

clear fluids (no >3mL/kg) up to 1 hr b4 anaesthesia, limited solids, formula, breastmilk up to 6hrs pre- anaesthesia

How well did you know this?
1
Not at all
2
3
4
5
Perfectly
267
Q

The commonest primary cause of death from anaesthesia airway events in the NAP4 report was

A

aspiration

How well did you know this?
1
Not at all
2
3
4
5
Perfectly
268
Q

A patient has bipolar disorder and is on long term lithium therapy. An analgesic which should be avoided is

A

NSAIDs (eg. indomethacin COX-1 selective & diclofenac COX-2 selective- increase SS lithium [], risks lithium toxicity)

should discontinue lithium 24hrs before surgery

also, metronidazole may incr ss lithium [] & risk lithium toxicity
ACE inhibitors
AII RBs
Diuretics (thiazides, potassium-sparing, loop)
all –> lithium accumulation w supratherapeutic concentrations

risks for lithium tox= change dosing regimen, acute renal failure, hyponatremia

purely excreted by kidneys, narrow therapeutic index, interferes w ADH, cardiac dysrhy, GI disturbance, tremor.

Prolong NMB & reduce Anaes requirement.
Stop lithium >=24hrs prep.

How well did you know this?
1
Not at all
2
3
4
5
Perfectly
269
Q

Benzatropine ameliorates the side effects of drugs that antagonise

A

D2 receptor
D2 receptor blocking drugs may cause acute dystonic reactions (involuntary contraction of extremities, face, neck, larynx- intermittent or sustained.
Thought to be due to imbalance of dopaminergic & cholinergic transmission

How well did you know this?
1
Not at all
2
3
4
5
Perfectly
270
Q

what are some drugs withOUT CYP2D6 contributing to metabolism?

A

hydromorphone (but hydrocodone metabolism IS by CYP2D6, as is tramadol, codeine, oxycodone, amitryptilline, methadone, beta blockers

How well did you know this?
1
Not at all
2
3
4
5
Perfectly
271
Q

The catheter type most likely to be associated with bloodstream sepsis per days insertion is

A

Femoral CVC, then IJ then subclavian

How well did you know this?
1
Not at all
2
3
4
5
Perfectly
272
Q

Other factors related to risk of CLABSI?

A

tunnelled lower risk
placement emergent vs non-emergent
lack of asepsis
skill of operator
catheter care, surveillance, infection control with use (eg. disinfect hub)
prolonged duration in situ

How well did you know this?
1
Not at all
2
3
4
5
Perfectly
273
Q

what’s a central line-associated bloodstream infection (CLABSI)

A

laboratory-confirmed bloodstream infection not related to an infection at another site that develops within 48 hours of a central line placement

How well did you know this?
1
Not at all
2
3
4
5
Perfectly
274
Q

what’s the order of pathogens responsible for CLABSI?

A

GP (eg. coagulase negative staph, enterococci, s. aureus)
then
GN (klebsiella, enterobacter, pseudomonas (more common if neutropenic or severe illness))
candida (more likely with femoral catheterisation, TPN, haematologic malignancy)

How well did you know this?
1
Not at all
2
3
4
5
Perfectly
275
Q

To where are PICC lines advanced?

A

superior cavo-atrial junction (2 vertebral bodies below carina)

How well did you know this?
1
Not at all
2
3
4
5
Perfectly
276
Q

what are atypical presentations of sepsis (eg. elderly or immunocompromise)?

A

hypotension, lethargy, fatigue, altered mental state

How well did you know this?
1
Not at all
2
3
4
5
Perfectly
277
Q

How usually treat MRSA? anti-staph that’s not MRSA? and how treat pseudomonas?

A

vancomycin
cefazolin
beta-lactam & aminoglycoside (eg. cefepime or carbapenem)

How well did you know this?
1
Not at all
2
3
4
5
Perfectly
278
Q

Rx for a pt with hysteroscopy syndrome?

A

3% NaCl 100mL

How well did you know this?
1
Not at all
2
3
4
5
Perfectly
279
Q

what’s operative hysteroscopy intravascular absorption (OHIA) syndrome?

A

fluid overload complications from operative hysteroscopies

How well did you know this?
1
Not at all
2
3
4
5
Perfectly
280
Q

example manifestations (& their management) of OHIA syndrome?

A

altered consciousness
acute pulmonary oedema (CPAP, frusemide, ?GTN): desaturation, creps on lung auscultation
metabolic acidosis (sodium bicarb), hyponatremia

How well did you know this?
1
Not at all
2
3
4
5
Perfectly
281
Q

measures to limit risk of OHIA syndrome?

A

isotonic electrolyte-containing distension media
bipolar electrosurgical instruments
closely monitor fluid status every 30 mins (net & total irrigation fluid amounts should be <3L & <8L, respectively (in these cases, the probability of OHIA is >10%); monitor fluid deficit (fluid overload considered when the fluid deficit is >1L with hypotonic solutions or >2.5L with isotonic solutions in healthy pts, based on expert opinion)
Minimise intrauterine pressure to reduce intravascular & intraperitoneal absorption

How well did you know this?
1
Not at all
2
3
4
5
Perfectly
282
Q

What causes metabolic acidosis?

A

increase in weak acids (eg. serum proteins, albumin, inorganic phosphate) or decrease in the strong ion difference

How well did you know this?
1
Not at all
2
3
4
5
Perfectly
283
Q

What’s the strong ion difference?

A

presence of an excess strong cations (Na+, K+, Ca++, Mg++) over strong anions (Cl-) (the normal value in plasma is 42mEq/L, strong anions or cations are those that exist in a fully ionised state in body fluids)

How well did you know this?
1
Not at all
2
3
4
5
Perfectly
284
Q

which approach should be used to quantify acid-base status in critically ill patients?

A

base excess is equivalent to Stewart’s SID approach

How well did you know this?
1
Not at all
2
3
4
5
Perfectly
285
Q

what causes the metabolic acidosis in DKA?

A

B-hydroxybutyrate & acetoacetate, strong anions produced from the hepatic metabolism of fatty acids

How well did you know this?
1
Not at all
2
3
4
5
Perfectly
286
Q

why is lactate a strong anion? which conditions may be associated with lactic acidosis?

A

lactate is more than 99% ionised

lactic acidosis may be observed in cardiogenic or hypovolaemic shock, severe HF, severe trauma, sepsis

How well did you know this?
1
Not at all
2
3
4
5
Perfectly
287
Q

What’s a cause of hyperchloraemic metabolic acidosis?

A

administration of fluids which reduce the strong ion difference (eg. NaCl which has no SID, cf plasma-lyte which has SID of 47.5

How well did you know this?
1
Not at all
2
3
4
5
Perfectly
288
Q

what’s normal SID?

A

42mEq/L

How well did you know this?
1
Not at all
2
3
4
5
Perfectly
289
Q

what’s cardiac index? normal?

A

CO / BSA
2.5-4 L/min/m2

How well did you know this?
1
Not at all
2
3
4
5
Perfectly
290
Q

What are the management principles for a metabolic acidosis?

A

treat the underlying cause of the acidosis
may give sodium bicarbonate if concern for suppressed cardiac function due to the metabolic acidosis

How well did you know this?
1
Not at all
2
3
4
5
Perfectly
291
Q

what’s the rationale for using sodium bicarbonate for a metabolic acidosis?

A

administering a solution with high SID may increase the pH, improving cardiac function

How well did you know this?
1
Not at all
2
3
4
5
Perfectly
292
Q

CXR findings in acute pulmonary oedema?

A

prominently increased opacity in both lung fields, increased hilar markings, figure 2 in useful tables/figures

How well did you know this?
1
Not at all
2
3
4
5
Perfectly
293
Q

actions if concerns arise for OHIA?

A

communicate w surgeons
ABG
prophylactic diuretics

How well did you know this?
1
Not at all
2
3
4
5
Perfectly
294
Q

initial treatment for symptomatic hyponatraemia (whether mild, moderate or severe)?

A

100mL bolus 3% saline, repeat twice more as needed if symptoms persist.
monitor the serum sodium hourly until it’s increased by 4-6mmol/L after which frequency of monitoring can reduce
General measures= identify & Rx underlyinc cause, identify drugs that may contribute, reduce intake of electrolyte-free water (impose FR, eliminate hypotonic IVT, incr dietary salt)
if SIADH or chronic hyponatremia consider loop diuretics, oral salt tabs & urea.

How well did you know this?
1
Not at all
2
3
4
5
Perfectly
295
Q

contraindications to blind NGT insertion in adults include:

A

BOS#
caustic ingestion or oesophageal stricture (risk perforation)
coagulopathy (epistaxis risk)
severe mid-face trauma (risk cribriform plate destruction)
recent nasal surgery
oesophageal varices

suspected epiglottis
emergency intubation/significant hypoxia

orogastric if coagulopathy or facial truama

How well did you know this?
1
Not at all
2
3
4
5
Perfectly
296
Q

what’s Le Fort I? (leforts are fig III)

A

separation of hard palate from upper maxilla
transverse # through maxilla & pterygoid plates @ level just above floor of nose

How well did you know this?
1
Not at all
2
3
4
5
Perfectly
297
Q

what’s LeFort II?

A

transects nasal bones, anterior/medial orbital walls/floor/inferior orbital rings, posterior maxilla & pterygoid plates

How well did you know this?
1
Not at all
2
3
4
5
Perfectly
298
Q

what’s Le Fort III?

A

separates maxilla from skull base (craniofacial dysjunction), traverses the nasofrontal suture, medial orbital wall, lateral orbital wall (zygomaticofrontal suture), zygomatic arch & pterygoid plates

How well did you know this?
1
Not at all
2
3
4
5
Perfectly
299
Q

for which drug should total body weight dosing be used in morbidly obese?

A

suxamethonium

How well did you know this?
1
Not at all
2
3
4
5
Perfectly
300
Q

how should propofol & thiopentone be dosed for maintenance & induction in morbidly obese pts?

A

LBW for induction, TBW for maintenance

How well did you know this?
1
Not at all
2
3
4
5
Perfectly
301
Q

on what dosing scalar should fent & remi be based in morbidly obese?

A

lean body weight

How well did you know this?
1
Not at all
2
3
4
5
Perfectly
302
Q

on what dosing scalar should NDMRs be based in morbidly obese?

A

ideal body weight

How well did you know this?
1
Not at all
2
3
4
5
Perfectly
303
Q

how to calculate ideal BW? what is it?

A

males ht - 100
females ht -110

the BW associated with maximum life expectancy for a given height

How well did you know this?
1
Not at all
2
3
4
5
Perfectly
304
Q

how to calculate lean BW?

A

males: 50 + 0.9 per cm above 150cm
females: 45 + 0.9 per cm above 150cm

How well did you know this?
1
Not at all
2
3
4
5
Perfectly
305
Q

disadvantage of dosing based on ideal BW?

A

assumes that everyone of a certain height should receive the same dose & doesn’t take body composition into account; in morbidly obese may lead to under-dosing, ideal BW will be < lean BW for a given height

How well did you know this?
1
Not at all
2
3
4
5
Perfectly
306
Q

strengths of using lean body weight?

A

more useful in morbidly obese as LBW increases as TBW increases, albeit not proportional (as TBW ultimately incr out of proportion to LBW)
correlates to CO (important in early distribution kinetics) & drug clearance

How well did you know this?
1
Not at all
2
3
4
5
Perfectly
307
Q

mg/kg dose of lignocaine IV for Biers block?

A

3

How well did you know this?
1
Not at all
2
3
4
5
Perfectly
308
Q

benefit of adductor canal block for TKR cf FNB?

A

comparable analgesia, less motor weakness

How well did you know this?
1
Not at all
2
3
4
5
Perfectly
309
Q

cardiovascular effects of hyperthyroidism

A

Increased resting HR
AF
incr LV contractility
incr CO (incr peripheral O2 needs & incr cardiac contractility)
incr blood volume (reduced MAP incr RASS activation & renal Na+ resorption), preload
decreased SVR & diastolic BP/afterload
may have systolic HTN
may develop high- or normal-output CCF
LVEF doesn’t appropriately increase during exercise, suggesting cardiomyopathy
Incr red cell mass (EPO synthesis promoted by T3)

How well did you know this?
1
Not at all
2
3
4
5
Perfectly
310
Q

cardiovascular effects of hypothyroidism?

A

endothelial dysfunction & impaired VSM relaxation–> incr SVR
diastolic HTN & incr afterload
reduced CO
slower resting HR

How well did you know this?
1
Not at all
2
3
4
5
Perfectly
311
Q

how do we calculate the confidence interval for low probability events?

A

“rule of 3’s”, so the 95% CI is zero + 3/n

How well did you know this?
1
Not at all
2
3
4
5
Perfectly
312
Q

what’s sick euthyroid?

A

low serum thyroid levels in a pt who is clinically euthyroid but sick from another cause; Rx is for the underlying illness vs thyroid replacement
Typically low or low-normal T3, T4, may have rel high rT3 (ie. low T3:rT3). Normal or low-normal TSH.

How well did you know this?
1
Not at all
2
3
4
5
Perfectly
313
Q

what are the blood test results for subclinical hypothyroidism?

A

high TSH, normal T3 &T4

How well did you know this?
1
Not at all
2
3
4
5
Perfectly
314
Q

what’s the smallest size ETT manufacturer recommends fit over aintree intubation catheter?

A

ID >=7.0mm

How well did you know this?
1
Not at all
2
3
4
5
Perfectly
315
Q

Which drugs may increase bleeding risk with DOACs?

A

Those that inhibit CYP3A4 incr risk bleeding with apixaban, eg. fluconazole, a moderate CYP3A4 inhibitor, has a 3.5x incr risk of bleeding on apixaban cf when having apixaban sans fluconazole (esp GI bleeding)- systemic fluconazole only.
Strong dual CYP3A4 & P-gp inhibitors can increase apixaban & rivaroxaban effect.
P-gp inhibitors can increase dabigatran effect.
Examples of strong CYP3A4 inhibitors: clarithromycin, ketoconazole, voriconazole)
Moderate CYP3A4 inhibitors: amiodarone, diltiazem, verapamil, cyclosporine, cimetidine, erythromycin, grapefruit juice
Pgp inhibitors: amiodarone, carvedilol, larrythromycin, itraconazole, ivacaftor, ketoconazole, quinidine

How well did you know this?
1
Not at all
2
3
4
5
Perfectly
316
Q

For which pts is dose reduction of apixaban recommended?

A

low CrCl (avoid if CrCl <15mL/min), body wt & higher age
Those taking strong dual inhibitors of CYP3A4 & P-glycoprotein

How well did you know this?
1
Not at all
2
3
4
5
Perfectly
317
Q

what inheritance is hereditary angioedema?

A

autosomal dominant

How well did you know this?
1
Not at all
2
3
4
5
Perfectly
318
Q

what are the signs/symptoms of hereditary angioedema?

A

well-demarcated angioedema without urticaria or pruritis (cutaneous)
may cause debilitating abdominal pain (GI), nausea or vomiting due to intestinal oedema, or life-threatening laryngeal oedema (upper airway)
symptoms usually take several hours to develop, resolve in 2-4 days sans Rx

How well did you know this?
1
Not at all
2
3
4
5
Perfectly
319
Q

what’s the etiology of HAE?

A

rare autosomal dominant condition
deficiency or dysfunction of the C1 esterase inhibitor which leads to excessive bradykinin, episodic increase in vascular permeability & angioedema

How well did you know this?
1
Not at all
2
3
4
5
Perfectly
320
Q

how is the presentation of HAE fundamentally different from the angioedema of allergic reactions?

A

not mediated by histamine or other mast cell mediators, not responsive to epinephrine, antihistamines or glucocorticoids, must be managed by replacing C1 inhibitor (C1 inhibitor concentrate infusion) or blocking production or functioning of bradykinin

How well did you know this?
1
Not at all
2
3
4
5
Perfectly
321
Q

What are some potential triggers for hereditary angioedema?

A

Dental & medical procedures, intubation/oral surgery/major dental work are particularly high risk, emotional stress, hormonal changes, infections, medications (incl oral contraceptives & ACE-inhibitors)

How well did you know this?
1
Not at all
2
3
4
5
Perfectly
322
Q

What Rx should be given to pts with hereditary angioedema prior to dental work, surgery as prophylaxis? (high risk procedures= those involving head & neck incl intubation or any airway/head& neck instrumentation)

A

Intravenous C1-INH concentrate (Berinert), 1-6hr before procedure, aiming for functional C1-INH level of >=50% of normal at the time of procedure (usually get this with 20IU/kg body wt)
2nd line if C1 inh n/a: attenuated androgens, danazol (androgenic hormone) in incr doses for 5 days pre & 3 days post procedure is an alternative if berinert n/a
3rd line= FFP if C1 inhibitor n/a, or solvent/detergent-treated plasma
The pt should have access to on-demand therapies as swelling often occurs 1 day or so AFTER procedure

alt icabitant is a bradykinin receptor antagonist, has short half life so not ideal prophylactic agent

How well did you know this?
1
Not at all
2
3
4
5
Perfectly
323
Q

What are adverse effects of pdC1-INH concentrate?

A

Headache, nausea, fever, anaphylaxis

How well did you know this?
1
Not at all
2
3
4
5
Perfectly
324
Q

when should surgery for pts with hereditary angioedema be scheduled?

A

Early in the day as swelling often happens a day or so AFTER procedure, airway symptoms can be slower to recognise if begin @ night

How well did you know this?
1
Not at all
2
3
4
5
Perfectly
325
Q

what do the Society of NeuroInterventional Surgery and the Neurocritical Care Society recommend for endovascular Rx of acute ischaemic stroke?

A

SBP >140 <180mmHg, DBP <105mmHg

How well did you know this?
1
Not at all
2
3
4
5
Perfectly
326
Q

what’s the maximum warm ischaemia time for procuring kidneys following donation for cardiac death?

A

60mins (from SBP <50mmHg)

How well did you know this?
1
Not at all
2
3
4
5
Perfectly
327
Q

maximum warm ischaemia time heart/liver/pancreas?

A

30 mins (if greater for liver, risks biliary stricture), (from SBP <90mmHg)

How well did you know this?
1
Not at all
2
3
4
5
Perfectly
328
Q

max warm ischaemia time lungs?

A

90mins (from SBP <50mmHg)

How well did you know this?
1
Not at all
2
3
4
5
Perfectly
329
Q

max cold ischaemia time various organs?

A

heart 4hrs
lung 6-8hrs
liver/pancreas 12hrs (DBD), 6hrs (DCD)
kidneys 18hrs (DBD), 12hrs (DCD)

How well did you know this?
1
Not at all
2
3
4
5
Perfectly
330
Q

what’s warm ischaemia time?

A

the time from treatment withdrawal to the start of cold perfusion of the donated organs

How well did you know this?
1
Not at all
2
3
4
5
Perfectly
331
Q

what’s the recommended cleaning protocol for a laryngoscope handle which has been used but which has no visible soiling?

A

washed with detergent & water but if contaminated, wash & disinfect (semi-critical devices)

How well did you know this?
1
Not at all
2
3
4
5
Perfectly
332
Q

what classification of equipment are laryngoscope blades considered & how should they be cleaned?

A

critical (penetrate skin or mucous membrane) so require sterilisation

How well did you know this?
1
Not at all
2
3
4
5
Perfectly
333
Q

what’s asepsis?

A

prevention of microbial contamination of living tissues or sterile materials

How well did you know this?
1
Not at all
2
3
4
5
Perfectly
334
Q

what’s disinfection?

A

inactivation of non-sporing organisms using thermal or chemical means

How well did you know this?
1
Not at all
2
3
4
5
Perfectly
335
Q

what’s sterilisation?

A

complete destruction of all micro-organisms including spores

How well did you know this?
1
Not at all
2
3
4
5
Perfectly
336
Q

what’s a critical device?

A

one that penetrates skin or mucous membranes, enter vascular system or a sterile space
these devices require sterilisation

How well did you know this?
1
Not at all
2
3
4
5
Perfectly
337
Q

what’s a semi-critical device?

A

semi-critical device will be in contact with intact mucous membranes or may become contaminated with readily transmissible organisms. they require high level of disinfection or sterilisation.

How well did you know this?
1
Not at all
2
3
4
5
Perfectly
338
Q

what’s a semi-critical device?

A

semi-critical device will be in contact with intact mucous membranes or may become contaminated with readily transmissible organisms. they require high level of disinfection or sterilisation.

How well did you know this?
1
Not at all
2
3
4
5
Perfectly
339
Q

what’s a non-critical device?

A

one that comes in contact with intact skin or doesn’t contact the pt directly, these devices require low level disinfection or cleaning

How well did you know this?
1
Not at all
2
3
4
5
Perfectly
340
Q

what are the feature of brown sequard syndrome?

A

lose ipsilateral motor power (CST), vibration/proprioception, light touch & contralateral pain/temp (beginning 1-2 segments below the lesion)

retain contralateral vibration/proprioception, ipsilateral pain/temp

hyperreflexia & spastic paralysis BELOW level of lesion ipsilateral (UMN lesion)
flaccid paralysis of muscles supplied by the nerve of the level of lesion (LMN affected)

if lesion above T1 will get ipsilateral Horner’s with involvement of oculosympathetic pathway

How well did you know this?
1
Not at all
2
3
4
5
Perfectly
341
Q

In a Blalock–Taussig shunt, blood passes to the pulmonary artery via what?

A

subclavian artery

How well did you know this?
1
Not at all
2
3
4
5
Perfectly
342
Q

how much does 10mm on the Y axis of ecg measure?

A

1mV (2 big squares)

How well did you know this?
1
Not at all
2
3
4
5
Perfectly
343
Q

What’s the most common arrhythmia in maternal cardiac arrest?

A

PEA (this was in 50%, followed by asystole in 25%. Only 12% had shockable rhythm)

How well did you know this?
1
Not at all
2
3
4
5
Perfectly
344
Q

Which agents are the most commonly implicated in periop anaphylaxis?

A

NMBAs

How well did you know this?
1
Not at all
2
3
4
5
Perfectly
345
Q

what’s central to the allergenicity of the reacting NMBAs?

A

positively-charged quaternary & tertiary ammonium ions

How well did you know this?
1
Not at all
2
3
4
5
Perfectly
346
Q

what proportion of pts with NMBA anaphylaxis have cross-sensitisation?

A

50%

How well did you know this?
1
Not at all
2
3
4
5
Perfectly
347
Q

how long after an anaphylactic reaction should skin testing be delayed? why?

A

4-6wks (possible immune refractory period & may get false -ve & only +ve results are considered valid during that time)

How well did you know this?
1
Not at all
2
3
4
5
Perfectly
348
Q

Are sIgE assays subject to the 4-6/52 refractory period as are skin tests? utility of this?

A

no
may therefore be useful if urgent surgery is required within this period following an anaphylactic reaction

How well did you know this?
1
Not at all
2
3
4
5
Perfectly
349
Q

do morphine & pholcodine have ammonium groups similar to NMBDs?

A

yes- at physiologic pH they have substituted ammonium groups similar to those present on NMBAs; morphine contains one while pholcodine contains two amine groups of which one is more protonated @ physiologic pH

How well did you know this?
1
Not at all
2
3
4
5
Perfectly
350
Q

are sIgE to morphine & pholcodine of value in the detection of antibodies to NMBAs?

A

Yes, but it’s unclear whether these assays have equal diagnostic value for investigating allergic reactions to different classes of NMBAs (thought to be more reliable in detecting sensitisation to benzylisoquinoline NMBAs)

How well did you know this?
1
Not at all
2
3
4
5
Perfectly
351
Q

management of intracranial aneurysm rupture?

A

MANAGEMENT DEPENDS ON WHETHER OR NOT THE ANEURYSM IS EXPOSED.
If exposed, goals are:
-create a bloodless field to facilitate clipping & protect the brain
–>induce hypoT- esmolol 10-20mg IV as needed to achieve MAP 50-60mmHg to reduce bleeding & facilitate clip placement
–>induce temporary flow arrest with adenosine 0.5mg/kg
–>reduce CMR with propofol 20-60mg IV (if using prop for maintenance, incr rate to 125-200cmg/kg/min- aim to achieve burst suppression on BIS)
-volume resuscitation may be necessary once the clip placed; IVT & blood aiming for euvolaemia & Hb >=80g/L
-once clip in place, lighten anaesthesia
if ruptures prior to exposure, may be difficult to detect (may be heralded but unexplained incr BP & ICP. rapid decision necessary re: Surg, imaging or angio. Support pt with:
-optimising CPP by allowing permissive HTN, aim CPP 50-70mmHg if ICP monitor, if not, MAP >90mmHg. vasopressors as needed to incr BP (NAdr pref)
-manage intracranial HTN
–> optimise O2 (aim PaO2 >80mmHg, PaCO2 32-38mmHg) & ventilation
–> osmotheray, mannitol 0.25-1g/kg or HTS 100mL 3% NaCl, aim serum Na <=155mmol/L or osmolality <320mmol/L
–> improve venous drainage with head elevation 15 degrees head-up, neutral position, tape vs tie of the ETT
beware that opening of ventriculostomy abruptly may worsen bleeding (acutely elevates transmural pressure gradient)
–> neuroprotection with propofol 20-60mg IV or incr rate to >125mcg/kg/min, titrate EEG to burst suppression

IAR occurs with an endovascular procedure for an aneurysm (uncommon, if so it’s most likely with coil deployment), reverse heparinisation with protamine.

How well did you know this?
1
Not at all
2
3
4
5
Perfectly
352
Q

if bleeding during endovascular coiling of an aneurysm, should coiling continue?

A

no, not until bleeding controlled

How well did you know this?
1
Not at all
2
3
4
5
Perfectly
353
Q

what may be signs of ruptured aneurysm during endovascular procedure?

A

sudden rise ICP
sudden incr BP or decr HR
extravasation of contrast

How well did you know this?
1
Not at all
2
3
4
5
Perfectly
354
Q

goals of management of bleed during endovascular coiling?

A

reduce coagulability (protamine to reverse heparin, IF REQUESTED BY RADIOLOGIST; 1mg protamine per 100 units heparin given)
reduce BP to level before HTN due to bleeding
reduce ICP (gentle hyperventilation, reverse trendelenburg, would’ve taped not tied ETT, consider mannitol or HTS)
control seizures
reduce CMR (prop)
only continue coiling once BP controlled

How well did you know this?
1
Not at all
2
3
4
5
Perfectly
355
Q

what are some risks of mannitol in renal failure?

A

retention may cause hyperosmolality & osmotic movement of water & K out of cells. May get vol expansion, hyponatremia, metabolic acidosis & hyperK.

How well did you know this?
1
Not at all
2
3
4
5
Perfectly
356
Q

Dose of protamine for heparin reversal?

A

1mg protamine per 100 units heparin

How well did you know this?
1
Not at all
2
3
4
5
Perfectly
357
Q

difference required for orientation of quincke or atraucan vs whitacre or sprotte?

A

the quince or atraucan require orientation of bevel along with the dural fibres so less likely trauma, while it doesn’t matter with Whitacre or sprotte

How well did you know this?
1
Not at all
2
3
4
5
Perfectly
358
Q

does early (<12 hrs) vs delayed (48hrs) renal replacement therapy in pts with sepsis & AKI result in survival benefit?

A

no

How well did you know this?
1
Not at all
2
3
4
5
Perfectly
359
Q

Patients with obstructive sleep apnoea undergoing surgery, have been shown to have an increased incidence of what?

A
  1. neurocognitive (childhood OSA may impair learning skills), psychiatric (eg. depression), endocrine (impaired glucose tolerance, dyslipidaemia, incr cortisol [], testicular/ovarian dysfunction), cardiovascular morbidity (HTN & Brady- and tachyarrhythmias are more common, pulm HTN, CHF & biventricular dysfunction incr risk of haemodynamic instability perioperatively- OSA IS AN INDEPENDENT RISK FACTOR FOR MI- dyslipidaemia, increase pro-inflammatory cytokines, endothelial dysfunction, incr platelet aggregation) & mortality in all age groups.
    they are @ incr risk of CVA with poorer outcome.

difficult intubation, obstructed breathing, OPIOID SENSITIVITY

Pulmonary complications
postop delirium
**POSTOP CARDIAC EVENTS ((Composite outcome of myocardial injury, cardiac death, heart failure, thromboembolism, AF & stroke) within 30/7 of surgery) = sig incr HR IN SEVERE BUT NOT MILD/MOD OSA

When are pts with obesity @ highest post risk?
Sleep architecture disturbances greatest on PN1 but breathing disturbances during sleep greatest PN3

Death & anoxic brain damage/other critical evnents within 24hrs. More likely if on ward vs ICU. More likely if not use CPAP.
Highest risk= SEVERE UNTREATED.

Minimise risk by:
Routine post O2
Minimise opioids
Extubatne awake
Incr intensity post-op monitoring (routine pulse ox or capnography)
Referral for sleep studies:
STOP-BANG 6-8 OR stop-bang 3-5 & MP 3-4 or neck >50cm2 or SpO2 <94% or bicarb >20 or NYNA III—> sleep studies
Non-supine positioning where possible
Continue CPAP if already on CPAP; those with severe OSA but not compliant w CPAP—> HDU invasive monitor. If not on CPAP or on CPAP + compliant OR mod risk based on screening or diff airway—> low-opioid & overnight SpO2

How well did you know this?
1
Not at all
2
3
4
5
Perfectly
360
Q

What are some pt factors associated with increased risk of BCIS? Surg factors?

A

increasing age
ASAIII-IV
male
poor physical reserve
cardiopulmonary dysfunction
pre-existing pulmonary HTN
PFO
OP
bony mets
hip fractures (esp pathological or intertrochanteric), warfarin use & diuretic use
surg factors: long-stem arthroplasty, prev uninstrumented femoral canal NOT revision surgery, ?femoral canal diameter >21mm

How well did you know this?
1
Not at all
2
3
4
5
Perfectly
361
Q

What are the 3 grades of BCIS? What’s bone ceme t

A

1: hypoxia SpO2 <94% & hypotension with SBP decrease >20% 2: SpO2 <88% or SAP drop >40% or LOC 3. cardiovascular collapse requiring CPR

Features BCIS: hypoxia, hypoT, arrhythnias, cardiac arrest

Reversible, time-limited
Limit risk by identifying high-risk pts (CEMENTED HIP ARTHROPLASTY), modify surg technique to reduce CV compromise (prep & dry canal, venting hole, bone-vacuum cement, avoid pressurisation, consider cementless)

Anaes: vigilance (prepare for cement with 100% O2, normal BP, open IVDT) to haem changes, R) heart protection, normovol, adequate MAP, art line, incr FiO2 during @ risk times (reaming, insertion cement, insertion prosthesis, reduction joint, release tourniquet), appropriate depth (not excess @ risk times), avoid N2O (air embolism).

Mx: early identification, 100% O2, aggressive resus, manage PAH & RV failure, pulm VD (prostacyclin) & inotropes (dob & milrinone, ephedrine), consider higher-level monitoring (TOE) & CVC
ICU postop.
Early post-op Ax (possible delirium w cerebral emboli)

How well did you know this?
1
Not at all
2
3
4
5
Perfectly
362
Q

According to National Audit Project (NAP) 5, the incidence of awareness during general anaesthesia for lower segment caesarean section should be quoted as

A

1:670

How well did you know this?
1
Not at all
2
3
4
5
Perfectly
363
Q

Piped O2 supply to major hospitals is sourced mainly from what?

A

VIE (vacuum insulated evaporator)

How well did you know this?
1
Not at all
2
3
4
5
Perfectly
364
Q

what’s 1 bar in KpA?

A

100KpA

How well did you know this?
1
Not at all
2
3
4
5
Perfectly
365
Q

The water capacity of an oxygen transport cylinder is 2 litres. The gauge is reading 150 bar. At an oxygen flow rate of 10 litres per minute, the number of minutes the cylinder will last is?

A

Boyle’s Law:
P1V1 = P2V2

P1= 150bar
V1= 2L
P2= 1bar (atmospheric)

so, V2= (2 x 150) / 1 = 300L

300L at 10 L/min = 30mins

How well did you know this?
1
Not at all
2
3
4
5
Perfectly
366
Q

What are the specifications of a normal transport size C cylinder?

A

2.9L water capacity
440L oxygen at 13700kPa

How well did you know this?
1
Not at all
2
3
4
5
Perfectly
367
Q

According to PS50, after an absence of more than 12 month from practicing clinical anaesthesia a re-entry program should be offered. The duration of the program for every year of absence would usually be at least

A

1 month per 12 months

How well did you know this?
1
Not at all
2
3
4
5
Perfectly
368
Q

Why is perioperative overheating most likely to cause worsening of symptoms of multiple sclerosis?

A

demyelinated axons are more sensitive to heat & symptoms can deteriorate with increase in temperature

How well did you know this?
1
Not at all
2
3
4
5
Perfectly
369
Q

does aspirin for primary prevention (in adult non-diabetic pts) reduce risk of cardiovascular mortality or all cause mortality? does it provide benefit on nonfatal stroke? does it increase the risk of major bleeding?

A

no
no
yes (incr (50%) risk of major nonfatal extracranial bleeding

likely may reduce nonfatal MI over 10 yrs
possible reduction colon Ca incidence over 20yrs

How well did you know this?
1
Not at all
2
3
4
5
Perfectly
370
Q

what does the ACC/AHA recommend regarding low-dose (75-100mg/day) aspirin for primary prevention of atherosclerotic cardiovascular disease?

A

consider it for adults aged 40-70 with high risk of ASCVD but NOT at incr bleeding risk
do not give it on routine basis for primary prevention of ASCVD in adults >70yo
don’t give it to adults at any age @ incr risk bleeding

How well did you know this?
1
Not at all
2
3
4
5
Perfectly
371
Q

what’s COPD GOLD3?

A

FEV1/FVC <70% & FEV1 30-49% predicted

How well did you know this?
1
Not at all
2
3
4
5
Perfectly
372
Q

what’s the maximum FiO2 that can be delivered through a Venturi mask? nasal cannula? non-rebreather?

A

60%
36%
85%

How well did you know this?
1
Not at all
2
3
4
5
Perfectly
373
Q

which agent has the highest capacity to absorb IR radiation?

A

?desflurane, since GWP is the product of radiative efficiency & atmospheric lifetime & N2O has much longer atmospheric lifetime than des yet lower GWP

How well did you know this?
1
Not at all
2
3
4
5
Perfectly
374
Q

when do most cardiac arrests after cardiac surgery occur?

A

within first 5 postop hours

How well did you know this?
1
Not at all
2
3
4
5
Perfectly
375
Q

what are some critical differences with cardiac arrest after open cardiac surg cf standard ACLS?

A

-external cardiac compressions aren’t immediately initiated (concern for disruption of the surgical repair)
-avoid administration of full 1mg epinephrine; may –> extremely high BP which may disrupt arterial suture lines; rather, if indicated at all, give 50mcg increments of epinephrine as needed with continuous reassessment
-avoid atropine for asystole or severe brady, instead, initiate pacing (rate of 90, DDD, if pacing wires available)
-if they are paced & in PEA, turn off pacing to “unmask” VF

if VF or pulseless VT, give 3 successive defibrillator shocks & 300mg amiodarone IV
initiate pacing for asystole or severe bradycardia- set pacemaker for dual chamber pacing (DDD mode) at 80-100bpm, max output voltage 20mA atrial & 25mA ventricle (deranged phys says max output)

if PEA, external cardiac compressions while opening chest (within 5 mins); do manual cardiac massage 100-120bpm.

internal defibrillator at 20J/s during attempts to treat VF.

How well did you know this?
1
Not at all
2
3
4
5
Perfectly
376
Q

what’s MEN2A? how differ from MEN2B?

A

characterised by medullary thyroid cancer, pheochromocytoma/paraganglioma, primary parathyroid hyperplasia

MEN2B no hyperparathyroidism

How well did you know this?
1
Not at all
2
3
4
5
Perfectly
377
Q

what may NF1 be associated with?

A

cafe au lait spots
peripheral neurofibromas
neurocognitive abnormalities
CNS tumors
soft tissue sarcomas
other tumors incl pheochromocytomas/paragangliomas

How well did you know this?
1
Not at all
2
3
4
5
Perfectly
378
Q

how do you calculate the est RVSP from TTE?

A

Bernoulli equation:
4v2 + RAP (using the continuous wave doppler trace through tricuspid valve), where v is the TR jet VTI

How well did you know this?
1
Not at all
2
3
4
5
Perfectly
379
Q

what are the physical properties of normal saline?

A

Na+ & Cl- 154, osmolarity 308mOsmol/L (considered isotonic), SG 1.0046 (plasma 1.02)

How well did you know this?
1
Not at all
2
3
4
5
Perfectly
380
Q

what are air bronchograms & what might they signify?

A

air-filled bronchi (dark) made visible by opacification of the surrounding alveoli, suggest a pathologic process going on filling alveoli w something other than air. they won’t be visible if the bronchi themselves are opacified (eg. filled w fluid).

Ddx:
pulmonary consolidation
pulmonary oedema
pulmonary haemorrhage
non-obstructive atelectasis
severe interstitial lung disease
neoplastic process
pulm infarct

How well did you know this?
1
Not at all
2
3
4
5
Perfectly
381
Q

how to differentiate causes of a complete hemithorax whiteout on CXR?

A

position of trachea.

pulled toward opacified side: pneumonectomy, total lung collapse (eg. endobronchial intubation), pulmonary genesis or hypoplasia

remains central: consolidation, pulm oedema/ARDS, pleural mass, chest wall mass, being on ECMO

pushed away: pleural effusion, diaphragmaticc hernia, large pulm mass, diaphragmatic rupture

How well did you know this?
1
Not at all
2
3
4
5
Perfectly
382
Q

According to the Australian and New Zealand Resuscitation Guidelines the immediate treatment for an adult conscious victim with a severe airway obstruction due to a foreign body inhalation is:

A

send for help

up to 5 back blows (Heel of hand middle of back btwn sh blades, aim is to relieve obstruction, check between each to see if dislodged the FB)
Infant head downwards across rescuers lap
if not successful

up to 5 chest thrusts, same hand position as CPR but thrusts slower rate & sharper. infant head down on back across rescuers thigh, adult sitting or standing. therafter, alternate 5 back blows & 5 chest thrusts.

How well did you know this?
1
Not at all
2
3
4
5
Perfectly
383
Q

what’s the mean (SD) lip-carina distance for males & females?

A

24 (2) & 22 (1)

How well did you know this?
1
Not at all
2
3
4
5
Perfectly
384
Q

what’s the length of the aintree catheter? Fr & ID? with which tubes can it be used? and scope? LMA?

A

56cm long

19Fr

ID 4.7mm

contains 15mm adapter & luerlock

can be used with an ETT ID >=7mm (as it has 6.5cm OD)

can be loaded onto fiberoptic scope with OD 4.2mm or smaller

LMA 2 or larger

How well did you know this?
1
Not at all
2
3
4
5
Perfectly
385
Q

length of Cook exchange catheter? French?

A

83cm, 11-19Fr

How well did you know this?
1
Not at all
2
3
4
5
Perfectly
386
Q

length of DLT exchange catheter? Fr?

A

100cm, 11-14Fr

How well did you know this?
1
Not at all
2
3
4
5
Perfectly
387
Q

length of bougie?

A

70cm

How well did you know this?
1
Not at all
2
3
4
5
Perfectly
388
Q

lesion causing L) homonymous hemianopia is due to lesion of what?

A

posterior cerebral artery, supplying the R) occipital lobe

How well did you know this?
1
Not at all
2
3
4
5
Perfectly
389
Q

which is the most commonly-affected cerebral artery for CVA?

A

MCA

How well did you know this?
1
Not at all
2
3
4
5
Perfectly
390
Q

what’s MCA syndrome?

A

contralateral hemisensory loss (UL, LL, lower 2/3 of face)
contralateral motor weakness, spasticity, hypertonicity/hyperreflexia
ipsilateral eye deviation & contralateral homonymous hemianopia
if dominant (usually L)-sided), Broca’s aphasia (difficulty forming words) if superior part of MCA, wernicke’s (difficulty understanding words & they may speak in fluent but unintelligible sentences & be unaware of this) if inferior part & if non-dominant, hemineglect/agosognosia/apraxia

How well did you know this?
1
Not at all
2
3
4
5
Perfectly
391
Q

along with contralateral homonymous hemianopia (due to occipital infarct), with what may a pt with PCA stroke present?

A

contralateral hemisensory loss, due to thalamic infarct, may also have memory impairment & decr consciousness

ACA: CL LL weak, executive dysfunction
brainstem: crossed sensory or motor findings, nystagmus, diplopia, vertigo, horner’s
cerebellar: ataxia, nystagmus, vertigo, nausea, headache, rapid deterioration consciousness

How well did you know this?
1
Not at all
2
3
4
5
Perfectly
392
Q

what is unilateral homonymous hemianopia & hemisensory loss, without motor manifestation, considered diagnostic of?

A

PCA territory infarct

How well did you know this?
1
Not at all
2
3
4
5
Perfectly
393
Q

What may rapid development of proptosis, tense eye, extra-ocular movement deficiencies & markedly decreased visual acuity suggest? major concern? treatment?

A

elevated IOP due to retrobulbar haematoma

visual loss

lateral canthotomy- decompressing the orbit, relieving pressure to prevent compartment syndrome & permanent vision loss

other Mx: acetazolamide, mannitol, emerg orbital decompression

don’t see chemosis with retrobulbar haemorrhage

How well did you know this?
1
Not at all
2
3
4
5
Perfectly
394
Q

how soon after development of orbital compartment syndrome must lateral canthotomy be performed before irreversible visual loss may occur?

A

2hrs, possibly less

How well did you know this?
1
Not at all
2
3
4
5
Perfectly
395
Q

which substance should be avoided in pts with a Hx of anaphylaxis to MMR vaccine?

A

gelofusin

How well did you know this?
1
Not at all
2
3
4
5
Perfectly
396
Q

what’s the most important complication of gelation-based colloids?

A

possibility of allergic reaction

How well did you know this?
1
Not at all
2
3
4
5
Perfectly
397
Q

for which children is MMR vaccine not advised?

A

those with known severe systemic allergic reaction to gelatin or neomycin

How well did you know this?
1
Not at all
2
3
4
5
Perfectly
398
Q

The anti-emetic action of aprepitant is via receptors for what?

A

Substance P (NK1 is the receptor, substance P is the ligand, aprepitant antagonises the receptor)

How well did you know this?
1
Not at all
2
3
4
5
Perfectly
399
Q

are opioids likely to impact MEPs? or ketamine?

A

neither

How well did you know this?
1
Not at all
2
3
4
5
Perfectly
400
Q

what’s the main cause of airway compromise following anterior C-spine surgery?

A

retropharyngeal OEDEMA

How well did you know this?
1
Not at all
2
3
4
5
Perfectly
401
Q

risk factors for airway compromise following anterior procedures on cervical spine?

A

multilevel surgery
blood loss >300mL
duration >5hrs
combined ant & posterior operation
previous Cx surgery

How well did you know this?
1
Not at all
2
3
4
5
Perfectly
402
Q

what are the 2 most common causes of metabolic alkalosis? management approach? further diagnostic approach if the cause unclear?

A

external loss of gastric secretions (vomiting, NG suction) or diuretics

management approach= to treat the underlying cause- eg. treat cause of vomiting, stop or slow removal of gastric secretions, stop loop or thiazide diuretics then correct the factors maintaining the disorder (eg. Cl- administration, repletion of Cl-, K+ & ECF volume will promote renal bicarb secretion & return plasma bicarb to normal).

if the metabolic alkalosis is due to persistent vomiting or gastric suction, drugs that reduce gastric HCl secretion may help (H2 blockers, PPIs)

if the ethology unclear from Hx/exam, measurement of urine chloride & Na may help as may looking at the potassium-

  • low urine Cl occurs with vomiting or NG suction (unless the pt also on diuretics), spot urine Na also often low in these pts due to volume contraction
  • if the pt is hypokalaemia, this is usually due to urinary potassium losses, even if the pt has concurrent vomiting, since there is little K+ in gastric secretions
How well did you know this?
1
Not at all
2
3
4
5
Perfectly
403
Q

what does acetazolamide do?

A

carbonic anhydrase inhibitor, promotes renal loss of bicarb along with sodium, water & potassium

How well did you know this?
1
Not at all
2
3
4
5
Perfectly
404
Q

for how long should SGLT2-i be withheld preop?

A

3 days; 2 days & day of OT

How well did you know this?
1
Not at all
2
3
4
5
Perfectly
405
Q

for how long should SGLT2-i be withheld preop for procedures involving 1 or more days in hospital or bowel prep? what about day stay procedures, including gastroscopy (no bowel prep)?

A

3 days; 2 days & day of OT

can withhold just for the day of the procedure but fasting before & after the procedure should be minimised

How well did you know this?
1
Not at all
2
3
4
5
Perfectly
406
Q

in which situations should I suspect eDKA?

A

pts taking SGLT2i who have one or more of:
• symptoms of abdo pain, nausea, vomiting, fatigue or metabolic acidosis – a normal or only modestly elevated plasma
glucose level does not exclude the diagnosis.
• finger prick capillary blood ketone (or blood beta-hydroxybutyrate) levels >1.0 mmol/L with or without hyperglycaemia
• low (negative) base Excess (BE) < -5mmol/l indicating metabolic acidosis on arterial or venous blood gasses.

How well did you know this?
1
Not at all
2
3
4
5
Perfectly
407
Q

what to do if a pt hasn’t stopped their SGLT2i as per guidelines?

A

course of action depends on urgency of procedure, pt factors eg. HbA1c (>9% indicates higher risk of insulin insufficiency & risk of eDKA in this setting)… if:

ketones <1 & BE > -5, there’s no ketosis or metabolic acidosis so consider proceeding w day surgery with hourly monitoring of ketones intra-op & 2-hourly postop until E&D normally. provide written post-D/C info. more extensive surgery requires considering goals of care, collaboration with endocrine & critical care.

if ketones >1 & BE > -5, ketosis without metabolic acidosis. get endocrine advice. it may be due to starvation (esp if the pt has HbA1c of <9%). could consider proceeding with periop insulin & dextrose infusions to mitigate risk of DKA.

if ketones >1 & BE < -5, ketosis with metabolic acidosis. strongly consider postponing non-urgent surgery, escalate w endocrine & critical care.

How well did you know this?
1
Not at all
2
3
4
5
Perfectly
408
Q

what’s the management of eDKA or DKA?

A

in collaboration with endocrine & critical care:
rehydration
IV insulin infusion (with added dextrose once BGL <15mmol/L)
hourly BGL, ketones & blood gas

How well did you know this?
1
Not at all
2
3
4
5
Perfectly
409
Q

what’s the best US probe for median nerve block?

A

high-frequency linear 10-14MHz

How well did you know this?
1
Not at all
2
3
4
5
Perfectly
410
Q

what does higher frequency confer on US?

A

better resolution but less penetration (ie. better for superficial)

How well did you know this?
1
Not at all
2
3
4
5
Perfectly
411
Q

what does lower frequency on US confer?

A

better tissue penetration but poorer resolution; allows visualisation at greater depth

How well did you know this?
1
Not at all
2
3
4
5
Perfectly
412
Q

what’s gain on ultrasound?

A

uniform amplification of the returning US waves; higher gain= higher signal of reflected sound waves & image whiter.

How well did you know this?
1
Not at all
2
3
4
5
Perfectly
413
Q

optimal depth for US images?

A

as shallow as possible to still see all the structures of interest, since the image quality varies inversely w penetration depth

How well did you know this?
1
Not at all
2
3
4
5
Perfectly
414
Q

what are the components of US that we can manipulate?

A

mode
depth
focus (at or 0.5cm below target nerve)
gain
frequency
doppler

How well did you know this?
1
Not at all
2
3
4
5
Perfectly
415
Q

for a pt with normal renal function, for how many days should apixaban be ceased prior to neuraxial block?

A

3 days

How well did you know this?
1
Not at all
2
3
4
5
Perfectly
416
Q

for how many days should dabigatran be withheld prior to neuraxial block?

A

5 days, however if there is NORMAL renal function (CrCl >=80mL/min) & no additional risk factors for bleeding (age >65, HTN, antiplts), may consider 3 days or if CrCl 50-79mL/min consider 4 days.

if CrCl <30mL/min, avoid neuraxial block if pt has been on dabigatran.

How well did you know this?
1
Not at all
2
3
4
5
Perfectly
417
Q

for how many days should rivaroxaban be held prior to neuraxial block?

A

3 days

How well did you know this?
1
Not at all
2
3
4
5
Perfectly
418
Q

for how many days should prasugrel be withheld prior to neuraxial?

A

7-10 days

How well did you know this?
1
Not at all
2
3
4
5
Perfectly
419
Q

for how many days should ticagrelor be withheld prior to neuraxial?

A

5-7 days

How well did you know this?
1
Not at all
2
3
4
5
Perfectly
420
Q

for how many days should clopidogrel be withheld prior to neuraxial?

A

5-7

How well did you know this?
1
Not at all
2
3
4
5
Perfectly
421
Q

for how many hours should we wait after catheter removal prior to restarting dabigatran, apixaban, rivaroxaban, ticagrelor & prasugrel? how about clopidogrel?

A

6hrs for all, immediately for clopidogrel provided no loading dose (in which case 6hrs)

How well did you know this?
1
Not at all
2
3
4
5
Perfectly
422
Q

with what would hypoglossal nerve damage manifest?

A

ipsilateral tongue deviation (LMN) or contralateral (UMN)

How well did you know this?
1
Not at all
2
3
4
5
Perfectly
423
Q

which cranial nerves DO decussate?

A

II, IV, VII & XII

How well did you know this?
1
Not at all
2
3
4
5
Perfectly
424
Q

how to convert hydromorphone to OMEs?

A

x5

How well did you know this?
1
Not at all
2
3
4
5
Perfectly
425
Q

what’s 12mg PO hydromorphone conversion to parenteral morphine?

A

20mg parenteral morphine

How well did you know this?
1
Not at all
2
3
4
5
Perfectly
426
Q

on what is a Fontan circulation dependent?

A

preload (? this may be achieved w decr volatile?) & pulmonary resistance

Keep Fontan pressure <20mmHg
Transpulmonary gradient <5mmHg
PVR <2WU/m2
for optimal circulation

How well did you know this?
1
Not at all
2
3
4
5
Perfectly
427
Q

what’s the most common cause of visual loss after spinal surgery?

A

ischaemic optic neuropathy (higher freq than pressure causing occlusion of retinal vessels).
ischaemic optic neuropathy causes are uncertain but may relate to low arterial pressure, low hct, long surgery, large IV fluid volume.

How well did you know this?
1
Not at all
2
3
4
5
Perfectly
428
Q

what’s ERAS?

A

multimodal perioperative care pathway aiming for early recovery for pts undergoing major surgery, includes pt optimisation, anaes, surg & OPD support. limit fasting, carb loading, education, normothermia, limit intraop IVT, multimodal analgesia, PONV & antibiotic prophylaxis, in/out IVC, d/c instructions & f/up plan.

How well did you know this?
1
Not at all
2
3
4
5
Perfectly
429
Q

What’s differential hypoxaemia & when can it occur?

A

is a complication specific to the use of V-A ECMO for severe cardiorespiratory failure, where the upper half of the body is perfused by blood of relatively low oxygen saturation from the heart, cf the lower body, where well-oxygenated blood is supplied via the peripheral femoro-femoral VA ECMO system.

How well did you know this?
1
Not at all
2
3
4
5
Perfectly
430
Q

what’s the most common complication of ECMO? then?

A

BLEEDING & COAGULOPATHY; 5-79% of ECMO patients
Answer= HAEMORRHAGE- particularly since most pts are on continuous anticoagulation. half of these haemorrhages relate to the cannulation site, arterial cannulation= highest bleeding risk.
Risk intracranial haemorrhage (so avoid HTN)

Thrombosis

then infection

How well did you know this?
1
Not at all
2
3
4
5
Perfectly
431
Q

what would the PFTs in restrictive lung disease of myasthenia gravis show?

A

reduced FEV1 & FVC, normal DLCO, ratio normal or high

How well did you know this?
1
Not at all
2
3
4
5
Perfectly
432
Q

what’s the factor to most effectively reduce mortality in early SAH?

A

prevent rebreeding by securing the aneurysm, ideally within 24hrs

How well did you know this?
1
Not at all
2
3
4
5
Perfectly
433
Q

which parameter of respiratory physiology decreases during pregnancy?

A

FRC, 20% by term (both RV & ERV decrease)

How well did you know this?
1
Not at all
2
3
4
5
Perfectly
434
Q

You are inserting a pulmonary artery catheter in an intubated patient prior to cardiac surgery, and a significant amount of blood appears in the endotracheal tube. The most appropriate specific initial management is to

A

aim to maintain oxygen & ventilation, identify site of haemorrhage, position pt with suspected bleeding side down (R)-side down decubitus) to limit soiling of “good lung”, isolate the non-bleeding lung to protect it from soiling & optimise oxygenation, most practically & rapidly with large-bore (at least size 8) single-lumen ETT & L) mainstem endobronchial intubation or DLT (less likely time for this) with bronchial cuff inflated, tracheal lumen clamped- also DLTs are difficult to insert w brisk bleeding & aren’t large enough for passage of a flexible bronchoscope w suction capability.
?+/- tamponade the bleeding lung with bronchial blocker such as Arndt endobronchial blocker (theoretical risk mucosal ischaemia)
?suction with bronchoscope- flexible is less efficient cf rigid, flexible can be used to instil topical vasoconstrictors or haemostat agents, ?cooled saline
volume resuscitate, manage hypoxaemia-related arrhythmia (primarily by treating cause- optimising gas exchange) manage any coagulopathy/bleeding diathesis/replace blood products
apply PEEP to the bleeding lung if possible
transport pt to medical imaging for diagnosis & embolisation if feasible

How well did you know this?
1
Not at all
2
3
4
5
Perfectly
435
Q

correct position of bronchial cuff with L)-sided DLT?

A

5mm distal to carina without herniation across it

How well did you know this?
1
Not at all
2
3
4
5
Perfectly
436
Q

if develop massive haemoptysis from a PA catheter, which lung likely to have haemorrhage?

A

L)

How well did you know this?
1
Not at all
2
3
4
5
Perfectly
437
Q

how long is a PA catheter usually?

A

110cm

How well did you know this?
1
Not at all
2
3
4
5
Perfectly
438
Q

How do I define massive haemoptysis?

A

any volume of blood that could obstruct the airway or cause significant haemodynamic compromise (“magnitude of effect” definition preferred over blood volume- ie. could be described as “haemoptysis that impairs gas exchange, causes haemodynamic instability or is >100mL/24hrs (the minimum volume reported to be life-threatening due to asphyxia))

How well did you know this?
1
Not at all
2
3
4
5
Perfectly
439
Q

what is the source of most cases of massive haemoptysis?

A

bronchial circulation (supplies bronchial tree @ systemic pressure)

How well did you know this?
1
Not at all
2
3
4
5
Perfectly
440
Q

disadvantage of rigid bronchoscope for pulmonary haemorrhage?

A

only gets to the level of main bronchi (may not get distal bleeding)

How well did you know this?
1
Not at all
2
3
4
5
Perfectly
441
Q

How may hyperbaric oxygen help with carbon monoxide poisoning?

A

reduces the half life of carboxyHb (from 90mins at normobaric, down to 30mins) so HBO can prevent neurocognitive deficits associated with severe CO poisoning

How well did you know this?
1
Not at all
2
3
4
5
Perfectly
442
Q

What’s normal carboxyhemoglobin?

A

<1.5%, may be up to 9% in smokers

How well did you know this?
1
Not at all
2
3
4
5
Perfectly
443
Q

What are indications for the use of HBOT in acute CO toxicity? (note: it’s NOT recommended for those with mild-mod CO toxicity, who should receive high flow O2 until CO<5%)

A

CO>25%
CO >20% if pregnant
LOC
severe metabolic acidosis (pH <7.1)
evidence of end-organ ischaemia (ECG changes, chest pain, altered mental status)

How well did you know this?
1
Not at all
2
3
4
5
Perfectly
444
Q

when is the greatest benefit of HBOT for CO poisoning?

A

if Rx initiated within 6hrs (benefit >12hrs after exposure is unproven)

How well did you know this?
1
Not at all
2
3
4
5
Perfectly
445
Q

what are the differential diagnoses of deranged LFTs in pregnancy?

A

intra-hepatic cholestasis of pregnancy
PET with hepatic impairment (highest proportion of deranged LFTs in pregnancy are due to this)
HELLP syndrome
acute fatty liver pregnancy

How well did you know this?
1
Not at all
2
3
4
5
Perfectly
446
Q

management of acute fatty liver of pregnancy?

A

assess for encephalopathy
manage coagulopathy (plt function tends to remain stable, unlike HELLP, however use of regional may be precluded by coagulopathy)
manage hypoglycaemia

expedite delivery once pt stabilised as high maternal & foetal mortality

Pt must be in HDU, particularly postop as symptoms can deteriorate post partum with worsening liver, renal function & coagulopathy for 48hrs

How well did you know this?
1
Not at all
2
3
4
5
Perfectly
447
Q

poor prognostic indicators with severe liver disease in pregnancy?

A

lactate >2.8mg/dL & presence of encephalopathy

How well did you know this?
1
Not at all
2
3
4
5
Perfectly
448
Q

when may regional anaesthesia be considered in pregnant women with liver disease?

A

mild, stable disease, balancing risks & benefits to mum & Bub

How well did you know this?
1
Not at all
2
3
4
5
Perfectly
449
Q

normal trend of LFTs during pregnancy?

A

ULN transaminases is reduced by 25% in all trimesters
ALP increases in the 3rd trimester due to placental production

How well did you know this?
1
Not at all
2
3
4
5
Perfectly
450
Q

what’s the pattern of LFT changes with intrahepatic cholestasis of pregnancy?

A

elevated ALT (>1.5x normal), elevated bile acids (>1.5x normal), bilirubin is usually NORMAL

How well did you know this?
1
Not at all
2
3
4
5
Perfectly
451
Q

what’s the pattern of LFT derangement in PET with hepatic impairment?

A

raised ALT (>2x normal), BA & bilirubin usually normal

How well did you know this?
1
Not at all
2
3
4
5
Perfectly
452
Q

what’s the pattern of LFT derangement in HELLP syndrome?

A

raised ALT (>2x normal), BA usually normal, Bilirubin elevated (>1.5x normal)

How well did you know this?
1
Not at all
2
3
4
5
Perfectly
453
Q

what’s the pattern of LFT derangement in acute fatty liver of pregnancy?

A

elevated ALT (>3x normal), elevated bilirubin (>4x normal), bile acids usually normal

How well did you know this?
1
Not at all
2
3
4
5
Perfectly
454
Q

What’s HELLP, symptoms & management?

A

haemolysis, elevated liver function (ALT >2x ULN, elevated bilirubin >1.5x ULN, bile acids usually normal) & low platelets

can present in late 3rd trimester, can also present post-partum
occurs in 4-20% of pts with PET

The BP elevation may be mild, as may be the proteinuria

Pt may c/o upper quadrant pain/epigastric pain (in which case the pt should have abdo imaging as hepatic haematoma or rupture are rare but significant complications ass’d w high (up to 60%) mortality)
LFT derangement may lag behind abdo pain so the LFTs should be repeated if not elevated
N&V is common

may c/o dyspnoea/chest pain & have tachypnoea due to concomitant metabolic acidosis

The haemolysis is not usually severe enough to cause anaemia
DIC occurs in about 20% of cases
The trend in platelets should be monitored, replace if active bleeding or for operative delivery/placement of invasive lines

acute renal failure is more common in HELLP than in other forms of PET & morbidity is often ass’d with AKI

Pts should be managed in HDU given the course can be unpredictable
complications= placental abruption, APH/PPH, pneumonia, liver haematoma, pulmonary oedema, ICH

TREATMENT = Stabilisation & deliver foetus, timing of which balances risks to mother & foetus

How well did you know this?
1
Not at all
2
3
4
5
Perfectly
455
Q

what usually recovers first after HELLP? thrombocytopenia or liver function?

A

liver function

How well did you know this?
1
Not at all
2
3
4
5
Perfectly
456
Q

when does AFLP usually present?

A

late in 3rd trimester (30-38wks & up to 4.7 postpartum)

How well did you know this?
1
Not at all
2
3
4
5
Perfectly
457
Q

what are some associations with acute fatty liver of pregnancy (AFLP)?

A

more common in women with multiple pregnancies, lower BMI & children with disorders of B-fatty acid oxidation

How well did you know this?
1
Not at all
2
3
4
5
Perfectly
458
Q

why is acute fatty liver of pregnancy (rare!) an important diagnosis for which it is worth developing a protocol?

A

maternal mortality is high, ranges from 20-50%
foetal mortality also high

20% have concurrent HELLP

How well did you know this?
1
Not at all
2
3
4
5
Perfectly
459
Q

how is the diagnosis of acute fatty liver of pregnancy made?

A

Swansea criteria, there must be @ least six of the listed clinical features in the absence of another explanation:

Vomiting
Abdo pain
Polydypsia/polyuria
Encephalopathy

Hypoglycaemia
Elevated bilirubin
Ascites/bright liver on US
Leukocytosis

Coagulopathy
Renal impairment (develops in about 90% of pts)
Elevated urate
Elevated transaminases
Elevated ammonia
Microvesicular steatosis on liver biopsy

Imaging (CT or MRI), liver biopsy or fat stain may support the Dx

Other complications include: pancreatitis, ARDS, metabolic acidosis & elevated lactate

Urgent delivery then Mx of liver failure

How well did you know this?
1
Not at all
2
3
4
5
Perfectly
460
Q

what is the pattern of platelet function in AFLP? can regional be done with AFLP?

A

tends to be stable, unlike with HELLP

it’s often precluded by the presence of coagulopathy but it has been described & may improve hepatic blood flow

ALT/AST all raised, bili, ammonia & WBCs can also be raised

How well did you know this?
1
Not at all
2
3
4
5
Perfectly
461
Q

What’s obstetric cholestasis, what’s the general approach to management & what are anaesthetic considerations?

A

pregnancy-related impairment of bile acid excretion

causes pruritus, typically affecting hands & feet (palms & soles, worse @ night)- may be severe but rarely requires HDU

LFTs typically show raised ALT & bile acids (both >1.5x ULN), normal bilirubin

vitamin K malabsorption may occur (steatorrhoea) so the pt should have coagulation (raised PT, INR) assessed before proceeding with regional anaesthesia (<1.4 within 24hrs acceptable, changes slowly)

neuraxial opioid may worsen pruritus but this must be balanced against need for effective pain control

obstetric cholestasis may be a marker of other liver disease (eg. AFLP, hepatitis) so the collection of symptoms should be considered

mum treated with ursodeoxycholic acid (a synthetic bile acid)- 300mg BD or TDS until delivery- timing of delivery depends on total bile acid level, hepatic function, severity of the pruritus balanced against foetal risks

How well did you know this?
1
Not at all
2
3
4
5
Perfectly
462
Q

two broad classifications of liver disorders in pregnancy- which more common?

A

gestational-related conditions (more common) or liver disorders incidental to pregnancy

How well did you know this?
1
Not at all
2
3
4
5
Perfectly
463
Q

example viruses that can cause hepatitis?

A

hep A, B, C, D, E & G, along with HSV, CMV & EBV

How well did you know this?
1
Not at all
2
3
4
5
Perfectly
464
Q

what’s more common? hep B or C?

A

hep B

How well did you know this?
1
Not at all
2
3
4
5
Perfectly
465
Q

which hepatits-causing viral infections have a worse course in pregnancy?

A

hep E & HSV

hepatic encephalopathy & hepatorenal syndrome caused by viral hepatitis have higher incidence during pregnancy

viral hepatitis ass’d with obstetric complications eg. PROM, IUGR, prem delivery

How well did you know this?
1
Not at all
2
3
4
5
Perfectly
466
Q

clinical features of viral hepatitis?

A

fever, nausea, jaundice

How well did you know this?
1
Not at all
2
3
4
5
Perfectly
467
Q

what’s the usual course of autoimmune hepatitis in pregnancy? management implications?

A

usually improves during pregnancy (as with many other autoimmune conditions)

management= immunosuppressive therapy which should be continued in pregnancy; AZA, cyclosporine & tacrolimus considered safe but not mycophenalate (contraindicated, teratogenic)

How well did you know this?
1
Not at all
2
3
4
5
Perfectly
468
Q

for drug-induced liver injury, what tends to be the clinical picture for methyldopa/paracetamol/highly active antiretroviral therapy? oestrogens/progesterones/amoxicillin/psychotropic drugs? trimethoprim/nitrofurantoin/carbamazepine?

A

-hepatocellular damage: raised transaminases & sometimes bilirubin
-cholestatic picture (raised ALP & GGT)
-mixed picture (hepatocellular & cholestatic)

How well did you know this?
1
Not at all
2
3
4
5
Perfectly
469
Q

considerations for the rare occasion when a cirrhotic pt is pregnant?

A

10% mortality rate

portal HTN worsens so oesophageal varicies= major bleeding risk (lower if Rx before pregnancy)- potential for strain makes LSCS preferred mode for delivery

use Bilirubin, INR & Cr for prognosis

How well did you know this?
1
Not at all
2
3
4
5
Perfectly
470
Q

implications for liver transplant pts in pregnancy?

A

generally favourable outcomes but higher risk pre-term delivery, LBW, PET & GDM.

must continue immunosuppression (low risk teratogenicity with commonly-used agents)

consider side-effects of immunosuppressants (neuropathy, electrolyte imbalance)

How well did you know this?
1
Not at all
2
3
4
5
Perfectly
471
Q

what’s budd-chiari?

A

obstruction to hepatic venous outflow causing ascites & hepatomegaly +/- R) UQ pain. most cases are due to thrombosis.

How well did you know this?
1
Not at all
2
3
4
5
Perfectly
472
Q

general management considerations for pregnant women with severe liver disease?

A

individualised care plan
liaise with liver unit, ICU (requires MDT planning & management)
monitor carefully for complications & identify those @ risk of fulminant hepatic failure (for which liver transplantation may be considered- best predictors for deterioration necessitating liver transplant or resulting in death= lactate >2.8mg/dL with encephalopathy)

How well did you know this?
1
Not at all
2
3
4
5
Perfectly
473
Q

what are some signs of worsening hepatic synthetic function?

A

prolonged INR
hypoglycaemia
hypoalbuminaemia
lactic acidosis

How well did you know this?
1
Not at all
2
3
4
5
Perfectly
474
Q

how does cirrhosis alter propofol’s pharmacokinetics?

A

It doesn’t. And it doesn’t reduce hepatic blood flow so it’s a suitable induction agent.

How well did you know this?
1
Not at all
2
3
4
5
Perfectly
475
Q

what are the NDMRs of choice in liver disease?

A

actrac & cistatrac; duration of roc unpredictable but can be reversed with sugammadex.

How well did you know this?
1
Not at all
2
3
4
5
Perfectly
476
Q

considerations for neuraxial in pregnant pts w liver disease?

A

severe disease coagulopathy & need for resus will make regional unsuitable

mild disease: balance risks based on plt count & +INR. only do epidural in very mild, stable disease (issues w placement, catheter removal & that all LA undergoes hepatic metabolism)

How well did you know this?
1
Not at all
2
3
4
5
Perfectly
477
Q

periop steroid replacement for pt @ risk of HPA axis suppression due to tertiary adrenal insufficiency (eg. pred >=5mg for >=4/52):

A

100mg IV hydrocortisone with 200mgIV/24hrs OR 6-8mg dexamethasone

How well did you know this?
1
Not at all
2
3
4
5
Perfectly
478
Q

what does hepcidin do?

A

an acute phase reactant.
binds to ferroportin, stopping release of ferritin from enterocytes & macrophages.

How well did you know this?
1
Not at all
2
3
4
5
Perfectly
479
Q

what may inhibit hepcidin production?

A

Anaemia. EPO & other erythrocyte stimulation agents, decr body Fe stores, hypoxaemia, incr erythropoietic demand

in contrast, infection, inflammation & malignancy may increase serum hepcidin levels, as does increase in body iron stores

How well did you know this?
1
Not at all
2
3
4
5
Perfectly
480
Q

Effects of N2O abuse?

A

a few hrs exposure= megaloblastic changes to bone marrow
more prolonged exposure= agranulocytosis
neurological symptoms from chronic B12 inactivation (resembles subacute combined degeneration of spinal cord, from damage to the posterior (fine touch/vibration/proprioception loss) & lateral (motor from lateral corticospinal tract) columns: general weakness, tingling and numbness in the hands and feet, and stiff limbs and may become irritable, drowsy, and confused.. may also get visual changes)

How well did you know this?
1
Not at all
2
3
4
5
Perfectly
481
Q

The Brain Trauma Foundation guideline for management of severe head trauma recommend the treatment of intracranial pressures greater than?

A

22mmHg (as they recommend treating >=22mmHg)
IIa: Treating ICP above 22 mm Hg is recommended because values above this level are associated with increased mortality

How well did you know this?
1
Not at all
2
3
4
5
Perfectly
482
Q

To perform regional anaesthesia suitable for a fourth toe amputation, it is essential to block the

A

posterior tibial (to block lateral plantar nerve)
also need
superficial peroneal nerve (L4-S1) to block dorsal surface

How well did you know this?
1
Not at all
2
3
4
5
Perfectly
483
Q

what are the roots for the superficial perineal nerve?

A

L4-S1

How well did you know this?
1
Not at all
2
3
4
5
Perfectly
484
Q

potential interventions for postictal agitation after ECT? is remi useful?

A

IV BZD, donepezil or propofol at the end of a seizure, dexmed may be useful if refractory, pre-med olanzapine helps

remifentanil is NOT useful for agitation after ECT

How well did you know this?
1
Not at all
2
3
4
5
Perfectly
485
Q

on gastric US, how does empty stomach appear?

A

small, flat & collapsed (2-3cm diameter), either round or ovoid in XC & often appears as “bull’s eye” target and the muscular propriae (hypo echoic) appears thick.

How well did you know this?
1
Not at all
2
3
4
5
Perfectly
486
Q

What’s appropriate prophylaxis for surgical TOP?

A

Doxycycline 400mg PO 60mins before procedure (has been administered 10-12hrs b4 procedure so can have w food to limit risk nausea) or 100mg PO 60mins b4 then 200mg PO, 90 mins postop
-NO evidence for adding metronidazole prophylaxis

How well did you know this?
1
Not at all
2
3
4
5
Perfectly
487
Q

what acid/base disturbance would you see with acetazolamide?

A

lower pH promotes bicarb loss kidneys
hypokalaemia, hyponatraemia

How well did you know this?
1
Not at all
2
3
4
5
Perfectly
488
Q

Of the following, the procedure that is most commonly associated with chronic pain after surgery is
risk factors for chronic post-surgical pain include:

A

surgical factors= surgical approach with risk nerve damage):
amputation (30-85%)
sternotomy (30-50%)
thoracotomy (5-67%)
mastectomy (11-57%, breast surgery 13-35%)
inguinal herniotomy (5-63%)
LSCS (6-56%)
knee replacement 19-43%

pt factors:
psychological vulnerability (eg. catastrophising), pre-op anxiety
female
younger age (adults)
pre-op pain, mod-severe, lasting >1/12
repeat surgery
prev opioid use (esp if inefficient)
worker’s comp
genetic predisposition

others:
poorly controlled postop acute pain
radiation therapy to the area
neurotoxic chemotherapy

For breast Ca surgery, PVB, local infiltration, IV lignocaine reduce incidence CPSP. sparing of intercostobrachial nerve doesn’t decrease chest wall hypersensitivity.

How well did you know this?
1
Not at all
2
3
4
5
Perfectly
489
Q

What’s CPSP?

A

CPSP is pain developing or increasing in intensity after a surgical procedure, in the area of the surgery, persisting beyond that time for expected wound healing (ie. 3 months) but not better explained by another cause (eg. infection, malignancy, pre-existing pain)

How well did you know this?
1
Not at all
2
3
4
5
Perfectly
490
Q

which anion contributes most to the anion gap?

A

albumin (other unmeasured anion is phosphate under normal conditions, critically ill= lactic & keto acids)

How well did you know this?
1
Not at all
2
3
4
5
Perfectly
491
Q

indications for prothrombinex?

A

prevention or control of bleeding in pts with factor IX deficiency (haemophilia B) ** but due to risk of thrombosis, better to use specific factor concentrate eg. monofix-VF
or
life-threatening haemorrhage associated with warfarin is off-label)

could also use for pts with deficiencies of factor II or X

How well did you know this?
1
Not at all
2
3
4
5
Perfectly
492
Q

where should the tip of a PICC line sit? risks if too high? too low?

A

at the cavoatrial junction

thrombus

arrhythmia

How well did you know this?
1
Not at all
2
3
4
5
Perfectly
493
Q

SBP goal during ext cardiac compressions in a pt after cardiac surg?

A

60mmHg

How well did you know this?
1
Not at all
2
3
4
5
Perfectly
494
Q

what’s moclobemide? anaesthetic implications?

A

MAO-A inhibitor, reversible

avoid indirect-acting sympathomimetics & pro-serotonergic drugs

Can withold (normal return to MAO activity in 24hrs)
Interacts with:
pethidine (norpethidine serotonergic), tramadol, dextromethorphan, Fentanyl causes efflux of serotonin & binds to serotonin receptors. methadone, oxycodone, fentanyl & tapentadol are intermediate risk opioids. Tramadol & pethidine high risk.
ABx: linezolid
MDMA (releases serotonin)
indirect sympathomimetics can precipitate fatal hypertensive crisis. slowly titrate direct (Adr/norad/phenyl) due to incr receptor sensitivity.
M relaxants: phenelzine decr plasma cholinesterase & prolongs sux. Panc releases stored NAdr.
LAs: avoid cocaine, care w Adr mix.
bzd, volatiles, antichol/nsaids all OK.

How well did you know this?
1
Not at all
2
3
4
5
Perfectly
495
Q

what are the major causes of cardiac arrest after cardiac surgery?

A

VF, cardiac tamponade & major bleeding

How well did you know this?
1
Not at all
2
3
4
5
Perfectly
496
Q

up until what time point is a patient considered a cardiac surgical pt (during which the algorithm for arrest includes resternotomy)? why?

A

10 days. thereafter, pericardium is more difficult to access due to adhesions so open resus more challenging

How well did you know this?
1
Not at all
2
3
4
5
Perfectly
497
Q

what’s the survival after in-hospital cardiac arrest? chance of success of a 4th stacked shock?

A

50%

10% so no point in continuing after 3 stacked shocks- then amiodarone (VF or VT), ext cpr w opening chest within 5mins

How well did you know this?
1
Not at all
2
3
4
5
Perfectly
498
Q

what’s the only cardiac arrest rhythm within 10/7 of cardiac surgery for which we’d immediately perform external cardiac massage?

A

PEA

How well did you know this?
1
Not at all
2
3
4
5
Perfectly
499
Q

why not atropine for post cardiac surgery arrest due to extreme bradycardia or asystole?

A

pacing is the treatment. atropine delays that.

How well did you know this?
1
Not at all
2
3
4
5
Perfectly
500
Q

what adjunctive measures should be taken for cardiac arrest post cardiac surgery?

A

take pt off ventilator, manually ventilate
increase FiO2 to 100%
drop PEEP to zero to optimise preload
exclude tension PTx
stop sedating infusions & other infusions
switch IABP to trigger mode (which assists CPR)

How well did you know this?
1
Not at all
2
3
4
5
Perfectly
501
Q

what are the different isoenzymes of monoamine oxidase (which breaks down NAdr)?

A

MAO is bound to the outer mitochondrial membrane. It breaks down NAdr, serotonin & dopaimine.

Monoamine oxidase A is found in GI mocosa, liver, noradrenergic & serotonergic nerves- moclobemide is reversible & specific to MAO-A
MAO-B is found in platelets, liver & dopaminergic nerves, selegiline is specific to MAO-B

Phenelzine is a traditional MAO inhibitor inhibiting MAO nonselectively & irreversibly (worse reaction), duration weeks

reversible moclobemide

How well did you know this?
1
Not at all
2
3
4
5
Perfectly
502
Q

with which type of drugs do MAO-Is interact?

A

indirect-acting sympathomimetics (direct still has incr effect but preferred since also metabilised by COMT so less dependent on MAO, ie. phenylephrine good, ephedrine & metaraminol avoid), due to an exaggerated response to increased NAdr release from nerve endings

SSRI or tricyclics and tramadol & pethidine (latter cause serotonin & norepinephrine reuptake inhibition)- excessive amount of serotonin (MAO inhibition results in reduced breakdown & increased activity of serotonin)

they also interact with foods high in biogenic ingested amines such as tyramine (hypertensive crisis may occur due to loss of protective effect of MAO-A in the gut)

orthostatic hypotension can occur

may get excitatory (inhibition of serotonin reuptake by pethidine) or depressive (decreased breakdown of pethidine)

How well did you know this?
1
Not at all
2
3
4
5
Perfectly
503
Q

preferred vasopressor for obstetrics & why?

A

phenylephrine- more rapid onset & short DO, ideal for continuous infusion, no adverse effect on foetal acid:base cf ephedrine

How well did you know this?
1
Not at all
2
3
4
5
Perfectly
504
Q

What’s nephrogenic diabetes insipidus? example causes?

A

decrease in urinary concentrating ability resulting from resistance to the action of ADH, either due to resistance at the ADH site of action or interference with the countercurrent mechanism

causes include hereditary such as V2 gene mutations, aquaporin-2 gene mutations, lithium toxicity, hypercalcemia, hypokalaemia

How well did you know this?
1
Not at all
2
3
4
5
Perfectly
505
Q

what’s central diabetes insipidus? causes? manifestations? lab results?

A

characterised by decreased release of ADH–> polyuria due to deficiency in ability to concentrate urine

idiopathic, primary or secondary tumours, infiltrative diseases, neurosurgery, trauma

polyuria, nocturia, polydipsia, may have neurologic symptoms related to the underlying neurologic disease, may develop decreased BMD (even those treated with desmopressin)

elevated serum Na (high normal except if pt can’t express or access (eg. postop) the hypernatremia may become severe)
elevated serum osmolality
dilute urine (osmolality <600msomol/kg) low in Na

How well did you know this?
1
Not at all
2
3
4
5
Perfectly
506
Q

how to treat central diabetes insipidus?

A

correct free water deficit (if the pt has chronic >48hrs) hypernatremia, only want to lower the serum sodium by 10mmol/L in 24hrs (3mL/kg electrolyte-free water lowers serum Na by 1mmol/L), while acute hypernatremia want to replace entire water deficit within 24hrs (5% dextrose in water 3-6mL/kg/hr up to 666mL/hr, monitoring serum Na & glucose 1-2 hourly until serum Na <145mmol/L then reduce rate to 1mL/kg/hr until normal Na+)
*as per RCH, use 0.9% NaCl w 5% dextrose to prevent hyponatremia!

increase activity of ADH, with desmopressin (ADH analogue); oral 100mcg (start with 0.5microg & titrated, intranasal 10mcg (equivalent to 0.5microg PO is 5mircog intranasal), subcut 1microg

low-solute diet (low-sodium, reduced protein) which may be combined with a thiazide

How well did you know this?
1
Not at all
2
3
4
5
Perfectly
507
Q

side effect of Rx with desmopressin?

A

hyponatremia–> symptoms such as nausea, vomiting, headache, lethargy, seizures

How well did you know this?
1
Not at all
2
3
4
5
Perfectly
508
Q

What’s the main advantage of using noradrenaline over phenylephrine for the prevention of hypotension as a result spinal anaesthesia for elective Casearean Section?

A

less maternal bradycardia; similar agars, hypotension & use of rescue vasopressors

How well did you know this?
1
Not at all
2
3
4
5
Perfectly
509
Q

what are manifestations of HF (hydrofluoric acid) burns?

A

Hx: glass etching, electronic industries, cleaning solutions- highly corrosive acid

Local injury + potentially fatal systemic reaction

Fluoride complexes with Ca++ & Mg++ –> hypoCa & hypoMg. hypocalcaemia may cause K+ efflux–> hyperKalaemia. May get prolonged QTc.
fluoride directly cardiotoxic (arrhythmias, primary cause of death with HF burns)
inhalation can–> severe pulmonary injury

How well did you know this?
1
Not at all
2
3
4
5
Perfectly
510
Q

A patient with a purely metabolic acidosis has a serum bicarbonate of 14 mmol/L and a lactate of 3.8 mmol/L. What’s the expected PaCO2?

A

1.5x (bicarb) +8

21 + 8 = 29

How well did you know this?
1
Not at all
2
3
4
5
Perfectly
511
Q

what’s the most common arrhythmia in maternal cardiac arrest?

A

PEA (50%), asystole 25%

How well did you know this?
1
Not at all
2
3
4
5
Perfectly
512
Q

complications of hyperbaric O2 therapy?

A

Claustrophobia
Hypoglycaemia
Middle ear barotrauma
Sinus squeeze
Oxygen toxicity seizure
Progressive myopia – typically reverses completely in days to weeks.
Cumulative pulmonary oxygen toxicity
Pulmonary barotrauma +/- air embolism
Exacerbation of congestive heart failure in patients with severe disease, due to:
a. Sinus bradycardia from stimulation of vagal activity and stimulation of a further measurable, nonoxygen dependent bradycardia that is associated with hyperbaric pressures.
b. Systemic vasoconstriction causing increased afterload.
Increased rate of maturation of cataracts with very long courses of HBOT.

How well did you know this?
1
Not at all
2
3
4
5
Perfectly
513
Q

advice for woman on hormonal contraception who gets sugammadex?

A

if taking the pill, follow “missed pill advice” on packaging leaflet
if using hormonal implant/innjection, use barrier contraception for next 7/7

How well did you know this?
1
Not at all
2
3
4
5
Perfectly
514
Q

what are some benefits of volatile-based anaesthesia compared to total intravenous anaesthesia for maintenance during cardiac anaesthesia?

A

Meta-analysis 2020 Anaesthesiology (RCTs 8200 participants): pts receiving volatile had lower incidences of one-year mortality & MI along with less need for inotropic medications & cardiac troponin release, higher cardiac output or index measurements & shorter extubation times

NEJM RCT 2019: 5400pts randomly assigned to VA vs TIVA, no difference in mortality at 1 year or other outcomes incl MI.

histological evidence preconditioning (no mortality benefit in vivo)

How well did you know this?
1
Not at all
2
3
4
5
Perfectly
515
Q

what are disadvantages of volatile anaesthesia for cardiac surgery? and of TIVA?

A

difficulty controlling the volatile uptake with different oxygenators & difficulty maintaining steady state plasma [] with variable FGFrates during different phases of CPB, poor correlation between oxygenator exhaust volatile [] & BIS
inefficient scavenging & risk of OT environment pollution

TIVA problem with changes in PPB capacity & variations in the free fraction of plasma propofol due to haemodilution, effects of hypothermia on hepatic clearance of propofol, absorption of propofol into the CPB circuit

How well did you know this?
1
Not at all
2
3
4
5
Perfectly
516
Q

how does hypothermia during CPB impact anaesthetic requirement? risk of this?

A

hypothermia reduces anaesthetic requirement & if using volatile, there’s greater uptake & plasma solubility during hypothermia. to avoid inadequate anaesthesia during rewarming, dose of IV or volatile should increase (blood propofol [] rapidly decreases during rewarming.

How well did you know this?
1
Not at all
2
3
4
5
Perfectly
517
Q

what’s the most sensitive test of liver synthetic function? why?

A

INR, tests extrinsic & common pathways & if prolonged, suggests impaired clotting factor synthesis or deficiency of vit K; it’s non-specific (eg. prol in warfarin use, DIC). The clotting factors have relatively short half-lives (eg. VII 4-6 days).

How well did you know this?
1
Not at all
2
3
4
5
Perfectly
518
Q

how is albumin useful in evaluating liver synthetic function?

A

marker of chronic liver impairment due to relatively long half life (20/7)

How well did you know this?
1
Not at all
2
3
4
5
Perfectly
519
Q

while very non-specific, how may anaemia & low platelets be a marker of impaired liver function?

A

liver produces TPO & 10% of EPO so low Hb & plt may occur w hepatocellular damage

How well did you know this?
1
Not at all
2
3
4
5
Perfectly
520
Q

why may high ammonia & low urea (non-specifically) suggest impaired hepatic metabolic function?

A

liver converts toxic NH3 to the less toxic urea (for excretion via the urea cycle)

How well did you know this?
1
Not at all
2
3
4
5
Perfectly
521
Q

along with high ammonia & low urea, high bilirubin, what’s another indicator of impaired hepatic metabolic function?

A

low glucose

How well did you know this?
1
Not at all
2
3
4
5
Perfectly
522
Q

what may be the findings with bilirubin with different forms of liver metabolic derangement?

A

pre-hepatic cause (eg. haemolysis, Gilbert’s) may have high unconjugated bilirubin given there’d be exhaustion of the liver’s conjugating capacity (most common causes unconj hyperbili are Gilberts, haemolysis, resolution of haematoma or portal HTN)

high direct bilirubin suggests biliary obstruction

mixed (both high) suggests hepatic disease & impaired hepatic uptake & conjugation of bilirubin

How well did you know this?
1
Not at all
2
3
4
5
Perfectly
523
Q

Where are AST & ALT found?

A

both are found in hepatocyte mitochondria so elevation suggests hepatocellular damage

AST is also found in heart, rbc, skeletal muscle

ALT therefore more specific to hepatocytes so elevated ALT:AST more suggestive of hepatocellular damage

How well did you know this?
1
Not at all
2
3
4
5
Perfectly
524
Q

aside from serum aminotransferases, what else may be elevated with hepatocellular injury?

A

LDH (found in hepatocytes, also in heart, pancreas, rbc, lungs, placenta so can be elevated if MI, Ca, haemolysis etc.

How well did you know this?
1
Not at all
2
3
4
5
Perfectly
525
Q

where are GGT & ALP found & what’s the significance of their elevation?

A

enzymes found in hepatocytes AND ductal cells along bile duct/canaliculus; high ALP & GGT suggest hepatobiliary disease (intra- and extra-hepatic cholestasis)

in isolation, GGT is non-specific (raised with ETOH)
Also, ALP is non-specific (bone, placenta)

How well did you know this?
1
Not at all
2
3
4
5
Perfectly
526
Q

what are the markers of cholestasis?

A

elevated conjugated bilirubin, GGT, ALP

How well did you know this?
1
Not at all
2
3
4
5
Perfectly
527
Q

What’s pacemaker nomenclature?

A

position 1: chamber paced (A, V or D (dual))
position 2: chamber sensed (A, V, D or O if sensing is turned off)
postion 3= response to sensing; inhibited, triggered (no clinical purpose), D (response can be I or T), O if no response to sensed events
position 4= rate modulation (eg. with accelerometer, responds to perceived increased physiological demand eg bicycling)
position 5= multi-site pacing

How well did you know this?
1
Not at all
2
3
4
5
Perfectly
528
Q

what does asynchronous pacing mean?

A

heart’s intrinsic conduction system & artificial pacemaker are firing independently of what the other is doing; the paced beat only generates a contraction if it occurs outside of the absolute refractory period. only used as a temporary mode during surgery (only use on pacing-dependent pt. risk of R on T (pacing spike occurring on a T wave))

How well did you know this?
1
Not at all
2
3
4
5
Perfectly
529
Q

what’s VVI mode & when is it useful or not?

A

has both ventricular sensing & pacing; if it senses ventricular conduction it will inhibit pacing, which avoids the risk of R on T. if a prolonged period has passed without ventricular activity, it will pace. no synchrony between the atria & ventricles, so risk having an atrial contraction during a paced ventricular beat (“pacemaker syndrome”), inadequate cardiac output, not useful for pts in sinus rhythm, more useful for a pt with AV block & chronic AF.

How well did you know this?
1
Not at all
2
3
4
5
Perfectly
530
Q

what’s AAI? indication? does it risk R on T?

A

theoretically a single lead in RA (but this is VERY RARE, since most pts with sinus node disease have some degree of AV block- it’d be more common to have dual chamber pacing wires & set it to AAI mode with the potential to have some ventricular pacing were AV block to occur), rather than sensing & pacing QRS complexes it senses & paces P waves.

used for pts with isolated sinus node disease without AV block.

even if it paces during the T wave doesn’t risk v fib as it’s pacing the A not V.

How well did you know this?
1
Not at all
2
3
4
5
Perfectly
531
Q

what’s the indication for VDD mode?

A

AV block without sinus node dysfunction- this mode will pace the ventricle but senses both A & V, response can be inhibit or trigger. eg. if senses a p wave but there’s AV block, it’ll trigger a ventricular depolarisation (appears as wide, abnormal QRS). if senses ventricular premature contraction it will inhibit ventricular pacing.

How well did you know this?
1
Not at all
2
3
4
5
Perfectly
532
Q

when is DDD mode used? what beats can be generated?

A

pt with AV block & sinus node dysfunction, in sinus rhythm. this is also the mode used for pacing in bradycardia arrest after cardiac surgery

Can be:

presence of a & v activity are both sensed so pacing inhibited to both

a activity is sensed hence activity inhibited & lack of v activity sensed hence it’s ventricular triggered (paced)

lack of a activity sensed hence it’s is paced & v activity is sensed hence inhibited

neither a or v activity sensed hence both triggered

How well did you know this?
1
Not at all
2
3
4
5
Perfectly
533
Q

what’s the lower rate limit of a pacemaker?

A

lowest rate at which the device will allow the heart to beat

How well did you know this?
1
Not at all
2
3
4
5
Perfectly
534
Q

what’s the maximum tracking rate for a pacemaker?

A

fastest rate at which the device will track the heart- it’ll track with the atrial rate up until the max limit set (typically 120-130bpm, may be higher in young/middle aged)

How well did you know this?
1
Not at all
2
3
4
5
Perfectly
535
Q

what’s the maximum tracking rate for a pacemaker?

A

fastest rate at which the device will track the heart- it’ll track with the atrial rate up until the max limit set (typically 120-130bpm, may be higher in young/middle aged)

How well did you know this?
1
Not at all
2
3
4
5
Perfectly
536
Q

what’s the AV delay setting on a pacemaker?

A

used in VDD or DDD modes- it’s the maximum time that it’ll allow for delay between either sensing or paced atrial depolarisation before it’ll trigger ventricular activity (if intrinsic activity isn’t sensed beforehand), usually 150-200ms but this can be modulated ie. if a paced atrial activity the AV delay will be longer, or the it can be rate adaptive with the AV delay shorter during perceived exertion, as in the case of a natural pacemaker

How well did you know this?
1
Not at all
2
3
4
5
Perfectly
537
Q

what’s the post ventricular atrial refractory period on a pacemaker?

A

a safety feature- minimum time after ventricular depolarisation paced or sensed, during which the pacemaker will ignore sensed atrial depolarisation- this prevents the pacemaker tracking excessive atrial impulses (eg. retrograde impulses) & causing dangerously high ventricular rates; typically 250-300ms & can be rate adaptive

How well did you know this?
1
Not at all
2
3
4
5
Perfectly
538
Q

placing a magnet over a ventricular pacemaker is expected to switch it to which mode?

A

VOO

How well did you know this?
1
Not at all
2
3
4
5
Perfectly
539
Q

which areas require analgesia for thoracotomy?

A

posterolateral thoracotomy (traverses 6 dermatomal levels, starting at T3 posteriorly & going to approx T8 anteriorly) provides good access to lung, oesophagus, mediastinal structures, descending aorta, diaphragm
anterolateral & axillary thoracotomies are used for specific lung resections (typically 5th ICS= lung resection), thoraco-abdominal for oesophageal (often @ 7th ICS for oesophageal), aortic or upper abdo, clamshell for bilateral (eg. lung transplant)
chest drains are usually inserted around T8-9 ICS post thoracotomy.

require coverage of the impulses from:
-skin & intercostal muscles
-parietal pleura (highly sensitive, from intercostal & phrenic nerves); visceral pleura insensate
-lung & mediastinum (carried via vagus nerve)
-also the lat dorsi (thoracodorsal, C6-8) & serratus anterior (long thoracic, C5-7)

How well did you know this?
1
Not at all
2
3
4
5
Perfectly
540
Q

at which level is a Tx epidural sited?

A

mid-point of dermatomal distribution

How well did you know this?
1
Not at all
2
3
4
5
Perfectly
541
Q

at which level is a Tx epidural sited?

A

mid-point of dermatomal distribution

How well did you know this?
1
Not at all
2
3
4
5
Perfectly
542
Q

at which level is a Tx epidural sited?

A

mid-point of dermatomal distribution

How well did you know this?
1
Not at all
2
3
4
5
Perfectly
543
Q

what’s the warm ischemia time for procuring heart/liver/pancreas? kidneys? lungs?

A

30 mins/ 60 mins / 90 mins

How well did you know this?
1
Not at all
2
3
4
5
Perfectly
544
Q

max cold ischaemia time various organs?

A

heart 4hrs / lungs 6-8hrs /liver or pancreas 6hrs DCD or 12 hrs DBD / kidneys 12hrs DCD or 18hrs DBD

How well did you know this?
1
Not at all
2
3
4
5
Perfectly
545
Q

whats the Rx for an adult developing seizures following a brachial plexus block with ropivacaine?

A

CONTROL SEIZURES with a BZD (midazolam 2-5mg (0.05-0.1mg/kg) or diazepam 2-5mg) or this (1mg/kg) or careful boluses of propofol (NOT if CV instability)
then intralipid (20% lipid emulsion), 1.5mL/kg over 1 min (this is 100mL in a 70kg adult), then an infusion of 15mL/kg/hr (1000mL/hr in adults)
if refractory, can do a max 2 further boluses, 5 mins apart, or can double the rate (30mL/kg/hr) after 3 mins

BUT the max cumulative dose is 12mL/kg (ie. for a 70kg, don’t give > 840mL)

How well did you know this?
1
Not at all
2
3
4
5
Perfectly
546
Q

what are some considerations for circulatory arrest with LAST?

A

recovery from LA-induced cardiac arrest may take >1hr
lidocaine should not be used as anti-arrhythmic therapy

How well did you know this?
1
Not at all
2
3
4
5
Perfectly
547
Q

what condition should be monitored for following administration of lipid rescue?

A

pancreatitis; daily lipase & amylase

How well did you know this?
1
Not at all
2
3
4
5
Perfectly
548
Q

What’s ANZTADC & WebAirs?

A

Aust/NZ tripatriate Anaesthetic Data Committee, representing 3 organisations (ANZCA, ASA, NZ Society of Anaes), developed the WebAirs online reporting database for adverse anaesthetic incidents, can link into hospital system assisting Anaesthetists to report, evaluate & receive info re: adverse incidents- the info can be fed back to the system for closing the QI loop.

How well did you know this?
1
Not at all
2
3
4
5
Perfectly
549
Q

why is propofol an unsuitable substitute for lipid emulsion?

A

significant cardiovascular depression with propofol

How well did you know this?
1
Not at all
2
3
4
5
Perfectly
550
Q

on what should intralipid dosing be based in extremely obese pts?

A

lean body weight

How well did you know this?
1
Not at all
2
3
4
5
Perfectly
551
Q

what does indocyanine green do to SpO2?

A

brief, artefactual reduction (typically to low 90s)

How well did you know this?
1
Not at all
2
3
4
5
Perfectly
552
Q

wavelength of absorption peak of red & IR radiation? and indocyanine green?

A

660 & 940nm

805nm (indocyanine green increases the absorption of light, interpreted as increased oxyHb & falsely elevating NIRS readings)

How well did you know this?
1
Not at all
2
3
4
5
Perfectly
553
Q

what accounts for the clinical manifestations of congenital diaphragmatic hernia?

A

pathologic effects of the herniated viscera on lung development- with lung compression there are decreases in bronchial & pulmonary artery branching & pulmonary hypoplasia ensues, then there’s muscular hyperplasia of the pulmonary arterial tree & risk of pulmonary HTN.

postnatally the infant often presents with resp distress

How well did you know this?
1
Not at all
2
3
4
5
Perfectly
554
Q

what’s a common endocrine finding in infants with CDH?

A

adrenal insufficiency

How well did you know this?
1
Not at all
2
3
4
5
Perfectly
555
Q

which therapy is most likely to decrease mortality in neonates with congenital diaphragmatic hernia? how do this?

A

lung-protective ventilation, they avoid further injury to damaged lung tissue & decrease mortality

limit peak Pinsp to 25cmH2O, keep PEEP 3-5cmH2O & allow permissive hypercapnia

How well did you know this?
1
Not at all
2
3
4
5
Perfectly
556
Q

what’s the role of high freq oscillatory ventilation in CDH?

A

reserved as a rescue when hypoxia & severe hypercapnia persist despite maximal conventional ventilation; as the initial mode of ventilation in CDH, HFOV has no difference in mortality but conventional ventilation has shorter time of ventilation, lower NO requirements, sildafenil & ECMO & less inotropic requirement.

How well did you know this?
1
Not at all
2
3
4
5
Perfectly
557
Q

why is CDH repair not necessarily considered a surgical emergency?

A

because the main risk/prognostic factor is the degree of pulmonary hypoplasia so emergency surgery confers little benefit
there’s much debate & little consensus re: optimal timing of surgery, prior to surgery the pt should have:
-normal MAP for gestation
-pre-ductal SpO2 consistently 85-95% on FiO2 <0.5
-lactate <3mmol/L
-UO >1mL/kg/hr

How well did you know this?
1
Not at all
2
3
4
5
Perfectly
558
Q

haemodynamic change expect to see in 1st 24hr following major (adult >10%TBSA) burn?

A

incr SVR

How well did you know this?
1
Not at all
2
3
4
5
Perfectly
559
Q

via what is an Ivor lewis oesophagectomy performed?

A

abdo phase: upper midline (T6-10) laparotomy or rooftop abdo incision (T8-9 bilaterally)
Tx phase: R) posterolateral thoracotomy at the 4th-5th ICS

How well did you know this?
1
Not at all
2
3
4
5
Perfectly
560
Q

what’s the recommended maximum cuff pressure for insufflating a classic LMA?

A

60cmH2O

How well did you know this?
1
Not at all
2
3
4
5
Perfectly
561
Q

what are the classic LMA sizes, pt weight ranges & max cuff volumes?

A

1 - neonate/infant up to 5kg - 4mL
1.5 - 5-10kg - 7mL
2 - 10-20kg - 10mL
2.5 - 20-30kg - 14mL
3 - 30-50kg - 20mL- max tube size 6
4 - 50-70kg - 30mL
5 - 70-100kg - 40mL
6 - adults >100kg - 50mL

How well did you know this?
1
Not at all
2
3
4
5
Perfectly
562
Q

what is the max size ETT that can fit in each of the ambu LMAs?

A

1 - 3.5
1.5 - 4
2 - 5
2.5 - 5.5
3- 6.5
4- 7.5
5 & 6 - 8

How well did you know this?
1
Not at all
2
3
4
5
Perfectly
563
Q

What’s the hunsacker mon-jet used for? benefits? how differ to laser jet double-lumen catheter?

A

trans-glottic ventilation

all infra-glottic ventilation options have the advantages of:
-minimising glottic movement
-ensuring debris is blown away from vs into the tracheobronchial tree with exp flow
-minimal entrainment of air (allows consistently delivered FiO2)
-can measure airway pressures & EtCO2

-hunsacker mon-jet has a basket to help secure it’s position within trachea during jet ventilation & limit risk of mucosal trauma however risk of shearing a laryngeal lesion on insertion which may seed tumour or virus (use of videolaryngoscope may limit this risk)
-laserket is also a double-lumen, laser-resistant tube for trans-glottic ventilation (and EtCO2 can be measured), has a rounded tip & no basket so may be less traumatic with insertion/removal but not stabilised in trachea.

How well did you know this?
1
Not at all
2
3
4
5
Perfectly
564
Q

What’s the hunsacker mon-jet used for? benefits? how differ to laser jet double-lumen catheter?

A

trans-glottic ventilation

all infra-glottic ventilation options have the advantages of:
-minimising glottic movement
-ensuring debris is blown away from vs into the tracheobronchial tree with exp flow
-minimal entrainment of air (allows consistently delivered FiO2)
-can measure airway pressures & EtCO2

-hunsacker mon-jet has a basket to help secure it’s position within trachea during jet ventilation & limit risk of mucosal trauma however risk of shearing a laryngeal lesion on insertion which may seed tumour or virus (use of videolaryngoscope may limit this risk)
-laserjet is also a double-lumen, laser-resistant tube for trans-glottic ventilation (and EtCO2 can be measured), has a rounded tip & no basket so may be easier to traverse the glottis but potentially more traumatic in the trachea.
-laserjet is smaller (3.4 vs 4.3mm ED) but either could interfere w access to posterior glottic lesion

How well did you know this?
1
Not at all
2
3
4
5
Perfectly
565
Q

According to international perioperative guidelines, what plasma ferritin concentration, among pts presenting for major surgery, is diagnostic of inadequate or low Fe stores?

A

<100microg/L

How well did you know this?
1
Not at all
2
3
4
5
Perfectly
566
Q

What’s the parasternal short axis view useful for on TTE?

A

info re: the overall contractility of the heart or regional abnormalities (since it images all 3 territories), volume status (eg. collapsibility of the ventricle), may see evidence of large pericardial effusion or intracardiac mass, deviation of the inter ventricular septum.
place L) of sternum, 3-4th ICS, marker to R) hip & US marker will be to L) of screen

How well did you know this?
1
Not at all
2
3
4
5
Perfectly
567
Q

what are the 3 views of focussed transthoracic echo?

A

apical
subcostal
parasternal

How well did you know this?
1
Not at all
2
3
4
5
Perfectly
568
Q

what’s the apical 4-chamber view useful for in focussed TTE?

A

comparing ventricle sizes- evidence of RHF or RV dilation (eg. if PE)

How well did you know this?
1
Not at all
2
3
4
5
Perfectly
569
Q

what are the 3 branches of the posterior tibial nerve?

A

medial & lateral plantar, calcanea

How well did you know this?
1
Not at all
2
3
4
5
Perfectly
570
Q

which nerves need to be blocked for complete anaesthesia for amputation of 5th toe?

A

sural, lateral plantar & superficial peroneal

How well did you know this?
1
Not at all
2
3
4
5
Perfectly
571
Q

which actions are provided by C8-T1 via the median nerve, NOT the ulnar nerve, thereby being the movements for distinguishing C8-T1 radiculopathy from ulnar nerve lesion (eg, cubital tunnel syndrome)?

A

the ulnar nerve does all intrinsic hand movements EXCEPT:
thumb ABduction (APB), thumb flexion (flexor pollicis brevis), thumb opposition (opponens pollicis), lateral 2 lumbricals; these are innervated by C8-T1 via the median nerve

How well did you know this?
1
Not at all
2
3
4
5
Perfectly
572
Q

which muscles provide shoulder external rotation? internal?

A

infraspinatus & teres minor

internal is subscapularis

abduction initiated by supraspinatus

How well did you know this?
1
Not at all
2
3
4
5
Perfectly
573
Q

what innervates infraspinatus, supraspinatus?

A

suprascapular nerve, C5-6

How well did you know this?
1
Not at all
2
3
4
5
Perfectly
574
Q

what innervates teres minor?

A

axillary nerve, C5-6

How well did you know this?
1
Not at all
2
3
4
5
Perfectly
575
Q

innervation supraspinatus?

A

supra scapular nerve, C5-6

How well did you know this?
1
Not at all
2
3
4
5
Perfectly
576
Q

what does corrugfator supercilli do? and orbiculares oculi?

A

eyebrow movements, eg. frown
close eyelids

How well did you know this?
1
Not at all
2
3
4
5
Perfectly
577
Q

why is the onset of non-depolarising neuromuscular blockade faster at the larynx & diaphragm (central muscles) vs periphery?

A

central with high perfusion, small fibre size (larynx moree so), despite the fact that these muscles have high proportion of type II fibres & relatively high density of nAChR, conferring some resistance to NDMBs.

adductor policis has predominantly type 1 slow twitch hence lower density nAChR, is more sensitive to NDNMBDs. The APM is more sensitive to nondepolarizing NMBAs, so recovery is delayed in comparison to central muscles (diaphragm, laryngeal muscles.

offset sequence:
diaphragm > larynx > corrugated supercilii (correlates well w larynx) > abdo mm > orbiculares oculists > geniohyoid (upper airway, very sensitive to NDNMBs) > AP (very sensitive)

How well did you know this?
1
Not at all
2
3
4
5
Perfectly
578
Q

what’s the management for an unstable pt with tachycardia?

A

synchronised DC shock, up to 3 attempts; 70-120J biphasic is the most common initial energy (except for AF, where 120-150J biphasic is recommended)

How well did you know this?
1
Not at all
2
3
4
5
Perfectly
579
Q

what synchronised DC cardioversion energy should be given for unstable tachycardia? should it be done under sedation or GA? what medication then give?

A

initial energy 70-120J biphasic (up to 3 attempts)

do under sedation or GA

if AF, 120-150J biphasic

give amiodarone 300mg IV over 10-20mins (dose reduction may be reasonable if also on digoxin), then 900mg over 24hrs (*or if the rhythm was torsades, give Mg++ 2g over 10 mins)

How well did you know this?
1
Not at all
2
3
4
5
Perfectly
580
Q

how to manage a stable pt w narrow complex regular tachycardia?

A

with continuous ecg monitoring:

vagal maneouvers (valsalva +/- leg lift or knees to chest, carotid sinus massage, diving reflex (cold ice on face))

adenosine 6mg rapid IV bolus; repeat 12 up to 2x more

once sinus rhythm restored, consider it’s probably re-entry PSVT, 12-lead ecg to be recorded, if recurs, repeat adenosine Rx, consider anti-arrhythmic prophylaxis

if sinus not restored, seek expert help

How well did you know this?
1
Not at all
2
3
4
5
Perfectly
581
Q

how to manage pt w narrow complex irregular tachycardia?

A

most likely AF

IV B blocker or digoxin

or if <48hrs onset, cardiovert with amiodarone 300mg IV over 20-60mins then 900mg over 24hrs (dose reduce if on digoxin)

How well did you know this?
1
Not at all
2
3
4
5
Perfectly
582
Q

what are the differentials for broad complex, regular tachycardia (stable pt)?

A

VT or SVT with BBB, former is Rx with amiodarone, latter with adenosine

How well did you know this?
1
Not at all
2
3
4
5
Perfectly
583
Q

what are the differentials for broad complex irregular tachycardia?

A

AF with BBB (if so, Rx as for narrow complex)

pre-excited AF (consider amiodarone)

polymorphic VT (eg. torsades, Mg++ 2g over 10 mins)

would seek expert help for this

How well did you know this?
1
Not at all
2
3
4
5
Perfectly
584
Q

In the POISE trial, what were the outcomes for pts with or at risk of atherosclerotic disease (RCRI 1 or 2) undergoing non cardiac surgery, receiving prophylactic B blockers DoS?

A

Reduced MI (which overwhelmingly contributed to a sig reduction in their composite outcome of CV death/nonfatal MI/non-fatal cardiac arrest), new clinically significant AF & cardiac revascularisation

BUT

increased:
total mortality
nonfatal stroke
clinically important hypotension (showing a significant link with death & stroke) & bradycardia

How well did you know this?
1
Not at all
2
3
4
5
Perfectly
585
Q

what’s the aseptic technique for central neuraxial blockade?

A

education to pt & staff re: the sterile field, thorough handwashing with surgical scrub solution, barrier precautions (cap/mask/gown/gloves/drape & eye protection, sterile drape), chlorhexidine in alcohol (0.5% chlorhexidine in alcohol, given that there’s no evidence of superiority of 2% vs 0.5% for antimicrobial superiority & clear evidence of the neurotoxicity of chlorhexidine. For chn <2yo, volume should be absolute minimum necessary while still ensuring antisepsis) applied moving in to out. Prior to skin asepsis which is allowed to dry completely before skin palpated or punctured. Meticulous measures must be in place to ensure the chlorhexidine does not reach CSF (eg. use of swabsticks, dyed solution, equipment covered or protected while the antiseptic is applied).

How well did you know this?
1
Not at all
2
3
4
5
Perfectly
586
Q

what’s the recommended skin decontamination for central VENOUS access?

A

2% chlorhex in ETOH

How well did you know this?
1
Not at all
2
3
4
5
Perfectly
587
Q

for how long does isopropyl alcohol provide antimicrobial activity? And chlorheexidine?

A

ETOH at least 6hrs, chlorhexidine more effective @ 24hrs.

How well did you know this?
1
Not at all
2
3
4
5
Perfectly
588
Q

for how long does isopropyl alcohol provide antimicrobial activity? And chlorheexidine?

A

ETOH at least 6hrs, chlorhexidine more effective @ 24hrs.

How well did you know this?
1
Not at all
2
3
4
5
Perfectly
589
Q

2nd line antiseptic solution forsaken prep before neuraxial blockade?

A

10% povidone iodine

How well did you know this?
1
Not at all
2
3
4
5
Perfectly
590
Q

what breaks down remifentanil?

A

non-specific plasma esterase’s

How well did you know this?
1
Not at all
2
3
4
5
Perfectly
591
Q

normal PCWP?

A

4-12mmHg

PAP 20-30/8-15mmHg

*PCWP>15mmHg ass’d w LV dysfunction

How well did you know this?
1
Not at all
2
3
4
5
Perfectly
592
Q

if a pt has R) heart Cath to Ix new-onset dyspnoea, PCWP 10mmHg & waveform showing PAP 74/28, most likely Dx?

A

PE, since PCWP normal it’s not related to L) heart & acute more likely PE vs empysema

How well did you know this?
1
Not at all
2
3
4
5
Perfectly
593
Q

infraclavicular blocks the BP at the level of the…

A

cords

How well did you know this?
1
Not at all
2
3
4
5
Perfectly
594
Q

is ecg essential to use for GA & major regional?

A

no but it must be immediately available & may be used if clinically indicated

PS18:
-breathing system MUST have O2 analyser
-GA/sedation MUST have pulse ox
-whenever auto ventilator, disconnection/vent failure alarm MUST be on & continuous & auto activated,
-CO2 monitor foall GA, avail for sedation
-ECG must be avail for all anaes; use if GA/major RA as indicated
-NIBP avail for all pts; when used, @ least 10 minutely. IABP avail
-BIS when clinically indicated
-for all pts undergoing GA or if using inhalational; must have inhaled anaes agent monitor
-temp monitor avail for all pts having GA, use it if using warmer
-quant NMT avail for all pts w NMB, use whenever extubating pts after NDNMBAs
-other equip (eg. oximetry) avail as indicated

How well did you know this?
1
Not at all
2
3
4
5
Perfectly
595
Q

what’s thrombin time? what factors do influence it? do warfarin & direct Xa inhibitors influence it?

A

measures the final step in coagulation, conversion of fibrinogen to fibrin.
usually 14-19secs.
prolonged if fibrinogen levels are low or if an agent inhibiting thrombin is present (eg. dabigatran, heparin/LMWH/bivalirudin)

warfarin & direct Xa inhibitors don’t influence thrombin time.

How well did you know this?
1
Not at all
2
3
4
5
Perfectly
596
Q

what’s prothrombin time? what influences it?

A

lab test measuring the extrinsic & common coagulation pathways.

used to monitor LT use of anticoagulant through INR, evaluate liver function & evaluate coagulation disorders; may be prolonged by vit K antagonists, direct Xa inhibitors, vit K insufficiency, liver disease (esp obstructive jaundice), DIC

normal range 11-13 seconds.

INR = (PTtest/PTnormal)^ISI

How well did you know this?
1
Not at all
2
3
4
5
Perfectly
597
Q

what’s aPTT, what influences it?

A

lab test of intrinsic & common pathways

best test for coagulation disorder, if performed properly it’s abnormal in 90% of pts with a coagulation disorder

used to monitor heparin, screen for haemophilia A (def VIII), B (lack of IX) & to screen for clotting inhibitors. Also prol in liver disease, deficiency of any coagulation factor other than VII, massive transfusion with plasma-depleted red cells, DIC.

N range 25-38 secs

How well did you know this?
1
Not at all
2
3
4
5
Perfectly
598
Q

what magnitude of impact does an airway obstruction have to be to produce the fixed airway obstruction pattern with truncation on both inspiration & expiration?

A

lesion must restrict trachea to <1cm

How well did you know this?
1
Not at all
2
3
4
5
Perfectly
599
Q

What ARE some complications from dural puncture and resultant intracranial hypotension? what do they NOT include?

A

seizure
cortical vein thrombus
subdural haematoma
stroke

NOT encephalitis

How well did you know this?
1
Not at all
2
3
4
5
Perfectly
600
Q

what are the segments of the R) middle lobe? lingual?

A

medial & lateral

superior & inferior

How well did you know this?
1
Not at all
2
3
4
5
Perfectly
601
Q

what are Sgarbossa’s criteria?

A

used to diagnose myocardial infarction in pts with L) BBB or ventricular paced rhythms

in such pts, its common to see “appropriate discordance” where abnormal ventricular depolarisation is followed by abnormal depolarisation (eg. ST depression/TWI with +ve complexes, STE & T waves normal hight in -ve complexes)

the 3 criteria to raises suspicion of infarction in pts with L) BBB:

  1. concordant STE >1mm in leads with +ve QRS (score 5)
  2. concordant ST depression >1mm in V1-3 (score 3)
  3. excessively discordant STE >5mm in leads with a -ve QRS (score 2)

a total score >= 3 has high specificity for diagnosing MI

How well did you know this?
1
Not at all
2
3
4
5
Perfectly
602
Q

signs of arterial retrobulbar haemorrhage?

A

rapid development proptosis
reduced visual acuity
deficiency in extra ocular movements
tense eye or other signs of raised IOP

How well did you know this?
1
Not at all
2
3
4
5
Perfectly
603
Q

for Parkland, when use 4mL/kg/%TBSA?

A

if electrical/inhalational burn, dehydrated or trauma

How well did you know this?
1
Not at all
2
3
4
5
Perfectly
604
Q

what’s the usual source of allergy to egg?

A

proteins in the white, not the yolk (propofol has egg lecithin, from the yolk)

How well did you know this?
1
Not at all
2
3
4
5
Perfectly
605
Q

do you give propofol to pts with egg anaphylaxis?

A

adults- yes, but be prepared for anaphylaxis
children, prob not (but would give if mild egg allergy)

How well did you know this?
1
Not at all
2
3
4
5
Perfectly
606
Q

idarucizumab reverses anticoagulant effect of?

A

dabigatran

How well did you know this?
1
Not at all
2
3
4
5
Perfectly
607
Q

During the 21st century, the dominant ozone-depleting substance emitted as a result of medical usage to date has been?

A

N2O

How well did you know this?
1
Not at all
2
3
4
5
Perfectly
608
Q

During the 21st century, the dominant ozone-depleting substance emitted as a result of medical usage to date has been?

A

N2O

How well did you know this?
1
Not at all
2
3
4
5
Perfectly
609
Q

do narcotics/opioids have impact on SSEPs? ketamine?

A

opioids don’t really, ketamine may increase SSEPs (but neither ketamine nor opioids impact MEPs)

How well did you know this?
1
Not at all
2
3
4
5
Perfectly
610
Q

does dexmed impact SSEPs or MEPs?

A

evidence is conflicting, MEPs may be lost @ higher doses, impact on SSEPs is minimal

How well did you know this?
1
Not at all
2
3
4
5
Perfectly
611
Q

how to calculate QTc?

A

QT / square root of the cardiac cycle in secs. therefore, if the HR is 60, R-R interval is 5 squares so the QTc is the same as QT (RR = 60/HR)

How well did you know this?
1
Not at all
2
3
4
5
Perfectly
612
Q

what condition ass’d w loss of a waves on CVP trace?

A

AF

How well did you know this?
1
Not at all
2
3
4
5
Perfectly
613
Q

what condition(s) ass’d w cannon a waves (fusion of a & c waves) on CVP trace?

A

junctional rhythm (atrial contraction occurs @ same time as ventricular so there’s a fusion of a & c waves; atria contracting against closed TCV so cannon a wave)
VT
ventricular pacing
*any conditions where there may be retrograde conduction of ventricular action potentials

complete heart block

these situations create a mechanical disadvantage for the cardiac output- expect a SBP drop- as the ventricle robbed of preload

How well did you know this?
1
Not at all
2
3
4
5
Perfectly
614
Q

what may the CVP trace look like in significant TCR?

A

fused C & V waves; backflow of blood obliterates the X descent

How well did you know this?
1
Not at all
2
3
4
5
Perfectly
615
Q

what’s ass’d with prominent A waves on CVP trace?

A

tricuspid stenosis (the Y wave is slow & lazy due to attenuated ventricular filling)

also seen w pericardial disease (eg. tamponade)- anything restricting myocardial compliance & restricting filling of the chambers

RV failure, pulmonary stenosis or pulm HTN may also cause this (anything that causes a loss of compliance beyond the TCV)

canon a waves in junctional rhythm, V-tach, 3rd degree block from RA contraction against closed TCV

in pericardial constriction the CVP would be raised & there’d be abrupt X & Y descent, while in cardiac tamponade the Y descent is prolonged

How well did you know this?
1
Not at all
2
3
4
5
Perfectly
616
Q

What 3 perioperative factors impair host defence against recurrence during cancer surgery?

A

-surgical stress response
-use of volatile anaesthetic
-opioids for analgesia

How well did you know this?
1
Not at all
2
3
4
5
Perfectly
617
Q

what was the crux of Sessler’s Lancet study (2019) re: recurrence of breast Ca following potentially curative surgery for breast Ca with regional anaesthesia-analgesia with PVB + propofol vs sevo & opioid? and their 2nd hypothesis re: regional reducing & persistent pain?

A

no sig difference in cancer recurrence (median follow-up 36 months)

no sig difference in incisional pain at 6 months

no difference in incisional pain @ 12 months

neuropathic breast pain didn’t differ with regional at 6 or 12 months

How well did you know this?
1
Not at all
2
3
4
5
Perfectly
618
Q

what does forest plot show?

A

graphically the studies included in a meta-analysis (statistical combo of numerical results from individual studies), demonstrate differences btwn studies & provide an estimate of overall result

central vertical line= line of equity. indicates no difference btwn intervention/control (risk ratio 1). To the L) indicates benefit, to the R) indicates no benefit (some the other way around)
horizontal scale @ bottom
n/N= number of events / total number of pts
%, weight of each trial in additional column to the R)
each study a blob, size proportional to study’s weight. CI extends horizontal from blob, if crosses line of equity, no difference.
diamond pooled analysis of all studies.

How well did you know this?
1
Not at all
2
3
4
5
Perfectly
619
Q

which nerves supply the ear?

A

auriculo-temporal from V3 (of trigeminal nerve)= tragus, anterior lobe & skin anterior to ear (to temporal area)
auricular nerve of vagus= inside ear
greater auricular= C2-3 = most of the helix, all of the ante helix, antetragus & earlobe
lesser occipital= C2-3 = tiny bit of upper/posterior helix, area posterior to ear

How well did you know this?
1
Not at all
2
3
4
5
Perfectly
620
Q

how manage suspected autonomic dysreflexia (eg. pt with C6 tetraplegia, remove bladder stones under GA, BP 166/88? most worrying complications? signs? risk factors?

A

emergency situation, requires rapid management
stop trigger (a stimulus below the level of the lesion, eg. bladder or bowel distension or constipation/pressure sores)-eg. relieve distension of hollow viscus
deepen anaesthesia
head up- taking advantage of orthostatic BP drop
administer 100% O2
rapid-onset, short-acting vasodilators (eg. GTN infusion 5mcg/min)- caution with longer-acting as risk hypoT when AD resolves. hypotensive effects of nitrates may be exaggerated in pts on sildefanil for erectile dysfunction.
treat arrhythmias with B-blockers, anticholinergics
treat myocardial ischaemia (eg. with GTN infusion)
invasive monitoring if doesn’t resolve quickly

the disordered SNS response can –> severe HTN, raised ICP & risk of seizures & ICH
cardiac complications= ischaemia, arrhythmias, pulm oedema

incr BP at least 20%
Brady- or tachycardia or heart block; may gen myocardial ischaemia/infarction/acute heart failure
headache, nasal congestion
flushing, sweating above the lesion (VD)
pallor/blanching/vasoconstriction below the lesion
chills
piloerection
may get ICH & seizures

risk of ADR is higher with higher level sessions (above T6)
incr frequency with more complete lesions
generally observed 1yr after injury but can occur within wks of injury

*due to risk of ADR, pts with high SCI may require deep GA or RA, even if they don’t have sensory function @ the surgical site
*ALWAYS ask pts with SCI Hx re: AD episodes- inciting events, regimens, prev surg & anaes Hx

How well did you know this?
1
Not at all
2
3
4
5
Perfectly
621
Q

How many domains on the Edmonton Frailty Scale? what are they? max score?

A

9

Overall health status (eg. number of hospitalisations in past year)
social support

functional dependence
continence
functional performance (TUG)

cognition
mood

nutritional status (weight loss)
medication use

17

How well did you know this?
1
Not at all
2
3
4
5
Perfectly
622
Q

What type of extinguisher for power board on back of anaesthesia machine?

A

CO2 (liquid & electrical fires)

How well did you know this?
1
Not at all
2
3
4
5
Perfectly
623
Q

what should occur if identify OT fire?

A

alert room & stop surgery, simultaneously briefly attempt to douse it (usually only effective in the first few seconds of the fire)
then execute plan, incl appropriate fire extinguisher, cease oxidising gases/fuel removal which can be instantly done, plan for evacuation, activate fire alarm (which activates visual & audible alarms, closes fire doors, notifies local fire dept)

How well did you know this?
1
Not at all
2
3
4
5
Perfectly
624
Q

best extinguisher for combustibles (paper, wood)?

A

water

How well did you know this?
1
Not at all
2
3
4
5
Perfectly
625
Q

best extinguisher for both combustibles & flammable liquids (eg. petrol)?

A

foam

How well did you know this?
1
Not at all
2
3
4
5
Perfectly
626
Q

extinguisher that can be used for combustibles, flammable liquids AND gases AND metals AND electrical equipment?

A

dry powder (but only up to 1000V for electrical)

How well did you know this?
1
Not at all
2
3
4
5
Perfectly
627
Q

extinguisher for flammable liquids AND ELECTRICAL EQUIPMENT?

A

CO2

How well did you know this?
1
Not at all
2
3
4
5
Perfectly
628
Q

what are the red sockets backed up by?

A

backup battery & generator (ie. they are UPS-the battery should be constantly engaged & be in immediate action, seeing supply through until the generator powers on which can take mins to half an hour)

How well did you know this?
1
Not at all
2
3
4
5
Perfectly
629
Q

what’s the benefit of blue power points?

A

they activate an alarm indicating loss of ground power

How well did you know this?
1
Not at all
2
3
4
5
Perfectly
630
Q

what’s NNT for analgesics? which analgesic may have the most favourable NNT for neuropathic pain?

A

number of pts that need to be treated to achieve @ least 50% pain relief in 1 pt cf placebo over 4-6hrs

Etiologies may be peripheral or central

Characterised by burning, p&n, tingling, numbness, electric shocks/shooting, crawling, itching, temp intolerance. allodynia or hyperalgesia.
DN4 questionnaire 7 pain discriminators & 3 exam findings; score of 4+ indicates neuropathic pain likely. S&S 83 & 90%.
Multi-D nonpharmacologic care & nonopioids (TCAs, SNRIs, gabapentanoids, topicals, transdermals)= first line. Trial 3-8 wks, change if no effect or side effects.
first line + tramadol (1st line acute neuropathic) & tapentadol= 2nd line
serotonin-specific reuptake inhibitors/anticonvulsants/NMDA antagonists & interventional therapies 3rd line
neurostimulation 4th line
low-dose opioids (no >90mg morphine equivalent units)= 5th line

TCAs NNT 3.6 for moderate pain relief, NNH 28 (9 for minor adverse effects). Risk falls, arrhythmias, orthostasis, urinary retention in elderly.
duloxetine/venlafaxine (SNRIs): NNT 6.4, NNH 11.8
gabapentin better evidence post-herpetic neuralgia & diabetic peripheral neuropathy, weak evidence other neuropathic pain conditions. NNT 6.3, NNH 25.6
pregabalin NNT 7.7, NNH 13.9
tramadol- NNT 4.4 (other ref combined NNT of 4.7)
Gabapentin (specifically postherpetic neuralgia)- NNT 5.9

membrane stabilisers eg. systemic lidocaine reasonable for acute neuropathic pain (based on evidence for use chrnoic neuropathic)
tramadol effective for neuropathicpaketamine useful for neuropathic pain after SCI
reasonable to use alpha-2 delta ligands in Mx of acute neuropathic pain; they have evidence for efficacy w chronic neuropathic pain.
bain

How well did you know this?
1
Not at all
2
3
4
5
Perfectly
631
Q

why may the splitting of S2 be increased with inspiration?

A

more blood fills the R) heart, delays systolic ejection, greater time until the pressure above the valve > the pressure in ventricle below.

How well did you know this?
1
Not at all
2
3
4
5
Perfectly
632
Q

why may the splitting of S2 be increased with inspiration?

A

more blood fills the R) heart, delays systolic ejection, greater time until the pressure above the valve > the pressure in ventricle below.

How well did you know this?
1
Not at all
2
3
4
5
Perfectly
633
Q

what’s unstable angina?

A

ischaemic symptoms suggestive of ACS but no elevation of troponin, with or without ECG changes of ischaemia (NSTEMI is the same but with trop rise)

How well did you know this?
1
Not at all
2
3
4
5
Perfectly
634
Q

what’s bullying?

A

unreasonable behaviour directed towards an individual or group that creates a risk to health & safety
it’s repeated & occurs as part of a pattern of behaviour

How well did you know this?
1
Not at all
2
3
4
5
Perfectly
635
Q

what’s harassment?

A

unwanted behaviour that offends, humiliates or intimidates a person & targets them on the basis of a characteristic set out in anti-discrimination law (eg. gender)

How well did you know this?
1
Not at all
2
3
4
5
Perfectly
636
Q

what’s discrimination?

A

treating a person with an attribute set out in legislation less fairly than a person who does not have that attribute.

How well did you know this?
1
Not at all
2
3
4
5
Perfectly
637
Q

what type of fluid should be administered to a pt with TBI?

A

warm, non-glucose-containing, isotonic crystalloid to maintain euvolaemia

How well did you know this?
1
Not at all
2
3
4
5
Perfectly
638
Q

why AVOID ALBUMIN in pts with TBI?

A

colloids may increase ICP in pts with altered BBBs as per the SAFE study (saline vs Alb fluid evaluation)- incr mortality among pts with TBI who got albumin, particularly among pts with severe TBI. Among TBI pts who received ICP monitoring, resus w alb ass’d w incr ICP.

How well did you know this?
1
Not at all
2
3
4
5
Perfectly
639
Q

risk for AAGA with cardiac surgery (NAP5)?

A

1:8,600

How well did you know this?
1
Not at all
2
3
4
5
Perfectly
640
Q

overall incidence of pt reports of AAGA in NAP5? how about with NMBD? and without?

A

1:19,000
1:8000 NMBD
1:136,000 without NMBDs

How well did you know this?
1
Not at all
2
3
4
5
Perfectly
641
Q

what does the “sail sign” suggest on CXR? other signs of this condition?

A

L) LL collapse- it’s a triangular opacity @ the posteromedial aspect of L) lung (retrocardiac sail sign)

edge of the collapsed lung may create a double cardiac contour

L) hemidiaphragm & desc aorta may be obscured but can clearly see L) heart border

How well did you know this?
1
Not at all
2
3
4
5
Perfectly
642
Q

CXR findings for R) ML collapse?

A

AP: obscures R) heart border
lateral: triangular opacity in anterior aspect of chest overlying cardiac shadow, apex @ R) hilum

How well did you know this?
1
Not at all
2
3
4
5
Perfectly
643
Q

CXR findings for R) ML collapse?

A

AP: obscures R) heart border
lateral: triangular opacity in anterior aspect of chest overlying cardiac shadow, apex @ R) hilum

How well did you know this?
1
Not at all
2
3
4
5
Perfectly
644
Q

how does serum GH usually respond to a glucose load?

A

in normal subjects, serum GH suppressed to <=1ng/mL within 2hrs of glucose load. the post-glucose values in acromegaly are generally >2ng/mL.

How well did you know this?
1
Not at all
2
3
4
5
Perfectly
645
Q

features of Apert syndrome?

A

premature closure cranial sutures–> limited growth skull (*raised ICP)
mid-face hypoplasia, choanal atresia, cleft palate, OSA (difficult BMV)
increased secretions
low-Cx fusion (reduced neck ROM)
may have fused tracheal rings (reduced ETT size)
beaked nose/small nasal passages
cardiac defects
PCKD
syndactyly (webbed/conjoined fingers/toes)

How well did you know this?
1
Not at all
2
3
4
5
Perfectly
646
Q

which drug has the highest rate of anaphylaxis (events per exposure)?

A

teicoplanin (16 per 100,000)
patent blue (14.6 per 100,000)
sux is 11 per 100,000 (*NDNMBDs have similar incidences of anaphylaxis so concern for anaphylaxis shouldn’t be a major reason for choice)
co-amoxiclav 8.8
roc 6 per 100,000
vanc 6
atrac 3 (miv also 3)
chlorhex 0.78 per 100,000

How well did you know this?
1
Not at all
2
3
4
5
Perfectly
647
Q

what does the brain trauma foundation guidelines recommend for BP guidelines?

A

SBP >=100mmHg for pts 50-69yo, >=110 for everyone else
CPP should be 60-70mmHg

How well did you know this?
1
Not at all
2
3
4
5
Perfectly
648
Q

what’s the goal for management of ICP in severe TBI?

A

<22mmHg (22mmHg= threshold for Rx)

How well did you know this?
1
Not at all
2
3
4
5
Perfectly
649
Q

what’s the Aldrete score used for?

A

suitability for discharge from stage 1 recovery
5 domains (ROCCA: respiration, O2 sats, circulation, consciousness, activity):
activity level
respiration
circulation (BP)
consciousness
O2 saturation by pulse oximetry
need score >9

further modified incl 5 additional elements useful during phase II prior to discharge home (FUPAD):
dressing
pain
ambulation
feeding
UO

Minimum PACU:
sedation 15min, GA 30min, LA could go immediately to stage 2

How well did you know this?
1
Not at all
2
3
4
5
Perfectly
650
Q

which population is most sensitive to restless legs? why may it be exacerbated perioperatively?

A

middle-aged women

worsened by immobilisation, blood loss, sleep deprivation, pregnancy, Fe deficiency, renal failure, withdrawal of their dopaminergic medications such as pramipexole (it’s dopa-responsive), neuroleptics (eg. droperidol, haloperidol, prochlorperazine, metoclopramide) & antihistamines (cyclizine, phenothiazines) & atypical antipsychotics (cloz, olanz, risp), antidepressants (TCAs, SSRIs) & tramadol, naloxone, naltrexone

For PONV in RLS, ondans

periop, schedule RLS pts am OT lists, encourage usual RLS meds, consider premed w BZD & opioids.
ketamine & ondansetron are fine.

may still get involunt LL movements after spinal or epidural but opioids added help.
GCS & calf compressors.
mobilise early, adequate opioids.
If sig blood loss, Fe supplements.

651
Q

along with dopamine agonist (eg. pramipexole), with what is RLS treated?

A

if not significantly impacting QoL, non-pharm (sleep hygiene, exercise, massage)
If sig QoL impact, gabapentin/pregabalin initial Rx. effective, low risk augmentation (aka symptom worsening).

Non-ergoline dopamine agonists pramipexole or rotigotine given @ lowest possible dose (lower risk augmentation) are first line meds; lower incidence of augmentation than ergoline agonists (cabergoline, levodopa) which aren’t used due to risk of valve, retroperitoneal pericardial & pleuropulmonary fibrosis
Fe supplementation (target >300microg/L in adults), opioid

652
Q

what drug can be given for pt on a dopamine agonist if the oral route not available? shortcomings?

A

apomorphine (subcut, can give hourly) or rotigotine patch (can give every 24hrs)

should give with anti-emetic (eg. ondansetron)

653
Q

what would be best to give a pt agitated w restless legs in PACU?

A

BZD or opioid (or pregabalin), mobilise asap if safe

RLS disorder of dopamine; some inherited AD

654
Q

most common type of perioperative stroke?

A

embolic

655
Q

what would you NOT expect to see in SIADH?

A

urine osmolality <100

656
Q

what are the first line drugs for lowering BP in pre-eclampsia?

A

labetalol, methyldopa

657
Q

2nd line drugs for lowering BP in pre eclampsia?

A

hydralazine, nifedipine, prazosin

658
Q

which anti-hypertensives are contraindicated pregnancy?

A

ACE-Is & ARBs- ass’d w foetal death & neonatal renal failure

659
Q

fastest flow can achieve through an EZI-IO?

A

100mL/min

660
Q

what does the 4th bottle in UWSD system protect against?

A

suction failure

661
Q

By how much does infrarenal X-clamp decrease renal blood flow? how much incr renal vascular resistance?

A

40%

75%

662
Q

what’s responsible for the coagulopathy resulting from intrahepatic cholestais of pregnancy?

A

vit K deficiency (malabsorption)–> deficiency of vit K dependent coag factors II, VII, IX, X

663
Q

which AF patients should not get NOACS?

A

mechanical heart valve
moderate-severe mitral stenosis

664
Q

which anticoagulants should pts with AF & end-stage kidney disease NOT receive?

A

rivaroxaban

dabigatran

665
Q

for pts with AF >=48hrs or unknown duration, for how long should warfarin (INR 2-3), dabigatran or direct Xa inhibitors be taken?

A

@ least 3/52 before cardioversion, 4/52 after

666
Q

at what CHA2DS2-Vasc score should anticoagulation commence?

A

> =2 for men
=3 for women

Can use HAS-BLED scoring (9 factor)s:
HTN
renal disease
liver disease
age >65
ETOH >=8x/week
prev stroke
prev major bleed/bleeding predisposition
uses other meds incr bleeding risk (NSAIDs, aspirin)
labile INR

> =3 is high risk >5% risk mjor bleeding

667
Q

what’s obesity hypoventilation syndrome?

A

Presence of AWAKE alveolar hypoventilation in an obese individual which can’t be attributed to other conditions associated with alveolar hypoventilation.

ie. pt presents with:
-obesity (BMI >30kg/m2)
-awake alveolar hypoventilation (PaCO2 >45mmHg)
-other causes of hypercapnia & hypoventilation have been excluded

668
Q

what are risk factors for OHS?

A

obesity (BMI >30kg/m2), particularly severe obesity (BMI >50kg/m2) where prevalence may be as high as 50%.
central obesity
reduced lung function due to obesity
reduced insp m strength
severe OSA (AHI >30/hr)

Male gender (unlike OSA) is NOT a risk factor for OHS
most present in 50s-60s

669
Q

how may pts with OHS present?

A

obese, hyper somnolent
most have severe hypoxaemic hypercapnic respiratory failure
R) heart failure from pulmonary HTN is common
may have facial plethora from polycthaemia
may have the Sx of OSA (STOPBANG)- in 90% of pts with OHS

-all have PaCO2 >45mmHg when awake on RA
-may have elevated bicarb (>27)- nonspecific (eg. dehydration, meds) & not 100% sensitive since other conditions (lactic acidosis, chronic hyperventilation) may lower the bicarb. A raised serum bicarb (>27) or base excess (>3mmol/L) in absence of another cause for metabolic alkalosis in an obese pt w PaCO2 <45mmHg may be an early indicator of OHS, should Ix.
-hypoxaemia (PaO2 <70mmHg) with normal A-a gradient (may be sl widened due to V/Q mismatch or parenchymal/vascular lung disease), severe nocturnal desaturation is common
-polycythaemia
-may have restrictive PFTs but these are nonspecific (may be useful for ruling out underlying causes of hypoventilation)
-ecg may show evidence of RVH from pulm HTN
-get an echo if evidence pulm HTN

CXR may show elevation of hemidiaphragms & heart enlargement (RVH or pericardial fat) but not gen nec preop

670
Q

what’s the obesity mortality risk score (OS-MRS)?

A

validated scoring system specific to obese pts undergoing bariatric surgery

BMI >=50kg/m2
male
systemic HTN
risk factors for PE
age >45

low risk (0-1)
intermediate 2-3
high 4-5

671
Q

intra-op plan

A

monitoring
equip
drugs
analgesia
other

672
Q

what’s the optimal position for placement of the catheter tip to provide continuous analgesia with an ESP block for post-thoracotomy analgesia?

A

T5 transverse process in the plane under erector spinae

673
Q

what’s clamped with the Pringle manoeuvre?

A

structures of porta hepatis:
hepato-duodenal ligament with the 3 structures that run within it: HA, PV, CBD
PV, R&L HA & R&L HDs

NOT the hepatic vein

674
Q

main cause of mortality in DKA in children?

A

cerebral injury (cerebral oedema)- still uncommon but has a 25% of mortality rate, is more common among chn than adults with DKA, chn with severe acidosis +/- severe dehydration, higher BUN (represents hypovolaemia), lower PaCO2, younger age & new onset are @ greatest risk

**50-80% of diabetes-related deaths in children are caused by cerebral injury

675
Q

what are symptoms of post polio syndrome

A

fatigue
cold intolerance
dysphagia
resp dysfunction
pain
muscle weakness

676
Q

anaesthetic implications of post-polio syndrome?

A

50% of the dose of NDMRs

poliovirus thought to damage the reticular activating system. the initial virus attacks anterior horn cells- surviving neurone sprout collaterals to reinnervate motor units but these are larger & fewer (ageing may –> overuse or decrease in the # of polio-affected motor neurone during ageing)

very sensitive to anaesthetic medications (sedative/hypnotics, NMBDs, opioids)

avoid sux (risk hyperkalaemia, prolonged doa, severe myalgia)
1/2 induction dose of NDNMBDs & preference short-acting agents, monitor quantitative TOFR
potential delayed awakening

consider bulbar dysfunction (laryngeal weakness or VC paralysis- FNE useful), sleep apnoea, restrictive ventilatory defect–> consider preop FNE, postop HDU

no risk worsening post polio syndrome symptoms with neuraxial (retrospective data) however it’s unknown if these pts more vulnerable to LA & if dose adjustment needed

careful considering of positioning (chronic pain risk)

care w analgesia- pts may have chronic pain but may be opioid sensitive (eg. risk OIVI if virus impacts RAS, if bulbar dysfunction)

cold intolerance- premwarm pt & OT, monitor temp, warm IVT, postop warming

plan for pre-ads, tertiary centre, post-op HDU or at least overnight monitoring

677
Q

what’s the ED95 of atracurium? how much to give?

A

0.25mg/kg, give 2x ED95 & get max blockade @ 2.5mins, DOA 40mins

atrac onset & duration of effect are predictable (hoffman elim spont deg @ physiological pH & temp), useful in critically ill pt (but hypothermia & cidosis slow metabolism of atracurium)

678
Q

ed95 cisatrac?

A

0.05mg/kg, more potent than actrac, need to give 4x ed95 (0.2mg/kg) for induction to get onset 2mins

679
Q

what intubating dose of atracurium should I give a post-polio pt?

A

half usual, so 0.25mg/kg induction

680
Q

most common cause of acromegaly?

A

somatotropin (GH-secreting) adenoma from ant pituitary- these also account for about 1/3 of hormone-secreting pituitary adenomas

681
Q

classify causes of acromegaly

A

Primary GH excess:
somatotroph-secreting pituitary adenoma
GH-cell carcinoma
familial syndromes (eg. MEN1 which is ass’d with GH-cell adenoma)

ectopic or iatrogenic GH excess:
pancreatic islet cell tumor
lymphoma
iatrogenic

GHRH excess:
central (eg. hypothalamic hamartoma)
peripheral (eg. pancreatic islet cell tumor, bronchial carcinoid, SCLC, adrenal adenoma, medullary thyroid carcinoma, pheochromocytoma)

682
Q

what’s MEN1, 2A & 2B?

A

MEN1: pituitary adenoma, parathyroid hyperplasia, pancreatic tumors

MEN2A: parathyroid hyperplasia, medullary thyroid carcinoma, pheochromocytoma

MEN2B: mucosal neuromas, marfanoid body habits, medullary thyroid Ca, pheochromocytoma

683
Q

what’s the composition of blood returned from ICS?

A

packed red cells suspended in normal saline with haematocrit approx 60%

684
Q

what 5 factors are independently associated with successful awake extubation in infants after volatile anaesthesia?

A

eye opening
conjugate gaze
facial grimace
spontaneous tidal volume >5mL/kg
purposeful movement

greater success the more factors present but minimal yield going from 2-3 vs a higher # of predictors (waiting for 5 predictors a good strategy for high-risk pts)

685
Q

is low end-tidal anaesthetic concentration associated with successful extubation?

A

no, surprisingly!

686
Q

what’s deep extubation?

A

extubation in a plane where the pts pharyngeal reflexes have been ablated (ie. surgical anaesthesia- stage III 2

687
Q

in children >=1yo, what are 2 factors associated with need for airway innervation after extubation?

A

URI
premed w midaz

in all kids, ETCo2 >55 also ass’d w post-extubation intervention

688
Q

which patients should be managed with targeted temperature management to 32-36deg C?

A

comatose pts (not following commands) who’ve achieved ROSC following OOHCA & initial shockable rhythm (strong rec, low q evidence), for @ least 24hrs

also SUGGESTED (wk rec, very low quality evidence)for non-shockable rhythms & in-hospital cardiac arrest, pregnant or haemodynamically unstable pts, pts undergoing coronary catheterisation or thrombolysis; only absolute CI is an AHD preventing aggressive care

689
Q

what are early & late findings of acute compartment syndrome?

A

pain out of proportion (early & common finding)- pain is the most important finding
pain with passive stretch of muscles of affected compartment (early finding)

persistent deep ache or burning
tense compartment with “wood-like” feeling

paraesthesias (within 30-120mins of ACS, suggests ischaemic nerve dysfunction)
muscle weakness (2-4hrs)
paralysis (later finding)
pallor uncommon

690
Q

what are the muscle compartments & their contents?

A

thigh:
anterior= quads, sartorius with the femoral & saphenous
posterior= hamstrings with sciatic
medial= adductors & gracilis w obturator

ant leg= tib ant/EHL/EDL/fibularis tertius with DPN
lat leg= fib long & brev with SPN
deep posterior leg= popliteus, FHL, FDL, tib post with tibial nerve
superficial post= gastrocs, soleus, plantaris (just mm)

anterior arm: bb/br/cb with ulnar & median nn
post arm: triceps & radial n
deep & superficial volar forearm (superficial FCR, PL, FCU, PT, FDS) with deep (FDP, FPL, PQ) median & ulnar nerve
dorsal forearm= BR, ECRL, ECRB, ECU, ED, EDM, APL, EPB, EPL, supinator & radial n

691
Q

what are the muscle compartments & their contents?

A

thigh:
anterior= quads, sartorius with the femoral & saphenous
posterior= hamstrings with sciatic
medial= adductors & gracilis w obturator

ant leg= tib ant/EHL/EDL/PT with DPN
lat leg= fib long & brev with SPN
deep posterior leg= popliteus, FHL, FDL, tib post with tibial nerve
superficial post= gastrocs, solens, plantaris (just mm)

anterior arm: bb/br/cb with ulnar & median nn
post arm: triceps & radial n
deep & superficial volar forearm (superficial FCR, PL, FCU, PT, FDS) with deep (FDP, FPL, PQ) median & ulnar nerve
dorsal forearm= BR, ECRL, ECRB, ECU, ED, EDM, APL, EPB, EPL, supinator & radial n

692
Q

A man presents with a fractured tibia. He has increasing pain in his leg, loss of sensation on the plantar surface of his foot and weakness of toe flexion. This is most consistent with a compartment syndrome of the leg in the

A

deep posterior compartment

693
Q

what lab test of coagulation does vit K def in intrahepatic cholestasis of pregnancy cause?

A

prolonged PT

694
Q

best position to evaluate gastric contents with US?

A

R) lat decubitus (R)-side down, better for smaller volumes)

695
Q

what’s anaemia for females? males?

A

<120g/L
<130g/L

696
Q

if a pt is not anaemic but ferritin <100mcg/L, what could consider?

A

could consider Fe therapy if the estimated Hb drop with surgery is >=30g/L

important to determine the cause & need for GI Ix if ferritin suggests Fe deficiency (<30microg/L)

697
Q

by how much do Hb & Hct increase after a unit of blood?

A

10g/L

3%

698
Q

if a pt is anaemic (Hb <130g/L males, <120g/L females) & ferritin <30mcg/L, action?

A

Fe therapy
Ix cause (d/w gastroenterologist GI Ix & their timing in relation to surgery)

699
Q

if anaemic & ferritin 30-100microg/L, action?

A

look @ CRP. If it’s raised, commence Fe consider the clinical context- eg. haematology advice, renal advice if CKD, consider Ix in liaison with gastroenterologist (*some Dx Fe deficiency if ferritin 30-100, Tsat<20% & CRP>5)

If CRP normal (or if the pt is anaemic with ferritin >100microg/L), it may be anaemia of chronic disease, inflammation or other cause of anaemia:
consider clinical context, renal function, MCV/MCH & blood film, B12/folate, retics, liver & thyroid function, haematology advice or renal (if CKD)

700
Q

if a penicillin skin test is positive, what proportion will react to cephalosporins? and carbapenems? monobactam?

A

2-3%
particularly first-get cephalosporins, negligible with 2nd

Cephalosporins most likely to cross-react w penicillins incl:
-cephalexin (1st gen, high risk)
-cefalcor (2nd gen) (both cefalexin & cefaclor have similar side chains to amoxicillin or ampicillin have similar side chains to amox/amp)
-ceftriaxone (3rd gen) (Ceftriaxone has similar side chains to cefotaxime, cefepime, cefuroxime)

safer (lack B lactam chain): cefazolin (1st gen), cefuroxime (2nd gen, but sim side chains to ceftriaxone), cefixime*(cefixime 3rd gen, least likely)

ceftazadime is a 3rd gen with same side chain as azotrenan & similar to cefuroxime
<1%

Cefazolin has NO common side-chains w other beta lactams so often can be tolerated if peniciallin allergy or cephalosporin

NO cross-reactivity with aztreonam (monobactam)

701
Q

is antibiotic prophylaxis indicated for insertion of an intra-uterine device?

A

no

702
Q

what fissures do the R) & left lungs have?

A

R) has horizontal & oblique
L) just oblique

703
Q

risk of AAGA under volatile withOUT NMB? vs with? and TIVA no NMB & with?

A

1:193,000 (for all GA sans NMB, 1:136,000)

704
Q

risk of AAGA under volatile withOUT NMB?

A

1:193,000 (for all GA sans NMB, 1:136,000)

705
Q

does methylene blue inhibit MAO?

A

yes- highly selective for MAO-A, may be ass’d w severe serotonin toxicity

706
Q

at what dose may methylene blue cause toxicity (paradoxical methaemoglobinaemia)?

A

5mg/kg
>7mg/kg- may cause haemolysis even if don’t have G6PD, may get potentially fatal serotonin tox >5mg/kg

707
Q

what is metHb? adverse effects?

A

ferric Fe (3+), which has stronger affinity for O2, L) shift of OHDC & decreases O2 delivery to tissues- functional anaemia

708
Q

what is metHb? adverse effects?

A

ferric Fe (3+), which has stronger affinity for O2, L) shift of OHDC & decreases O2 delivery to tissues- functional anaemia

709
Q

A woman is having a potentially curative primary breast cancer resection. Compared with a sevoflurane and opioid technique, using a regional anaesthesia-analgesia technique with paravertebral block and a propofol infusion will result inwhat?

A

Less nausea in PACU and day 1 but similar btwn groups day 2
a meta-analysis from 2018 suggests that PVB reduces the risk of CPSP at 12 months, a
subsequent RCT has confirmed these findings of reduced chronic postmastectomy pain with PVB
use at 3 mth (OR 0.51; 95%CI 0.28 to 0.94) and at 6 mth (OR 0.48; 95%CI 0.25 to 0.94) (Qian 2019
Level II, n=184, JS 5).Pain Book 2020
BUT this question about Sessler study-
no sig difference in cancer recurrence (median follow-up 36 months)
no sig difference in incisional pain at 6 months
no difference in incisional pain @ 12 months
neuropathic breast pain didn’t differ with regional at 6 or 12 months

710
Q

normal blood volume adult & child? infant? neonate? term pregnancy?

A

70mL/kg ideal body weight (60mL/kg if >65yo)
80mL/kg
90mL/kg
100mL/kg

711
Q

management of narrow complex tachycardia?

A

if definitely identified as AV=node independent (eg. sinus tachycardia), treat cause (anaemia, hyperthyroidism, anxiety, pain, hypovolaemia,, depth of anaesthesia, infection, PE, coronary ischaemia)
if it’s atrial tachycardia, synchronised cardioversion if unstable (50-100J). Consider Mg++, K+, amiodarone
if it’s AF & unstable, synchronised cardioversion (120-200J). if it’s stable, treat the cause (eg. electrolytes, infection, anaemia), rate control (B blockers, dig (if HF)), cardiovert if <48hrs (amiodarone), consider anti-coagulation
A flutter: synch CV (25-50J), drugs= digoxin, B blockers, flecainide, amiodarone

If AV-node dependent- regular tacchycardia-
vagal manoeuvres (CS massage, valsalva, diving reflex)
NO verapamil or adenosine if WPW (could use flecainide for them)
junctional tachy= amiodarone or flecainide

persistent= B blockers, diltiazem, verapamil

712
Q

energy for synch CV for different tachyarrhythmias?

A

narrow complex regular 50-100J
narrow irregular 120-200J
wide regular: 100J
wide irregular: defibrillator (200J)

713
Q

minimum time to wait after stroke before proceeding with surgery is?
MACE risk @ different time intervals after ischaemic stroke?

A

9 months

MACE risk post ischaemic stroke?
<3 months: 15%
3-6/12: 6%
6-12/12: 3%
>12/12: 2.5%

risk levels off after 9 months

714
Q

is there clinical value in doses above 1000mg IV elemental iron?

A

no

715
Q

what’s the required info for calculating parenteral Fe dosage?

A

body weight (kg)- use ideal BW if obese
Hb
concentration of elemental Fe in the product

716
Q

benefit of ferric carboxymaltose?

A

can give as a single dose over 15mins (start slowly)

717
Q

which electrolyte may drop with ferric carboxymaltose?

A

phosphate

718
Q

what’s the preferred maintenance fluid for paediatrics nbm?

A

0.9% Nsaline with glucose 5%

could consider addition of potassium if normal baseline electrolytes & renal function but has to be in premixed bags

719
Q

how calculate child’s fluid deficit?

A

[premorbid wt (kg) - current wt (kg)] x 1000

720
Q

what fluid bolus use for shocked child?

A

10-20mL/kg 0.9% NaCl as fast as possible

721
Q

how to rehydrate paeds?

A

if moderate, enteral (oral or NG) preferred
may require IV if severe or if can’t tolerate enteral

first estimate degree of dehydration & calculate fluid deficit:
mL= (premorbid-current wt kg) x 1000
if wt unknown, % dehydration x wt (kg) x 10

if the fluid deficit is <=5%, replace in the first 24hrs
if it’s >5%, replace more slowly- 5% in first 24hrs & remainder over next 24hrs if still needed.

replace ongoing losses as measured based on previous 1-4hours; generally replace GI losses with NaCl 0.9% + KCl 20mmol/L

total fluid= rehydration + maintenance + ongoing losses

722
Q

what are risk factors for myotoxicity from local anaesthetics?

A

bupivacaine
higher LA []
prolonged exposure to LA

723
Q

what do currents passing directly through the heart need to be before causing micro shock?

A

50-100 microamps (0.05-0.1 amps), so max leakage current in equipment to avoid micro shock is 10microamps

724
Q

Non-anaesthetist practitioners wishing to provide procedural sedation should have training in sedation and/or anaesthesia for a minimum of how long?

A

3/12 FTE

725
Q

which oral diabetic agent agent has the highest risk of periop hypoglycaemia in fasted pts?

A

**sulfonylureas (-azides)

pioglitazone= thiazolinedione, improves glycaemic control but uncertain mort & complications (hence is a 3rd line agent). Ass’d. w wt gain & Na/fluid retention; unsuitable in mod/severe CCF. Improves insulin sensitivity. reduce circulating fatty acid concentrations.

Sitagliptin= DDP-4 inhibitor, DDP-4 metabolises incretins (which stimulate insulin;l GLP-1 & GIP) so you get more endogenous insulin. can cause hypoglycaemia but not common & incr risk if combine w sulphonuylurea

Metformin: acts by inhibition of mitochondrial 1 complex, decr hepatic glucose production, intestinal glucose absn & incr peripheral glucose uptake & utilisation. Doesn’t impact insulin secretion so no hyop.

Acarbose limits the breakdown of intestinal oligosaccharides (inhibits intestinal alpha glucosidase); therefore slows glucose uptake & lowers post -prandial glucose [], may promote weight loss (promotes GLP-1). Flatulence, diarrhoea, abode pain but not hypos

Sulfonylrea cause insulin secretion (stimulate pancreatic beta cells), can cause hypoglycaemia
meglitinides similar mechanism but weaker binding & more rapid dissociation

GLP-1 agonists incretin mimetics, promote endogenous insulin release but in a glucose-dependent manner so don’t tend to cause fasting hypoglycaemia. weight loss. improves LV function.

726
Q

what’s the treatment & infection control requirements for pts with carbapenemase-producing enterobacteriacae (CPE)?

A

ceftazidime-avibactam

It’s a GN bacteria- with carbapenemases

Place them on contact precautions, single room, own toilet, for the duration of hospital stay & maintained indefinitely for future hospitalisations, given prolonged colonisation & limited Rx options. also standard measures (hand hygiene, minimise use of invasive devices, antimicrobial stewardship).

screening high-risk pts to detect rectal colonisation.

don’t need droplet precautions.

727
Q

blocking sciatic nerve spares

A

medial calf/arch of foot
knee jerk
quadriceps

728
Q

regarding healthcare research, what do the SQUIRE guidelines describe?

A

standards for quality improvement reporting excellence; FOR REPORTING COMPLETED WORK, they are used as guideline for authors reporting on systematic, data-driven efforts to improve the quality, safety & value of healthcare.
aim to improve completeness & transparency of reporting quality improvement work.

729
Q

after anaesthesia, pts should

A

be encouraged to breastfeed as normal when feeling alert & well, exercise caution (observe infant for signs of abnormal drowsiness or resp depression) if sedative drugs were used & baby up to 6 wks old, esp if the mother showing signs of sedation.
codeine should NOT be used in breastfeeding women.

730
Q

How to manage insulin perioperatively?

A

highly individual- pts with T1DM or insulin-treated T2DM are at higher risk of ketoacidosis if basal insulin not supplied yet risk hypoglycaemia if insulin excess.

endocrine advice

Basal only:
generally, if once-daily & safe morning glucose levels don’t need to adjust
If the pt’s insulin has been adjusted to be in normal or low-normal or if BGL runs low, reduce dose by 20%.
If the once-daily insulin is in the evening, typically reduce it by 20%

if it’s twice-daily, may be able to continue as long as basal dose correctly calculated. reduce both doses by 10-25% if concern about hypoglycaemia.

basal and prandial:
generally omit prandial insulin from commencement of fasting
if basal insulin in the morning, pt to take 1/5-2/3 their TOTAL morning insulin as basal to prevent ketosis during the procedure
if basal in evening, reduce basal the night before by 20% (unless mane BGLs high)
if basal 2x/day, reduce evening before by 20% (unless mane BGLs high in which case give full dose) & give 1/2-2/3 TOTAL morning dose as basal insulin on the morning of surgery

premixed, generally reduce dose night before by 20% & mane by 50% but if BGL generally <12 mane, omit morning

if continuous pump & it’s OK for the procedure, can continue their basal insulin rate. If pump has to be discontinued, administer basal (according to programmed insulin settings) 2-3hrs prior to pump discontinuation.

731
Q

immediate management of a neonate with suspected duct-dependent congenital heart disease?

A

prostaglandins infusion (start low dose at 5ng/kg/min if think the duct still open, in neonatal collapse if think the duct has closed, start higher 20ng/kg/min to a max 100ng/kg/min), target SpO2 75-85%- must maintain the PDA with prostaglandin infusion, to provide systemic perfusion (adverse effects incl apnoea, hyperthermia, hypotension)

732
Q

how can congenital adrenal hyperplasia present & how to mange?

A

can present as shock with salt loss crisis (requires IV saline, hydrocortisone, correction of electrolytes)

733
Q

what to always consider with paediatric presentations?

A

non-accidental injury- low threshold for referral to child protection professionals

734
Q

what are the most common causes of neonatal collapse?

A

congenital heart disease
sepsis
metabolic disorders

735
Q

how may a neonate with duct-dependent CHD present?

A

cyanosis
hypoxia despite O2 therapy
tachypnoea without lung pathology
pulmonary oedema (late)

hepatomegaly
cardiomegaly
absent femoral pulses
pathological murmur
lactic acidosis unresponsive to initial resuscitation

736
Q

what criteria point towards sepsis in the newborn? risk factors for neonatal sepsis?

A

temp (>38.5 or <36)
HR >180 or <100bpm
RR >50
leucs >34 x10^9/L

hypotension NOT included (it’s a late finding)

preterm birth
VLBW
chorioamnionitis
PROM >24hrs before delivery

737
Q

what may point to a metabolic disorder?

A

metabolic acidosis, lactic acidosis, hyperammonaemia, hypoglycaemia

738
Q

how may abusive head injury present (due to damage of bridging veins)?

A

ICH (most commonly subdural), retinal haemorrhage, encephalopathy

739
Q

what should I be prepared for with induction in a neonate?

A

bradycardia- prepare w atropine 20microg/kg

740
Q

what []O2 would be used for a pt with suspected PDA?

A

FiO2 30-40%, as higher may cause excessive flow through lungs which incr PVR

741
Q

indications & considerations for newborn intubation?

A

**impending resp arrest
if running high-dose prostaglandin infusion for duct-dependent circulation, @ risk apnoea
neuroprotection- ICH or cerebral oedema
loss of airway reflexes due to depressed consciousness

anticipate difficult airway
anticipate bradycardia- atropine 20microg/kg prepared

pre-O2 with FiO2 1.0 unless PDA-dependent lesion- in which case FiO2 30-40% (high []–> excessive flow through lungs which may incr PVR)

pre-induction resuscitation with IVT boluses (10mL/kg) & consider inotropes, prepare adrenaline 1microg/kg as bolus, consider Adr infusion if pt required >40mL/kg fluid. if shock unresponsive to fluids & inotropes, 6-hourly hydrocortisone 2.5mg/kg.
IV access asap- IO if difficult IV. if pt <1 wk old, umbilical vein may be patent for venous access’=
if need central access, avoid IJ if suspect single ventricle circn (pt may get a Fontan), avoid femoral if suspect transposition great vessels (atrial septostomy is usually via femoral vein.
MAP goal for term neonate is 40mmHg. in premature, MAP in mmHg is to the corrected GA.
circulation time may be prolonged. induce with ketamine, fentanyl, NMBDs
desaturation likely so consider gentle vent prior to intubation

generally robertshaw (for neonatal/infant, gentle shallow blade curve to lift epiglottis indirectly) or miller (straight blade with shallow curved tip to lift epiglottis directly)

tube size <28 wks is 2.5
28-34wks is 3
>34 wks is 3.5
term is 3.5-4
depth- not accurate to do diam x3 for size ETT<3 (only if size >=3)

depth size 2.5 is 5.5cm
6cm from 25 wks

size 3 is 6.5cm

7.5cm if size 3.5

9cm if size 4

target SpO2 >95% if sepsis or raised ICP, target pre-ductal (R) hand) & post-ductal (either foot) 75-85% in PDA-dependent CHD (post-ductal lower due to mixing of pulm blood through PDA)

NGT may be required

Hb 100 until 7 days of age, 75 after 8 days of age.

broad-spectrum ABx
keep warm
manage seizures
correct electrolytes & glucose

COMMUNICATION WITH PARENTS- THEY MAY FIND WRITTEN RESULTS HELPFUL

742
Q

key principles of management for pt with duct-dependent CHD?

A
  1. keep PDA open for systemic perfusion
  2. balance the flow btwn systemic & portal circulations, incr PVR or decr SVR so to avoiding a large proportion of CO going to the pulmonary circulation- important for circulations in parallel (incr prom flow means decr systemic flow which compromises myocardial perfusion & –> tissue hypoxia); can maintain “balanced circulation” by avoiding high FiO2, aiming SpO2 80%, PaCO2 37-45mmHg.applying modest (4-6cmH2O) PEEP
743
Q

risk of prolonged (>5/7) ABx if low possibility of sepsis?

A

necrotising enterocolitis, incr incidence late-onset sepsis, incr mortality

744
Q

what 4 tests look at if concerned about a metabolic disorder in neonate?

A

abg
ammonia
glucose
lactate

if abnormal, support normal physiology, consult an expert in paediatric metabolic medicine

745
Q

characteristics of congenital adrenal hyperplasia? management?

A

one of the more common metabolic causes of SUPC

can result in insufficient mineralocorticoid & corticosteroid (important for maintaining salt/water balance & circulating volume)
ass’d with:
hypoglycaemia
hyponatremia
hyperkalaemia

poor cardiac function with decreased responsiveness to catecholamines–> shock

fluid boluses of saline 20mL/kg reAx after each bolus
correct hypoglycaemia with dextrose
early administration of hydrocortisone aids correction of electrolyte abnormalities
aggressively treat hyperkalaemia (incl Ca gluconate)
consult paediatric endocrinologist

746
Q

what does hyperammonaemia, esp w resp alkalosis, suggest? and with resp acidosis?
how hyperammonaemia present in neonate?
how to manage?

A

urea cycle defect
with resp acidosis, more likely secondary & suggestive of organic acid disorder

presents with lethargy, poor feeding, tachypnoea, hepatomegaly, seizures, coma- Rx within 24hrs essential to limit neurological damage

stop all protein intake, administer fluids with dextrose & intralipid, use ammonia scavenger meds (sodium benzoate or arginine)
consider haemodialysis esp if ammonia [] excessive

747
Q

how hypoglycaemia present in neonate? risks? Rx?

A

reduced tone & alertness, convulsions, sweating, arrhythmias

risk neurological damage

absolute level for treatment depends on post-conceptional age, wt & clinical condition- generally <2.0mmol/L in term neonate needs Rx

10% dextrose 2-3mL/kg initial bolus. if require high amounts of dextrose, may need CVC (peripheral vascular irritation, fluid overload)

748
Q

risk factors for neonatal sepsis?

A

preterm birth, VLBW, chorioamnionitis, PROM >24hrs before delivery

749
Q

what’s the 3-point cessation strategy developed by the smoking cessation taskforce of the ASA?

A

Ask about tobacco use- even if known- emphasises that tobacco a significant issue
Advise quitting- outline specific periop risks
Refer

750
Q

what ARE effective (risk ratio cf placebo) pharmacotherapies for smoking cessation perioperatively? and behavioural?

A

nicotine (1.55 all types, inhaler 1.9, patch 1.64)- 1st line, combo better, patch start high then wean
varenicline (2.24)- champix, also 1st line; start 2/52 before smoking cessation to occupy receptors, gradual up-titration
buproprion (1.64)- zyban- atypical antidepressant, start 1-2wks before smoking cessation
clonidine & nortriptyline also help
individual (1.57)
group (1.88)

Pharmacotherapy + counselling incr quit rates 50-70% periop

751
Q

what’s the success rate of each quitting attempt?

A

4-7%
success tends to be grater in pts having surgery

752
Q

why may smokers have higher requirements for vet & roc?

A

smoking induces liver CYP450 enzymes.

753
Q

how does smoking impact opioid requirements?

A

smokers have higher opioid requirements & experience more postop pain.

754
Q

what’s the atmospheric lifetime of N2O?

A

110yrs

755
Q

target SBP for cardiac arrest after cardiac surgery?

A

60mmHg

756
Q

What’s major trauma as per injury severity score? what’s the incidence of traumatic coaguloapthy with that ISS?

A

> 15

67%

757
Q

which is the peripheral nerve most commonly injured in TKR? and which is the most common neurologic complication?

A

infrapatellar branch of saphenous- poorly localised ant knee pain sl more medially, possibly referred to medial calf

peroneal nerve palsy

758
Q

how does peroneal nerve palsy manifest? in which procedures is it a risk?

A

foot drop, weakness DF & eversion
paraesthesia/numbness over dorm of foot & lateral shin

TKR esp if significant valves deformity or flexion contracture

damage to lumbosacral trunk (btwn L4&5) w descent of foetal head. May occur during forceps delivery. esp short stature & large baby. less commonly CPN damage.

759
Q

why are myopics at higher risk of globe perforation with peribulbar? how to limit risk?

A

incr risk of staphyloma (15% if axial length 27-29mm, 60% if axial length >31mm)
30x higher risk of needle damage to globe if inferotemporal peribulbar

MEDIAL CANTHUS approach or sub-Tenon’s are safe

760
Q

what classes of drugs to avoid with methylene blue?

A

serotonergic medications such as SSRIs, since methylene blue is a potent MAO inhibitor

761
Q

why is UFH the preferred agent for bypass? steps for a pt with Dx HIT?

A

measurability, reversibility, low cost, familiarity

ideally delay surgery until anti-PF4/heparin antibodies aren’t present OR can treat with pre-op plasma exchange & then use intraop heparin just for the procedure.
could give IVIG prior to heparin to reduce activity of heparin/P4 antibodies
use an alternative- eg. bivalirudin, as it has relatively short life. drawback= unable to reverse so bleeding risk (although short 25min half life if normal renal function. can be used in hepatic insufficiency. expensive. Protocols using bivalirudin during cardiac surgery with & without CPB have been established.

762
Q

where & how deep should airway exchange catheter be inserted?

A

must remain above carina
don’t insert beyond 25cm in adult (DAS= not beyond 26cm @ lips)

Mean lip-carina distance 25cm males, 21cm females

763
Q

most cook AECs are 83cm long (11, 14, 19Fr, for exchange of ETT size 4, 5 or 7 & larger).. what’s the other size?

A

also 45cm one, 8fr for ETT size 3 or larger

764
Q

what’s the time taken for plasma levels to decrease by half following removal of nor span patch?
how long does it take to achieve steady state after application of the patch?

A

12 hrs

3 days (for a 7-day patch, 24hrs for a 3 day patch)

765
Q

what causes metformin-related lactic acidosis?

A

inhibition of hepatic gluconeogenesis (metformin inhibits mitochondrial transport chain complex 1, decreasing hepatic gluconeogenesis from lactate, pyruvate & alanine so there’s excess lactate & substrate for lactate production)
it tends to occur if metformin overdose or if pts have kidney or hepatic insufficiency or other disease causing AKI & impaired metformin elimination.

766
Q

how does metformin work?

A

decreases insulin resistance
decreases hepatic glucose output (inhibits gluconeogenesis & glycogenolysis), enhances peripheral glucose uptake

767
Q

what are absolute and relative contraindications to regional anaesthesia for the eye?

A

absolute: pt refusal, LA allergy, infection/marked orbital inflammation
relative: myopic, unable to lie flat, children, communication difficulties, bleeding diathesis, previous scleral buckling or space-occupying lesions within the orbit

768
Q

innervation of knee joint

A

Femoral L2-4 (from saphenous & muscular branches)
Common peroneal (L4-S2) & tibial (L4-S3)
obturator L2-4 (posterior knee)

769
Q

Severe (grade III) anaphylaxis dose

A

50microg
range 50-100microg, 200microg if no response.
if >3 boluses given, start Adr infusion, peripherally 3microg/min=3mL/hr

770
Q

How’s sugammadex cleared?

A

excreted renally, unchanged; not recommended if CrCl <30mL/min
sugammadex CAN be removed via high-flux haemodialysis

771
Q

Factors likely to improve cerebral O2 sats

A

Intervene if desat >20%
optimise venous drainage
increase DO2:
-Augment MAP w vasopressor (MAP 110% normal to enhance collateral flow through CoW)
-FiO2 100%
-optimise CO (HR x SV)
Surgical placement of carotid shunt
Strategies to limit carotid vasospasm, selective perfusion coronaries
-Optimise Hb

reduce O2 consumption: manage any temp/seizures, adequate depth

772
Q

Normal bowel calibre
What to do if postop bowel distension, no mechanical obstruction?

A

small bowel <3cm
large bowel <6cm
appendix <6mm
caecum <9cm

For ileus:
-exclude mechanical obstruction or other secondary causes (CT abdo w oral contrast S&S 90-100% for distinguishing ileus from complete SBO & may identify secondary causes of ileus).
-correct reversible causes (prevention better than cure, ie. ERAS measures incl mid-Tx epidural or TAP block, minimally invasive vs open OT, opioid antagonist, multimodal analgesia, minimal length incisions, minimal bowel handling, limiting fluids. Routine NGT placement & use of COX2s are HARMFUL (incr risk anastamotic leak)- can use nonselective)
-check electrolytes (incl K+ & Mg++)
-multimodal analgesia (minimises opioid, non-selective NSAIDs & paracetamol)
-IVT- normovolaemia
-dietary restriction- bowel rest (sips of clear fluids, liquid diet if BS)
-selective NGT if N&V
-serial monitoring. &abdo exams (additional imaging if doesn’t improve. in48-72hrs)

discontinue meds that ay decrease colonic motility, avoid laxitives
could have rectal tube
encourage mobility, prone w hips elevated, alternate L) & R) each hour
could use colonoscopic decompression
can use parasympathomimetic drugs to enhance gut motility (eg. neostigmine, erythromycin)

773
Q

Large doses of sugammadex can potentially lead to

A

BRADYCARDIA
anaphylaxis (dose-dependent)
delay reestablishment of neuromuscular block w steroidal NMBAs
other adverse effects:
cardiac arrhythmias (marked brady)

774
Q

The initial management for a seizure during an awake craniotomy is

A

irrigating the brain with ice-cold saline

occasionally: BZD, anti-epileptics or re-sedation w airway control are required

775
Q

The rate of drainage of cerebrospinal fluid via a lumbar drain is NOT influenced by the

A

LEVEL OF INSERTION

Is influenced by:
height of drainage bag
height of bed
changing patient position supine to sitting
larger bore catheter
rate of CSF production
length of the catheter

776
Q

The minimum recommended duration between the stroke and surgery is

A

9 months

777
Q

A 6-year-old patient (140 cm, 24 kg, BSA 0.97m2) is on hydrocortisone 15 mg/day. Perioperative glucocorticoid supplementation is

A

Supplementation required if >10mg/m2 hydrocort equivalent FOR >=1 MONTH

Major procedures:
2mg/kg IV hydrocortisone @ induction followed by infusion:
0-10kg: 25mg/24hrs
11-20: 50mg/24hrs
>20kg:
prepubertal 100mg/24hrs
pubertal 150mg/24hrs

postop: 2mg/kg IV or IM 6-hourly or wt-based infusion (0-10kg 25mg/24hrs, 11-20 50mg/24hrs, >20kg prepubertal 100mg/24hrs, pubertal 150mg/24hrs.
Once oral intake, double usual dose 48hrs postop then reduce to normal over 1 week.

Minor: 2mg/kg @ induction, once enteral feeding, postop double oral dose for 24hrs.

So this kid, major or minor surgery 48mg @ induction

778
Q

Two half lives rivaroxaban

A

22-26hrs (say 24hrs)
for elderly, riva half life 11-13hrs
young 5-9hrs

DOACS half lives
Dab 12-14
Riva 5-9 young, 11-13 elderly
Api 8-15

779
Q

Pulse pressure variation is defined as

A

PPmax-PPmin/PPmean over a respiratory cycle or other period of time (often averaged over 3+ breaths)
It reflects where the heart is on the frank-starling curve, denoting fluid responsiveness. PPV 13-15% ass’d w volume responsiveness.
Limited to mech vent pts w >=8mL/kg TV, sinus rhythm, no spont trigger.

Stroke volume variation is (SVmax/SVmin)/SVmean. SVV >10% correlates with fluid responsiveness.

780
Q

Proportion of blood donors ANZ who are CMV seropositive

A

76% (higher in females & older age groups)

“CMV safe”= leucodepletion or antibody testing. of donor blood, provides risk reduction for transfusion-transmitted infection. leucodepletion residual risk CMV 1 in 13,575,000.
Should offer to all pregnant women, regardless of CMV status.

CMV= a human herpes virus that can. be transmitted horizontally (bodily fluids) or vertically (postnatal infection through breastfeeding or congenital infection. if primary maternal infection in pregnancy).
In most, a mild non-specific or asymptomatic illness & develop an immune response, seropositive 6-8wks after contracting virus. remain potentially infectious for life (dormant infection) esp in mononuclear white cells & their precursors.

781
Q

In critically ill patients undergoing mechanical ventilation, energy dense enteral nutrition (1.5 kcal/mL/kg)compared to routine (1 kcal/mL/kg) enteral feeding provides

A

no difference in 90 day all cause mortality
incr vomiting (NNT for vomit=31)
required more prokinetics & more insulin
it provides higher residual gastric volumes

TARGET trial: energy-dense vs routine enteral nutrition in critically ill, mechanically ventilated pts
Multi-centre, double-blind RCT Aust/NZ
1mL/kg/hr of 1.5kcal/mL based on ideal body weight- approx 1900kcal/day
1mL/kg/hr 1kcal/mL approx 1300kcal/day
energy dense= higher fat & carbo, similar protein
Enteral nutrition is preferable. to parenteral (reduced gastric erosion, bacterial translocation & infectious complications

782
Q

A 56-year-old patient presents with exertional syncope. The most likely diagnosis is

A

Hypertrophic cardiomyopathy: syncope in up to 25% of pts w HCM, may be due to dynamic LVOTO or arrhythmias, exertion ischaemia. Autosomal, most common form. of inherited cardiac malformation (1:500 adults)
Syncope: transient loss of consciousness due to a period of inadequate cerebral nutrient flow (usually due to drop SBP & inadequate cerebral perfusion). Spontaneous & self-limited.
AS: a presenting symptom in 10% of pts w symptomatic severe AS; rare presentation unless valve critically stenotic. Exertion. High mortality if untreated so AVR generally indicated.
Myocardial ischaemia (—> bradycardia & arrhythmias)= rare cause of syncope (1%)

783
Q

A patient presents with sepsis-induced hypoperfusion or septic shock. The minimum suggested volume of intravenous crystalloid to be administered over the first three hours as outlined in the Surviving Sepsis Guideline is

A

-30mL/kg IV crystalloid within the first 3hrs of resus

Other recommendations:
-Rx & resus immediately
-use crystalloids as first-line fluid, balanced vs N saline
-if large vols, use alb
-dynamic measures to guide fluid resus
-guide resus to decrease lactate
-cap refill to guide resus as an adjunct

-Target MAP >=65mmHg

-Use norepinephrine as first-line vasopressor
-If on Norepinephrine but inadequate MAP, add vasopressin rather than escalating norepinephrine.
-If inadequate MAP despite those 2, add epinephrine. (*NE >=1microg/kg/min avoided- “decatecholaminisation”, high doses NE can compromise host immune system, promote bacterial growth, may induce myocardial injury & oxidative stress, incr mortality, peripheral ischaemia. Commence vasopressin when NAdr requirement 20microg/min.
Vasopressin 20units/mL vial. 20units in 20mL glucose 5% (vaso is compatible with NaCl 0.9%), final concentration 1unit/mL. Start at 0.6u/hr. wean no >0.6u/hr every 15 mins. usual range 0.6-2.4U/hr, high doses risk ischaemic side effects (peripheral, mesenteric). dedicated central line). Adr 1000mcg/50mL, 5mL/hr if urgent, if extra hands: Epi risks tachy, incr myocardial O2 consumption, hyperglycaemia, lactic acidosis, peripheral & splanchnic ischaemia. 0.5-2mcg/kg/min (3mg/50mL is 60mcg/mL)
-If septic shock & cardiac dysfunction w hypoperfusion despite adequate volume & MAP, add dob to Nepi or use Epi alone.
-Dobutamine 250mg (20mL) made up to a volume of 42ml with 5% glucose (can also use hartmann’s or NaCl). final [] 6mg/mL, start @ 2.5mcg/kg/min.
-Use art line over NIBP asap
-start vasopressors peripherally @ first, to avoid delay in CVC

-IVABx within 1hr for sepsis. w septic shock, seek sources & refine from empiric
-If possible sepsis but no shock, time-limited Ix, ABx within 3hrs
-if high risk MRSA, use ABx w MRSA cover empirically
-if high risk MDR organisms, 2 ABx w GN cover empirically (until susceptibilities known)
-daily assessment & de-escalation as indicated

-If septic shock & ongoing vasopressor requirement, give IV corticosteroids
-restrictive over liberal transfusion
-use stress ulcer prophylaxis if risk factors for GI bleeding, use pharmacologic VTE prophylaxis (LMWH), only use mech if no pharmacological
-If septic shock & AKI, use RRT if definitive indication
-target BGL <10mmol/L w insulin infusion
-use early enteral nutrition. if poss (within 72hrs)
-don’t use: sodibic for hypoperfusion-induced lactic acidaemia but DO use it for severe metabolic acidemia with AKI, IV vit C, IVIG, polymixin B hemoperfusion

-If sepsis-induced hypox resp failure, HFNO over NIV
-ARDS: low tidal vol (6mL/kg) over high TV (>10mL/kg), severe ARDS upper limit goal of plateau pressure 30cmH2O, mod-severe ARDS high PEEP over low PEEP
If sepsis-induced severe ARDS, do use traditional recruitment maneuvers
prone ventilation >12hrs/day if mod-severe ARDS sepsis-induced, intermittent vs continuous invusion NMBAs
-VV ecmo if mech vent fails if infrastructure & experience in place

-Discuss goals of care

-Institutional performance improvement program for sepsis incl screening high risk pts.

784
Q

When using the ECG to time intra-aortic balloon counterpulsation, balloon inflation should occur at the

A

Inflation middle T wave (onset of diastole, closure of the AV, ideally @ dicrotic notch), electrical activity precedes mechanical by a few ms.

Deflation peak of R wave (onset of systole, just before AV closes, just before upstroke on art pressure waveform)

If using pressure waveform (eg. arrhythmias), inflation w dicrotic notch, deflation just before upstroke arterial pressure waveform.

IABP provides aortic counterpulsation, augments diastolic flow during diastole (increases DPTI (as increasees ADP),, incr Cor blood flow), reduces afterload during systole, improves LV performance (reduces LV tension time, reduces LV O2 consumption).

contraindications:
ABSOLUTE:
significant AR, aortic dissection, clinically significant aortic aneurysm repair/stent, uncontrolled sepsis
helium: less resistance to inflation/deflation (low density laminar flow), easily absorbed if balloon rupture

relative:
Uncontrolled bleeding
Known atheroma or severe atherosclerosis
Severe PAD that can’t be pretreated w tenting
LVOTO
Inability to achieve adequate timing

Complications:
Vascular:
Limb ischaemia, visceral ischaemia
Vascular laceration
Major haemorrhage

Non-vascular:
Haemolytic, insertion site (usually groin) infection, peripheral neuropathy, balloon rupture (helium embolisation, thrombus formation), cholesterol embolisation, CVA
Sepsis

insertion 2-3cm distal to L) SC artery, @ carina

When inflated shouldn’t be >80-90% diam Desc aorta

785
Q

A 45-year-old male received a heart transplant one month ago. He develops a new supraventricular
tachyarrhythmia without hypotension during a gastroscopy. The most appropriate therapy is

A

valsalva won’t work, can use adenosine reduced dose (1.5mg >=60kg or as per AHA, 3mg

analgesia, correct other reversible causes (eg. correct electrolytes)

If irregular, B blocker cautious of bradycardia risk (digoxin not likely to be effective, could use amiodarone if evidence HF)

diltiazem & verapamil are contraindicated tacrolimus & cyclosporine (can impair metabolism of CYP3A & incr drug levels renal tox.
cardioversion (if adverse features: shock, syncope, myocardial ischaemia, heart failure); DCCV 120J

786
Q

A 60-year-old man remains unconscious after an isolated head injury. The systolic blood pressure (in mmHg)
should be kept above

A

100mmHg

787
Q

A 34-year-old for a diagnostic laparoscopy has a height of 158 cm and a weight of 120 kg (BMI 48 kg/m2). For induction of anaesthesia, appropriate drug dosing includes

A

SOBA:
This guy’s ideal BW 55.4kg, lean BW 65.2kg, Adj40BW (IBW + 0.4(TBW-IBW)) 81.2kg

propofol induction lean BW 2-3mg/kg 130-200mg
maintenance TBW or Adj40BW
suxamethonium 120-180mg (1-1.5mg/kg actual BW)if RSI
rocuronium 40-79mg (0.6-1.2mg/kg LBW)
opioids LBW & adjust doses down (more sensitive to OIVI), ie. fent 30-130microg 0.5-2microg/kg

788
Q

Suxamethonium may be safely given to patients with (list of neuromuscular diseases given)

A

myaesthenia gravis (relative resistance, dose increased; 2mg/kg for RSI)

AVOID in:
Hereditary:
Pre-junctional disorders, eg. charcot-marie-tooth, Fredrich’s ataxia
Postjunctional disorders, eg. duchenne & becker muscular dystrophy, myotonic dystrophies, hyperkalemic & hypokalemic periodic paralysis
Acquired:
Pre-junctional eg. motor neurone, in multiple sclerosis IF the pt is mobility-limited, guillian barre due to muscle weakness (even after recovery. of neurological deficit)
NMJ: Eaton-lambert
post-junctional:
-critical illness polyneuropathy

-postpolio

Friedrich’s ataxia:
Autosomal recessive
Due to altered mitochondrial proteins—> iron accumulation.
Ataxia, sens neuropathy, cardiomyopathy (arrhythmias, HF), DM
Degeneration of DRG, post columns, spin-cerebella & corticospinal tracts.
BULBAR
KYPHOSCOLIOSIS
SUX AVOIDED (hyperK risk)
INCR SENS TO NDMRS

GBS:
Acute inflammatory demyelinating peripheral polyneuropathy- viral or back infection
Asc progressive m weakness, ANS dysfunction, areflexia
25% require mach vent.
Asp risk BULBAR
Resp insufficiency (may need post vent)
ANS dysfunction- harm instability, ANS hyperreflexia type reactions
SUX CI (hyperK)
Sens to NDMR
Incr risk VTE
Neuropathic pain common
IVIG, PEx
NOT steroids
Neuraxial controversial; document deficits

DMD:
XLR
Becker= milder form Duchenne
Asp risk bulbar, GI motility
Poss macroglossia
Sux/VA contraindicated (rhabdo/hyerK)
Poss osa, sensitiviety to sed/hyp
Pulmonary HTN/RV failure
Rest lung disease from scoliosis, Resp m weakness
Risk Perioperative Resp failure, impaired cough, recurrent aspirations
DCM (tall R waves precordial, incr R:S, deep Q in I, aVL, V5-6)
MR common (due to LV dilation_
Conduction/arrhhyhtmias
May have mild cog impairment
May be on ACE-I, B block, steroids

789
Q

In a burns patient, the blood concentration of propofol is

A

higher during initial hypodynamic (“ebb”) phase

DECREASED during later (>48hrs) hyperdynamic (“flow”) phase, so higher doses are needed (bolus & infusion) to maintain therapeutic [] from 3-4 days post burn. Hepatic & renal Cl increase since reduction in alb increases free propofol fraction & plasma prop []s are then lower for a given dose regimen.

790
Q

WFNS for GCS 10 & no motor deficit

A

4

1=15
2=13-14 no motor def
3=13-14 w motor def
4= 7-12 +/- motor deficit
5= 3-6 +/- motor deficit

791
Q

High-risk criteria for pre-eclampsia (justifies prophylactic aspirin)

A

High risk factors:
-prev preg w hypertensive disease
-T1 or 2 DM
-chronic HTN
-kidney disease
-autoimmune disease w potential vascular complications (antiphospholipid, SLE)

Mod risk factors (require 2 of these to justify aspirin)
-first preg
-multifoetal gestation
age >40
-interpreg interval >=10yrs
-FHx (mother or sister) PET

792
Q

Haemodynamics of a pt w pre-eclampsia & FGR @ 30/40

A

incr SVR & CO
also inc SV, incr inotropy, reduced diastolic function

793
Q

A 15-year-old patient with a known prolonged QT interval has a ventricular tachyarrhythmia while being monitored postoperatively in the postanaesthesia care unit. The patient is alert, orientated and without chest pain but feels unwell. The best initial management is

A

IV Magnesium
Correct metabolic/electrolyte derangement
remove inciting medications
consider overtrive atrial pacing or IV isoprenolol

794
Q

A 15-year-old patient with a known prolonged QT interval has a ventricular tachyarrhythmia while being monitored postoperatively in the postanaesthesia care unit. The patient is alert, orientated and without chest pain but feels unwell. The best initial management is

A

IV Magnesium 25-50mg/kg (likely 2g at 15yo)
Correct metabolic/electrolyte derangement
ensure not taking QT prolonging drugs
?consider overdrive transvenous atrial pacing 100-140bpm or IV isoprenolol (which would worsen the pt if VT misdiagnosed as TdP) start 2microg/min & titrate to HR 100bpm if the pt doesn’t respond

pts w TdP typically B block, consider L) heart symp denervation, pacing to shorten QTC, ICD.
avoid sotalol, amiodarone, droperidole, fluox, proca, flucon, flecainide, METHADONE.

If give Mg++ for torsades, ensure potassium above 4.5
trans-venous pacing if resistant to Mg++.

props safe. midaz fine. avoid ketamine as sypathomimetic.
SUX prolongs QTc!
all volatiles prolong QTc.

Unstable: promptsynch CV with 2g IV Mg++
if pulseless (TdP degen to VF), CPR, defib, give 2g IV Mg++, NO amiodarone. instead, give IV lignocaine Lignocaine 1mg/kg IV/IO (adults & paeds)

795
Q

An eight-year-old child with sickle cell disease is scheduled for emergency fixation of a fractured radius. Her haemoglobin is 80 g/L. The most appropriate management is

A

Discuss w haematology, but guidelines= top-up transfusion, aiming for Hb >100g/L, pt-specific blood (ABO, Rh, Kell, HbS -ve without any other relevant antibodies), doesn’t have to be done pre-op (could be intra), depending on haematologist advice

796
Q

MEPs minimal effect from

A

opioids
ketamine
nitrous (mildly depress)

cf:
incr w etomidate
mild depr w props/barbiturates
volatiles depress ++

797
Q

amiodarone for refractory VF paeds?

A

5mg/kg (after 3rd shock)

798
Q

The correct blood collection tube for a mast cell tryptase test is a

A

if it’ll take >1hr for the sample to get to the lab, needs to be refrigerated so get it to the lab ASAP
Serum (serum separator) or plain tube
(serum gel or lithium heparin gel)

799
Q

All patients over 70 years of age having received either spinal or general anaesthesia at Hospital X are
reviewed 3 years later to assess cognitive function. The aim of the study is to determine whether exposure to
general anaesthesia or spinal anaesthesia impacts cognitive function. This trial design is best described as a

A

cohort study

800
Q

Conditions associated with acromegaly

A

NOT abdominal aneurysm
Primary GH excess:
functioning pituitary macro adenoma
MEN1 (pancreas islet, pituitary, parathyroid)

ectopic: lymphoma, pancreatic islet-cell tumour

GHrH excess:
from hypothalamus
bronchial carcinoid
SCLC
adrenal adenoma
phaeochromocytoma

Difficult airway (macrognathia, prognathism, macroglossia, frontal & nose enlargement, prominent upper incisors)
Excess upper airway soft tissue—> prone to OSA (in up to 70%!), laryngoscopy challenge
HTN
LV hypertrophy
IHD, arrhythmias, heart block, biventricular dysfunction, cardiomyopathy, myocarditis, myocardial fibrosis
Elevated pulmonary pressures
Difficult vascular access
peripheral nerve entrapment (eg. carpal tunnel)
Visual field defects & mass effects
kyphoscoliosis
proximal myopathy
arthropathy
osteoporosis
hyperhydrosis
colonic polyps/bowel Ca
T2DM/glucose intolerance

801
Q

A patient in the intensive care unit has ventricular fibrillation two hours after her coronary artery bypass graft procedure. Recommended immediate management does NOT include

A

Do NOT immediately commence CPR (can delay for shockable rhythms 1 min)
Do NOT give full 1mg adrenaline

Initially DO do 3x stacked shocks 150J (re-Ax between each one)
300mg amiodarone IV

If no improvement, open chest (resternotomy within 5 mins); closed cardiac massage, internal defib 20J
Manually vent w ambu bag FiO2 100%, 30:2, discontinue sedation
If IABP, switch triggering mode to arterial tracing
50microg boluses of Adr

802
Q

The sensor on a NIM (Nerve Integrity Monitor) endotracheal tube used for thyroid surgery directly records

A

Monitors movement of the VCs on endotracheal tube–> EMG activity vocal cords (internal laryngeal muscles)

803
Q

A drug that is contraindicated for a patient with a history of heparin-induced thrombocytopaenia is

A

prothrombin complex concentrates (4-factor, 3-factor inactivated but NOT the activated PCC (with FAIBA; no heparin)
UFH, LMWHs (enoxaparin, danaparoid), heparin flushes, heparin-bonded catheters, some hematopoietic stem cell products, some TPN, some forms of IV medications

804
Q

The estimated proportion of human induced climate change attributable to nitrous oxide is

A

7%

GWP 365 over 100yrs (CO2 1, des 2540, iso 510, nitrous 365, sevo 130. iso does have some (0.01) ozone depletion potential but short life so minimal impact, sevo & des no ODP as nil chlorine)
atmospheric life 114yrs

805
Q

Regarding cardiopulmonary exercise testing before major surgery, oxygen pulse is the

A

VO2/Hr (oxygen consumption/HR, mL/beat)
represents product of SV & arterial-venous oxygen difference, surrogate for stroke volume
should increase at the start of exercise & plateau@ highest predicted value

806
Q

The antiemetic that interferes with the effectiveness of oral hormonal contraception is

A

aprepitant

Metabolism accelerated by drugs incr liver microsomal enzyme activity (phenytoin, rifampin, carbamazepine)
Griseofulvin
St John’s wort

807
Q

Despite an interscalene block being performed preoperatively for arthroscopic rotator cuff repair, a patient
wakes up with posterior shoulder pain. The most appropriate procedure to consider would be a nerve block of
the

A

suprascapular nerve

suprascapular nerve= majority of supply to shoulder joint
blockade is more effective than shoulder LIA, less effective than interscalene block

Supply to should joint (terminal branches)= primarily from suprascapular & axillary, with minor contributions from subscapular, musculocutaneous & lateral pectoral

808
Q

Intraoperative lung protective ventilation strategies include all of the following EXCEPT

A

*recruitment maneuver sans PEEP

-Low TVs (6-8mL/kg predicted BW)
-low PEEP (start at 5cmH2O, individualise to avoid incr in driving pressure (Pplat-PEEP) while maintaining low VT
-low pplat (max 30cmH2O)
-low driving pressure (max 15cmH2O; Pplat-PEEP)
-recruitment manoeuvres if needed, lowest effective pressure (Pplat 30-40cmH2O non-obese, 40-50cmH2O obese, shortest effective time, fewest # of breaths; they have limited benefit without sufficient PEEP).
-Ramp pt before induction (HOB >=30 degrees) & if no contraindication, before loss of spont vent use NIPPV or CPAP to limit atelectasis). Aggressively avoid obstruction during induction (eg. NPA).
-After intubation, FiO2 <-0.4, use lowest possible FiO2 to achieve SpO2 >=94%.
-No specific controlled vent mode is recommended.

-target PaO2 55-80mmHg, SpO2 88-95%.

-position to avoid ZPEEP during emergence, don’t suction tracheal tube just before extubation. If high FiO2 for extubation (>0.8), consider CPAP with low FiO2 immediately after to reduce risk resorption atelectasis.

Aim= improve mechanics of breathing & resp function, prevent PPCs.

-Dedicated score should be used for risk evaluation of PPCs (main risks age >50yr, BMI >40, ASA >2, OSA, preop anaemia, preop hypoxaemia, emergency or urgent surgery, ventilation duration >2hrs, intraop haemodynamic impairment & low SaO2)

-To evaluate the effect of interventions, assess the impact on respiratory system compliance with constant tidal volume.

809
Q

An absolute contraindication to transoesophageal echocardiography is

A

Perforated viscus
oesophageal stricture
oesophageal tumour
oesophageal perf/lac
oesophageal diverticulum
active GI bleed
recent oes or gastric surgery

relative:
Hx radiation neck/mediastinum
Hx GI surgery
recent GI bleed
dysphagia
neck mobility restriction
hiatal nernia (symptomatic)
oesophageal varices
coagulopathy
active oesophagitis or PUD

810
Q

A 25-year-old ASA (American Society of Anesthesiologists) physical status classification I patient develops
seizures five minutes after receiving a brachial plexus block with ropivacaine. Of the following, the most
suitable initial intravenous treatment is

A

Benzodiazepine, prop

811
Q

You are performing femoral venous cannulation in an obese man under ultrasound guidance. The image
quality is suboptimal as the vein is deep. The best way to improve the image quality is to

A

Curvilinear probe, lower frequency
increase depth
setting. a focal zone
adjust gain; excessive or inadequate blurs tissue boundaries. Increasing gain below the focus can help
Doppler

812
Q

A patient is bleeding and her ROTEM displays a Fibtem A5 of 2 mm (normal > 4 mm). The most appropriate
treatment is

A

fib conc 1g/25kg or cryo 1u/5kg

813
Q

After ceasing smoking, a patient’s immune function has effectively recovered to normal after

A

6 months

ANZCA PS12
1. 5.1 Quitting smoking for one day will lower carboxyhaemoglobin and nicotine levels and could
be expected to improve tissue oxygen delivery.
2. 5.2 Quitting smoking for as little as three weeks has been shown to improve wound healing.
3. 5.3 Quitting smoking for six to eight weeks results in sputum volumes that are not increased
compared to non-smokers, and improved pulmonary function.
4. 5.4 Immune function is significantly recovered by 6 months after quitting smoking.

814
Q

The most consistent risk factor for postoperative vomiting in children is

A

Age >=3yo
surgery >=30mins
personal or FHx PONV/personal Hx motion sickness
strabismus & tonsillectomy

815
Q

Recirculation is a cannula position complication specific to the use of

A

VV ECMO
reinfused oxygenated blood withdraawn through rainage cannula, doesn’t pass through systemic circulation, reduced systemic oxygenation & reduced efficiency.
Improve by ncr distance btwn cannulae, echo & fluoroscopic guidance so reinfusion jet towards TCMdrainage cannula less -ve pressure

816
Q

The composition of blood returned to the patient from intraoperative cell salvage shows

A

packed red cells suspended in saline, hct approx 60% (50-70%)

trace amounts of plt/CFs/anticoagulant/microaggregates/other cells/contaminants aspirated

817
Q

The diabetic medication that, as part of its therapeutic effect, significantly prolongs gastric emptying is

A

GLP1 agonists have slowing gE as part of their therapeutic effect
DULAGLUTIDE does slow gastric emptying but effects are attenuated over time.
sitagliptin does.
GLP-1 agonists (exenatide, semaglutide): slow gastric emptying (–> nausea & vomiting), reduce postprandial glucagon & food uptake, enhance glucose-dependent insulin secretion.
Don’t usually cause hypoglycaemia.

DDP-4 inhibitors (sitagliptin & other gliptins) inhibit DDP-4 from inhibiting GLP-1, effect is more modest than GLP-1 antagonists but they do slow gastric emptying & don’t generally cause hypoglycaemia unless combined. w other therapies that do.

Amylin analogues also slow gastric emptying (pramlintide).

alpha glucosidase inhibitors (eg. acarbose) prolong the breakdown of polysaccharides to monosaccharides, reducing postprandial glucose levels. main side effect= flatulence.

818
Q

The most likely side effect observed in the post anaesthetic care unit after the use of dexmedetomidine is

A

hypotension

819
Q

When using cardioversion to revert a patient in atrial fibrillation to sinus rhythm, the direct current shock is
synchronised with the ECG to coincide with the

A

Peak of the QRS (highest point of the R wave or just after)

820
Q

Adverse effects of the use of sodium-glucose co-transporter 2 inhibitors in the perioperative period do NOT
include

A

DOES include:
-UTI (incl urosepsis, pyelonepritis), vulvovaginal candidiasis
-euglycaemic ketoacidosis
-bone fractures (osteoporotic mechanism)
-incr falls risk (BP lowering from osmotic diuresis, intravascular volume contraction)
-lower limb amputations (peripheral ischaemia)
-AKI controversial; may be related. toother factors, dose adjust if GFR <60

821
Q

The electrolyte abnormality most associated with an increased risk of laryngospasm is

A

hypocalcaemia

822
Q

A 21-year-old patient with a history of schizophrenia on quetiapine develops tremor, restlessness,
hyperreflexia, nausea and vomiting in the post-anaesthesia care unit following an emergency laparoscopic
cholecystectomy. Her heart rate is 80 / minute, blood pressure 130/90 mmHg, and her temperature is 37.0°C.
The most likely diagnosis is

A

serotonin syndrome

823
Q

The normal axial length of the globe of an adult eye is

A

22-25mm

824
Q

The Glasgow Coma Score of a patient whose best responses are: opening eyes to pain, making
incomprehensible sounds, and withdrawing from pain is

A

8

825
Q

A non-obese adult patient is administered a target-controlled propofol infusion for more than 15 minutes, with
a constant target plasma concentration of 4 μg/ml propofol. Compared to the Marsh model, the propofol dose
given by the Schnider model will be a

A

Smaller initial bolus, smaller cumulative dose

826
Q

You have been managing a case of malignant hyperthermia in an 80 kg man and have given a total of 400 mg
of dantrolene (Dantrium). The amount of mannitol you have also administered is

A

60g (3g mannitol per 20mg vial dantrolene)

827
Q

A child with well controlled dysrhythmias has an ASA (American Society of Anesthesiologists) Physical Status
classification of at least

A

II

828
Q

According to ANZCA PS54(A), an anaesthetic machine requiring electrical power must, in the event of mains
power failure, be able to operate under battery backup power for a minimum of

A

30 mins

829
Q

You are inserting a pulmonary artery catheter in an intubated patient prior to cardiac surgery and a significant
amount of blood appears in the endotracheal tube. The most appropriate specific initial management is to

A

Turn bleeding side down, advance tube down the side of the non-bleeding lung

830
Q

category A analgesics pregnancy

A

paracetamol, codeine, dihydrocodeine

831
Q

A thoracic regional technique that will NOT provide analgesia for sternal fractures is a

A

PECS 1 block (doesn’t cover anterior ramus intercostal nerves 2-6)

PVB & TEA ARE effective, as is parasternal (btwn pec maj & int intercostal mm & transversus thoracis plane block (btwn transversus thoracic & intercostal m, level T4-5 intercostal space).

832
Q

A 72-year-old man with peripheral vascular disease presents for a femoral angioplasty and is currently taking
aspirin. Regarding the perioperative management of his aspirin,

A

continue, despite , the large randomized POISE-2 trial found that perioperative aspirin increases bleeding risk but does not improve cardiovascular or mortality outcomes for non-card surg

833
Q

A patient with acute right heart failure secondary to acute myocardial infarction is likely to have a/an

A

inferior infarct
RCA lesion proximal to the origin of major RV branches

Acute LV failure & APO
Echo:
-incr RAP >10mmHg
incr RAP:PCWP ratio >0.8
reduced CI
REDUCED PAPI <=0.9
decreased TAPSE <17mm but may be inaccurate w RWMA

Pulmonary artery pulsatility index (PAPi): calculated from ratio of pulmonary artery pressure to RA pressure (it’s (PASP-PADP/RAP)). predicts RV failure after inf MI & LVAD implantation. lower PAPi ass’d w mortality, MACE, HF & hospitalisations.

834
Q

You are called to recovery to review an 80-year-old woman post neck of femur fracture fixation performed
under general anaesthesia with a fascia iliaca block. She has a history of mild dementia. She has become
confused and agitated after initially being cooperative and pain-free. The most appropriate drug therapy to
manage her is intravenous

A

First-line for postop delirium= Ax & Mx underlying cause (infection, pain, dehydration, metabolic derangement, constipation, urinary retention) & NON-PHARMACOLOGICAL INTERVENTIONS

If severely agitated, after Rx of reversible causes & redirection/reorientation, small dose haloperidol (0.5-2mg), since it doesn’t worsen the overall course of delirium (preferred over a benzo)

DEXMEDETOMIDINE INFUSION: NNT=10 for postop delirium

Melatonin (takes 30-45mins to start working, low 3-33% POBA, t1/2 1hr, anxiolysis, sleep disturbance, delirium, anxiety, pain, emergence agitation, myocardial protection; regulating circadian rhythm, anti-inflammatory & anti-oxidative)

Postop delirium: common & serious. Incr LoS 2-3 days, 30-day mort 7-10%, incr risk
significant functional decline & need for care facilities on discharge

A cognitive disturbance characterised by acute & fluctuating impairment in attention & awareness

Risk identification & peri-op risk reduction= the most vital part of post-op delirium management (multi-d, organisation-wide), since limited Rx options once delirium established

Risk assessment, risk reduction & rescue Rx= cornerstone

Risks:
-Pt: poor baseline functional status, sensory impairment (visual, hearing), neurological/cardiac/respiratory/metabolic diseases
-Higher ASA
-Surgical: #NOF up to 70% risk postop delirium!
-Emergency/complex procedures requiring postop ICU admission
-Metabolic derangement
-Pain (both pre-op & postop; preop pain is 1.5-2x higher risk postop delirium)

Risk scores exist (eg. combo of MMSE & APACHEII) however validated for specific populations weakens translatability.

Effective peri-op interventions:

Non-pharmacological (family member re-orient, early mobilising, avoiding excessive bed rest/fasting/malnutrition)
-Reorientation measures (non-pharmacological); minimising staff change/pt transfer, introduction of staff, natural light, time-keeping devices, reminding pts about previous events, future planning; Reorientation alone can reduce the incidence of overt delirium by 40%! Cognitive exercises, vision & hearing optimisation, sleep optimisation, mobilisation, hydration & nutrition. Implement as part of multicomponent interventions (reduce incidence of delirium with OR 0.4 (NNT=14.3)

-Avoiding periop polypharmacy

-Avoid prolonged fluid fasting (>6hrs= 10.6x OR postop delirium)

-Pre-op complex geriatric assessment (CGA & OPTIMISATION- eg. orthogeriatric teams; multi-D approach to systematically evaluate & address often complex care needs in older pts, CGA-based periop care improves postop outcomes)

-AVOID intraop BZD (2.4x higher risk postop delirium) & gabapentinoids (independent risk factors for postop delirium). TCAs & scopolamine also incr risk.

-DoA monitoring (needs further evidence)

-multimodal opioid-sparing analgesia, regional & NEURAXIAL important (neuraxial= independent predictor of delirium risk reduction)

-paracetamol reduces delirium risk in cardiac surgery pts (NNT=5.6)

-Parecoxib reduces delirium risk in ortho pts (NNT= 20)

-DEXMEDETOMIDINE: neuroprotective. Intra-op dexmedetomidine infusion, postop dexmed. Reduces risk postop delirium w OR 0.35.

Postop:
non-pharmacological
melatonin or ramelteon (melatonin agonist)
dexmed, olanzapine & risperidone. but not haloperidol (overall value of antipsychotics unclear due to risk of complications)
high-dose dexamethasone (in card surg)

multimodal analgesia
Those which may help (need further studies):
TIVA vs VA- no diff, low quality evidence
subhypnotic doses intraop ketamine show no difference, low quality evidence
BIS guided anaes
minimally invasive surgery
avoidance of hypothermia
strict BP control
goal-directed fluid therapy
restrictive transfusion

Postop delirium usually postop days 2-5
previously BZD & antipsychotics were used for hyperactive/agitated delirium but BZD may worsen symptoms & antipsychotics don’t reduce length of symptoms, don’t reduce adverse outcomes ass’d w delirium & there are some concerns regarding safety of antipsychotics- incr M&M!

rapid assessment: confusion assessment method
For a diagnosis of delirium by CAM, the patient must display:
1. Presence of acute onset and fluctuating discourse
AND
2. Inattention
AND EITHER
3. Disorganized thinking
OR
4. Altered level of consciousness

IF give antipsychotics (best avoided as high anti-cholinergic affinity, effective @ sedation but may prolong delirium & POCD): haloperidol or risperidone 0.5mg po single, olanz 1.25mg single dose

835
Q

In preschool-aged children having tonsillectomy under general anaesthesia, delirium is more likely with the
use of

A

volatile (esp des, sevo & iso no diff, halothane better than sevo)
opioids
poorly controlled pain

Factors ass’d with LOWER incidence of emergence delirium:

-propofol cf sevoflurane (propofol effective to reduce emergence agitation either as TIVA, as an infusion throughout or 3mg/kg over 3 mins @ the end (transition), or as a bolus 1mg/kg @ the end (TIVA greater magnitude of benefit for EA reduction). Obv the 3mg/kg over 3 mins prolongs emergence (8mins vs 4mins if 1mg/kg)

-administration of fentanyl (intranasal 2mcg/kg, useful both for prevention or Rx of EA if IVC dislodged; or 1mcg/kg IV @ end of sevo anaes; fent higher PONV rate than alpha agonists or props)

-alpha-2 agonists (clonidine or ESPECIALLY dexmed; dexmed results more impressive & consistent wrt reducing EA, reding rescue analgesia, reducing PONV & non clinically sig (minimal) incr emergence time; doses in Costi blue book for deemed: 0.15, 0.3, 0.5, 1mcg/kg early or late in anaes or as infusion +/- loading dose; 95% effective dose for reducing EA for tonsillectomy or adenoidectomy is 0.38mcg/kg. Dexmed premed performs better or equal to midaz for induction outcomes (parental separation, mask acceptance) & reduces incidence of EA. 1.5mcg/kg 45mins pre-induction.

-Dexmed superior to propofol bolus 1mg/kg @ end of case.

-ketamine either as a premed or bolus 0.25mcg/kg @ end of Anaes may reduce EA.

-midaz premed doesn’t impact EA but IV midday @ the end may reduce EA.
-propofol during or @ the end of anaesthesia

*An intra-op DEXMED dose reduces incidence of emergence delirium in chn (OR 0.22). Clonidine also effective, OR 0.5 (but delayed emergence/hypotension limit usefulness w higher doses eg. 2mcg/kg). Doses ranged dexmed 0.15-4(!)mcg/kg, clonidine 1.5-2mcg/kg, IV post-induction. No differences in time spent in recovery or time to discharge. Less need for rescue analgesia.

For reducing risk sevo EA, prop/fent/dexmed/clonidine/halothane/ketamine/IV midaz @ the end/analgesia/N2O washout may all be effective- ?reduce time for sevo emergence effective?

Dexamethasone 0.2 mg kg−1 before operation has also been found effective in reducing ED

A technique known as ADVANCE (Anxiety-reduction, Distraction, Video modelling and education, Adding parents, No excessive reassurance, Coaching, and Exposure/shaping) performed in the waiting room and at induction has shown to reduce the incidence of ED. Techniques to reduce parental & child anxiety.

Parental presence @ emergence doesn’t improve EA (but does improve post-hospitalisation behavioural change).

For a smooth calm emergence:
-adequate IV analgesia
-emergence on props (also reduces airway reactivity & PONV)
-consider dexmed if cost allows (premed in anxious child, IV intra-op)

Preschool age children & pre-op anxiety incr risk of both emergence delirium & post hospitalisation behavioural change.
Other risk factors emergence agitation: preschool, pre-op anxiety, temperaments (eg. Poorly adaptable), ophthal/ENT procedures, sevo or desflurane, inadequate analgesia.
Post-hospital behavioural change occurs in >50% of children undergoing a GA, usually short-lived, 5-10% may last up to 12 months.
Risk factors= underlying anxiety in child or parent, prev bad hospital experience, emergence delirium, pre-school age. Parent presence upon eye opening in recovery reduces incidence of post hospital behaviour change.

Emergence delirium= disassociated conscious state where child is unaware of his or her surroundings
PAED (paediatric anesthesia emergence delirium) scale= the most widely used (& is “validated”). 5 items, 0-4. Research supports threshold PAED >12.

PAED scale score eye contact/purposeful actions/awareness of surroundings/restlessness/inconsolability
Hypoactive delirium: child unaware of surroundings, quiet & withdrawn; most screening tools may not identify it (Cornell Assessment of Paediatric Delirium, CAP-D, is used in ICU & RCH investigating it in PACU
Hyper-active: motor agitation/restlessness, movements not purposeful, can’t be consoled, no eye contact, don’t recognise ppl/things.

836
Q

A 72-year-old woman on aspirin presents to her ophthalmologist for follow-up three days after you performed
a transconjunctival peribulbar block for cataract surgery on her left eye. She complains of painless periorbital
swelling, erythema, and mild chemosis which started the day after surgery but is improving. She had a
peribulbar block three weeks ago for surgery on the other eye. The most likely diagnosis is

A

conjunctival chemosis (superficial haemorrhange)
Superficial haemorrhage may produce unsightly periorbital haematoma but is not vision-threatening.
Not uncommon postop.
May be delayed hyalase reaction.
retrobulbar haemorrhage, if arterially based, may cause sudden bleeding, palpable dramatic increase IOP, globe proptosis, upper lid entrapment, may compromise optic nerve’s vascular supply & impair vision.
Urgent ophthal consult, consider lat canthotomy or paracentesis. continuous ecg as oculocardiac reflex may occur.

837
Q

You have diagnosed anaphylaxis in an eight-year-old girl having an appendicectomy. She weighs 20 kg and
has refractory bronchospasm despite an adrenaline (epinephrine) infusion running at 15 mcg/min. The
recommended initial dose of salbutamol (100 mcg/puff) via metered dose inhaler is

A

12 puffs (>6yo), ie 1200mcg

(6 puffs <6yo)

adjuncts:
salbutamol IV infusion 1mcg/kg/min
Mg 50% (500mg/mL): 50mg/kg, max 2g, over 20 mins, aka 0.1mL/kg)
aminophylline 10mg/kg over 1 hr (max 500mg)
hydrocort 2-4mg/kg (max 200mg)

838
Q

For a skewed distribution of data the best measure of dispersion of data is the

A

median, paired with interquartile range or other %-based ranges

Normal distribution, use mean with standard deviation; 68% of the data falls within 1STD, 95% within 2 & 99.7% within 3STDs

839
Q

Blockade of the superficial cervical plexus includes the

A

C2-4

skin btwn anterior scalene & SCM

lesser occipital
greater auricular
transverse cervical
supraclavicular

840
Q

Of the following, the substance LEAST likely to cause lactic acidosis is

A

Substances that DO:
epinephrine
sulfonylureas (gliclazide)
biguanides (metformin)- rare
inhaled B agonists
linezolid
nucleoside reverse transcriptase inhibitors (NRTIs, used for HIV)

propofol infusion syndrome

841
Q

You will anaesthetise a 39-year-old woman for a laparoscopic cholecystectomy. She has a history of
mastocytosis and has never had an anaesthetic in the past. A drug which you should avoid is

A

Drugs with potential for mast cell degranulation
morphine
codeine
pethidine
buprenorphine

atracurium
mivacurium
LMWHs
NSAIDs & aspirin MAY trigger mast cell degranulation (depends on the pts)
sux is UNLIKELY to cause non-allergic mast cel degranulation but is the commonest cause of allergic anaphylaxis during anaesthesia so avoid it.
avoid amethocaine (an ester LA)

Rare disorder w pathologic accumulation of mast cells in tissues

Other factors:
pre-ads
assess previous anaes records; avoid any known trigger factors (eg. for the pt). as well as drugs known to trigger mast cells
pt should have a medic alert
ACE inhibitors should be stopped several wks before planned surgery (on advice. of GP/allergy specialist). While B blockers may blunt BP supporting effect of epinephrine, generally OK in usual clinical doses; could change to another class a few wks before surgery if taken for BP but generally continue if for Cor artery disease or HF.
ARBs generally continued.

multi-D preop preparation & assessment (dermatology, immunology (pt should be assessed re: potential triggers preop & discussion btwn immunologist & anaes re: planning. skin prick testing may be unreliable), surgical & anaes; *bowel manipulation may trigger mast cell degranulation)
Team brief to ensure other OT staff aware. of potential triggers (eg. contact pressure, antiseptic solutions (pt should have allergy testing if there’s concern re: prev reactions to chlorhex or iodine), latex-containing equipment)
limb tourniquets relatively contraindicated
temperature control: avoid hyper/hypothermia
avoid excessive SNS activation:
-anxiety (preop anxiolysis)
-pain (consider regional)
-ensure adequate depth
-give all drugs slowly & in the minimum dose required.
stabilise mast cells w inh salbutamol & 100mg hydrocort immediately preop
consider sodium cromoglycate in asthmatic pt (ie. for pts already taking it, but no evidence it should be commenced. it may improve diarrhoea & abdo pain in mastocytosis but doesn’t have high systemic bioavailability so may not prevent hypotension etc.
-art line, 2x large IVCs under light sedation
-BIS, temp probe, 5-lead
-warm the OT, warm the IVT, upper & lower FAWDs, pressure points
-discuss w radiologist before any contrast
-postop HDU. Day case not appropriate.

proliferation of mast cells (cutaneous eg. urticaria pigmentosa, esp in chn), most adults have systemic involvement (esp bone marrow)
symptoms relate to mast cell degranulation; release. of vasoactive mediators–> anaphylactic-like reactions
can have severe adverse events, a previous uneventful anaesthetic doesn’t guarantee future uneventful

Do not need. to repeat tryptase pre-op but can do for baseline

premed w antihistamine not evidence-based but could use in principle

CAN use:
fent
remi
midaz
props
volatile
roc/vec & cis
paracetamol
dexamethasone
neo/glyco
regional (no evidence, neuraxial or nerve blocks should be avoided)
cophenylcaine is fine (an alternative to amethocaine)

842
Q

test for a condition which has a prevalence of 1 in 1,000 has a sensitivity of 100% and a specificity of 90%.
The probability of a patient who receives a positive result actually having the condition is

A

1% (PPV)

(TP=1, TN 900, FP 99)

843
Q

Anterior spinal artery syndrome would NOT result in

A

loss of vibratory sense or proprioception below level of lesion

It DOES result in:
Loss of motor function below level of lesion (initially flaccidity, later spasticity)
initial loss of deep tendon reflexes, later hyperreflexia
clonus
loss of pain/temp sensation
bladder & bowel dysfunction
May get hypotension from ANS dysfunction
sexual dysfunction

844
Q

The amount of intravenous potassium chloride required to raise the plasma potassium level from 2.8 mmol/L
to 3.8 mmol/L in a normal adult is approximately

A

200mmoL.
administering 10 mEq of potassium for every 0.1 mEq/L desired increase in serum potassium.

*if the serum potassium level is >3, 100-200mmol are required. if the K+ is <3, need 200-400mmol

845
Q

A raised (> 140% predicted) single-breath diffusing capacity of the lung for carbon monoxide (DLCO) can be caused by

A

altitude
asthma
polycythemia
Severe obesity
pulm haemorrhage
L)-R) intra-cardiac shunt
mild L) heart failure (incr capillary blood volume)
exercise just prior to the test- incr CO
mueller manoeuver
supine
bronchodilator

846
Q

You are performing a focused cardiac ultrasound in the postanaesthesia care unit on a patient who is
hypotensive for unclear reasons. His heart rate is 100 beats/min. The left ventricular velocity time integral is
10 cm. The left ventricular outflow area is 3 cm2. The left ventricular ejection fraction is 25%. The right
ventricular systolic pressure is 40 mmHg. The inferior vena cava diameter is 20 mm. The estimated cardiac
output is

A

CO= HR x SV
SV= LVOT CSA x VTI
3000mL/min=3L/min

*VTI units are velocity/time, so in cm (cm/second per second)

847
Q

Of the following, the congenital condition LEAST commonly associated with obstructive sleep apnoea in
children is

A

Conditions that ARE ass’d:
Apert
Down syndrome
Pierre robin
Treacher Collins
Crouzon
mucopolysaccharoidosis
Neuromuscular syndromes
Prader-willi
Arnold-chiari syndrome
Achondroplasia
Spina bifida

848
Q

An open Ivor-Lewis oesophagectomy is performed via a

A

Abdo phase via laparotomy (upper midline T6-10 or “rooftop” T8-9) (first stage)
Then thoracic phase: posterolateral R) thoracotomy, 4th-5th interspace or thoracoscopy.
Ivor Lewis better for Ca of lower 3rd oesophagus. Limited by high risk reflux, if leak @ intrathoracic anastamosis, high M&M risk.

3 stage involves thoracotomy or thoracoscopy first, laparotomy then neck incision (L) preferred since reduces risk of injury to RLN). Neck anastamosis easier to manage if anastomotic leak, lower risk GORD, more extensive proximal resection margin.

Transhiatal: upper midline laparotomy & L) neck incision

849
Q

Of the following, the procedure that is most commonly associated with chronic pain after surgery is

A

amputation
thoracic wall (thoracotomy)
inguinal herniotomy

850
Q

Management of status epilepticus

A

first line= benzodiazepines (lorazepam preferred intravenous; may have duration as long as 4-12 hrs as less redistribution to adipose). time to maximum effect up to 2mins.
midazolam (rapid- often terminates seizures in <1min) pref for IM, nintranasal or buccal; 0.15mg/kg IM/IV (max 10mg paeds), 0.3mg/kg buccal/IN (max 10mg)
diazepam pref for PR (effect as early as 10 seconds, duration <20mins, but stable @ room temp).

2nd line= non-BZD antiseizure meds (even if convulsions have ceased)
levetiracetam
fosphenytoin (prodrug, more rapid IV admin, no need for IV filter, lower potential for tissue or cardiotoxicity) & phenytoin (20mg/kg IV/IO loading, over 20mins max rate 50mg/min, monitored, NOT for age <1 mth), keppra 40mg/kg IV/IO max 3g, dilute to 50mg/mL & infusion over 5mins.
valproate

3rd line phenobarbital

can also give propofol insion; 2.5mg/kg IV/IO then 1-3mg/kg/hr
midaz infusion 1mcg/kg/min

Can use:
ketamine
isoflurane
thiopentone

not calcium

851
Q

RELIEF trail TBL:

A

restrictive (5mL/kg on induction, 5mL/kg/hr intraop, 0.8mL/kg/hr @ least 24hr postop) vs liberal (10mL/kg induction, 8mL/kg/hr intraop, 1.5mL/kg/hr @ least 24hr postop) not ass’d w higher disability-free survival @ 1 yr but was ass’d w higher AKI.

852
Q

A 45-year-old man is ventilated in the intensive care unit and is in a critical state. His pulmonary artery wedge
pressure is 26 mmHg, cardiac index is 1.7 L/minute/m2 and his PaO2/FiO2 ratio is 200 mmHg. A decision is
made to place him on extracorporeal membrane oxygenation. The most appropriate mode is

A

ECMO= use of modified heart-lung machine to provide resp, circulatory (or both) support, for days to weeks. For reversible but severe causes of resp faiure or cardiogenic shock refractory to conventional treatment.

Contraindicated if advanced malignancy, severe chronic organ failure, severe brain injury, non-recoverable pathology

Cannulae can be placed under direct vision or percutaneous (eg. Guided by US or fluoroscopy).
Centrifugal pumps (reliable, easy to care for). Gas exchanger (more efficient CO2 removal than O2 addition), newer devices less resistance to blood flow, less traumatic to blood, less thrombotic.
Blood passed through membrane for gas exchange, warmer, return to blood.

V-VA, provides support for both pulmonary & cardiac function, has a second venous cannula

V-V (central or peripheral) for non-pulmonary gas exchange (support for resp failure). Indicated for hypoxic resp failure ANY cause w expected mort >50%, eg. PF<80. Eg. ARDS (primary eg. infection, aspiration), secondary (eg burns), pulm haemorrhage

V-A ECMO (central or peripheral) for both gas exchange & haemodynamic support (provides organ perfusion despite cardiac failure. unloads RV failure (cannulae drain from RA & arterial returns to aorta to aorta) but not LV. eg. for ACS, cardiac arrest w CPR from reversible cause, toxidromes, acute cardiomyopathy, cardiogenic shock due to PE
complication= harlequin/differential hypox where LV pumps poorly O2 blood to upper body & coronary, cannula supplies well oxygenated to lower half body, put pulse ox on R) hand (comp of peripheral VA ecmo)

V-PA ECMO provides support for RV failure AND non-pulmonary oxygenation; blood bypasses the RV, often used to support RV failure following LVAD insertion.

Complications:
bleeding
thromobsis
infection
neurological catastrophe
differential hypoxia
access insufficiency, recirculation, vascular injury, circuit breach (air embolism, blood loss), thrombus, pump failure

853
Q

A 48-year-old man is day two post-laparoscopic high anterior resection. He has used 42 mg of intravenous
morphine in the past 24 hours. You wish to start him on oral tapentadol immediate release. The most
appropriate equianalgesic dosage would be

A

400mg (378mg)/day

854
Q

During an infraclavicular approach to the brachial plexus, the tip of the needle is positioned closest to the

A

posterior cord (passes from ceph to caud, under axillary artery, close to posterior cord)

855
Q

pin index position C size cylinder medical oxygen

A

2,5

nitrous 3,5
CO2 1,6
air 1,5
4

entonox single pin (7) in the centre

856
Q

In a previously normal patient with cardiac failure secondary to acute pulmonary embolism, the best choice of
vasoactive agent for initial treatment is

A

Norepinephrine

857
Q

You are reviewing a primigravida at 32 weeks gestation with a Fontan circulation in the anaesthetic
preassessment clinic. Peripartum care should avoid the use of

A

PVR: nitrous, PROSTAGLANDIN ANALOGUES & ERTOG ALKALOIDSValsalva
Dramatic drops preload: rapid onset neuraxial, cautious oxytocin (infusion best)
Should labour L) lateral
Avoid PPH :)

858
Q

Fontan circulation, avoid:

A

Prostaglandin analogues & ergot alkaloids
incr PVR
drops in preload
myocardial depression

misoprostil OK

859
Q

Cyclooxygenase type 2 inhibitors (COX-2) in pregnancy are considered

A

unsafe

860
Q

Compared to a normothermic patient, a patient with mild intraoperative hypothermia (35.0oC) will have

A

lower anaesthetic requirement & prolonged anaesthetic drug effects.
Higher risk coagulopathy/bleeding/transfusion requirements, reduced CMRO2, impaired wound healing & incr infection risk

reduced sens/gain of resp system to hypox/hypercapnia
reduced chest wall compliance/ventilation
shivering incr CO2 production & O2 consumption
vasoconstriction followed by VD
tachy followed by brady
incr O2 demand & reduced supply (ischaemia)
arrhythmia
incr PVR (RV failure)

861
Q

The 2012 Berlin definition of the acute respiratory distress syndrome (ARDS) defines moderate disease as one with a PaO2 / FiO2 ratio (in mmHg) of

A

101-200mmHg

(severe is <=100mmHg, mild >200 but <300mmHg). Ventilator settings that include PEEP >=5cmH2O

862
Q

Normal (0.9%) saline has the physical properties of

A

Osmolarity of 308mOsmol/L
pH 5.5

Sodium 154mmol/L
Chloride 154mmol/L

No antimicrobial agents

863
Q

The medication most strongly associated with an acute primary hypotensive reaction following transfusion of
blood products is

A

ACE inhibitor (due to bradykinin excess)

864
Q

A patient has return of spontaneous circulation (ROSC) but remains unresponsive after cardiac arrest. ANZCOR Guidelines recommend all the following measures EXCEPT

A

SHOULD do:

-TARGETED TEMPERATURE MANAGEMENT (33-36degC for @ least 24hrs) FOR ADULT CARDIAC ARREST (in-hospital any rhythm, out of hospital either shockable or non-shockable) PTS WHO REMAIN UNRESPONSIVE AFTER ROSC

-Avoid & treat fever post TTM. Should have temp measured (bladder). keep sedated at least 40hrs while undertaking TTM (but aim to extubate early as possible if neurologically appropriate). Regular paracetamol. Actively cool if temp 37.7. Avoid shivering (regular paracetamol, target Mg >0.8mmol/L, consider buspirone (serotonin agonist). if shivering starts, dexmed or props or remi, Mg++ 1.2-1.6mmol/L target. tier 3= fentanyl boluses, muscle relaxant.

-they recommend haemodynamic goals should be made but that must be pot-specific & inadequate data to suggest a threshold; the pts normal SBKP, at least 100mmHg
-12-lead ecg; immediate angio & PCI if STEMI or NEW LBBB

-Avoid hypoxia & hyperoxia (aim SpO2 94-98%)

-kee[ PaCO2 physiologic; extrapolated from TBI they say 35-40mmHg

-Treat BGL >10mol/L w insulin but avoid hypoglycaemia

-Continue an infusion of any antiarrhythmic that restored a stable rhythm during resus (eg. lignocaine 2-4mg/min or amiodarone 0.6mg/kg/hr for 12-24hrs), may be reasonable to commence an antiarrhythmic drug if one wasn’t used to get ROSC from a shockable rhythm.

-Treat seizures but DO NOT GIVE ROUTINE SEIZURE PROPHYLAXIS, start seizure maintenance therapy after the first event, exclude potential precipitating causes

-emergency cardiac catheterisation in comatose pts with ROSC after OHCA of suspected cardiac origin

-evaluate pts for resus-related injuries

NOT seizure prophylaxis

865
Q

1 MAC of sevoflurane affects the sensory evoked potential signal for spinal surgery by

A

dose-dependent incr latency, increase conduction time & reduce amplitude

866
Q

A patient is anaesthetised from the awake state to a state of surgical anaesthesia with propofol or a volatile
anaesthetic. As the depth of anaesthesia increases, the patient’s electroencephalogram (EEG) will show
oscillations that are of

A

greater regularity, higher amplitude, lower frequency
alpha waves (spindles) of surgical anaesthesia disappear, dominated by delta & theta waves
burst suppression–> flat

867
Q

Created by the Global Initiative for Chronic Obstructive Lung Disease (GOLD 2017), the numerical GOLD
classes 1 to 4 are classes of severity for chronic obstructive pulmonary disease (COPD). These classes are
based on an assessment of the

while the alphabetical classes A-D are based on

A

severity of airflow limitation, categorises based on (in pts w FEV1/FVC <0.7), FEV1
>=80
>=50-80
>=30-50
<30

2x2 table:
exacerbations 0 or 1 (not leading to admission) A or B
>=1 leading to admission or >=2 exac C or D>
-mMRC dyspnoea score 0-1 & COPD assessment tool score <10 A or C, mMRC >=2 or CAT >=10 B or D

868
Q

Predictors of difficult sedation (agitation or inability to complete the procedure) of patients undergoing
gastroscopy do NOT include

A

Specifically wrt agitation etc, DOES include:
anxiety
chronic psychotropic use
(NOT ETOH abuse or chronic opiate/BDZ use)

Surgical: prolonged procedure >1hr, difficult cannulation, pancreatic duct dilation/contrast, biliary sphincterotomy, haemorrhage or duodenal perf
Pt: obesity, COPD, OSA, severe acute illness, ileus, ascites, ETOH excess, CVD, elderly, pancreatitis, higher ASA
Anaesthetic: bolus vs TCI

(from UTD general risks converstion MAC to GA: Hypoxia/airway obstruction, aspiration, unplanned surgical extension
Pt factors: inability to tolerate MAC, BMI >35, male, ASA IV
other: non-anaes)
thorough pre-op Ax crucial

869
Q

A patient presents for endoscopic retrograde cholangiopancreatography (ERCP) with a history of previous
post-ERCP pancreatitis. The management most likely to reduce the likelihood of pancreatitis is

A

Rectal NSAID (indomethacin, diclofenac and valdecoxib all worked – Meta analysis)

prophylactic stent (only experienced endoscopists)

Pharmacologic prophylaxis: rectally administered NSAID (indomethacin suppository 100mg or diclofenac 100mg immediately before ERCP)

minimising trauma to biliary orific (limit cannulation, guidewire-assisted cannulation, only manipulate pancreatic duct if essential)

870
Q

A 72-year-old patient is undergoing resection of an anterior skull based tumour using a combined endoscopic
and frontal craniotomy approach. Seven hours into the procedure she has a large diuresis of pale urine and
you suspect she may have developed diabetes insipidus. The most appropriate test result to confirm your
diagnosis in this setting is a

A

Urine osmolality low (urine SG therefore low; they don’t quite correlate as SG considers the weight of the solute (therefore number & size), while osmolality just the number of particles. SG generally increases with osmolality. It can be falsely elevated (eg. Glucose) but there are no causes of falsely low urine SG, SG <=1.003 indicates maximally dilute urine (<=100mOsmol/kg)
serum sodium would be high (as would serum osmolality), in contrast. toC SW where hyponatraemia & low plasma osmolality

Impractical:
can measure the urine osmolality before&after desmopressin (with central DI it’ll rise, urine osmolality staying low= nephrogenic DI)
Plasma vasopressin (low) is infrequently available

871
Q

The amount of fresh frozen plasma that needs to be administered (in mL/kg) to increase plasma fibrinogen
levels by 1 g/L is

A

> 30mL/kg (so risk volume overload, TRALI & MOF)

cf fib conc which gives 1g/50mL

872
Q

Unsupported ventilation in a non-anaesthetised patient with long-standing tetraplegia is improved when

A

supine vs upright (diaphragm positioning more optimal)
abdominal binders worn when sitting (no benefit when supine)

loss. ofintercostals, lacf abdo tone. tocreate fulcrum for chest expansion w inspn, diaphragm flattened (lost mechanical advantage), w inspiration & dipahragm contraction chest wall sucked in (paradoxical chest wall movement).
supine FVC & FEV1 are larger supine cf seated, down to injury level of T1.
w time, spasticity develops so chest wall more rigid, less likely to collapse.

above C3: ventilator dependent
C3-5 depends on effect on diaphragm & accessory mm
C6-8: may require intermittent NIV. accessory neck mm. &intact diaphragm help. exhale passive, cough impaired.
thoracic: inefficient cough otherwise little resp compromise.
Cx spine lesions develop restrictive vent deficit.

873
Q

Drug classes demonstrated to reduce mortality in chronic heart failure with reduced ejection fraction include
all of the following EXCEPT

A

DIGOXIN

Loop diuretic (for symptoms but not mortality benefit)

Drugs that DO reduce M&M in HFrEF:

ARNi (degrades neprolysin, which breaks down ANP & BNP, both agents which reduce blood volume. neprolysin also degrades bradykinin).
ACE-I
ARB
Cardiospecific B blockers: bisoprolol, carvedilol, SR metoprolol
MRA (spironolactone or eplerenone), provided eGFR>30 & K<5
hydralazine + nitrate combo
SGLT2i reduce hospitalisation for HF & reduce cardiovascular mortality, irrespective of T2DM presence
omega 3 polyunsaturated fatty acid supplement mortality benefit

Initial pharmacologic therapy for HFrEF= combo diuretics, RAS inhibitor (ARNI, ACE-I or ARB) AND B-blocker

Secondary= MRA & SGLT2-I
in preg, hydralazine + nitrate= vasodilator therapy of choice, since ACE-I & ARB & sacubitril-valsartan contraindicated.

Thiazolinediones & DDP-4 inhibitors & NSAIDs worsen HF symptoms

874
Q

The use of intraoperative dexamethasone for tonsillectomy

A

REDUCES TIME TO FIRST ORAL INTAKE.
Single dose IV intra-op dexamethasone:
-Decreases N&V in first 24hrs post
-Decr time to first oral intake
-decr post pain
may incr risk postop bleeding but degree of risk unknown, likely low, reduces PONV (single intraop dose in chn half as likely to vomit), reduces swelling in oropharynx or soft palate

875
Q

When using an endotracheal tube in an adult, the highest recommended cuff pressure to avoid mucosal
ischaemia is

A

30cmH2O (20-30cmH2O) cuff pressure for mucosal ischaemia is lower. in shocked patients)

876
Q

A 55-year-old man with no past history of ischaemic heart disease is three days post-total hip replacement
surgery. He has an episode of chest pain that sounds ischaemic, began at rest and lasts 30 minutes before
resolving fully. There are no ECG changes nor troponin rise. The diagnosis is

A

UA

877
Q

According to the ANZICS Statement on Death and Organ Donation 2021, circulatory determination of death in
the context of organ donation requires the absence of evidence of circulation for at least

A

5 mins

878
Q

six-year-old child weighing 20 kg presents to hospital two hours after sustaining a burn to 25% of her body.
Appropriate fluid management should include 1000 mL Hartmann’s solution in the next

A

6hrs

879
Q

Dabigatran differs from rivaroxaban and apixaban because it inhibits

A

thrombin

880
Q

The recommended antibiotic prophylaxis for surgical termination of pregnancy is

A

doxycycline 400mg w food 10-12hrs before OR doxycycline 100mg orally 60mins prior to procedure then 200mg 90mins after.

881
Q

A patient has severe hypokalaemia and is in cardiac arrest. The Australian Resuscitation Council
and the New Zealand Resuscitation Council recommend intravenous potassium should be given
as

A

5mmol bolus KCl IV

882
Q

75-year-old man has a loud ejection systolic murmur detected on clinical examination before a
joint replacement. A focused transthoracic echocardiogram (TTE) detects a calcified aortic valve
with a peak aortic jet velocity of 3 m/s. The peak gradient across the aortic valve is

A

36mmHg

883
Q

The current ANZCA guidelines for preoperative fasting of adult patients state that studies have
shown that it is safe to administer

A

up to 400mL clear fluids 2 hrs prior to OT, limited solid food up to 6hrs

prescribed meds with a sip of H2O within 2hrs prior to anaesthesia

884
Q

A two-year-old boy with a history of respiratory tract infection one week previously has just
undergone squint surgery. His airway was managed with a size 4.5 mm cuffed endotracheal tube.
The surgery was unremarkable. Twenty minutes after extubation he is awake and appears
anxious, with stridor and a visible tracheal tug. His oxygen saturation is 96% on room air. The best
initial management of this child is to administer

A

Adr neb over 5-10mins (OR 0.5mL/kg 1:1000, max 5mL)
analgesia
cool humidified mist (postextubation stridor; mild cases where stridor only)

prophylaxis IV dex 0.15mg/kg (if croup stridor, 0.6mg/kg)

885
Q

postextubation stridor risks:

A

tightly fitting ETT, leak pressure >25cmH2O
chn <4yo (smaller airway lumen)
multiple intubation attempts
longer intubation
head & neck srugery (frequent position changes)
URTI, recent croup
coughing. onextubation
subglottic stenosis

prophylaxis:
IV dex prior to extubation (0.15mg/kg)
sedation. &pain control to prevent crying/agitation
cool humidified mist if stridor mild only
racemic epinephrine moderate

886
Q

You place a paravertebral catheter for postoperative analgesia at the level of T5 in an adult patient
prior to a thoracotomy. Two minutes following the injection of 0.75% ropivacaine 10 mL, the patient
becomes bradycardic, hypotensive and apnoeic. The most likely cause of the complication is

A

intrathecal; high spinal (total spinal=LOC)

887
Q

A 54-year-old woman has a laryngeal mask airway inserted for a surgical procedure. The following
day she complains of tongue numbness and abnormal taste over the posterior third of the tongue.
The most likely site of the nerve injury is the

A

Glossopharyngeal nerve IX

888
Q

The washing process of modern cell savers for intraoperative blood salvage removes all the following EXCEPT

A

just left with red cells suspended in saline, FOETAL RED CELLS are left
Removes: free Hb, plts, plasma, microaggregates, anticoagulants
thrombogenic products (FDPs), complement proteins, leucocyte enzymes, SOME inflammatory mediators

889
Q

A patient with a haemopneumothorax has a chest drain in situ, which is attached to a three-bottle
underwater seal drain apparatus. The system is attached to wall suction at -80 cmH20. This will
cause

A

risks RPE
lack of oscillation suggests lung re-expanded
usual initial suction -10- -20cmH2O, adults & paeds
spont air leaks, use the minimum suction for re-expansion; initially water seal only (no suction); add suction -10cmH2O if incomplete PTx resolution, incr only as needed, increase if persistent air leak.
if the tube is for fluid drainage, -20cmH2O good starting point, incr as indicated to aim for full lung expansion. LARGE PDIFFERENTIAL PRESSURE GRADIENT SHOULD BE AVOIDED DURING LUNG EXPANSION TO PREVENT RE_EXPANSION PULMONARY OEDEMA.. If large effusion, initially no suction, this may decr risk RPOEDEMA..
Traumatic PTx: immediate 20mL/kg (or 1500mL or >3mL/kg/hr indicates thoracotomy. rapid fluid removal w large effusions ass’d w RPE.
if no air leak, clamp tube if pt gets cough/chest pain/dyspnoea or O2 desat (limit risk RPE). wait for symptom resolution before resuming drainage.
Risk of RPO only when the lung starts to inflate, so can drain larger amounts if mediastinal shift CL to the effusion.
excessive suction risks “air steal” & hypoxaemia, perpetuates leaks, damage lung if trapped in catheter

890
Q

You inadvertently place a 7.5Fr central venous catheter into the carotid artery of a patient
undergoing an emergency laparotomy for peritonitis. The best course of management is to

A

> =7Fr OR vessel dilated w sheath: leave in place, safe removal urgently discussed w vascular surgeon. Delay non-urgent surgery unless risk cancelling > risk proceeding. If proceed, leave needle/sheath in situ until end of operation (consider heparinisation if poss). Urgent vascular & radiological consult w immediate imaging to Ax injured site.

Leaving arterial catheter in place w prompt repair carries less M&M than catheter removal w pressure (ass’d w major stroke, death).

Carotid generally direct exploration/catheter removal/arterial repair, while sites not easily accessible surgically (subclavian) require endovascular (stent, vascular closure device, tract embolisation).
Should NOT permit prolonged arterial cannulation; if immediate Rx not possible, consider heparinisation.

Zone III (from 1cm above clavicle & below), require sternotomy or endovascular repair. Zone II (from 1cm above clavicle to angle of mandible) can be repaired w direct pressure/open cutdown/endovascular. Zone III angle of mandible to BoS.
Remove small (22-25g) needles from carotid artery, apply pressure 5-10mins to prevent haemorrhagic complications, clinical Ax of bleeding/haematoma. Image to exclude sig complicaitons. Neuromonitor 24hrs postop (embolic stroke a risk if significant carotid atherosclerotic disease). For pts w Hx carotid atherosclerotic disease, >1 puncture or significant haematoma, could postpone elective surgery w 24hr neurol follow-up.

Our quad lumen is 8.5Fr, 20cm.

891
Q

A patient in atrial fibrillation with a CHA2DS2-VASc score of 2 has presented for elective hip
surgery. Warfarin had been ceased for four days preoperatively and on the day before surgery the
international normalized ratio (INR) was 2.1. The best course of action at this point is to

A

postpone ELECTIVE surgery until cause established (discuss w Haematology) & corrected, w PO vit K 1-2mg PO.
other options (day of surgery) PTx (15-30IU/kg (30 in this case (INR 1.5-2.5, target <1.4) 50 if life-threatening bleed)- prothrombotic risk, 4F contains heparin) or FFP (150-300mL if given w PCC for life-threatening bleed, 15mL/kg if PCC n/a) or (if day before surg, vit K 1-2mg PO only generally sufficient).

892
Q

An anaesthetised patient is ventilated and has standard monitoring plus a central venous line. As
surgery commences, the line isolation monitor alarms, indicating a potential leakage current of
greater than 5 mA from one of the power circuits in use. The most appropriate action is to

A

Turn off the last piece of equipment to be plugged in- that’s usually the culprit of the fault in the system

893
Q

In the World Maternal Antifibrinolytic (WOMAN) trial, tranexamic acid administration within three
hours of birth reduced the

A

Death due to bleeding
give within 3hrs of birth

894
Q

The dose of hydrocortisone that has equivalent glucocorticoid effect to 8 mg dexamethasone is

A

200mg

895
Q

Preperitoneal pelvic packing is a surgical treatment of haemorrhage from a/an

A

haemodynamically unstable pelvic fracture.
Place pads in retroperitoneal space.
Leave. insitu 24-48hrs.
Controls VENOUS & bony source of bleeding (unlikely to control arterial haemorrhage)
can use concurrent. w ex fix.

896
Q

The gauge pressure on a gas cylinder does NOT necessarily represent the contents remaining if
the cylinder is filled with

A

Nitrous oxide

Sits below it’s critical temp (36degC) existing as vapour in equilibrium w liquid phase

897
Q

Maintaining a CO2 pneumoperitoneum at a pressure of 15mmHg is most likely to lead to

A

Reduced venous return, reduced cardiac output

The initial incr IAP from 5mmHg first causes incr VR & CO
As IAP rises (10-20mmHg), VR & CO decrease. Incr SVR means that @ IAP 10-20mmHg, BP ISQ or incr.
From >20mmHg, VR, CO & BP all decr

898
Q

gel supraglottic airway device recommend a

minimum patient weight

of

A

2kg

Size selection for I-Gel
* Size 1.0 = 2 - 5 kg
* Size 1.5 = 5 - 12 kg
* Size 2.0 = 10 - 25 kg
* Size 2.5 = 25 - 35 kg
* Size 3.0 = 30 - 60 kg
* Size 4.0 = 50 - 90 kg
* Size 5.0 = > 90 kg

899
Q

gel supraglottic airway device recommend a

minimum patient weight

of

A

2kg

Size selection for I-Gel
* Size 1.0 = 2 - 5 kg
* Size 1.5 = 5 - 12 kg
* Size 2.0 = 10 - 25 kg
* Size 2.5 = 25 - 35 kg
* Size 3.0 = 30 - 60 kg
* Size 4.0 = 50 - 90 kg
* Size 5.0 = > 90 kg

900
Q

A 60-year-old woman presents for thrombectomy with left lower leg ischaemia. She has not
received any medications since presentation, and takes none at home. The sole abnormality on
laboratory testing is an activated partial thromboplastin time (APTT) of 52 seconds. The most likely
cause of the raised APTT is

A

Antiphospholipid syndrome (lupus anticoagulant)
Pts have venous/arterial thrombosis +/- adverse pregnancy outcome, persistent aPL antibodies. Can be primary or with other condition eg. SLE, RA, systemic sclerosis, Bechet’s.
Pt often presents w livedo reticular, digital ischaemia/gangrene, sequelae of DVT, heart murmur, neurological abnorms suggesting prev stroke

Other DDx= heparin exposure, haemophilia A & B, factor XII or XI def, artefact

901
Q

In adults the spinal cord usually extends from the brainstem to the level of the inferior margin of the

A

L1 vertebral body

902
Q

A risk factor for the development of torsade de pointes is

A

Hypomagnesemia, hypokalaemia, hypocalcaemia, Brady
Meds prolonging QT: methadone, droperidol, amiodarone + dig, TCA overdose
Pt factors: congenital long QT syndrome, female, renal or liver failure

903
Q

The EXTEM plot from a ROTEM sample is shown. The most appropriate treatment for this

bleeding patient is

A

EXTEM A5 <=35, platelets
EXTEM CT >=90, factors (FFP)

Test tube shape: thrombocytopenia, give platelets
Inverted martini: fibrinolysis (TxA)
Wine glass: factor deficiency (give FFP)
Champagne flute: fibrinogen deficiency (give cryo/fibrinogen)
Brandy snifter= normal

Extem= TF + CaCL2 + polybrene, extrinsic pathway
Fibtem= Extem + cytochalasin D, inhibits plt activity, analysis of fibrinogen activity
Aptem= Extem + aprotinin, excludes hyperfibrinolysis
Intem= CaCl2 + ellagic acid, intrinsic pathway, APTT equivalent

904
Q

Following the initial subarachnoid haemorrhage from a ruptured aneurysm, the patient is at

greatest risk of rebleeding during the following

A

6hrs (UTD: maximal risk in first 2-12hrs), 5-10% rebleed within the first 72hrs

905
Q

The sensory innervation to the larynx above the vocal cords is provided by the

A

Internal branch superior laryngeal nerve

906
Q

The

oral morphine equivalent of tapentadol 50 mg (immediate release) is

A

15mg

907
Q

Jet ventilation for shared airway surgery is traditionally delivered at pressures in atmospheres of

A

0-4atm

908
Q

In comparison with fresh frozen plasma, cryoprecipitate contains an increased concentration of factor

A

1 (fibrinogen; approx 15mg/mL in cry vs 3mg/mL in FFP) & VIII

1 unit of cryo (20-30mL) has
150->300mg fibrinogen
>80IU factor VIII
50-75 units factor XIII
100-150U vWF
fibronectin

1 unit FFP (200-300mL)
Has 1.14IU/mL factor VIIIc (ie < cry)
600mg fibrinogen

909
Q

year
-
old boy is in

refractory ventricular fibrillation.
The recommended dose of amiodarone is

A

5mg/kg (80mg using (age+4)x2)

910
Q

year
-
old child with severe life
-
threatening anaphylaxis and no intravenous access, the

recommended initial dose of intramuscular adrenaline is

A

150microg (0.15mL of 1:1000)
* Age > 12 years = 500 microgs (0.5 mL of 1:1000 Adrenaline)
* Age 6-12 years = 300 microgs (0.30 mL of 1:1000 Adrenaline)
* Age < 6 years = 150 microgs (0.15 mL of 1:1000 Adrenaline)
* Age < 6 months = 0.01 mg/kg or 10 microgs/kg (0.01 mL/kg of 1:1000 Adrenaline)

911
Q

Abnormal Q waves are NOT a feature of the electrocardiogram in

A

Pathological Q waves ARE a feature in hypertrophic > dilated cardiomyopathy
STEMI (they are a sign of myocardial necrosis, may take minimum 2hrs (sometimes up to 24hrs) to appear.
LV aneurysm
Pathological Q waves are >1mm wide, >2mm deep, >25% depth QRS, seen in V1-3
They are NOT a feature of dig tox (freq VEBs, high degree AV block, atrial tachycardia); cf digoxin effect (“sagging” st depression & TWI (reverse tick), PR pool, U waves, J point depression)

912
Q

The fourth position of the international pacemaker (NBG) code represent
s the

A

presence of rate modulation (O=none, R= rate modulation present)

913
Q

A 30-year-old parturient presents in labour. She has a history of Addison’s disease fromautoimmune adrenalitis and has been taking prednisolone 6 mg daily for ten years. Onpresentation the patient is given hydrocortisone 100 mg intravenously. The most appropriate steroid replacement regimen the patient should receive during labour is

A

200mg IVI over 24hrs

914
Q

When using cardioversion to revert a patient in atrial fibrillation to sinus rhythm, the direct current shock is synchronised with the ECG to coincide with the

A

R wave, to avoid R on T phenomenon

915
Q

A woman with atrial fibrillation has no valvular heart disease. According to AHA guidelines, oral anticoagulants are definitely recommended if her CHA2DS2-VASc score is greater than or equal to

A

3
2 for males

For females scoring 2 & males scoring 1, decision for OAC depends on non-gender risk factor; if it is age 65-74, use chronic OAC.

916
Q

PDPH can be ass’d w greater likelihood of:

A

Cranial nerve (abducens) palsy
Cortical vein thrombosis
Cerebral venous sinus thrombosis
Cranial-subdural haematoma
Seizures
Stroke
persistent headaches, chronic LBP, depression, bacterial meningitis
NOT encephalitis or Sheehan syndrome

917
Q

Of the following, the drug most likely to cause pulmonary arterial vasodilation with systemic arterial vasoconstriction when used in low doses is

A

vasopressin

918
Q

When fresh frozen plasma is administered to treat hypofibrinogenaemia in a bleeding patient, the volume required to achieve an increase in plasma fibrinogen concentration of one gram per litre is

A

> 30mL/kg

919
Q

You review a patient before major bowel surgery. Using the American Heart Association/American College of Cardiology consensus guidelines, you assess him as being at intermediate risk of a perioperative adverse cardiac event. When explaining this to the patient, this best translates to a numerical risk in the range of

A

1-5% (intraperitoneal, intrathoracic, CEA, head & neck, ortho, prostate)

High risk is >5% (aortic & major vascular, peripheral vascular)
Low risk <1% (endoscopy, superficial, cataract, breast, ambulatory)

920
Q

IABP inflates at… deflates at…

A

middle of T wave
peak of the R wave

921
Q

The most clinically useful indicator of effective ventilation during neonatal resuscitation is an
improvement in

A

HR >100

922
Q

In the awake term neonate the systolic arterial blood pressure is normally approximately

A

80 (GA x2)

923
Q

Predictors of successful awake extubation after volatile anaesthesia in infants do NOT include

A

DOES include:
Eye opening
Conjugate gaze
Purposeful movement
Vt >5mL/kg
Facial grimace

Movement other than cough/purposeful movement
RR 16/min
FiO2 0.35, SpO2 >97%
Etsevo <0.2, des <1%
Positive laryngeal stimulation test (return of sport vent <5s after gentle stimulation of glottis by cephalic, caudal movement of ETT in pt spont venting @ emergence)

924
Q

A man underwent a heart transplant 12 months ago. A drug or therapy which is likely to result in an
exaggerated effect in him is

A

AVN blocking drugs (esp adenosine, verapamil)
sympathomimetics (direct-acting; Adr, dobutaime, increased adrenoceptor density)

925
Q

A 30-year-old woman has had a free flap operation of eight hours duration. She received an
intraoperative remifentanil infusion and was given 10 mg morphine 30 minutes before the end of
the operation. In recovery her pain score has increased from 6/10 on arrival in recovery to 9/10 in
spite of a further 10 mg of intravenous morphine. The most likely diagnosis is

A

opioid induced hyperalgesia

926
Q

Propofol infusion syndrome is characterised by all of the following EXCEPT

A

Splenomegaly or metabolic alkalosis

Is characterised by:
Cardiac: heart failure, pulmonary oedema, widened QRS, bradycardia, VT, VF, systole, brugada-like ecg changes (coved STE V1-3)
Vascular: hypotension
MSK: rhabdo (elev CK)
Metabolic: acidosis (lactic, due to impairment normal oxidative phosphorylation), hyperkalemia, lipidaemia
hepatomegaly & elevated liver enzymes
Renal failure

Risk >4mg/kg/hr for >-48hrs

927
Q

Of the following, the drug with the LEAST effect on serum potassium is

A

calcium chloride

Drugs that DO impact:
insulin
salbutamol
bicarb (only in setting of acidaemia)

spironolactone
suxamethonium
magnesium
frusemide

928
Q

The underlying trigger for the development of acute traumatic coagulopathy is

A

tissue injury & hypoperfusion–> protein C pathway

929
Q

Differential hypoxia is a syndrome characterised by lower arterial oxygen saturation in the upper
body. It is a complication specific to the use o

A

Peripheral V-A ECMO.
pts receiving peripheral VA-ECMO are dependent on retrograde flow to delivery oxygenated blood to the upper body. If they’ve got concomitant resp failure, risk poorly oxygenated blood preferentially being delivered to the myocardium & brain, risking hypoxic injury.

*should avoid peripheral V-A ecmo in pts receiving ECMO for resp failure. Should measure SO2 in both hands of pts receiving peripheral V-A ECMO (the phenomenon is developing if lower SO2 readings in R) hand cf L).

930
Q

Under the NEXUS criteria, requirements to clear the cervical spine of trauma patients without
radiographic imaging include all of the following EXCEPT

A

NEXUS: for pts <60yrs if satisfy all of the 5:
-Ax for absence of intoxication
-Ax that the pt is alert & oriented
-Ax no painful distracting injury
-Absence of midline tenderness
-Absence of focal neurology

931
Q

A 54-year-old woman had a laryngeal mask airway inserted during anaesthesia. The next day she
reports hoarseness. On indirect laryngoscopy the right vocal cord is in a paramedian position and
is lower than the left vocal cord. The most likely site of the nerve injury is the right

A

recurrent laryngeal nerve

932
Q

A patient undergoing robotic prostatectomy with volume-controlled ventilation has the following ventilatory measurements (Vt 600mL, PIP 36, Pplat 32, PEEP 8, autoPEEP 4). The static compliance is

A

Compliance = 600 / (32 - 12) = 30

933
Q

When compared to the interscalene block, the supraclavicular block has the advantage that

A

Less ulnar sparing
less phrenic nerve block

934
Q

A ten-year-old child (weight 30 kg) presents to the emergency department in status epilepticus. He
has received one dose of 15 mg midazolam buccally prior to his arrival. According to Advanced
Paediatric Life Support Australia (APLS) guidelines the next drug treatment should be intravenous

A

APLS states if no vascular access, IM midaz 0.15mg/kg OR buccal/intranasal 0.3mg/kg (max 10mg).
Then IV or IO 0.15mg/kg
while preparing levetiracetam 40mg/kg or phenytoin 20mg/kg

5mins: midaz 0.15mg/kg IM or buccal/IN 0.3mg/kg (both max 10mg)
at 10 mins, IV/IO midaz 0.15mg/kg
prepare keppra or phenytoin
15mins keppra 40mg/kg max 3g over 5mins or phenytoin 20mg/kg over 20 mins
20 mins give the other or phenobarbitone (20mg/kg max 2g paeds)
25mins RSI

935
Q

In long-term use of nonsteroidal anti-inflammatory drugs, the risk of thromboembolic complications
is lowest with

A

aspirin prevents, naproxen low risk

NSAID’s and haematological complications

COX-1 (anti-platelet function)

Constitutional

Homeostatic

Inhibition of platelet function

COX-2

Inducible

Pain, Inflammation, Fever, Vascular Permeability, inhibit prostacyclin (which inhibit plt activation) & stimulate thromboxane which promotes plt aggregation.

Descending order from COX-1 > COX-2 selectivity

Ketorolac = COX-1 selective

Aspirin = COX-1 selective

Naproxen = COX-1 selective

Ibuprofen = COX-1 selective

Diclofenac = COX-2 selective

Celecoxib = COX-2 selective

Meloxicam = COX-2 selective

Valdecoxib = COX-2 selective

936
Q

A 74-year-old man presents for a femoral popliteal artery bypass procedure for peripheral limb
ischaemia. Regarding its role in modifying his perioperative cardiovascular risk, clonidine

A

Administration of low-dose clonidine (or aspirin) in patients undergoing noncardiac surgery did not reduce the rate of the composite outcome of death or nonfatal myocardial infarction; it did, however, increase the risk of clinically important hypotension and nonfatal cardiac arrest, aspirin incr risk bleeding

937
Q

Aspirin POISE II

A

Administration of aspirin before surgery and throughout the early postsurgical period had no significant effect on the rate of a composite of death or nonfatal myocardial infarction but increased the risk of major bleeding.

938
Q

A 45-year-old man presents on the day of surgery for an elective inguinal hernia repair. He is well
but is noted to be mildly jaundiced. He takes simvastatin for hyperlipidaemia and has no other
medical history. He consumes about three standard drinks of alcohol per day and does not
smoke. He briefly experimented with illicit drugs more than ten years ago. His laboratory results
show: (supplied) The most likely diagnosis is
Show elevated bilirubin, normal conj bili, other LFTs normal incl albumin

A

Gilbert’s (inborn error of bili conjugation)

Autosomal recessive decr activity of uridine diphsophate glucuronosyltransferase

Bilirubin levels incr w stress/dehydr/fasting/infection

939
Q

The most common cause of maternal mortality in women with preeclampsia is

A

ICH OR eclamptic seizures (which may –> haemorrhage & cardiac arrest)

940
Q

Ehlers Danlos IS ass’d with

A

6 major subtypes, range from minor to life-threatening, mutations related to collagen proteins, AD or AR, features incl:

chronic pain

Ophthalmic: risk retinal detachment, globe rupture

Fragile skin
Hypermobile/unstable joints (C-spine, TMJ)
risk premature spondylosis
tissue fragility subtypes risk dural rupture & headache similar to PDPH. Some have Tarlov cysts (CSF-filled perineural cysts).

some may have muscle weakness
Atrophic scars, easy bruising

PTx risk (vascular EDS)

scoliosis

Cardiac involvement (MR, MVP, AR)
Possible POTS (esp w hypermobile variant)

Vascular EDS may have arterial rupture–> X-match for vascular EDS or those w positive bleeding Hx or for surg factors
most have some abnormality of plt aggregation (lab tests often normal)

risk organ rupture.

LA resistance, resistance to opioids has been reported.

NOT intellectual disability

Little evidence-based knowledge to guide Mx.

check for bleeding Hx
screen multisystem complications
check airway difficulties (spondylosis, A-O instability, TMJ subluxation, mucosal fragility, scoliosis)

facility

consider discussion of open vs laparoscopy in vascular EDS
Potential difficult airway if C spine instability, spondylosis or kyphosis, TMJ laxity
consider pre-load w IVT & early vasopressors if POTS
gentle +++ airway instrumentation, consider smaller ETT & regular cuff pressure checks (risk mucosal damage)
Minimise airway pressure (risk PTx)

ideally AVOID tourniquets- risk massive compartment bleeding & haemorrhagic shock
Use non-invasive monitoring where possible (care w repeated NIBP). in vascular EDS avoid invasive wherever possible; use US. If need CVC, IJ pref over subclavian, limit use dilators, US mandatory.

care w taping (shear forces, fragile skin)
positioning/padding pressure points, protect eyes, avoid brachial plexus traction.

blood strategy
CONSIDER DDAVP (incr vWF & VIII levels)
Consider TxA
Cell salvage for pt/surg risk factors

consider difficult/failed neuraxial risk (scoliosis, prev spine surgery, ineffectiveness of LAs. SOME CONSIDER NEURAXIAL CONTRAINDICATED IN VASCULAR EDS (risk haematoma/nerve injury if tissue fragility); individualise. If considering, pre-intervential MRI to help rule out spine pathology (eg. Tarlov cysts).
Peripheral blocks; US mandatory (risk vascular puncture) ?consider avoiding if vascular EDS. Tissue scarring may impact the spread of LA. May impart LA resistance (eg. EMLA).

Risk uterine rupture, delayed wound healing any mode delivery.
anticipate severe bleeding. X-match, cell saver, DDAVP prophylaxis. Consider elective LSCS in high risk EDS subtypes.
vascular subtype if in labour (?), consider PCA vs neuraxial. care w nitrous as spon PTx risk in some pts. If neuraxial, discuss extra PDPH (expert opinion), haematomoa risk.

Postop early mobilisation limit risk of immobility, thorough monitoring bleeding site haematoma risk. monitor pts w POTs for CV instability. PONV prophylaxis (risks spont oes rupture).

Staff looking after the pt to be aware of potential complications relevant to the pts subtype (compartment syndromes, vascular rupture, PTx, organ rupture; most common vascular EDS). Low threshold for Ix.

941
Q

The effect of a drop in patient core temperature from 37 C to 34 C is to

A

decr CMRO2 21degrees
slow cardiac conduction, risk brady, arrhythmias, reduced CO, decr SVR, circulatory collapse
pulm oedema, L)-shift OHDC, slow/shallow/irrecgular breathing
hyperkalaemia, acidosis
coagulopathy, thrombocytopaenia

MILD hypothermia:
hypERglycaemia
Incr SVR & sig reduction cardiac index so MAP only decreased slightly
Incr platelet adhesion

942
Q

According to the Third International Consensus Definitions for Sepsis and Septic Shock (Sepsis-3),
sepsis is defined as

A

A life-threatening organ dysfunction caused by a dysregulated host response to infection

943
Q

According to the 6th National Audit Project, the likelihood that a patient who reports an allergy to
penicillin has a true allergy is approximately

A

10%

944
Q

A 70-year-old man undergoes a stress echocardiogram as part of his preoperative preparation
before a total hip replacement. If he has clinically significant coronary artery disease, the earliest
indicator during his test is most likely to be

A

Regional wall motion abnormalities (endocardial hypokinesia, akinesia or diskinesia)on echo

(flow heterogeneity–> metabolic changes–> altered mechanical function–> then ecg & pain. ie. regional malperfusion is the most sensitive clinical marker of ischaemia, chest pain the least).

945
Q

Regarding the Australian and New Zealand categorisation system for prescribing medicines in
pregnancy, Category C medicines are ones which

A

are ass’d w incr harm. to foetus which may be reversible, no evidence congen malformations

Category A= taken by large number of preg & women of child-bearing age, no evidence direct or indirect harm to foetus
B1= limited number. no evidence harm. animal studies not demonstrated incr risk harm.
B2= limited number. no evidence harm. animal studies inadequate/lacking.
B3= limited number. no evidence harm. animal studies incr occurrence foetal harm.
C= ass’d w incr harm which may be reversible. no evidence congen.
D= incr risk foetal malformations or irreversible damage, may also have adverse pharmacological effects.
X= contraindicated. High risk permanent damage.

946
Q

An adult male patient has a haemoglobin level of 80 g/L and his blood film shows a reticulocyte
count of 10%. These findings are compatible with

A

accelerated haematopoiesis (with active bone marrow), eg. from:
-HAEMOLYSIS (destruction, also see decr haptoglobin, incr LDH, incr bilirubin, incr RDW) which may be:
-immune (transfusion reaction, RhO)
-toxins (clostridial)
-thrombotic microangiopathy (haemolytic anaemia)
-traumatic (direct, shear stress, heat damage)
-G6PD (XLR, where haemolytic anaemia occurs in response to infections, stressors, hypothermia, acidosis, fava beans, aspirin (exposure to oxidative drugs (avoid drugs causing oxidative stress & those that can induce metHb)) can–> haemolytic crisis)–> fever, dark urine, abdo/back pain, pallor). Avoid hypothermia, manage infection, limit surgical stress, avoid acidosis, avoid methylene blue. Prepare for blood loss (haemolysis risk). first on list, limit stress of fasting, aggressive glycaemic & temperature control. Generous analgesia. The pts may take vit E or folic acid
-other haemolytic anaemias (sickle cell, thalassaemia)
-DIC
DDx for raised retics= blood loss

normal retics 0.5-2%

Other DDx of reticolocytosis (if no haemolysis)
recovery from episode of bleeding (or ongoing bleeding),
repletion of Fe, B12, folate, EPO, recovery from bone marrow insult (eg. infection, parvovirus), medicaiton or ETOH

Unsafe drugs. forG6PD:
acetazolamide
co-trimoxazole
dapsone
diclofenac
diazepam
genatmicin
lignocaine
methylene blue
maxalon
nitrofurantoin
nitroprusside
penicillin
prilocaine
quinolones (eg. cipro)
rasburicaise
sulfonamides
toludine blue
phenazopyridine

947
Q

A straight laryngoscope blade is likely to be more useful than the Macintosh blade when
performing direct laryngoscopy in patients with all of the following EXCEPT

A

Straight useful for:
-macroglossia
-large front incisors/irregular teeth
-large floppy epiglottis
-useful for pathologies where don’t want to use much force or neck extension (mac blade requires maximum neck extension)
-larynx fixed from scar/trauma/mass effect
-short thick neck or morbid obesity

NOT post column

Macintosh 90 deg
Kessel 110deg (often used w short handle)
McCoy moveable distal tip
Magill straight blade U cross section
Miller straight w curved tip

948
Q

Prolonged paralysis associated with mivacurium is most appropriately managed with

A

Time

Can give FFP however risk > ben when administering blood product

949
Q

The mechanism of action of tranexamic acid is to inhibit the formation of

A

inhibits plasminogen–> plasmin hence fibrinogen–> fibrin

950
Q

Local anaesthetics may exacerbate symptoms in patients with

A

multiple sclerosis (denervation)

951
Q

According to the international consensus statement on uterotonic agents during caesarean section
published in 2019, the suggested initial bolus dose of oxytocin to be administered after delivery of
the fetus during an elective caesarean section is

A

1IU, 2.5-7.5IU/hr infusion (if required after 2 mins, further 3IU bolus over >30s)

Intrapartum caeser: 3IU oxytocin over >=30s, infusion 7.5-15IU/hr

alt is carbetocin 100microg over >=30s

952
Q

A 45-year-old woman is being assessed for liver transplantation. In order to determine the severity
of her liver disease the Model for End-stage Liver Disease score is derived using the international
normalised ratio, serum bilirubin and

A

Creatinine

MELD Na also uses sodium. If the pt ahs been dialysed twice within preceding 7 days, value for Cr becomes 400.

953
Q

A risk factor for postoperative nausea and vomiting in adults is age less than

A

50

954
Q

A patient has a known IgE-mediated allergy to penicillin. The cephalosporin with the lowest risk of
allergic cross-reactivity is

A

completely dissimilar side chains:
ceftazidime (3rd gen)
`cefazolin
cefuroxime
cefixime
cefmetazole
LOWEST risk 3rd gen cephalosporins
(cephalexin, ceftriaxone, ceflacor possible)

955
Q

The abnormality shown in this image (image of shoulder shown) is LEAST likely to be caused by
an injury to the

A

thoracodorsal (supplies lat dorsi actions on GHJ vs scapula)
serratus anterior IS long thoracic nerve
trapezius (XI- spinal accessory nerve). rhomboids (dorsal scapular nerve)
levator scap (C3-4 & dorsal scapular nerve (C5))

956
Q

Complex regional pain syndrome is NOT characterised by

A

CRPS= debilitating condition, most often in limb followlin injury/trauma/surgery, 0.5-2% post-traum incidence.

Dx criteria= regional pain, disproportionalte to initial trauma, skin colour & temp changes, oedema, vasomotor & sudomotor changes, motor dysfunction & trophic changes.
CRPS1 (prev RSD)= sans obvious nerve injury
CRPSII (prev “causalgia”) with confirmed nerve lesion.

acute phase: inflammatory dominance.
OR cold phenotype.
pain restricted to the affected limb hyperalgesia.

May transition to chronic phase.
inflammation diminishes, centrally-driven symptoms prevail (eg. sensory, motor symptoms & fear avoidance.
paediatric more readily treated with physical/rehab therapy

changes to areas distal to injury

IS characterised by:
pain: burning/stinging/tearing generally deep
hypoaesthesia/hyperalgesia/allodynia
arious types of sensory abnormalities are common in CRPS. Some patients have evidence of hyperalgesia, allodynia, or hypesthesia on examination . Sensory disturbances are usually distal in the limb, sometimes in a stocking/glove pattern
oedema
changes to areas distal to injury
skin changes (incl colour, sweating, trophic changes (hair growth, skin atrophy))
temp changes
functional motor impairment related to pain
preceding OT/injury

No clear unified pathological explanation, may involve cortical reorganisation. Latest thought= an immunoneurological disorder, with 4 components proposed: local tissue damage, pain processing, ANS/endocrine & immune (time course of homeostatic disturbance in that sequence).

painful condition, continuing (spont/evoked) regional pain (not in dermatome, may be stocking/glove-like distribution), seemingly disproportionate in time or degree to usual course of any known trauma or other lesion. regional (not nerve territory or dermatome), usually distal predominance. variable progression.
Type 1- no evidence peripheral nerve injury
Type II has peripheral nerve injury present.

Mx:
Limb desensitisation techniques have not had efficacy validated.
Opioids & NSAIDs no proven benefit, concern opioids worsening
NO biers (no evidence)
Calcitonin no evidence benefit

Early diagnosis & intervention improve prognosis, those progressing to chronic phase poorer prognosis. Psych symptoms in first uyea ass’d w worse outcomes if untreated.
Individualised.
Best evidence= early oral corticosteroids 6 wk course, single IV bisphosphonate infusion.
Absence of specific CRPS validated treatments so standard neuropathic pain Rx often used eg. Gabapentin, pregab, duloxetine (sari), tca, IV lignocaine & ketamine.
Multimodal: CBT. Limited evidence but reasonable.
MOTOR IMAGERY: documented benefit.
Sympathetic nerve blocks
Radio frequency lesioning of symptoms chain limited evidence efficacy in early CRPS.

inflammatory: CS
Regional osteopenia (acute CRPS1): bisphosphonates
psych: CBT
Pain/hyperalgesia/allodynia: alpha2delta ligands, TCAs
SNS-mediated pain: SNS blocks, ketamine
Chronic refractory: spinal cord or DRG stimulation
Central motor symptoms: botox, intrathecal baclofen

957
Q

endometrial ablation

A

Anaes:
ABx not routinely administered
LA/regional or GA.NSAIDs 1 hr preop to limit uterine contractions
Generally regional better tolerated

958
Q

The outer diameter of an Aintree Intubation Catheter is

A

6.5mm
ID 4.7
size 7=smallest ETT

56cm long

use over flexi bronch 4mm ID or less, eg. ambu slim 3.8mm which can fit ETT size 5, DLT size 37, ambu regular is 5 so too big, that scope fits 5.5 ETT or 39 DLT

ultrathin bronch 2.2mm (no suction channel)
storz 2.3mm fits 2.5ETT, 35Fr DLT
2.8mm fits ETT size 3, DLT 37Fr
3.there’s an adult olympus 4.1mm ED

959
Q

Extended life plasma is thawed fresh frozen plasma which can be stored at 2 to 6 C for a
maximum period of

A

5 days

960
Q

Red man syndrome as a consequence of vancomycin administration is caused by

A

mast cell degranulation (histamine release from mast cells & basophils; non immune-mediated)
influenced by dose of vancomycin infused & rate of infusion

961
Q

A 35-year-old patient is undergoing a diagnostic laparoscopy. Three minutes after insufflation of
CO2 her oxygen saturation falls to 85%. You note decreased air entry on the left side of her chest.
Lung ultrasound on the left reveals lung pulse and no lung sliding. The best first action is to

A

Pull back ETT 1-3cm (endobronch intubation following capnoperitoneum).

lung pulse= rhythmic movement of pleura in synchrony w cardiac rhythm, indicates opposition of parietal & visceral pleura.

962
Q

Bowel preparation prior to elective colorectal surgery is associated with

A

facilitates intraop colonoscopy

963
Q

The part of the lung that is typically divided into apical, anterior and posterior segments is the

A

R) UL
middle lobe has lat & medial

964
Q

The most common cause of bilateral blindness following spinal surgery and anaesthesia is

A

ION

965
Q

This image is an apical four chamber view obtained by transthoracic echocardiography. The artery
that supplies the area indicated by the arrow is the

A

LAD= apex, RCA= RV; all else is shared

966
Q

A patient has undergone a multilevel cervical spine fusion and upon emergence from anaesthesia
reports complete visual loss. Fundoscopic examination shows a pale optic disc with
haemorrhages. This supports a diagnosis of

A

Central retinal artery occlusion: retinal whitening, cherry red spot, oedema, segmental flow of retinal blood vessels (boxcarring).

Post ION= initially normal optidc disc & fundus, later pallor. RAPD.

967
Q

An asymptomatic 65-year-old male with squamous cell carcinoma of the left lung has been
referred for assessment of suitability for lung resection. There is no evidence of spread on
computerised tomography scanning. PaCO2, electrocardiogram, full blood count and electrolytes
are normal. His SpO2 on room air is 95%. His forced expiratory volume in one second is 2.3 litres
(predicted 3.3 litres) and forced vital capacity is 3.4 litres (predicted 4.4 litres). The most
appropriate course of action is to

A

proceed to lobectomy (fine if FEV1 > 1.5L) or pneumonectomy (fine. ifFEV1 >2L

(if it was < these thresholds, can have surgery if % ppo FEV1 or TLC was >40%

If not, proceed to exercise testing; crack on if VO2max >15mL/kg/min

If not, consider other options

968
Q

Findings associated with massive pericardial tamponade include

A

Beck’s triad: hypotension, raised JVP, muffled HS
tacchycardia
Pulsus paradoxus (10mmHg incr in SBP. w inspiration)
Kussmaul sign: elevated JVP during inspiration
Signs/symptoms obstructive shock
Ecg: low voltage, electrical alternans
CXR: enlarged globular heart
echo: pericardial effusion (large is >20mm), R) heart affected before L), collapse of cardiac chambers, IVC dilation, incr resp variation in intracardiac blood flow, excessive shift of IVS w spont ins, functional effects
dyspnoea, orthopnoea, chest discomfort, cool peripheries, low UO, diaphoresis, anx

969
Q

The first-line drug recommended by both the Australian Resuscitation Council and the New Zealand Resuscitation Council to treat severe cyanide poisoning is

A

Hydroxycobalamin 5mg over 15 minutes, repeat up to max dose 15mg

Cyanide is not a common form of poisoning, however, should be considered in closed house fires or industrial fires

Cyanide reversibly binds to ferric ions within the mitochondria causing anaerobic metabolism (inhibition of oxidative phosphorylation)

Rapid development of tissue hypoxia and profound mydriasis, ALOC, seizures, coma and death

HAGMA with serum lactate > 8.0 mol/L

Early treatment with hydroxycobalamin 5 mg over 15-minutes can be life-saving

Activated charcoal 50g

Carbon monoxide inhalation improves with time and 100% oxygen administration

970
Q

Complications of severe anorexia nervosa (body weight < 40% ideal) include all of the following
EXCEPT

A

Hypercalcaemia

Does include:
Hypoglycaemia
hypokalaemia
Immunodeficiency (WCC, neutrophils or leukocytes)
metabolic alkalosis
Cardiac complications: sinus brady (decr BMR), orthostatic hypotension, arrhythmias (AV block, prolonged QTc, sinus arrest, wandering pacemakers, nodal escape beats, SVT, VT)ST depression, TWI, MV prolapse, pericardial effusion, congestive cardiac failure, cardiomyopathy, impaired myocardial contractility
hypotension
hypOcalcaemia, hypokalaemia, hypoMg++, hypoCl, ypocalc, hypophlsphataemia (impairs myocardium, reduces threshold for arrhythmias)
renal calculi
enlarged salivary glands, dental caries, oesophagitis/gastritis
risk refeeding
thrombocytopenia
may have cognitive impairment
seizures
stress fractures
poor wound healing
Osteopaenia leading to fractures
cold intolerance, vulnerable to pressure injuries
often co-existing psychiatric conditions.

971
Q

A patient requires a peripherally inserted central venous catheter. Electrocardiographic (ECG)-
aided tip localisation is used to site the tip of the catheter. The initial ECG from the catheter is
shown. The ECG when the catheter is placed appropriately will be

A

Highest deflection of the P wave= cavo-atrial junction

972
Q

You are asked to review a patient two days after a surgically difficult total knee replacement that
was undertaken under tourniquet. The anaesthesia and analgesia technique used was spinal
anaesthesia in combination with an ultrasound-guided adductor canal block and high-volume local
anaesthetic infiltration by the surgeon. The patient complains of a new onset of leg weakness on
the operative side. The nerve LEAST likely to be involved in this weakness is the

A

saphenous purely sensory
obturator out of the way

973
Q

When using an endotracheal tube in an adult, the highest recommended cuff pressure to avoid
mucosal ischaemia is

A

30cmH2O

974
Q

A drug which does NOT increase the defibrillation threshold in a patient with an implanted
cardioverter defibrillator is

A

Drugs that decrease DFT (K+ channel blockade):
sotalol

Those that incr (sodium channel blockade):
amiodarone (in chronic stage, decreases in acute stage!)
lignocaine
verapamil
venlafaxine
atropine
diltiazem
flecainide

no change = procainamide

975
Q

This posteroanterior chest X-ray shows enlargement of the

A

LA if double density sign

976
Q

Your patient has been administered 50 mL of oral 5-aminolevulinic acid hydrochloride (Gliolan)
three hours prior to her scheduled craniotomy for resection of a glioblastoma. Care should be
taken perioperatively to avoid the adverse effect of

A

photosensitivity (avoid direct light incl bright OT light/sunlight for 24hrs)/burn risk
there’s also risk lactic acidosis.

These pts also get hepatic dysfunction (bili, AST, ALT, GGT all. go up)
anaemia, thrombocytopaenia, leukocytosis, thromboembolism
elevated amylase
N&V
hypotension, reduced PVR

Gliolan is C/I in porphyria, pregnancy, if hypersensitive to ALA (haem precursor)

977
Q

Anaesthesia-induced rhabdomyolysis differs from malignant hyperthermia in that it is NOT

A

Characterised by rigidity
Responsive to dantrolene

978
Q

You are planning to perform a peribulbar block and wish to check the axial length of the eye prior to proceeding. The average axial length of the globe in adults as measured by ultrasound is

A

25mm23.5mm (22-25mm)

979
Q

Of the following clinical conditions, difficult intubation is LEAST likely to be associated with

A

Down syndrome (mask vent might be difficult, laryngoscopy gen fine as MO normal, laryngoscope displaces large but soft tongue)

-Apert
-Hurler (mucopolysaccharoidosis type 1)- frequent URTI, chronic OM & OSA, dysmorphic features, limited reck ROM, macrocephaly, coarse facial features, depressed nasal bridge, short stature, , flexion/joint deformities, pectus
hepatomegaly, fatty infiltration
mitral valve prolapse, MR
-treacher collins
-pierre robin
all difficult intubation

980
Q

A 25-year-old man suffers a 30% total body surface area burn. A physiological change expected
within the first 24 hours is

A

incr PVR
hypodynamic phase initially:
incr SVR, PVR, reduced CO, decr SvO2
subsequent incr vasc permeability, hyperdynamic & hypermetabolic flow phase

981
Q

Somatosensory evoked potentials (SSEPs) are used to monitor spinal cord function during
scoliosis surgery. They are LEAST affected by

A

opioids (NMBDs augment them)

982
Q

Suxamethonium may be safely given to patients with

A

cerebral palsy

983
Q

A 65-year-old man presents to the preadmission clinic two weeks prior to his total knee
replacement. His blood results include haemoglobin 100 g/L, ferritin 20 μg/L and normal C-reactive
protein. The best course of action is to

A

Defer surgery @ least 4wks, Fe infusion (or oral Fe & OT 12wks later)

984
Q

A patient’s glomerular filtration rate is estimated at 35 mL/min/1.73m2. The patient’s chronic kidney
disease can be classified as Stage

A

IIIb

985
Q

To allow cardiopulmonary bypass in a patient with heparin resistance, fresh frozen plasma may be
administered in order to increase the level of

A

ATIII

986
Q

A 63-year-old woman is to undergo an elective total hip replacement. Her past medical history
includes hypertension, stroke, type 2 diabetes mellitus, chronic atrial fibrillation and chronic renal
impairment with an estimated creatinine clearance of 46 mL/min. Her medications include
dabigatran 150 mg bd for stroke prevention. Perioperatively, her dabigatran therapy should

A

ve witheld 96hrs if purely surgery,

(normal renal fn 48hrs, 50-80 72hrs, 30-50 96hrs, <30 120hrs)

987
Q

A 50-year-old man with carcinoid syndrome having a resection of a peripheral hepatic metastasis
develops a sudden fall in BP from 110/70 mmHg to 85/50 mmHg without significant bleeding. The
most appropriate management is

A

octreotide; 20-50microg IV boluses to effect (or as per UTD, 500microg followed by 50-200microg/hr infusion

988
Q

A man has symptomatic carbon monoxide poisoning. His pulse oximetry (SpO2) and arterial blood
gas (PaO2) would be expected to show

A

SpO2 low (close to 100%), PaO2 generally normal since it reflects O2 dissolved in blood & this process isn’t affected by CO.

989
Q

Venous air embolism during frontal craniotomy is most likely to arise from the

A

Superior sagittal (where the incision for frontal crani is)
Occurs when pressure gradient across vein; relatively -ve in vein cf atmospheric, particularly in noncollapsible venous structures. spont vent incr risk.most often during incision or dissection around vascular structures.

990
Q

St. John’s wort (herbal medicine Hypericum perforatum) will reduce the effects of

A

It induces CYP3A4
so, reduces effects of amiodarone, warfarin (2C9), MTx, cyclosporine, HIV protease inhibitors, alfentanil, midaz, lignocaine, some NSAIDs.
Digoxin minor CYP3A4 metabolism. Theophylline some CYP3A4 metabolism.

It works by inhibiting serotonin, NAdr, dopamine re-uptake, risks serotonin syndrome.

991
Q

You are about to anaesthetise a 25-year-old man for an open appendicectomy. He has a history of
tricuspid atresia for which he has had a Fontan procedure. An important goal in managing his
ventilation under anaesthesia is to ensure

A

main goals: maintain preload, limit PVR, maintain contractility
-ve pressure is ideal (spont vent) to limit drops in preload however if procedure requires paralysis, aiming to avoiding excessively high Paw, avoid atelectasis; LOW RR WITH SHORT Tinsp, low PEEP, relatively low TVs but ensure low-normocapnia
need to preserve myocardial contractility

992
Q

According to the ‘Fourth Consensus Guidelines for the Management of Postoperative Nausea and Vomiting (PONV)’ published in 2020, multimodal PONV prophylaxis should be implemented in adult patients

A

with 1 or more risk factors

1-2: 2 agents
>2 3-4

paeds:
No risk factors: 0 or ordans or Dex
1-2 dex & ondans
2+: dex, ondanks, consider TIVA

993
Q

When using ROTEM thromboelastometry, the APTEM test is used to assess

A

hyperfibrinolysis

994
Q

Of the following, the drug that is LEAST likely to provide effective analgesia following paediatric
tonsillectomy is

A

Clonidine

PROSPECT: include paracetamol, NSAIDs, dexamentasone w opioids as rescue.

995
Q

You are anaesthetising a patient for implantation of an automated implantable cardioverter
defibrillator. The patient is a 48-year-old with dilated cardiomyopathy and pulmonary hypertension.
The preoperative echocardiogram report states that the estimated pulmonary artery systolic pressure is 55 mmHg, and that there is mild right ventricular systolic dysfunction. To avoid worsening right ventricular function during induction, it would be best to consider using

A

spont vent
vasopressor to defend RV perfusion (eg. vasopressin, NAdr) BUT don’t want to overload dilated ventricle so initrope (dob)

PAH mild 20-40, mod 41-55, seere >55

996
Q

A 36-year-old man complains of left calf pain for two weeks. His pain is worse on walking but not
completely relieved by sitting or lying down. On examination, he has mild weakness of left big toe
extension. The most likely finding on MRI would be

A

L4/5 peripheral disc bulge w impingement

997
Q

A normal sized six-year-old girl has a haemoglobin of 70 g/L following surgery. The volume of
packed red blood cells that you would plan to infuse to raise her haemoglobin to 80 g/L is

A

100mL (desired increment Hb x body wt x 0.5)
but if >20kg, just give 1 paeds unit (60mL)

*4mL/kg incr Hb by 10
10mL/kg incr it by 30

998
Q

he train-of-four (TOF) ratio above which the majority of anaesthetists will NOT be able to visually
detect fade on TOF stimulation is

A

40%

999
Q

A 24-year-old man with Wolff-Parkinson-White syndrome is having anaesthesia for a knee
arthroscopy. During the procedure he develops the following rhythm. His blood pressure is 100/65
mmHg. The most appropriate treatment is

A

if haem stable, throught to be antidromic VT, vaslalva. no adenosine or CCB. can give procainamide.

1000
Q

A high mixed venous oxygen saturation (SvO2) is most likely to be associated with

A

incr supply (high FiO2, hyperbaric)
reduced demand (hypothermia, NMB)
high flow states (hyperthyroidism, severe liver disease)
severe sepsis may see high SvO2 as high CO &low O2 extraction; aim SvO2 >70%
cyanide toxicity

1001
Q

causes of 4th HS

A

HCM, HTN, AS, MR; reduced myocardial compliance; forced atrial contraction & raised LVED

1002
Q

In a patient with tetraplegia who develops autonomic dysreflexia, the expected
haemodynamic response is

A

HTN from splanchnic (T5-9) VC (below level of lesion; if injury T6 or higher, get splanchnic)

1003
Q

venous air embolism considered highest for

A

seated crani (100%)
LSCS (40%)
hip arthroplasty (30%)
ant Cx discectomy 10%

1004
Q

Intraoperative cell salvage is contraindicated in

A

pheochromocytoma
pt refusal
HIT if heparin-containing anticoagulnat
discontinue it where temporary contamination of surgical field w solutions that may cause red cell lysis or drugs/subsances that shouldn’t be given intravenously eg. cement, iodine, topical clotting agents. Wash the area w 0.9% NaCl before resuming ICS.
Discuss sickle cell w haematologist on case by case basis. Chance salvaged blood returned to pt may sickle.
avoid from grossly contaminated fields & manufacturers contraindicate it if bowel contaminationunless catastrophic haemorrhage, , should be able to use for part of procedure w LDF & broad-spectrum ABx.

1005
Q

A 25-year-old woman is administered two doses of aprepitant for postoperative nausea and
vomiting after a sleeve gastrectomy. She normally takes the oral contraceptive pill. You
should advise her to use alternative contraception for the next

A

28 days

1006
Q

A patient who usually takes oral morphine 50 mg bd develops a bowel obstruction and
experiences withdrawal symptoms. They may be described as having

A

physical dependence, predictable & physiological. increased dose. stop/reverse/reduce= withdrawal

Tolerance: increasing dose required for same effect

Dependence: withdrawal symptoms on reducing/abrupt cessation

Addiction: aberrant drug seeking behaviours (departing from accepted standard) despite risk of psychological/physical/social harm

1007
Q

The image below on the left shows a normal central venous pressure (CVP) trace. The CVP
trace in the image below on the right is most consistent with

A

Fused C& V waves= tricuspid regurg

1008
Q

A 26-year-old man is brought into the Emergency Department four hours after an accidental chemical exposure during crop spraying. His clinical signs include bradycardia, vomiting, diarrhoea, coughing, miosis and weakness. A drug which is NOT recommended during his resuscitation and treatment is

A

Suxamethonium, since the organophosphate inhibit acetylcholinesterase so prol NMB. Can use NDNMBs but may be less effective due to competitive inhibition @ NMJ (likely to need incr doses). The Rx for cholinergic toxicity includes pralidoximine; cleaves bond btw organophosphate & acetylcholinesterase, reactivating it

1009
Q

A 76 year old woman who is spontaneously breathing through a tracheostomy tube with an inner cannula becomes acutely breathless. Despite application of high flow oxygen, her respiratory rate is 40 breaths per minute and her SpO2 is 82%. The next most appropriate step in her airway management is to

A

remove inner cannula- re-assess. Pass suction catheter if established trache, re-assess.
Deflate/reinflate cuff (in case herniated over end of trache, re-assess

1010
Q

A new volatile agent is developed. The property it shares with sevoflurane that will enable it to
be used in a sevoflurane vapouriser and deliver an accurate concentration is its

A

SVP

1011
Q

The diffusing capacity of the lungs for carbon monoxide (DLCO) is likely to be decreased with

A

factors affecting membrane SA eg. emphysema (standing lower DLCO vs supine)

factors affecting membrane diffusion barrier eg. CCF, pulmonary oedema, interstitial fibrosis, sarcoidosis

factors causing lack of pulm blood volume eg. pulmonary HTN or PE, valsalva

DLCO generally incr during exercise, may not incr adequately if pulm vascular bed reduced by emphysema

Factors affecting uptake by Hb: anaemia
CarboxyHb
supplemental O2

1012
Q

The use of erythropoietin before major surgery results in

A

less transfusion, more thrombosis (hence benefits offset by risks incl mortality & thrombotic complications so NICE recommend that EPO shouldn’t be used in surgical pts to reduce blood transfusion.
It stimulates rbc proliferation

1013
Q

The risk of postoperative respiratory failure in myasthenia gravis is increased by the administration of

predictors of need for postop ventilation in myaesthenia:

A

GENTAMICIN- aminoglycosides
CCBs
Mg++
all contribute to muscle weakness, potentiation of actions of NMBDs

VC<2.9L, pyridostigmine >750mg/day, Dx >6yrs ago, coexisting pulmonary disease

1014
Q

The risk of postoperative respiratory failure in myasthenia gravis is increased by the administration of

predictors of need for postop ventilation in myaesthenia:

A

GENTAMICIN- aminoglycosides
CCBs
Mg++
all contribute to muscle weakness, potentiation of actions of NMBDs

VC<2.9L, pyridostigmine >750mg/day, Dx >6yrs ago, coexisting pulmonary disease

1015
Q

A patient with known type 3 von Willebrand disease presents with persistent epistaxis. First-
line medical therapy should include

A

TxA

1016
Q

ANZCA fasting guidelines classify all of the following as clear fluids EXCEPT

A

strained broth (pulp-free juice, black coffee, cordial, carbo rich fluids (clear) specifically developed for periop use, all clear fluids)
NOT soluble fibre, milk-based drinks & jelly.

1017
Q

The most common cause of mortality in children with diabetic ketoacidosis is

A

cerebral oedema

1018
Q

anorexia nervosa may be ass’d with ecg findings (& is NOT usually ass’d with resting tachycardia)

A

bradycardia
prol QTc (from hypocalc, hypoMg++, drugs, starvation)
ST depression
TWI

AV blocks
sinus arrest
nodal escape beats
wandering atrial pacemakers
supraventricular tachycardia
ventricular tachycardia

1019
Q

You have been asked to provide general anaesthesia for a complex thoracic endovascular
aortic aneurysm repair. After the placement of a lumbar drain the recommended safe time
before the administration of intravenous heparin is

A

1 hr
(can remove after 204hrs & normal aPTT)

Aim CSF pressure <=10mmHg

MAP aim >85mmHg

Aim SCPP pressure >75mmHg

COPS protocol for reduced MPEs or SSEPs:
-CSF drain status
-O2 delivery (ensure Hb >120, CI >2.5L/min/BSA)
Patient status:
mean BP >90mmHg
SCPP>80mmHg (MAP-CSFP)
cognitive status

1020
Q

Painless post-operative visual loss with preserved pupillary reflexes is most likely due to

A

Posterior cerebral ischaemia or CORTICAL BLINDNESS

ION: most common cause POVL, painless but pupillary light reflexes absent, pupil dilated
CRAO: RAPD, pupil reflexes sluggish/absent, painless
corneal abrasion: painful, altered visual acuity generally not visual loss
acute angle closure glaucoma: painful, fixed mid-dilated pupils
retinal detachment painless, PLR maintained unless significant retinal death

1021
Q

With regard to the risk of postoperative surgical-site infection, 8 mg dexamethasone
administered intraoperatively has

A

does not increase risk SSI

1022
Q

With regard to the risk of postoperative surgical-site infection, 8 mg dexamethasone
administered intraoperatively has

A

does not increase risk SSI

1023
Q

With regard to the risk of postoperative surgical-site infection, 8 mg dexamethasone
administered intraoperatively has

A

does not increase risk SSI

1024
Q

evidence of PTx on lung US

A

lung point (distinction where normallung to absent (may or may not be present)
absent lung pulse
incr clarity A lines
lack of B lines
lack of sliding pleura

other conditions:
B lines fluid or fibrosis (interstitial syndrome)
hepatisation (consolidation or atelectasis)

Z lines= short, broad comet tails arise from pleural line but don’t reach distal edge of screen. Normal or in pts w PTx, don’t erase A lines & don’t move w lung sliding.

1025
Q

Local anaesthetic-induced myotoxicity is most likely to be associated with

A

ADDUCTOR CANAL infusionsHigher []s of LA (but >0.375%, ropi >0.75%)
Bupivacaine
Longer duration of exposure to LA
Eg. Continuous adductor canal infusion
proximity to muscle plane

1026
Q

The most common cause of cor pulmonale is

A

COPD. generally, it’s mild pulm HTN (mean PAP <=40mmHg

1027
Q

What’s post-capillary PH?

A

haemodynamically defined as mPAP >20 mmHg and PAWP >15 mmHg

1028
Q

risk factors for local anaesthetic systemic toxicity:

A

hypoxia & acidosis
hypercapnia

pre-existing heart disease, extremes of age, frailty, conditions causing mitochondrial dysfunction (eg. cartinine deficiency), liver or kidney disease (delayed LAST by depressing LA metabolism or distribution); reduce dose 10-20% in renal impairment. Liver impairment single dose blocks unaffected, repeated bolus or continuous infusion doses should be reduced.
cardiac failure esp susceptible to LA-induced myocardial depression & arrhythmias + lower liver & renal perfusion slows metabolism & elimination.
Smaller pt size, paeds (neonates & infants reduced AAG levels, chn have incr elemination half life; risk accumulation w continuous infusions), pregnancy (lower AAG, accelerated perfusion), lower AAG

bupiv (racemic higher risk than levo), low CC:CNS (bup 2 vs 7.1 lidocation), higher dose, intrinsic VD (bupiv, ?clin sig?)
subcut
continuous vs single administration

site: some (interscalene, stellate ganglion) risk intravasc injection. highly vascular (increased perfusion @ site of injection).
toxicity highest to lowest= IV, subcut, mucosal, tracheal, intercostal, caudal, Lx epidural, brachial plexus, sciatic, topical.

LOWER risk LAST with US guidance (less risk vascular injury, lower volumes used), frequent aspiration, incremental injection, ?epinephrine, and higher alpha1 acid glycoprotein levels (which have high affinity for amide LAs, reducing risk toxicity), ?intrinsic vasoconstricor (levo, ropiv)

1029
Q

The medical laser LEAST likely to cause eye injury is

A

CO2; 10.6microm long wavelength (this is long), shortest penetration. used for cutting & coagulation of sort tissue. Photo-thermal effect.
lasers in visible & near IR range have greatest potential for retinal injury (400-550nm wavelength).

Holmium:YAG: photo-mechanical effect. for tissue abloation or lithotripsy. Very intense but brief- explosive expansion of tissue.
Argon: photo-dissociation, non-thernmal, corneal reshaping.

1030
Q

The main advantage of using noradrenaline (norepinephrine) over phenylephrine for the prevention of hypotension as a result of spinal anaesthesia for elective caesarean section is

A

less maternal bradycardia

also improve CO
?concerns re: foetal acidosis not borne out in evidence
no diff in N, V & apgar scores

1031
Q

electrical fires extinguish w

A

CO2

1032
Q

number of segments in lower lobe L) lung

A

4

R) UL 3
R) ML 2
R) LL 5

L) UL 4 *incl sup & inf lingular
L) LL 4
*the R) lung has 2 lobes & 10 segments, the L) has 2 lobes & 8 segments however for calculating PPO, 5 in L) LL

1033
Q

A man who had successful treatment of a germ cell tumour ten years ago presents for
laparoscopic appendectomy. Your intraoperative management should consider

A

Lowest FiO2 possible (titrate SpO2 88-92%); these pts often took bleomycin which may–> pulm damage progressing to fibrosis & lifelong risk O2 tox. Sx nonspecific, incl dry cough, breathlessness, pleuritic cp, fever.
O2 therapy can induce & exacerbate bleomycin lung injury. High []O2 incr risk.
Pre-Ax Hx & exam, CXr, ABG, lung CT, PFTs or bronchoscopy (depending on findings). O2 titrated for SpO2 88-92%.
may have intersitial shadowing, airwpace shadowing, risk PTx/pneumomediastinum.

1034
Q

The muscle or muscle group with the greatest resistance to the action of non-depolarising
neuromuscular blocking agents is the

A

diaphragm

the central muscles with good supply have fast onset (larynx, diaphragm). however while diaphragm & orbicularis oculi (reflects diaphragm well) have fast offset, larynx has slow recovery as sensitive to NDMRs. Add pol poor blood supply, slow onset & offset.

1035
Q

A peripheral intravenous cannula is being inserted in the forearm of a man having a
hemicolectomy. The skin asepsis preparation NOT suitable for this procedure is

A

70% alcohol (which is suitable for only 24hrs)

Use 2% chlorhex in 70+% EtOH ethyl or isopropyl, or povidone iodine 10% in 70% ethyl EtOH remaining on skin at least 2min
tincture of iodine, iodophor or chlorhex).

1036
Q

n pulmonary function testing the presence of airflow limitation is defined by a post-
bronchodilator FEV1/FVC ratio less than

A

70%

*may overDx COPD in elderly, perhaps should only be seen as clinically important if < LLN (z score <-1.64)

1037
Q

may overDx COPD in elderly, perhaps should only be seen as clinically important if < LLN (z score <-1.64)

A

LBW
Adj40 for maintenance

1038
Q

QRS complexes in antidromic vs orthodromic AVRT

A

antidromie wide complex

1039
Q

A 45-year-old man has LFTs showing high bilirubin, high transaminases (AST 2x>ALT), low alb, most likely Dx

A

ETOH liver disease

ALT>AST: substance abuse, HIV, HBV, HCV
ALP>2x normal: cholestatic (and conj bili high, uncong bili normal)
low alb: systemic illness, malnutrition

1040
Q

The Vortex Approach to airway management does all of the following EXCEPT

A

use 4 lifelines; 3x lifelines
max 3 attempts @ each unless game-changer, @ least 1 attempt by most experienced clinician
for each lifeline consider: manipulations, adjuncts, different size/type, suction/O2 flow, muscle tone
CICO status escalates w unsuccessful best effort @ each lifeline

1041
Q

Suxamethonium causes a sustained contraction of the extraocular muscles for up to

A

10 mins

it incr IOP avg 4-8mmHg, peaks 1-2 mins, lasts duration of nm blockade

1042
Q

59-year-old lady presents for elective coronary artery graft surgery. She has a pulmonary artery catheter inserted with the waveforms displayed below. Her cardiac output is 4.5 L/min. Her mean pulmonary artery pressure is 33 mmHg. The most likely explanation for the waveforms seen is that she has

A

COPD (mild pulm HTN, ?prob not elev PCWP)

1043
Q

A patient with a history of paroxysmal atrial fibrillation and chronic obstructive airways disease
develops a wheeze intraoperatively which resolves with administration of salbutamol via the
endotracheal tube. Soon after, he develops rapid atrial fibrillation with a ventricular rate of 120
beats per minute, a BP of 90/60 and an ETCO2 of 40mmHg. His regular medications are
inhaled salbutamol, inhaled salmeterol and digoxin 125mcg daily. The next most suitable
treatment is

A

Amiodarone 150mg over 10mins then infusion (onset <48hrs, lower dose as on digoxin)

Unstable would be if reduced LOC, CP, SBP<90, HF

1044
Q

A man with atrial fibrillation has no valvular heart disease. According to joint American Heart
Association (AHA), American College of Cardiology (ACC) and Heart Rhythm Society (HRS)
guidelines, oral anticoagulants are definitely recommended if his CHA2DS2-VASc score is
greater than or equal to

A

2 in males
3 in females

1045
Q

he advantage of the Mapleson E circuit in paediatric anaesthesia is due to its

A

low resistance (valveless, no resistance to airflow, no points of possible mechanical failure)
minimal dead space (if flow rate high enough!)

valveless, low R- used in paeds up to 30kg
The FGF is near pt
need high FGF 2-3xMV to prevent rebreathing is needed (need to ensure that after the exp pause the next time they take a breath they won’t be re-breathing expired gas or inhaling atmospheric air)
no reservoir, can’t manually ventilate w +ve pressure or feel compliance
no valves means no PEEP
can’t scavenge waste

1046
Q

A 50 year old man has the following pulmonary function test result: (Results displayed) The
diagnosis is most consistent with

A

Pulmonary hypertension

In patients with PAH, the primary cause of a low DLCO is a reduction in the pulmonary capillary blood volume, whereas in patients with IPF- PH
FEV1/FVC>70 restrictive, <70 obstructive, look at post-bronchodilator result

Reversibility (12% change is the cutoff)

FEV1 - gold 1 (>80%), 2 (50-80%), 3 (30-50%), 4 (<30%)

TLC for restrictive dz (80/50/30), mild/mod/sev/critical

DLCO (adj Hb, VA)

1047
Q

When performing a paediatric pain assessment, the five elements assessed to obtain the
FLACC score are

A

face
legs
activity
cry
consolability

1048
Q

Local anaesthetic blockade of the sciatic nerve results in loss of function of all of the following
EXCEPT

A

Knee extension

1049
Q

Methylene blue may be used in the treatment of all of the following conditions EXCEPT

A

methaemoglobin reductase deficiency

1050
Q

A respiratory effect of high flow nasal oxygen therapy is

A

reduced RR

provides humidification & warming (assists secretion clearance, decr atelectasis, limits airway surface dehydration)
improved pt comfort if hypoxaemic resp failure.
reduce subjective feelings. of dyspnoea.

Can deliver FiO2 of up to 1 in moderate resp distress (since can match higher insp flow). measured nasopharyngeal FiO2 close to that set on the device unless pt grossly tachypnoeic.

REDUCES deadspace; washes out CO2, provides anatomical O2 reservoir in nasopharynx & oropharynx.
Reduces WoB (reduces airflow resistance in nasopharynx as splints upper airway).
CPAP effect; UA distending pressure 3.2-7.4cmH2O w mouth closed; +ve Paw, incr end-exp lung vol, improves alveolar recruitment. distending pressure transmitted to the lower airways to generate PEEP (depends on mouth being closed so PEEP variable)..

1051
Q

The size 5 i-gel® supraglottic airway is recommended for patients who weigh over

A

90kg, max ETT=8

4: 50-90kg, max ETT=7

3: 30-60kg, max ETT 6

2.5: 25-35kg, ett F

2: 10-25kg, max ETT 5

1.5: 5-12kg, ETT 4

1: 2-5kg , max ETT 3

(On iGel)

sizes 2-4 can take a 12Fr suction or NGT, size 5 can accomodate 14Fr

1052
Q

A normal 75 kg term parturient may be expected to have a total blood volume of

A

7500mL

1053
Q

he intrinsic muscles of the larynx do NOT include

A

DOES include:
cricothyroid
posterior & lateral cricoarytenoids
transverse arytenoids
oblique arytenoid
thyroarytenoid

NOT suprahyoid

1054
Q

Of the following, the deficit that DOES NOT result from damage to the common peroneal
nerve is

A

knee extension

CPN is L4-S2

damage DOES cause impaired:
knee flexion (short head piceps)
eversion, dorsiflexion, great toe E, sensory anterolat leg, dorsum foot

provides genicular branches to knee joint, lat cut nerve to calf, sural communicating branch, then SPN (L5-S2) & DPN (L4-S2)

1055
Q

A patient has numbness and weakness in her hand postoperatively. You are trying to
distinguish between an ulnar nerve lesion and a C8-T1 radiculopathy. You can diagnose a
C8-T1 radiculopathy if she has weakness

A

aBductor pollicis brevis which is from recurrent branch of median nerve C8-T1

ulnar nerve C8-T1 does:
FCU
medial half FDP
hypothenar mm
medial 2 lumbricals
adductor pollicis
interossei of hand
palmaris brevis

1056
Q

In a patient with anaemia of chronic disease, of the following the most likely to be elevated is

A

Ferritin

inflamm markers
TIBC & transferrin rise in Fe deplete states, fall if inflammation or Fe overload

1057
Q

The muscle or muscle group with the greatest sensitivity to the action of non-depolarising
neuromuscular blocking agents is/are the

A

larynx

Sequence of recovery (fastest to slowest):
diaphragm>laryngeal muscles>corrugator supercili>abs>orbicularis oculi>geniohyoid>add pollicis

1058
Q

he intubating dose of atracurium in a patient with post-polio syndrome should be

A

50% of dose

1059
Q

A man presents with a fractured tibia. He has increasing pain in his leg, loss of sensation on the
plantar surface of his foot and weakness of toe flexion. This is most consistent with a compartment
syndrome of the leg in the

A

deep posterior compartment of leg

1060
Q

The direct physiological effects of electroconvulsive therapy include

A

Initial PSNS 10-15secs; brady/hypoT or even asystole
Sustained SNS response (peak 3-5mins, rise SBP 30-40%, rise HR >20%, incr myocardial O2 consumption; ventricular dysfunction for up to 6hrs after Rx)

Also:
incr IOP
incr intragastric pressure
nausea
myalgia, general ill-ease
dental damage
oral cavity damage (tongue, gums)

1061
Q

The direct physiological effects of electroconvulsive therapy include

A

Initial PSNS 10-15secs; brady/hypoT or even asystole
Sustained SNS response (peak 3-5mins, rise SBP 30-40%, rise HR >20%, incr myocardial O2 consumption; ventricular dysfunciton for up to 6hrs after Rx

Also:
incr IOP
incr intragastric pressure
nausea
myalgia, general ill-ease
dental damage
oral cavity damage (tongue, gums)

1062
Q

Preferred fluid type paeds IV maintenance:

A

For 1 month +: 0.9% NaCl w glucose 5% (consideration of 20mmol/L KCl in pre-mixed bags, once normal baseline electrolytes & renal function have been confirmed)
neonates 10% glucose +/- additional NaCl

1063
Q

mnemonic for lumbar plexus

A

Iliohypogastric (T12-L1)
Ilioinguinal (T12-L1)
Genitofemoral (L1-2)
Lat fem cut (L2-3)
Obturator (L2-4)
femoral (L2-4)

1064
Q

The atmospheric lifetime of nitrous oxide (in years) is approximately

A

100 (it’s 114), des is 9-21

1065
Q

What are PRAEs?

A

Coughing

Bucking

Breath Holding

Laryngospasm

Bronchospasm

Stridor (Post-Extubation)

Obstruction

Aspiration

Desaturation

LRTI

Atelectasis

1066
Q

The risk of a perioperative respiratory adverse event in a child is least likely to be increased by

A

Pt:
age <1
preterm
atopy or FHx atopy, wheezy child
Tobacco exposure
URTI within 2/52
obesity/OSA/sleep disordered breathing
craniofacial abnormalities/airway abnormalities

Anaes:
non-paediatric specialist
volatiles (inhalational induction incr risk)
inadequate depth
oxy, midaz, airway topicalisation
ETT>SGA>FMV

Surgical:
airway (esp adenotonsillectomy), emergency, stimulating

1067
Q

The most likely cause of hip adduction in a patient undergoing transurethral resection of a bladder
tumour is

A

obturator nerve, L2-4, which travels alongside posterolateral bladder wall before exiting pelvis]

innervates add long & brevis, adductor magnus gets dual innervation from obturator & sciatic.

NDNMBAs & obturator nerve blocks help prevent or minimise adductors spasm

1068
Q

Chronic recreational use of nitrous oxide may lead to

A

reduced synthesis methionine & tetrahydrofolate
Vit B12 deficiency & peripheral neuropathy
neuro resembling subacute combined degeneration of the cord; myelopathy & peripheral neuropathy; limb weakness, ascending or I/M distal numbness & tingling with imbalance/ataxia, impaired proprioception, reduced grip strength
skin hyperpigmentation
vascular disease from hyperhomocysteinaemia
teratogenic
megaloglastic changes in bone marrow- agranulocytosis
bone marrow failure after days
risks increased in B12 deficiency (ETOH, vegan, elderly, malabsorption)

1069
Q

hen commencing treatment of proximal deep vein thrombosis or pulmonary embolus, factor Xa
inhibitors (apixaban, rivaroxaban) are preferred to dabigatran or warfarin because they do not require

A

bridging

dabigatran is 150 mg twice daily (after an initial 5 to 10 days of parenteral anticoagulation)
rivaroxaban 15mg BD for the first 3 wks then 20mg daily
apixaban 10mg BD for first 7 days then 5mg BD, down. to 2.5mg BD after >=6/12 Rx

1070
Q

Apgar

A

Appearance: 0=blue/pale, 1=acrocyanotic, 2=all pink
Pulse: 0=absent, 1=<100, 2=>100
grimace: 0=no response, 1=minimal response to stimulation, 2=cough/cry (prompt response to stimulation)
activity: 0=absent, 1=weak (flexed UL/LL), 2=strong
respiration: 0=absent, 1=irregular/slow, 2=good/crying

1071
Q

The lung ultrasound finding most consistent with atelectasis is three or more

A

B lines between rib spaces

Lung ultrasound:
A lines= horizontal lines below pleura; present both in normal lungs & PTx
M mode lung sliding shows “seashore sign”
“Barcode sign” indicates lack of lung sliding
Interstitial syndrome: characterised by B LINES. These artefacts are generated by juxtaposition of alveolar air & septal thickening (fluid or fibrosis). Normal lungs have occasional B lines but up to 2 between 2 adjacent ribs is normal. 3 OR MORE BETWEEN RIB SPACES are pathological. Present in any disease affecting interstitial (eg. Pulmonary oedema). Very severe oedema causes hyper echoic confluent pattern (white lung)

1072
Q

Hydrofluoric acid:
First aid

A

Calcium gluconate 2.5% gel to contaminated skin, 15-minutely
Remove clothing & jewellery, bag & remove from care areas
Wash, incl scrubbing under nails. Staff to wear PPE (incl chemical resistant gloves)
Flush eyes w 0.9% saline

Pts @ risk fluorosis (which manifests as hypocalcaemia, w prol QT) need continuous cardiac monitoring
Mx arrhythmias or arrest w 10% calcium gluconate 0.5mL/kg up to 60mL, IV sodibic 1mmol/kg, IV mg sulfate 50%
Continue to correct hypocalcaemia
Discuss w toxicologist
Inhalation, neb 1mL Ca gluconate in NaCl

1073
Q

If group A RhD negative fresh frozen plasma is not available for use in an A RhD positive patient, of
the following your next best choice should be

A

AB+

1074
Q

transhiatal oesophagectomy is performed via a

A

Abdo (gen laparoscopy) & L) neck incision.
don’t need OLV, if need a CVC, place on R). don’t need to turn the pt.

1075
Q

ANZCA choosing wisely recommendations

A
  1. Avoid routinely performing preoperative blood investigations, chest X-ray or spirometry prior to surgery, but instead order in response to patient factors, symptoms and signs, disease, or planned surgery.
  2. Avoid ordering cardiac stress testing for asymptomatic patients prior to undergoing low to intermediate risk non-cardiac surgery.
  3. Avoid administering packed red blood cells (blood transfusion) to a young healthy patient with a haemoglobin of ≥70g/L who does not have on-going blood loss, unless the patient is symptomatic or haemodynamically unstable.
  4. Avoid initiating anaesthesia for patients with limited life expectancy, at high risk of death or severely impaired functional recovery, without discussing expected outcomes and goals of care.
  5. Avoid initiating anaesthesia for patients with significant co-morbidities without adequate, timely preoperative assessment and postoperative facilities to meet their needs.
1076
Q

Consideration for same-day discharge in an ex-premature infant after orchidopexy for undescended
testis would be suitable at a minimum postmenstrual age of

A

54 wks

term infants only if reached PMA 46 wks

(*pt must be medically fit, risks discussed w carers & parents on individual basis)

1077
Q

The implemention of comprehensive multidisciplinary geriatric assessments in the peri-operative
period has been shown to

A

-improve M&M in hip fracture
-be cost-effective, reduce LoS & delays secondary to cancellations
-for vascular, reduced LoS & complications incl delirium, cardiac/bladder/bowel

Overall: likely. to have positive impact on postop outcomes eg. medical complications & LoS, in older pts undergoing elective & emergency surgery.
mortality, LoS, financial cost.

1078
Q

The implemention of comprehensive multidisciplinary geriatric assessments in the peri-operative
period has been shown to

A

-be superior to traditional scoring systems (eg. ASA & POSSUM) in identifying pts @ highest risk of adverse outcomes
-elective ortho: improved LoS, reduced pressure sores, better pain scores, higher rates early mobilisation.
-identifies issues affecting periop course that wouldn’t likely have been detected w standard preop Ax.
-improve M&M in hip fracture
-be cost-effective, reduce LoS & delays secondary to cancellations
-for vascular, reduced LoS & complications incl delirium, cardiac/bladder/bowel
-improve communication w pts & their families

-overall literature inconclusive; CGA periop is predominately assessment in absence of interventions.
Overall: likely. tohave positive impact on postop outcomes eg. medical complications & LoS, in older pts undergoing elective & emergency surgery.
mortality, LoS, financial cost.

Blue book: CGA very strong evidence of reduced LoS.

1079
Q

An 84-year-old woman with dementia presents for surgery for a breast lump. She lives in a care facility and is accompanied by the nurse manager from the facility and her son. Neither have a written legal authority to act on her behalf. Regarding consent for her surgery

A

Advanced health directive
Guardian appointed by civil & administrative tribunal
Power of attorney from AHD
Statutory attorney in following order: spouse (close & continuing relationship), unpaid carer, close adult friend or relative
Public guardian

May only carry out care without consent if reasonably consider the pt has impaired capacity, it must be carried out urgently (imminent risk to life or health or must occur to prevent significant pain or distress) & it’s not reasonably practical to get consent from someone who may give it under guardianship & administration act or powers of attorney act. This includes providing life-sustaining measures eg. CPR. “Blood transfusion is not a life-sustaining measure”.

1080
Q

In patients without other co-morbidities, bariatric weight loss surgery is indicated when the body mass
index (kg/m2) is greater than

A

40

> 35 if associated obesity illness

pts also must have made reasonable attempts @ other WL techniques, age 18-65, capacity to understand risks & commitment ass’d w the surgery, no psych. or drug probs

1081
Q

CVP findings of tamponade & TR

A

tamponade= loss of Y descent
TR= huge V wave, C&V fuse w blunted X descent

1082
Q

Of the following, the device that delivers the greatest flow when using ‘Level 1® Fast Flow Fluid
Warmer’ rapid fluid infuser system is a (list of intravascular catheters)

A

8.5Fr RIC 1L in 46 seconds. 6.5cm. 600mL/min in other ref. One study max flow rate 1.2L/min

sheaths from 4Fr up to 11fR, lengths from 5.5 to 23cm. 8.5Fr sheath 1.17L/min max flow rate (more variable than RICC)

MAC has 9 Fr distal lumen (& 12-gague prox lumen for incompatible fluid
admin. has chlorhex. no catheter it is 500mL/min.
Variable reports- absolute max 1.2L/min

Vascath 13.5Fr, 400mL/min

14g IVC 250/400
16g 150/300
18g 100/150
20 70/100
22 35/70
24 20mL/min

1083
Q

Local anaesthetic systemic toxicity does NOT manifest as

A

initial manifestation may be cardiovascular collapse/cardiac arrest.
CNS: initial excitatory (perioral tingling, tinnitus, slurred speech, light-headedness, tremor, confusion/agitation, excitation culminates in generalised convulsions). depressive phase= coma & resp depression.
cardio: 3 phases: HTN & tacchy initially, then myocardial depression & hypotension, terminal phase= peripheral vasodilation, severe hypoT & arrhythmias incl sinus brady, conduction blocks, ventricular tachyarrhythmias, asystole.

other LA toxicities:
anaphylaxis: more common w esters. methyl-paraben or metabisulphite preservatives may be the cause.
metHb may occur w prilocaine (its metabolite o-toludine)- avoid eutectic mixture of LA in those <1yo receiving MetHb-inducing drugs (phenytoin, sulphonamides, benzocaine)

1084
Q

According to the ANZCA ‘Guideline on infection control in anaesthesia’, skin preparation prior to
central neuraxial blockade should be performed using

A

Chlorhexadine 0.5% in ethanol

1085
Q

The risk of major bleeding in patients taking direct oral anticoagulants (DOACs) is NOT significantly increased by commencing administration of

A

Likely anything but:
SSRIs
antiplatelets
amiodarone
fluconzole
rifampin
phenytoin

a couple of studies suggest that atorvastatin & digoxin diltiazem NOT ass’d with incr risk major bleeding

Pharmacokinetic interaction not considered as important as pharmacodynamic:
Concurrent use of drugs that incr plasma levels of DOACs by inhibiting P-glycoprotein (eg. verapamil, amiodarone, quinidine) or CYP3A4 (for riva or apix) don’t incr risk of major bleeding, however should avoid drugs which are strong inhibitors (eg. antifungal azoles, cyclosporine).

From a study looking at bleeding risk:
amiodarone, simvastatin, atorvastatin, verapamil, digoxin, diltiazem (this is the only one which significantly lowered)

Presumed based on Pk:
ezetimibe & lipid-lowering drugs except simvastatin w dabi
metronidazole & ciprofloxin
antacids
gabapentin, pregabalin, lamotrigine
Probably COX-2 selective NSAIDs; they have little to no inhibitory effect upon platelet function (eg. COXIB)

Paracetamol

Should use PPIs for pts on anticoagulation & high bleeding risk (eg. NSAIDs, aspirin, high bleeding risk scores).

Concurrent use of antiplatelet drugs or SSRIs (which inhibit plt aggregation) WAS associated with increased risk of major bleeding (100% incr w antiplatelets, 70% w SSRIs)
Antiplatelets inhibit plt aggregation (primary haemostasis), DOACs inhibit fibrin formation (secondary haemostasis)

1086
Q

The management of a patient who has experienced a cardiac arrest within 10 days of cardiac surgery
should NOT routinely include

A

atropine
full-dose epinephrine
external cardiac compressions

1087
Q

A third heart sound at the apex may be heard with

A

children
pregnant
well-trained athletes
systolic heart failure (overly compliant myocardium–> dilated LV), eg. CCF most common cause, DCM, mitral prolapse, TR, L)–> R) shunts.

1088
Q

In cardiac surgery, volatile-based anaesthesia compared to total intravenous anaesthesia

A

no diff death from any cause @ 1 yr or 30/7, no sig diffs btwn groups in secondary outcomes incl MI.
Volatile may facilitate earlier extubation.

1089
Q

What’s hyperkalemic periodic paralysis & consideration?

A

mutations in sodium channel
precipitated by hyperKal, rest after exercise, stressful situations, possibly hypoglycaemia
avoid cholinesterase inhibitors, SCh, K+. Maintain normothermia & normoglycaemia.
Rx: glucose, insulin, epinephrine, B-agonists, Ca++

1090
Q

myotomes

A

C4: shoulder shrugs

C5: shoulder abduction and external rotation; elbow flexion

C6: wrist extension

C7: elbow extension and wrist flexion

C8: thumb extension and finger flexion

T1: finger abduction

L2: hip flexion

S1: hip extension

L3: knee extension

L5: knee F

L4: ankle dorsiflexion

L5: big toe extension

S1: ankle plantarflexion

S4: bladder and rectum motor supply

1091
Q

Considering emergency front-of-neck airway access, the major blood vessel that is most likely to lie
anterior to the trachea above the sternal notch is the

A

brachiocephalic artery

1092
Q

In the treatment of persistent mucosal bleeding in patients with von Willebrand disease type 3,
desmopressin (DDAVP) is

A

not indicated. never useful.
Rx persistent mucosal bleeding w TxA. some type 1 & some type 2 (d/w haem), DDAVP 0.3mcg/kg. biostate (vWF & F8 concentrate 2:1 dosed on F8 u/kg) is indicated in type 3 w bleeding or pre-op in type 1 not responsive to DDAVP or if bleeding not controlled w DDAVP & to Rx bleeding in type 2 & 3. Each vial has 50IU/mL FVIII & 100IU/mL vWF..
DDAVP generally not for chn <3yo as risk hyponatremia & seizures (relatively CI if prev seizure disorders). restrict fluid to approx 80% maintenance in first 24hrs of DDAVP.

1093
Q

In the treatment of persistent mucosal bleeding in patients with von Willebrand disease type 3,
desmopressin (DDAVP) is

A

not indicated. never useful.
Rx persistent mucosal bleeding w TxA. some type 1 & some type 2 (d/w haem), DDAVP 0.3mcg/kg. biostate (vWF & F8 concentrate 2:1 dosed on F8 u/kg) is indicated in type 3 w bleeding or pre-op in type 1 not responsive to DDAVP or if bleeding not controlled w DDAVP & to Rx bleeding in type 2 & 3. Each vial has 50IU/mL FVIII & 100IU/mL vWF..
DDAVP generally not for chn <3yo as risk hyponatremia & seizures (relatively CI if prev seizure disorders). restrict fluid to approx 80% maintenance in first 24hrs of DDAVP.

1094
Q

A 48 year old male is day two post-laparoscopic high anterior resection. He has used 42 mg of
intravenous morphine in the past 24 hours. You wish to start him on oral tapentadol immediate
release. The most appropriate equianalgesic dosage would be

A

42mg IV x3 to get OME (126mg)), then x 3.33 to get approx 400mg
IR tapentadol QID therefore 100mg; FPM calculator advises 25-50% dose reduction (not all opioids equal effect) so probably 50mg QID best approach.

1095
Q

A patient with a purely metabolic acidosis has a serum bicarbonate of 14 mmol/L and a lactate of 3.8
mmol/L. The expected PaCO2 is

A

29 (Winter’s formula 1.5x bicarb +8 (+/-2)

1096
Q

The part of the lung that is typically divided into superior, medial, anterior, lateral and posterior
segments is the

A

R) LL

1097
Q

Positive sIgE to morphine is a marker of antibodies to what?

A

quaternary ammonium component of NMBDs

1098
Q

How to work out predicted PaCO2 for a metabolic alkalosis?

A

(0.7 x HCO3-) + 20

1099
Q

How to work out predicted bicarb for a resp acidosis?

A

1mmol/L for every 10mmHg elev CO2 acute, 4 chronic

1100
Q

How to work out predicted biarb for resp alkalosis?

A

2mmol/L for every 10mmHg drop CO2 acute, 5 chronic

1101
Q

How calculate RVSP from CVP & velocity across TCV?

A

4v2 + RAP

1102
Q

How calculate RVSP from CVP & velocity across TCV?

A

4v2 + CVP

1103
Q

In maternal cardiac arrest the most common arrhythmia is

A

PEA 50%
asystole 25%

1104
Q

What’s the rule of 3s for calculating confidence interval for events w low probability?

A

3/n (ie. if 100pts, 3/100=0.03)

1105
Q

apnoea is?

A

20 seconds, less if brady/hypoxic
risks: young, prematurity, Hx apnoea, lung disease, NEC
postop monitoring HR 100, SpO2 94%, attend, confirm apnoea, stimulate, delay discharge if apnoea

1106
Q

how much blood need to transfuse to get Hb from 70-80g/L?

A

4mL/kg packed red cells raises Hb by 10

1107
Q

which drugs may cause tissue necrosis if extravasated?

local irritation?

A

amiodarone
atracurium
ketamine
phenytoin
thio

rocuronium

1108
Q

alternative to polysomnography in paeds?

A

overnight oximetry; contiuous, mod-severe if @ least 3 clusters of desat events SpO2 <90%

1109
Q

alternative to polysomnography in paeds?

A

overnight oximetry; contiuous, mod-severe if @ least 3 clusters of desat events SpO2 <90%

1110
Q

You are part of an international humanitarian aid mission. You have packed sevoflurane but the only
local vaporiser is isoflurane specific with a maximum output of 5%. If you added sevoflurane to the
isoflurane vaporiser the maximum sevoflurane output percentage would be approximately
(Sevoflurane saturated vapour pressure 160mmHg, isoflurane 240mmHg)

A

3%

1111
Q

how is remi metaboliised

A

non-specific tissue & plasma esterases

1112
Q

buprenorphine patch half time B
How many days reach steady state?

fent patch mean terminal half life?

A

12 hrs
3 days SS
fentanyl patch

17 hrs
time to analgesic effect 12-24hrs after patch on

1113
Q

electrolyte derangement in Conn’s

A

hypoK, hyperNa, NORMAL BSL (doesn’t impact glucose), alkalosis, HTN

1114
Q

equivalent dex to pred 10mg/day

A

1.5-2mg

1115
Q

clinical signs hypomagnesaemia

A

tetany (trosseau, chovstek), muscle spasms/cramps, muscle weak (“lemonade legs”), involunt movements

reversible blindness, vertigal nystagmus

rarely resp m weakness

confusion/delirium, seizure, comas

prol PR, QT, prone to atr/vent ectopy, torsades
more prone. to dig tox

often concurrent hypoCa & hypoK

1116
Q

Normal renal blood flow

Normal brain blood flow
Normal brain O2 consumption

Normal blood O2 content

A

1200mL/min

750mL/min or 50mL/100g/min
3.5mL/100g/min

200mL per litre oxyHb, 3mL dissolved
so the blood O2 content is 2mL as oxyHb per 10mL blood

1117
Q

Normal renal blood flow

Normal brain blood flow
Normal brain O2 consumption

Normal blood O2 content

A

1200mL/min

750mL/min or 50mL/100g/min
3.5mL/100g/min

200mL per litre oxyHb, 3mL dissolved
so the blood O2 content is 2mL as oxyHb per 10mL blood

1118
Q

If a TBI pt has a cerebral angiogram showing cerebral perfusion 15mL/100g/min (normal 50) & O2 consumption 3.5mL/100g/min (normal level), what’s it consistent with?

A

ischaemia; DO2 @15mL/100g/min would be 3mL/100g/min

1119
Q

absolute contraindication to ECT

A

raised ICP

1120
Q

Diluted thrombin time measures anticoagulant activity of

which DOAC can the PT be used to test?

A

dabigatran (but not suitable for monitoring; very low plasma levels–> long or unmeasurable TT)

rivaroxaban; normal PT suggests unlikely a sig rivaroxaban effect

1121
Q

what’s lymphoedema @ recommendations re: IV access & BP?

A

may arise following r/a axillary LN
generalised abN collection protein-rich IF, ass’d w oedema & altered tissue structure
4 stages subclinical (may be subjective but minimal tissue change, stage 0) to overt (stages I-III)

discomfort, compression bandages, poor healing
PGA 18 appendix 1: no lymph: no CI to NIBP or pIVC on affected arm & art line if clinically indicated
WITH lymphoedema: alt site where practical but no absolute CI to using affected limb for monitoring & IV access, monitoring the PIVC in accordance w clinical standards

1122
Q

What proportion of difficult airways are unanticipated?

A

> 90%

1123
Q

What’s difficult mask ventilation?

Grades. of MV difficulty?

A

when best efforts produce inadequate or low EtCO2 (grade C or D)

A= plateau
B= no plateau but >10mmHg
C= no planeau, EtCO2 <10mm Hg
D=no Et CO2

Describe how achieved (1, 2 hands, guedel, NP)

1124
Q

what’s absolute humidity? relative? OT humidity recommendations?

A

amount of water vapour per volume of air. relative= amount of water vapour in air relative to max that could be contained in the air @ a particular temp, a proportion, relative vs absolute important when discussing thermal comfort.

Temp 21-24deg C, relative humidity 50-60%

1125
Q

fidelity in health care

A

faithful or loyal- to patients & to professional standards, ie. follow guidelines

1126
Q

PICC position in paediatrics:

A

either in distal third of SVC or @ cavo-atrial junction. From lower limb terminate in IVC. UL may move 2.2 rib spaces w arm movement.
tracheal bifurcation.

1127
Q

How to diagnose atlantoaxial instability on latral Xray?

retropharyngeal abscess or significant trauma

teardrop #

what’s the cobb angle for scoliosis?

A

ant tip of vertebral body

> 4mm between atlas & odontoid peg inflexion

widening of prevertebral soft tissue; <1/2 with ie at C2 should be <=7mm wide, C6 should be <1 width ie <=14mm chn, <22mm adults. If swelling ant to C3 exceeds 3mm, suspect C2 fracture.

> 10deg

1128
Q

Colours for different infusion routes

A

intrathecal or epidural yellow
regional is white + yellow/black margin
subcut beige
IV blue
IA red
enteral green inh white
misc pink
central venous line white w blue border

1129
Q

How to calculate ARR?

A

% or rate without Rx (A)- % with Rx (B)

A-B= X%

NNT = 1/0.0X

1130
Q

cylinder w grey shoulders

A

CO2

air= black & white
helium brown (heliox brown & white)
oxygen= white
argon=green
nitrous= blue

1131
Q

goals of ventilation with fontans

A

limit intrathoracic pressure; low RR with short insp time (ie limit time that PiT is raised), low PEEP, limit TVs to 5-6L/kg, avoid incr PVR, important to avoid hypercarbia; simple procedures spont vent best as long as avoid hypercarbia.

1132
Q

Part of the heart most likely injured with penetrating chest injury?

A

RV

1133
Q

Part of the heart most likely injured with penetrating chest injury?

A

RV

1134
Q

volatile w least impact on CBF?

A

sevo

1135
Q

electrolytes with rhabdo:

A

hyperkalaemia, hyperphosphatemia, hyperuricaemia, hypOcalcaemia

renal failure from myoglobinuria, metabolic acidosis

1136
Q

Medical therapy vs TAVI decr 30-day risk of

A

stroke

1137
Q

What’s the osmolality of glycine?
sorbitol?
mannitol?

A

200

165
275

1138
Q

Volatile analysis in most anaesthetic machines is done via which method?

A

Infrared absorption spectroscoscopy

-molecules with dissimilar atoms absorb IR & convert the energy into molecular vibration. Vibration frequency depends on molecular mass & atomic bonding. Can determine gas by the [] of a wavelength absorbed (molecules absorb IR @ specific wavelengths); Beer-Lambert law (Beer: amount. of light absorbed directly proportional to [] of the solute. Lambert path length.

Refractometer: Raleigh: prisms split light. difficult for breath-breath analysis but for calibration & envt gas exposure.

Piezoelectric eg. quartz, can’t differentiate btwn vapours (but fast resposne)

Raman: expensive but fast. almost as accurate as mass spectrometry.

Mass spectrometry: long response & delay times, costly but very accurate- research.

1139
Q

What does prothrombinex VF contain?

A

II, IX, X, ATIII, heparin, sodium/phosphate/citrate/chloride (small amounts 7 & 5)

1140
Q

What may reduce chance of inserting epidural catheter into BV?

A

lateral position, injecting saline

1141
Q

Describe AIC

A

56cm 19Fr semi-pliable
accommodates fibreoptic scope (it has an ID of 4.7mm_
2x rapifit connectors; one for standard 15mm ID bag, one for jet ventilation so can oxygenate @ any time, or swivel connector
7.0mm ETT is the smallest that can accommodate AIC
=

1142
Q

How long does it take for insulin to improve hyperkalaemia?

A

begins in 10-20mins, peaks 30-60, lasts 4-6hrs. benefits for hypokalaemia aren’t sustained long however the insulin levels can remain high enough to cause hypoglycaemia. Therefore, give ongoing glucose following the initial bolus & monitor every hour.

1143
Q

What incr risk w eye block?

A

30x higher risk of needle damage to globe if inferotemporal peribulbar

MEDIAL CANTHUS approach or sub-Tenon’s are safe

1144
Q

What incr risk w eye block?

A

30x higher risk of needle damage to globe if inferotemporal peribulbar

MEDIAL CANTHUS approach or sub-Tenon’s are safe

1145
Q

Baby with FTT, loud systolic murmur, weak femoral pulses

A

Not CoaA as this has weak femoral pulses
Most likely VSD L)–>R) shunt, since PDA usually continuous (apex is more likely MR though), VSD may develop diastolic if large

1146
Q

murmur grades:

A

1 faint, not all posn
2 faint, all posn
3 mod loud, no thrill
IV load & palpable thrill
V very loud, thrill, hear w steth slightly off chest
VI loudest, heard with steth off chest

1147
Q

PDA manifestations

A

small (Qp:Qs <1.5) may be asymp, continuous flow murmur
mod (Qp:Qs 1.5-2.2, exercise intolerance, mod L)–>R) hsunt incr LA & vent fvolume load
large Qp:Qs >2.2: LV volume overload, eventual rise PAP & may reverse shunt & eisenmenger syndrome. FTT, poor feeding, resp distress.
adult: systolic ejection murmur, eisenmenger incl cyanosis & clubbing.

1148
Q

PDA manifestations

A

small (Qp:Qs <1.5) may be asymp, continuous flow murmur
mod (Qp:Qs 1.5-2.2, exercise intolerance, mod L)–>R) hsunt incr LA & vent fvolume load
large Qp:Qs >2.2: LV volume overload, eventual rise PAP & may reverse shunt & eisenmenger syndrome. FTT, poor feeding, resp distress.
adult: systolic ejection murmur, eisenmenger incl cyanosis & clubbing.

1149
Q

why do some diathermy pas have 2 separate electrodes on the pt?

A

to measure impedance of the neutral pad & make sure it’s safe (ie. that it’s not coming off & risking burning skin)

1150
Q

what happens to cisatracurium out. of fridge?

A

loses 5% potency per month @ 25 degrees. so once remove from fridge, use within 21 days.approx 99% efficacy

1151
Q

how long to use roc out of fridge?

A

60 days (some say 12 weeks!)
atrac? discard. @end of day
panc 3 months
sux 30 days

1152
Q

For which type of vWD is DDAVP contraindicated?

A

type 2b: they bind too well to plts so DDAVP promotes the clumping & thrombocytopaemnia

2a can’t bind properly to plts to form clot
type 3 complete lack vWF
type 1 get low levels of vWF & low FVIII, can give DDAVP if responder

1153
Q

If a pt has AAI pacemaker but has 2nd degree heart block, what’s happening?

A

wrong type of pacemaker; they have AV conduction issue (AAI inhibits impulse if senses native atrial activity, if none then atrial pacing is initiated. used in sinus dysfunction with intact AV conduction)

1154
Q

What was the most common cause of airway problems/complications in the ICU, reported in NAP4?

A

tracheostomy-related problems (tracheostomy dislodgement led to half of all cases of death & brain damage)
tracheal tube displacement/misplacement & failed intubation were equal

1155
Q

therapeutic serum Mg++ target for pre-eclampsia?

A

1.7-3.5mmolL

1156
Q

Benefits of ITM

A

improves analgesia & is opioid-sparing for up to 24hrs after major surgery incl abdo, ortho, spinal, cardiothoracic

adding ITM to IT bup/fent prolongs pain relief after labour

addint IT fent & morph to spinal prolongs time first analgesic request after CXS

ITM vs periopheral regional for LL arthroplasty offers similar analgesic benefits but incr adverse effeics

ITM has higher incidence of OIVI, pruritis & PONV cf PCA opioid

pruritis w IT opioids. isdose-dependent, can effectively treat & prevent w 5HT3 antagonists in non-obstetric but only for Rx (not prevent) in obs
naloxone reduces pruritis & nausea but not vomiting.

adding IT magnesium prolongs analgesia in non-obs!

IT clonidine + morph sl prolongs analgesia & opioid sparing

should use lowest effective dose of ITM (no consistent dose-responsiveness for analgesia but incr adverse effects dose-related)

monitor for OIVI 18-24hrs after ITM.

adverse: OIVI, pruritis, N&V, urinary retention (minor OT), reactivation oral herpes simplex labialis

1157
Q

When’s the peak resp depression after ITM 200-600mcg?

A

3.5-7.5hrs (late resp depression can occur 3.5-12hrs, peak 6 hrs)

1158
Q

what causes red man syndrome secondary to vanc?

A

mast cell degranulation (NOT IgE mediated)

1159
Q

types of hypersensitivity reactions?

A

1= IgE mediated (anaphylaxis)
2= Ab mediated (IgM or IgG bind to cells–> destruction)
3= complex formation
4= delayed response, T-cell mediated

1160
Q

main proglem in pts w IgA deficiency in anaes?

A

anaphylaxis to blood products

1161
Q

shelf life of cryo?

A

23 months when store @ -25
it has VIII, fibrinogen, XIII & vWF & fibronectin

1162
Q

how long to give cryo?

A

within 6 hrs of thawing or 4hrs of pooling.

1163
Q

Volume of B & C cylinders
pressure storing full?

A

170 & 490L

approx 14000kPa (about 2000psi)

1164
Q

How many ppl for propofol sedation?

A

3; proceduralist, sedationist, assistant to both (the assistant to the practitioner administering sedation must be exclusively available to that person @ induction & emergence & during procedure as required; if GA is intended, person specifically to assist the anaesthetist required throughout

1165
Q

How to give calcium for hyperK?

how rapidly other agents work to LOWER K+?

A

10mL of 10% (13.6mEq in CaCl cf 4.6 in Ca gluconate)
over 2-3 mins, cardiac monitoring
can repeat after 5 mins
acts within mins, 30-60mins duration

insulin 10U w 50mL 50% glucose drops it by 1mEq/L, onset 10-20mins, peak 30-60, DOA 4-6hrs

salbutamol lowers it by 0.5-1.5mEq/L, peak effect within 30 mins infusion, 90 mins neb.

1166
Q

Relative contraindications mediastinoscopy?

A

SVC syndrome. severe tracheal deviation, Cerebrovasc disease, severe C spine disease w limited neck ext, prev chest RTx, aneurysm.

1167
Q

Onset time for trali?

A

usually 1-2 hrs (it’s sudden onset hypoxaemic resp insufficiency); symptoms may be delayed as long as 6hrs.
fever, chills, dyspnoea, tachy, tachypnoea, hypoT, hypoxaemia, noncardiogenic bilat pulm oedema w resp failure.

1168
Q

At what level can detect fade?

A

most can only detect if TOFR <=0.4

1169
Q

What are the components of the neonatal facial pain scale?

A

brow bulge
deep nasolabial fold
eyes squeezed shut
open mouth
taut tongue
chin quiver
purse lips

NOT mouth closed

cry
requires O2 for SpO2>95%
incr vital signs
expression
sleeplessness

1170
Q

What’s the best natural frequency setup for art line?

How. toincr natural freq?
what’s damping, what contributes & optimal damping coefficient?

SO… what do air bubbles do to art line system?

A

high; at least 8x fundamental frequency (pulse rate); if lies close to nat frequency, resonates & excess amplification

incr nat frequ by having smaller length, less 3 way taps/attachments, lower compliance of cannula, lower density fluid, incr diam cannula, limit bubbles/cots.

Damping= delay in response time due to anything reducing the energy in an oscillating system & reduces the amplitude of oscillations. Most damping due to frictional resistance in the fluid. some damping necessary but excess or inadequate impairs output. optimal DC is 0.7. Over-damped (max amp under-est & min amp over-estimated, MAP unaffected, damping reduces nat frequ) damping with clots/bubbles, vasospasm, taps. Under-damped overshoots, SBP overestimated.

lower natural frequency of the system, lower MEASURED SYSTOLIC PRESSURE

1171
Q

medial scapular movement during interscalene block is sec to stimulation of the

A

dorsal scapular nerve (for lev scap/rhomboids)

1172
Q

blue urticaria is complication of

A

patent blue injection

1173
Q

which aspect insert subtenon & why?

A

inferonasal; away from site of surgery & away from insertion of sup & inf obliques

1174
Q

Where does the neoborn cord terminate? HENCE what’s the most cephalad IV space for spinal to limit risk SC damage?

A

L3-4

1175
Q

Where does the dural sac terminate at birth? and by end of 1st yr?

Where’s the intercristal line in neonates?

A

S4, to S2

L5-S1

it gets up to L3/4 in adults (hence it’s L5 in young chn)

1176
Q

How does pregnancy impact MS disease activity?

A

it decreases it, postpartum ass’d w increase MS activity.

postpartum relapses more likely if incr relapse rate in the year before preg or incr relapse during pregnancy.

exclusive breastfeeding ass’d w reduction postpartum MS relapse.

1177
Q

What’s adenosine indicated for?

A

stable narrow-complex SVT.

1178
Q

St John’s wort med interactions

A

potentiate effect of clopidogrel, sedatives, antidepressants

reduces effect of digoxin, warfarin, anticonvulsants

1179
Q

why commence insulin during fasting in T1DM?

A

limit catabolism (limit insulin resistance, lipolysis & protein catabolism which –> hyperglycaemia or ketosis.

1180
Q

why commence insulin during fasting in T1DM?

A

limit catabolism (limit insulin resistance, lipolysis & protein catabolism which –> hyperglycaemia or ketosis.

1181
Q

signs of porphyria (acut intermittent) DON’T include:

A

hypotension

They DO include:
acute neurovisceral attacks:
-abdo pain, back pain, leg pain
-nausea, vomit, constipation
-psychosis, confusion, seizures, hallucinations, insomnia, dep/anxiety/agitation
-tachycardia/palpitations, HTN, rarely arrhythmias
-peripheral neuropathy: generally motor, distal, mild sens symptoms, may progress to motor paralysis involving resp & pharyngeal, bladder dysfinction
hyponatraemia

photosensitive skin chnages

1182
Q

Stroke rates for CEA GA vs LA?

A

similar.
no stat sig difference @ 30 days for CVA, MI, death or LoS GALA & cochrane.

1183
Q

threshold for microshock

A

100microamps

1184
Q

How often give Adr for asystolic arrest?

A

every 4 mins (every 2nd 2-minute CPR round, during which about 5 rounds of 30:2)

1185
Q

CNS changes w ageing:

A

neuronal atrophy
limited neuro reserve incr risk POCD or POD
reduced CBF (atherosclerosis0

1186
Q

Paracetamol loading dose PO/IV & PR?

A

old notes 20mg/kg & 40mg/kg but latest RCH website:
paracetamol just 15mg/kg & PR 15-20mg/kg (max 24hr dosing 90mg/kg/day >1 month but 60mg/kg/day if birth-1 month)

Dose on IDEAL body weight

1187
Q

Brown sequard

Central cord syndrome

Conus medullaris

Cauda equina

A

ipsilateral motor, vibration, light touch (dorsal columns) & proprioception/position
contralateral pain& temp & crude touch

segmental loss pain & temp, weakness often greater in ULs than LLs

bladder & rectal dysfunction, saddle anaesthesia

asymmetric multiradicular pain, LL weakness & sensory loss, bladder dysfunction

1188
Q

some stable fractures

A

anterior wedge
clay shovelers
transverse process
UNILATERAL facet
burst # vertebral body
isolated #s articular pillar & vertebral body

1189
Q

some stable fractures

A

anterior wedge
clay shovelers
transverse process
UNILATERAL facet
burst # vertebral body
isolated #s articular pillar & vertebral body

1190
Q

why chn <8 higher risk of axial C spine #?

A

larger occiput relative to bodies
cervical spine fulcrum higher
weaker Cx muscles & incr ligament laxity (incr mobility of upper Cx spine)
immature vertebral joints & horizontal articulating facets, facilitate sliding of upper Cx spine
vulnerable growth plates
spinal cord more elastic so may get SCIWORA

1191
Q

Paed C spine clearance can be done based on Hx & exam if:

A

-GCS 15
-no neuro deficit
-no neck pain, diff neck ROM, midline tendernes, palpable step-off, substantial chest/abdo/pelvis injury
-mechanism not high-risk (MVA>89kph, axial load)
-no anatomical predisposition (DS, spondylitis, prior C-spine injury)

1192
Q

Paed C spine clearance can be done based on Hx & exam if:

A

-GCS 15
-no neuro deficit
-no neck pain, diff neck ROM, midline tendernes, palpable step-off, substantial chest/abdo/pelvis injury
-mechanism not high-risk (MVA>89kph, axial load)
-no anatomical predisposition (DS, spondylitis, prior C-spine injury)

1193
Q

O2 flush

A

30-70L/min with 45-60PSIG

1194
Q

pressure conversions

A

1atm=1bar=760mmHg=100kPa=14psi

1195
Q

refractory Mx anaphylaxis

A

call for help

remove triggers (eg. synthetic colloid)

invasive monitoring (art, CVC, TOE)

resistant hypoT:
-additional IV fluid 50mL/kg
NAdr 3-4mcg/min
Vasopressin 1-2U then 2U/hr
if neither avail, phenyl. ormetaraminol
Glucagon 1-2mg 5-minutely (counteract. Bblock)
ECMO

resistant bronchospasm:
consider where’s the tube? circuit? device malfunction? tension PTx
continue Adr infusion (which starts once done 3 boluses Adr), add:
salbutamol 12 puffs (1200mcg)
bolus 100-200microg then infusion 5-25mcg/min
Mg++ 2g (8mmol) over 20 mins
consider inhalational or ketamine

1196
Q

avoid tourniquets in

A

PVD
sickle cell
AVF
care ++ Hx VTE

1197
Q

AKI:

A

Abrupt decrease in kidney function—> retention of urea & other nitrogenous waste products & dysregulation of extracellular volume & electrolytes.
Criteria:
-decr UO to <0.5mL/kg/hr for 6hrs
-incr serum Cr >=1.5x baseline presumed to have occurred within 7 days
-incr Cr 26.5micromol/L within 48hrs

KDIGO criteria:
Stage 1: 1.5-1.9x Cr baseline within 7 days OR incr Cr >=26.5mmol/L in 48hrs OR UO <0.5mL/kg/hr for 6-12hrs
Stage 2: 2-2.9x baseline Cr OR UO <0.5mL/kg/hr for >=12hrs
Stage 3: 3x baseline Cr OR incr Cr 353 OR UO <0.3mL/kg/hr for >=24hrs or anuria >=12 hrs OR initiation of RRT OR in can <18, eGFR <35

1198
Q

sensitivity of methods to detect VAE during neuroanaesthesia

A

TOE (high sens 0.02mL/kg air)
Precordial doppler 2nd best (0.05mL/kg air)
then PA cath 0.24mL/kg
TCD also high
then ETH2 or ETCO2 both mod
ecg 1.25mL/kg
oes steth 1.5mL/kg

1199
Q

DECREASED levels of WHAT support diagnosis of AFE?

A

C3-C4 spec 100%

1200
Q

hyalase []

A

25U/mL

1201
Q

Is a MAOi OK for ECT?

A

yes but avoid indirect sympathomimetics eg. ephedrine/metaraminol

1202
Q

What can be given down an IO?

A

naloxone
atropine
vasopressin (adults)
Adr
Lignocaine
Incr doses 3-10x for ETT route

1203
Q

Intraosseous:

A

45mm (humerus, excessive tissue)
25mm (>40kg)
15mm (3-39kg)

15g (Ezi-IO)

should use if unable to gain IV access in life-threatening situation within short period of time (max 90 seconds 2 attempts paed cannulation in APLS)

Avoid:
over infection
over site of previous attempts (risk etravasation through old hole, compartment syndrome)
local indwelling metalwork
over limbs w possible prox #s
over fracture (compartment syndrome risk)
osteogenesis imperfecta

sites:
proximal humerus, 1cm above surgical neck (@ greater tubercle, which is poorly developed in small children)–> pts hand on abdo & elbow adducted, most prominent aspect of greater tubercle, 1cm superior to surgical neck.preferred in resus as infusion enter circulation fastest rate (except sternum bhut that requires specialised device). prox humerus straight to SVC, bypass pelvic & abdo vessels, important in trauma w abdo/pelvic injuries. child humeral shaft may be easier to identify.
2 fingers below patella & 1-2cm medial to tibial tuberosity in adults
prox tibia: 2-3cm below tib tub, <2yo tib tub might not be developed, in which case go 3cm distal & 1cm medial to lower aspect patella.
distal femur: 2-3cm above lat condyle.
distal tib: 3cm prox to med mall.

@ least 1 black line (5mm from hub) should be visible following insertion to ensure needle is sufficient length

infusion more painful than insertion; infuse lignocaine 2% prior to flushing the cannula w 10mL n/saline. drap bloods before flushed.

need pressure bag.

any med/fluid that can be infused through central line can be infused IO. same dose & onset.

prox humerus about 80mL/min, tibia 15mL/min, sternum (specific cannula, >12yo only), up to 120mL/min
*UTD: max flow rates under pressure 265mL/min at tbial, 150mL/min @ humerus (while nunmbers not under pressure have tibia as lower rate)

remove within 24 hrs (earlier if alt access gained), earlier if erythema, swelling, evidence extravasation.

rarely osteomyelitis, fracture, necrosis epiphyseal plate, compartment syndrome, extravasation.

IO good correlation for Hb, Cl, glucose, Cr, alb, urea
poor correlaton for plt, CO2, Na, K, Ca++ (potassium correlation worse than sodium)

1204
Q

Which med may contribute to VAE during reaming bone?

A

nitrous

1205
Q

what can an immunosuppressed pt not donate?

A

bone marrow, blood

1206
Q

What’s FVL?

A

FV resistent to aPC (activated protein C), a natural anticoagulant that degrades Va; therefore more prone to clotting. (factor V procoagulnat CF amplifies thrombin production).
Autosomal dominant. most pts (99%) heterozygous
only 5-10% of heterozygotes get VTE in their lifetime
VTE recurrence in heterozygotes cf those lacking the variant 1.4x OR.

1207
Q

By how much does factor V leiden homozygous incr risk of postop DVT?

A

20x

1208
Q

best method to prolong apnoeic oxygen in obese

A

head upmay reduce time. todesat by 50 secs
CPAP non-significant incr in time to desat (fRC goes back to pre-CPAP levels once anaesthetised & CAPAP removed)

1209
Q

Why reduce infusion doses morphine infant?

A

immature BBB, immature renal function (doesn’t reach adult value of 25%u ntil 1 year, slows elim of renally cleared drugs & prol DOA active metabolites)

1210
Q

CS5 ecg

A

RA below manubrium, LA v5, LL ground (R) hip/lower costal margin. monitors lead 1, observe flutter waves & detecting P waves in wide complex tacchys
Useful for anterior wall ischaemia

1211
Q

Failure rate cannula cric NAP4?

A

60%

1212
Q

straight port of multilumen adapter for arndt is for

A

fibreoptic

1213
Q

how many vials of dantrolene @ remote hospital?

A

36
at least 24 for anaes location where triggering anaes performed, larger hospitals or remote 36
20mg per vial, Reconstitute in 60mL sterile water
dose 2.5mg/kg, can repeat 10-15minutely until crisis over

1214
Q

What are ABNORMAL CV examination findings in pregnancy?

A

orthopnoea, chest pain, sudden onset breathlessness

diastolic murmur (soft systolic murmur normal)

S4 (S3 normal)

JVP >2cm (up to 2cm normal)

persistent tachy (HR rise by 10-20bpm normal)
pleural effusion
marked peripheral oedema or breathlessness (mild normal)

1215
Q

How long until normal plt funciton after chronic diclofenac?

A

1-2 days

1216
Q

Fast scan

A

pericardial
pelvic
R) flank (periphepatic)
L) flank (perisplenic)

1217
Q

what’s wall pressure(pipeline)

A

400kPa

1218
Q

most common cause litigation against anaes

A

dental damage

1219
Q

Rx for Fe def within 6 wks of surgery?

A

IV Fe

1220
Q

expected rise in plt from 1 unit pooled leucodepleted plts in 70kg pt?

A

20-40

1221
Q

what’s a phase 3 trial?

A

EFFICACY: large RCT, 300-3000 pts, usually required before public access.
Phase 0= exploratory (animal, pilot)
Phase 1= safety & tolerability (20-80 volunteers)
Phase 2= dose finding (20-200, observational)
Phase 4= effectiveness
5= comparative effectiveness

1222
Q

When switching between opioids, once converted to the 24-hour OME dose, how work out how much to give?

A

Apply dose reduction 25-50% to allow for incomplete cross-tolerance (closer to 50% if frail/elderly)

1223
Q

how dose prn breakthrough opioid during a titration process?

A

1/12th to 1/6th of total daily opioid dose

1224
Q

when preccordial thumb appropriate?

A

witnessed, monitored, pulseless VT if defib not immediately avail.
ineffective & not recommended for VF

1225
Q

when may V/Q be matched?

A

PE + pulmonary infarction
COPD (often vent defect more pronounced than perfusion
Pneumonia
hiatus hernia
tumors
pleural effusions

1226
Q

hard palate nerve supply

A

nasopalatine (anterior), greater palatine (posterior)

1227
Q

risks for LL compartment syndrome

A

procedure >5hrs
lithotomy
trendelenburg
external compression lower legs
compression iliac vessels

systemic hypotension/blood loss/hypovolaemia
PVD
obesity

1228
Q

Eaton lambert

A

weakness proximal/axial/girdle muscles. ass’d. wSCLC. sensitive to NDMRs.

1229
Q

According to ANZCA PG 31, level 2 check involves:

A

Beginning of each list.
Ultimately the responsibility of anaesthetist but may be delegated to a suitably trained person.

-Service label device & sub-assemblies.

-high-pressure:
O2 cylinder pressure, content sufficient, no leak, can turn on/off
gas supply lines pressure
turn reserve cylinder off

-low pressure:
turn on each gas, observe appropriate operation of flow indicator
verify function of O2 supply failure warning

-vaporisers:
liquid levels
filling ports sealed
correct seating, locking & interlocking
circuit leaks w each vaporiser on & off
leaks upstream of CGO (low pressure system bulb)

-breathing systems:
manually check assembly
CO2 colour & change if needed
ensure maintains a pressure >30cmH2O w flow 300mL/min
manually ventilate breathing bag, watching unidirectional valves as handbag ventilated, then check for easy spill (simultaneously squeeze both)
ceheck auto ventilation with the breathing bag, confirm disconnection & high pressure alarms

-scavenging system properly connected, not blocked

-emergency ventilation : bag & oxygen

-IV & other apparatus to use (suction, TIVA)

Document level c2 check

1230
Q

Action if sedation score 2?

A

half bolus of PCA, cease background infusion, hourly SS until <2 for @ least 2hrs

1231
Q

rate of rise of CO2 w apnoea? and THRIVE w muscle relaxant?

A

3.4mmHg/min
During THRIVE, eTCO2 incr 0.15kPA (1.1mmHg as 1kPa=7.5mmHg) per min, so eTCO2 incr 11mmHg over 10 mins, however PaCO2 incr 0.24kPA (or 1.8mmHg) /min, so incr 18mmHg/10 mins

1232
Q

can you do sub tenons w pterygia?

A

don’t cut through them; 6 vs 2 layers so hard to access space, vascular so may get haemorrhage. push it out of the way.

must avoid (rel CI) if Hx scleral disease w possible scarring & friability of sclera. scleral buckles/adhesions hinder dissection or spread of anaes, risk globe perf
avoid if infection or trauma, prev extensive vitreo-retinal surgery, sub tenon’s in same quadrant, Surgery requiring complete akinesia, surgery where conj haemorrhage -vely compromises outcome, severe ocular pemphigoid

1233
Q

Pressure (for research) of abdo compartment syndrome

A

20mmHg (no threshold to predict depends on person perfusion etc to get IAH-induced new organ dysfunction)

intra abdo HTN sustained pressure >12mmHg

1234
Q

indications of ECMO for cardiac shock

A

CI <2L/min/m2 & hypoT <90 despite adequate IV volume, high dose inotropes & IABP, eg. ACS, sepsis w profound cardiac depression, pulm embolism w cardiogenic shock/cardiac failure

pulmonary: severe ARDS eg. P/F <80

1235
Q

what happens when put magnet on AICD?

A

defib off, no change pacing

1236
Q

limiting TURP syndrome

A

<60mins resection, exp operator, height <60cm & <1-1.5L, aovoid glycine, anticipate if prostate >50g

1237
Q

LMA complications

A

sore throat
neuropraxia lingual nerve: taste/sensation
hypoglossal & RLN (cuff)
RLN complete= VC paramedian. if partial. &only abductors damaged, VC midline position (more dangerois). dyspnoea, voice change, dysphagia, aspiration

1238
Q

Poor prognostic indicators in SAH

A

Most important predictive factors for acute prognosis after SAH:
-LoC & neuro grade on admission
-Age (inverse correlation)
-Amount of blood on initial CT head (inverse correlation)

-hypoxaemia
-hyperglycaemia
-renal insufficiency
-fever
-anaemia

-cardiac complications, while often reversible, ass’d w poor outcomes
-neurogenic pulmonary oedema may –> hypoxaemia, contributing to cerebral hypoxia & poor outcomes.
It does have high early mortality (approx 20% die prior to hospital)

Early mortality:
rebleeding
vasospasm
delayed ischaemia
hydrocephalus
cerebral oedema
electrolyte abnormalities
incr ICP
seizures
cardiac complications

Later:
cerebrovascular events
MI

1239
Q

Most common cause foot drop after prolonged labour

A

compression of lumbosacral trunk; L4&5 w descending fetal head. May also occur w forceps delivery. most common mother short stature large baby.

may also occur w stirrups.

1240
Q

What determines the degree of cyanosis in ToF?

A

degree of RVOTO

1241
Q

Best TOE view for ischaemia detection

A

transgastric mid-papillary short-axis view (gives idea about portion of the territories of all 3 main coronary arteries perfusing the LV)

1242
Q

CKD stages

A

I >90
II 60-89
IIIa 45-59
IIIb 30-44
IV 15-29
V <15

1243
Q

avg duration symptom free w balloon decompression for trigeminal neuralgia

A

5 yrs, MICROVASCULAR DECOMPRESSION (w crani) 10 yrs

1244
Q

Risk of blood transfusion reaction with group specific ABO + Rh matching, but not cross matched?

By how much does ABO- and Rh- typing (but not cross match) reduce the risk of a transfusion reaction?
How much does screening?
and cross-matching?

A

0.2%

99.8%- ie. typing reduces risk of transfusion reaction to 0.2%
99.94%
99.95%

To screen, add “standard” blood cells (with known, significant non-ABO-Rh antibodies) to receipient’s serum.
To cross, add donor blood to recipient’s serum.

1245
Q

Risks of infectious complications

A

Hep B 1:250,000
Hep C 1:1mil
HIV 1:2mil

1246
Q

How much potency does sux lose with a month out of the fridge?

A

2%/month at room temp
8%/month at 37deg
in the fridge it’s 0.3%/month

1247
Q

Risk of anaphylaxis recurring post roc anaphylaxis is greatest with:

A

sux (44%)
vecuronium (40%)
atrac (20%)
panc (19%)
cisatrac (5%)

1248
Q

post resp tract illness, UL/LL weakness, Dx?

A

Guillian-Barre
Pre-junctional, acute, progressive, paralytic neuropathy due to autoimmunity in response to infection
hyporeflexia, symmetrical ascending weakness

1249
Q

Blood picture for acute phase response:

and ACD

Thalassaemia

A

elevated ferritin
low Fe, transferrin/TIBC, tsat

Hb decr, MCV normal to decr
elevated ferritin (or normal)
low Fe, transferrin/TIBC, tsat
CRP is elevated

Hb decr, MCV decr
elevated (or normal) ferritin
Serum Fe is normal or increased
transferrin normal
transferrin saturation normal

1250
Q

minimum flows for a case with a pt w MH susceptibility

A

10L/min

1251
Q

with charcoal filters in, FGF needed?

A

10 L/min for 90 mins
After 90 mins, can reduce FGF to 3 L/min

Before filters on, remove vaporisers, flush machine with O2 or air, 10L/min, 90 seconds, with 2L test lung
change full breathing circuit & soda lime while maintaining flushing @ 10L/min
insert activated charcoal filters on insp & exp ports, keep FGF going 10L/min for 90 mins from commencement of anaes, then can reduce FGF to 3L/min
can use ACFs @ 3L/min until 12 hrs have elapsed from commencement of anaes. single-use.

1252
Q

preparing OT

A

workstation
add “susceptible to MH” to surgical safety checklist & make awll aware of precautions required
move all VAs *& sux from room & trolley
Same PACU

1253
Q

types of errors

A

slip= action not taken out as intended/planned
lapse= missed actions or omission
violation= deliberate illegal
mistakes= error due to faulty plan/intention (but doing what thought was right @ the time)
sentinel event= adverse event resulting in death or serious harm

1254
Q

In which tumours should dexamethasone be avoided?

A

AML, ALL, NHL, CNS tumours

1255
Q

If anaphylaxis to roc on skin testing but -ve to atrac, sux & vec, which to use?

A

atrac, since it has lower risk anaphylaxiscf sux or vec

1256
Q

Can methylene blue cause serotonin syndrome?

A

Yes

1257
Q

most common organism in septic arthritis

A

staph aureus

1258
Q

dose of sugammadex for pt with LBW of 60kg but actual BW 110kg, PTC 1-2

A

440mg (4mg/kg actual BW)

also give 4mg/kg if TOFC 1, 2mg/kg if TOFC 2-4

1259
Q

upper lip sensory innervation

A

infraorbital

1260
Q

most reliable way of determining neonatal heart rate?

A

ECG; potential to reduce inappropriate interventions based on falsely low HR estimates based on pulse ox or auscultation but unclear if outcomes improved by early ecg
*as per ANZCOR, HR monitored by oximeter should be intermittently checked by ecg or ausc

auscultate precordium with stethoscope (more reliable than cord palpation)
base of umbi cord preferable to other palpation locations (hard to palpate in newborns, absence of pulses= unreliable sign

Pulse oximetry reliable display within 30 seconds of application, ecg even more quickly
pulse ox great as also gives info about oxygenation

Newborn HR soon after birth should be 130 (varies 110-160)

1261
Q

which virus has the highest rate of being in packed rbcs?

A

hep B

1262
Q

volume of LA caudal for orchidopexy

A

1mL/kg

1263
Q

what does N2O do to the BIS?

A

no change; incr high freq & decr low freq components

1264
Q

Longest time to environmental degradation

A

N2O>des>halo>iso>sevo
GWP100 Des>iso>N2O>sevo>CO2

1265
Q

which nerve is most often not paralysed w peribulbar?

A

medial rectus

1266
Q

Best pain score assessments different age groups

A

FLACC 2 months-8yrs & up to 18 if cognitive impairment

Self-report of pain is preferred when feasible & is possible to an EXTENT from 4yo (depend on cognitive & emotional maturity)

Revised faces pain scale/Wong baker faces 3-12 yrs (Wong Baker preferred, valid & reliable, but the smiling & crying anchor faces may lead to confounding w affect)

VAS 8 & up

Coloured anchor scale validated & reliable for acute pain Ax, recommended >=8yo, slider from white graduations to deep red

NRS-11 has been validated & is reliable; chn <8yo may need screening tasks

> =12, can use multidimensional eg. McGill pain questionnaire

Neonates:
-modified pain assessment tool (mPAT)= observational, involves behavioural (eg. sleep pattern) & physiological responses to painful stimuli (eg. RR, HR, SpO2, BP) & includes nurse’s perception
-Premature infant pain profile (acute procedural pain, post-op pain)
-CRIES: crying, requires O2 for SaO2 >95%, incr vital signs, expression, sleeplessness

APMSE:
4-5yo use simplified-FPS or pieces of hurt (four chips)
some 5 & >=6yo: NRS-11 or FPS-revised
>=8yo: CAS & VAS

1267
Q

Time following initiation of pneumoperitoneum for PaCO2 to reach plateau?

A

15-40mins

1268
Q

Management for B-blocker OD?

A

Resus, fluid
IV glucagon 50mcg/kg (up to 10mg) then 2-10mg/hr
Ca++ gluconate
vasopressor
high-dose insulin euglycaemic therapy (w dextrose to avoid hypos)
intralipid if refractory to standard measures

1269
Q

risk factors for desat on OLV?

A

Anaes:
-R) lung collapsed (it’s 10% larger, shunt is smaller if L) lung collapsed)
-poor sat on 2 lung vent, esp when lateral
-supine during OLV

Pt:
-normal PFTs pre-op (pts w severe airflow limitation on pre-op spirometry tend to have better PaO2 during OLV than pts w normal spirometry)
-restrictive lung disease
-V or Q % higher in the operative lung pre-op

1270
Q

complications of oral bowel prep do NOT include:

A

hepatic failure

complications DO include:
-hypovolaemia
-hypokalaemia: partly due to incr GI loss of secreted K+ w hyperosmotic & stimulant preparations, incr urinary loss due to hyperphosphaturia w use of sodium phosphate
-phosphate nephropathy
-hypocalcaemia
-hypermagnesaemia (picolax, citrafleet), esp if CKD

iso-osmotic
polyethylene glycol (PEG): iso-osmotic (eg. GoLytely- 4L), don’t cause fluid/electrolyte shifts mut may incr plasma volume & exac HF. Doesn’t damage colonic mucosa.
avoid solutions with ascorbic acid if G-6 P defic
Use them in older adults or those w renal insufficiency, end-stage liver disease or electrolyte imbalances or taking diuretics
low [] Na so risk hyponatraemia

hyperosmotic:
sodium sulfate-based: poorly absorbed anion, doesn’t produce significant fluid & electrolyte shifts, s3L
sodium phosphate tablet (osmoprep) is the only tablet form. however, black box warning: fluid shifts, hyperphosphate, hypoCa,K,hypERN, renal damage incl phosphate nephropathy so avoid if NYHA II/IV, renal insuffic, severe liver disease, incr risk for electrolyte abnorm, pts w IBD or diarrhoea of unknown etiology shouldn’t get these preparations- mucosal damage.

avoid sodium picosulfate in pts w HF, renal insufficiency, ESLD or electrolyte issues as it’s a stimulant laxative, potential for electrolyte shifts, higher risk hyponatremia cf PEG solutions.

1271
Q

if the glass toped chamber of flow meters is broken, can the gas flow be stopped?

A

No

1272
Q

ICP children?
infants?

A

3-7mmHg
1.5-6mmHg

1273
Q

Minimum CPP for TBI in infants?

CPP goal for 0-5yo, 6-17yo & adults?

A

40mmHg

40-50mmHg 0-5yo, 50-60mmHg for 6-17yo, 60-70mmHg adults
BTF:
III.1. Treatment to maintain a CPP at a minimum of 40 mm Hg is suggested.
III.2. A CPP target between 40 and 50 mm Hg is suggested to ensure that the minimum value of 40 mm Hg is not breached. There may be age-specific thresholds with infants at the lower end and adolescents at or above the upper end of this range.

1274
Q

For pts w TBI & refractory intracranial HTN, early decompressive craniectomy decr ICP.. what else?

A

reduces days of mechanical ventilation & days in ICU but not days in hospital & ass’d w more unfavourable outcomes @ 6/12.

1275
Q

steps to allow a pt with tracheostomy to talk

A

deflate tracheostomy cuff, insert fenestrated piece, insert one0way valve

1276
Q

Rate of PONV w one drug is 36%, with the other 12%. What’s the NNT?

A

4

1277
Q

Endocarditis prophyl indicated for dental extraction if pt has:

A
  1. Hx IE
  2. Prosthetic heart valve or prosthetic material used for cardiac valve repair (eg. annuloplasty rings & cords)
  3. cardiac transplant w valvulopathy (consult cardiologist)
  4. RHD
  5. Congenital heart disease, a) if un-repaired cyanotic defects incl palliative shunts & conduits or b) repaired defects but residual defects @ or adj to a prosthetic patch or device (which inhibit endothelialisation) or c) repaired but within 6 months (still endothelialising)
1278
Q

shelf life platelets

A

5 days, 20-24deg

1279
Q

serotonin syndrome cf NMS

A

SS= acute, hyperreflexia/myoclonus, dilated pupils, may have diarrhoea

cf NMS which is slower onset, muscle rigidity/hyporeflexia, pupils no change, may have paralytic ileus

1280
Q

in biomedical ethics, what’s utility?

A

the satisfaction or economic advantage gained from the outcome that results from a particular decision

1281
Q

gague cannula with EZ-IO?

A

15g

1282
Q

Features pointing to VT:

A

-QRS >0.14s
-Concordance of the QRS complexes in the chest leads
-Fusion beats (presence confirms VT, absence doesn’t exclude it)
-Capture beats (result in a narrow QRS, their presence confirms a Dx of VT but their absence doesn’t exclude it)
-extreme axis, or mean frontal plane axis changing during the tachycardia (esp >40 deg L) or R)
-evidence of atrioventricular dissociation is diagnostic of VT (absence of evidence AV diss’n doesn’t exclude)
-evidence of a prev MI incr likelihood of VT

1283
Q

punch throat- what look for on soft tissue XRay neck?

A

thyroid & hyoid fractures

1284
Q

Most appropriate test to Dx ruptured thoracic aorta?

A

CT angiogram (97-100% sensitivity)
CXR 98% NPV if normal

1285
Q

Where Glenn shunt go? Fontan?

A

SVC to R) PA
SVC + IVC to R) PA +/- fenestration to RA
incidations: tricuspid atresia, pulm atresia, hypoplastic R) or L) heart

1286
Q

Who are the only ppl who donate for FFP, cryo, apheresis platelets?

A

males (women of child bearing age risk developing ABs through exposure to foetal blood- the Abs canthen react w the recipients neutrophils in pulm vasculature to produce an immune response)

1287
Q

Bradycardia algorithm

A

ABCDE; monitor SpO2, BP, pulse, 12-lead ecg
apply O2, IV access
correct reversible causes (eg. electrolytes)

If any adverse features (syncope, shock, heart failure, myocardial ischaemia):

atropine 500microg. If satisfactory response (or no adverse features)

consider if the pt is @ risk of asystole (recent asystole, mobitz II, CHB w broad QRS, ventricular pause >3s)–> if no, observe

If yes, interim measures= atropine 500microg IV
isoprenaline 5microg/min IV
adrenaline 2-10microg/min IV
alternative drugs (aminophylline, dopamine, glucagon (if B blocker or CCB overdose) or glycopyrrolate can be used instead of atropine)

arrange transvenous pacing

1288
Q

% tracheal narrowing before UAO picture manifests on spirometry?

A

75%

1289
Q

A likely cause of bulb (low pressure leak test) failing to stay collapsed for 30s?

A

vaporiser incorrectly seated on backbar
leaks around agent filling device
fracture in gas piping
cracked rotameter flow tubes

leaks in high & intermediate pressure systems usually from defective valves, connectors, hanger yokes

1290
Q

What’s antiphospholipid syndrome, disease ass’ns, pregnancy-associated features & non-obs features? lab manifestations?

A

autoimmune disease of persistent aPL antibodies, recurrent venous & arterial thrombosis or adverse pregnancy outcomes
May be primary or secondary (ass’d with connective tissue diseases (RA, SLE, systemic sclerosis, Behcet’s))

Manifests:

obs
-recurrent miscarriage
-stillbirth/IUFD
-severe PET <34/40
-severe IUGR
-chorea gravidarum (involunt movements, altered speech/affect)

non-obs
-recurrent venous/arterial thrombosis
-livedo reticularis
-cardiac murmurs
-neurol features transient amaurosis fugax, TIA, CVA
-digital ischaemia
-thrombocytaemia, haemolytic anaemia

persistent aPL antibodies (lupus anticoagulant), prol aPTT, false +ve syphillis test, thrombocytopenia haemolytic anaemia

1291
Q

how long do P2Y12 inhibitors take to reach peak effect?

A

several days

Can restart P2Y12 inhibitors 12-24hrs after neuraxial block (no loading dose)

Stop clopi os ticag 5-7 days before neuraxial (prasugrel 7-10). need to do plt function tests if hold for only 5 days. P2Y12 assay similar when clopi stopped 5 days vs 7 (indicates <10% inhibition)

1292
Q

How long into maternal arrest resus should proceed until perform resuscitative hysterotomy?

A

4 mins (baby out @ 5)

1293
Q

How long into maternal arrest resus should proceed until perform resuscitative hysterotomy?

A

4 mins (baby out @ 5)

1294
Q

Cause late decels?
CTG interpretation

A

cord compression (late decels indicate contractions in presence of hypoxia eg. cord compression) - early decels normal physiologic response to minor head compression (raised ICP)

CTG big square= 1 minute

DR: define risk (gestation, are they in labour, events until now)

C: are theire contractious, regularity

BR: baseline rate
Normal FHR 110-160bpm
Baseline FHR: mean FHR when stable, excl acc & dec, define over 5-10 mins
incr by maternal fever, hypovolaemia, foetal arrhythmia, infection, distress. reduced rate if slep (low normal), drugs, congenital heart block.

Variability: sign of intact foetal neural circuitry. Baseline variability: minor fluctuation in baseline FHR, determined as diff btwn highest peak & lowest trough in FHR over 1 min segments, normal variability 6-25bpm, reduced 3-4, absent <3, increased (salutatory) >25bpm

Accels: transient incr FHR >=15bpm above baseline, lasting 15 secs

Decels: transient episodes of dec FHR 15bpm & lasting @ least 15 secs *late decels may be <15bpm from baseline
-early decels: benign. Ass’d w sleep cycle, often @ 4-8cm Cx dilation. caused by mild head compression, normal physiological response to mild incr ICP. Uniform shape, mirror contraction.
-variable decels: intermittent, significance depends on overall clinical picture.
-complicated variable decels: have non-reassuring additional features indicating likelihood of hypoxia eg. Rising baseline HR or fatal tacchy, reduced or absent variability, onset of nadir after peak contraction, large amp/longer duration decel
-prolonged decels: dec >15bpm 90secs to <5mins
-late decels: uniform, repetitive dec FHR, slow onset med to late contraction, nadir >20secs after peak contraction, end after contraction (don’t recover from contraction). Caused by contractions in presence of hypoxia. Or suggests some degree of placental insufficiency

-sinusoidal: smooth & regular, 2-5 cycles w amplitude 5-15bpm around baseline rate. lack baseline variability, no accels. Typically reflects severe anaemia (Hb <50) or peri-arrest
Preterm: higher baseline HR, access are less amplitude & shorter duration

1295
Q

newborn life support compressions to breaths?

A

3:1

1296
Q

How to tell if a pt on propranolol for prol QTc is on effective Rx?

A

no HR >130 w exercise
no change QT interval w valsalva

1297
Q

CHADS2 score

A

CHF
HTN
Age >=75
DM
Stroke/TIA 2

1298
Q

Dose enoxaparin 120kg prophylactic?

A

60mg daily;
-50-90kg 40mg daily
-91-130kg 60mg daily
-131-170kg 80mg daily
>171kg 0.5mg/kg

1299
Q

Dose enoxaparin 120kg prophylactic?

A

40mg BD (if that not an option, 60mg daily)
<50kg 20mg daily
-50-100kg 40mg daily
-100-150kg 40mg BD (or if BMI 41-60 as per latest QH)
>150kg 60mg BD
QH: if BMI >60, specialist advice

If eGFR 15-29, enoxaparin 20mg s/c daily
<15 don’t use LMWH

1300
Q

DLCO severity classification

A

Z score preferred, if N/A, DLCO >140% pred is abnormally high
76-140% pred is normal
61-75% mild impairment
41-60% mod impairment
<40% severe impairment

1301
Q

Significance of MVV on PFTs

A

suggests:
insufficient neuromuscular reserve
abnormal resp mechanics
inadequate effort

1302
Q

Significance of MVV (largest volume moved in & out of lungs in 10-15s) on PFTs

A

suggests:
insufficient neuromuscular reserve
abnormal resp mechanics
inadequate effort

eg. myaesthenia gravis. may also be heralded by decr FVC, normal or high residual volume, resuded max insp/esp pressures. MVV good as a dynamic test of ehnduratce.

1303
Q

most likely side effects of lumbar sympathetic block

A

Genitofemoral neuralgia
haematoma
intravascular injection
intrathecal or epidural injection
perforation viscera

1304
Q

Troponin rise in SAH in what % of pts?

A

17-28%

CKMB in 37%
LV dysfunction in 8-30%
Most severe form cardiac injury ass’d w SAH: stunned myocardium (reversible LV systolic dysfunction, cardiogenic shock, pulm oedema)

1305
Q

Likelihood hep C transmission w hollow needle?

A

2% (1.8)
HBV 30%
HIV 0.3%

1306
Q

Procedural risk factors for emergence delirium in children?

A

adenontonsilectomy

otorhinolaryngological
strabismus
tonsillectomy

1307
Q

Hyperkalaemia management:

A

ABCDE, card monitoring, IV access
life threatening (muscle weakness/paralysis, cardiac conduction abnormalities or arrhythmias)= hyperkalaemic emergency. Stop K, Rx with rapidly-acting therapies. IV Ca++ if ecg changes or serum K >6.5, insulin (50% glucose 50mL with 10U actrapid over 10-15mins), & aremove K+ from body (eg. haemodialysis)

If not life threatening but serum K >6.5, do the same as per emerg.

> 5.5, consider if significant renal impairment, ongoing tissue breakdown (eg. rhabod) or ongoing K absorption (eg. GI bleeding); if so manage as per emergency

If just renal impairment or pt needs K+ optimised for surgery, lower K promptly (eg haemodialysis if ESKD, discontinue meds causing hyper K, sodibic if metabolic acidosis, diuretics if hypervolaemic, or cation exchanger)
Otherwise, lower slowly: dietary modification, diuretics, bicarb if metabolic acidosis, reversal of triggers/contributors, resonium.

1308
Q

ED. &ID of CGO

A

22, 15mm

1309
Q

most common direction of atlanto-occipital subluxation?

A

anterior

1310
Q

what’s intra-osseous least accurate for measuring?

A

potassium (varies 25%, as does Cr, CO2) also not good for Na (varies by 5%), Ca++ (varies by 10%)

1311
Q

weakness finger adduction due to palsy of..

A

ulnar

1312
Q

praecordial thump done when

A

monitored pulseless VT, no immediately avail defib

1313
Q

superior oblique muscle action

A

primarily intorsion
also depression, abduction

1314
Q

annual stroke risk not on anticoag w different CHA2DS2-Vasc

A

0=0.2
1=0.6
2=2.2
3=3.2
4=4.8
5=7.2
6=9.7
7=11.2
8=10.8
9=12.2

1315
Q

NHYA classes

A

1= no symptoms w normal phys activity, normal functional status
2= mild symptoms normal physical activity, slight functional limitation, comfort @ rest
3= moderate symptoms normal phys activity, marked limitation functional status, comfortable only @ rest.
4= severe symptoms, features of HF w minimal physical activity (even @ rest), severe limitation functional status

1316
Q

which nerve provides sensation to the lateral border of lower lip?

A

mental nerve (branch of inferior alveolar from mandibular)

1317
Q

reflex arc for occulocardiac?

A

afferent via long/short ciliary of ophthalmic branch trigeminal nerve then vagus efferent

1318
Q

airway choice for a child with current URTI for closed reduction of # 2 days ago

A

spoont vent facemask

1319
Q

airway choice for a child with current URTI for closed reduction of # 2 days ago

A

spoont vent facemask

1320
Q

order of pressure measurements highest to lowest

A

Atm
bar
psi
kPA
mmHg
cmH2O

10atm= 1000kPA
10atm= 147psi
10atm=7600mmHg
10atm= 10000cmH2O

1321
Q

what to give to correct INR 2.1 for a pt needing urgent OT

A

prothrombinex 50IU/kg

1322
Q

what does given sux to an awake pt do to BIS?

A

decrease- 2 stage, initial small decr then several mins later, sharper decr to lower values.

1323
Q

new trache (8hrs ago), think dislodged. action?

A

intubate from mouth

1324
Q

hypoxic, distressed, distended neck VV just after pneumonectomy (R)-sided), immediate Mx?

A

place L) lateral

1325
Q

What’s a positive endotracheal tube cuff leak test?

A

> 110mL w cuff deflated has 98% NPV for post-extubation stridor

1326
Q

paraesthesia lifting ULs, m wasting, weak radial pulses

A

thoracic outlet syndrome

1327
Q

FiO2 during neonatal resus?

A

0.21
only up to 100% if HR <60 & commencing compressions
once compressions finished, titrate SpO2 to target oals

1328
Q

what’s background radiation?
how much in CTPA?
CXR (AP & lateral)

A

2.5mSv/annum
15mSv
0.1

1329
Q

why give mannitol during renal transplant?

A

Given just before removal of vessel clamps, reduces requirement for post-transplant dialysis but doesn’t improve long-term graft funciton in absence of adequate hydration

1330
Q

impact of pre-op aspirin (coronary artery surgery), ATACAS

A

neither incr risk of bleeding nor risk thrombotic complications nor death

1331
Q

impact of TxA coronary surgery ATACAS

A

reduces blood product usage, lower bleeding risk, no difference thrombotic complications or death (30 day), possibly seizure link

1332
Q

Delayed ischaemia most common afterSAH at what timeframe

A

4-10 days (or 3-14)

1333
Q

musculocutaneous flap cap refill time <1sec, management?

A

60mg daily

1334
Q

CPR, what % pre-arrest CO is achieved by effective CPR?

A

20-30%

1335
Q

How much sodium in mmol from 2L hartmann’s & 1L NaCl0.9%?

A

410 ((130x2) + 150)

1336
Q

numb chin, most likely neuropraxia

A

mental nerve, from inf alveolar)

1337
Q

sub tenon’s which muscle most likely to be missed

A

sup oblique

1338
Q

no water in 3rd bottle of UWSD (the suction bottle) will result in

A

inability to apply -ve pressure to pleural cavity

1339
Q

DDx of normal T3/T4 & high TSH?

A

thyroxine non-compliance, subclinical hypothyroidism

(autoimmune shows elevated TSH but low T3/T4)

1340
Q

cumulative max dose intralipid

A

12mL/kg

1341
Q

laryngomalacia

A

Most common congenital cause UA obstruction
Usually within 2/52 of birth, insp stridor, worse feeding or lying supine.
Generally conservative Mx better 6-9months later; if severe Ax rigid branch, secondary lesion in 20%. 5-10% surgical Rx.

1342
Q

laryngomalacia

A

Most common congenital cause UA obstruction
Usually within 2/52 of birth, insp stridor, worse feeding or lying supine.
Generally conservative Mx better 6-9months later; if severe Ax rigid branch, secondary lesion in 20%. 5-10% surgical Rx.

1343
Q

normal PCWP

A

4-12

1344
Q

Suction catheter size for 4-4.5 ETT?

A

8Fr
2.5=5Fr
3-3.5=7

1345
Q

peak resp depression post 300mcg ITM

A

3.5-7.5hrs (peak is 6, late can be up to 12hrs)

1346
Q

dosing adenosine SVT paeds

A

50mcg//kg inc by 50mcg/kg q2m max 250mcg/kg

1347
Q

NSAID decr PONV rate w opioid PCA by

A

25%

1348
Q

Comparing means between 2 groups (parametric)

Looking for a difference between paired groups where don’t want to lose the pairs (parametric)

Compare continuous outcome in 2 independent groups (non-para)

> 2 groups (parametric)

Comparing 2 alt interventions

CATEGORICAL

2 groups, <20 ppl

Multiple groups, many subjects

A

T test
(non-par= Wilcoxin rank sum (aka Mann-whitney U test))

Paired T-test
(non-parametric wilcoxin signed rank)

Wilcoxin rank sum (aka Mann-whitney U test)

ANOVA
(non-par=Kruskal Wallis)

Bland Altman

Fisher’s exact

Chi squared (can use for the other categorical but less exact than Fisher’s)

1349
Q

Designing a clinical trial

A
  1. Define research question:
    -hypothesis, study type
  2. Protocol:
  3. Background- lit review. Review what’s been done, set scene.
  4. Hypotheses: formed as a statement (to disprove)
  5. Methods:
    P: inclusion/excl
    I
    C
    O (specific & measurable)- primary= sample size etc based on this

allocation, data recording

Analyse & store data

How address safety issues/protocol breatches, analysis (having statistician useful from the beginning

  1. Funding
  2. Ethics & governance (legal regulations & quality assurance)

Once ethics approved or waived, collect data. Ethics: chairperson, lay person, someone involved in professional care/counselling, lawyer, 1-2 ppl with current experinence w the proposals

Analyse

Submit to journal

Pilot:
test out the methods/procedures, identify issues, to establish effect & sample size

Confounder= a variable independently associated with intervention & outcome

Bias= systematic error in the study resulting in incorrect estimates of the association. Usually methodological issue.

Selection bias: some groups over-represented
Recall bias
Observation bias (Hawthorne effect: when pts being watched, consciously or subsonsciously impacts their behaviour
Confirmation bias
Publication bias: not report -ve trails

1350
Q

Forest plots:

A

summarise findings of a meta-analysis
Small box= point estimate (eg. the mean, size represents # of participants). horizontal line= confidence interval (95%)
Vertical line= line of no difference.
Diamond= cumulative result of all studies (cumulative point estimate)

1351
Q

Indication for MgSO4 in preg pt w SBP 140mmHg

A

Eclampsia prophylaxis
MgSO4 for foetal neuroprotection if <30/40

1352
Q

why are dose-response curves logarithmic?

A

compresss the interesting part, btwn 20-80% response, where relationship usually linear

1353
Q

max pressure manujet can deliver

A

3.5atm

1354
Q

escitalopram drug interactions

A

tramadol & codeine: risk serotonin syndrome
metoprolol: CYP2D6 substrate so plasma levels incr (escitalopram CYP2D6 inhibitor)- codeine, tramadol also CYP2D6
omeprazole CYP2C19 inhibitor so will incr plasma concentrations of escitalopram

1355
Q

Which NSAIDs interfere w aspirin antiplatelet efficacy?

A

ibuprofen, naproxen, indomethacin

1356
Q

Which NSAIDs are ass’d w lowest risk thromboembolic events?

A

Naproxen

Diclofenac & the COX-2 selectives highest risk

1357
Q

Risk factors for propofol infusion syndrome:

A

YOUNGER AGE
severe head injury/acute neurological injury
sepsis
high catechol & GC levels
low CHO:high lipid intake
inborn errors FA oxidation

1358
Q

why are oral antigistamines eg. cetirizine better side effect profile cf IV promethazine?

A

promethazine can worsen hypoT

1359
Q

What incidence of trauma induced coagulopathy is an ISS >15 (major trauma) associated with?

A

67%

1360
Q

what was the aSOFA score out of? scores?

A

/3, altered mentation, SBP <100, RR>22

1361
Q

Adverse effects SSRIs

A

Serotonin syndrome
CYP inhibition (may reduce efficacy of codeine, tramadol CYP2D6 substrates)
bleeding risk (unclear if ass’d w transfusion)
atrial arrhythmias (5-HT receptors in atria)
VT (QT prol)
Delirium

1362
Q

Which drugs contain sulfonamide moiety?

A

thiazide diuretics, gliclazide, frusemide, sulfasalazine, amiodarone, diazoxide, indapamide, celecoxib BUT none contain arylamine group SO unlikely that cross-reactivity exists btwn sulfonamide ABx & other sulfonamide drugs

If pts have an allergic reaction to one cdrug there’s a stronger chance of allergic reaction to other drugs, even if unrelated.
SO, pts with a Hx of allergy to a non-arylamine sulfonamide drug need not be considered contraindicated to have arylamine sulfonamide drugs.

Benzoaine & procainamide DO have arylamine group, theoretical ana[hylaxis risk.

1363
Q

Depth of vent tube in 3rd bottle UWSD =

A

-ve pressure generated when suction applied to the system

1364
Q

Dose of prothrombinex for INR reversal pre-op

A

Depends on pt initial & target INR & if bleeding; INR 1.5-2.5 & target 1.4-2, 15units/kg. If INR >3.6 & want it <=1.3, 50IU/kg

1365
Q

FiO2 w 1,2,3,4L/min O2 flow

A

0.24, 0.28, 0.32, 0.36
The FiO2 w hudson 6L/min 0.6, 8L/min 60%

1366
Q

Max time tourniquet inflated prior to period of deflation?

A

120mins (90-120mins- pt-dependent, corresponds to depletion of ATP stores), every 30 mins incr risk nerve injury 3-fold)

1367
Q

Bier’s block minimum time til cuff deflation

A

Nysora: 30 mins
20-30 mins; should inflate @ least 20 mins to avlid LA toxicity w deflation (not evidence-based). If had to inflate before 30 ins, gradual deflation/reinflation to proloong LA washout.
Double cuff, on then elevate UL, inflate to 100mmHg above SBP, pulse goes, keep intralipid nearby.

1368
Q

SAFE study TBL?

A

no difference 4% alb cf 0.9% NaCl, avoid 4% alb in head injuries (incr 28 day mort)

1369
Q

DBS procedural considerations

A

Identification of the device & location of extension cable, liaison with the clinician managing the device, establish the conseequences of having the device off, if deactivating device= severe symptoms, oral meds started before turning the device off. If turned off during procedure, turn on after surgery & interrogate postop.

Potential interactions:
-electrocautery may burn neural tissue around electrodes or reprogram it- turn it off for electrocautery procedures, bipolar safer (similar to considerations w ICD & SC stimulators). If momnopolar, ground pads as far as poss from generator, lowest possible energy. NO short-wave diathermy modalities (radio frequency currents that head electrodes)
-Artefacts/interference on ECG
-safety of cardiac defies not established in DBS; if required, paddles as par as poss from generator, lowest energy, check generator after
ECT, RF nkeuroablation & peripheral nerve stimulation likely safe if pulse generator off & stimulator probes as far as poss from generator.
-MRI may cause incr temperature of electrodes & brain injury, device reprogramming; follow manufacture instructions.

1370
Q

Does fondaparinaux cuase HITS?

A

No- while it binds antithrombin III, doesnt have the extra chain like heparins do, doesn’t cause HITs

1371
Q

knee flexion myotome

A

S2

1372
Q

Ketamine POBA

A

20%

1373
Q

benefits of chlorhex vs iodine for skin prep for CNB?

A

bactericidal (@ high doses) vs bacteriostatic, faster onset &longer duration of action, efficacy in presence of blood, lower incidence skin reaction, less colonisation epidural catheter
it has MORE neurotox vs iodine

1374
Q

Half life gabapentin? So when reach steady state?

A

5-7hrs
24hrs

1375
Q

What’s the strong ion difference?

A

Difference in the concentrations between strong cations (Na+, K+, Ca++, Mg++) & strong anions (Cl- & SO4-); strong ions are those that dissociate completely in a particular solution (this case, @ blood pH 7.4). SID normal human plasma is 42.

1376
Q

Different types of categorical data

A

Nominal: each category has no impact on the other
Ordinal: natural progression in the data

1377
Q

What’s the mean?

A

Average. is the measure of central tendency for reporting normal data (where it’s also the median & the mode)
median is middle point (50% data above, 50% below), most useful measure of centraltendency for skewed data.
mode= most frequently appearing.

1378
Q

What proportion of values lie within +/- 1 std of mean? 2Std? 3?

A

68%
95%
99.7%

1379
Q

What’s the central limits theorem?

A

The sampling distribution of any statistic will be normal or near-normal if the sample size is large enough; this allows normal-based tests to be used for analysis of means.

1380
Q

How to calculate 95% CI?

A

sample mean +/- 1.96x SE

1381
Q

if p<0.05, what proportion of studies show benefit when there isn’t one?

A

1 in 20

1382
Q

if p<0.05, what proportion of studies show benefit when there isn’t one?

A

1 in 20

1383
Q

What are tests for relations between 2 dimensions of data?

A

Parametric tests: correlation, linear regression
Non-para: spearman’s rank correlation

1384
Q

Steps for determining sample size

A

First consider: what’s the effect we are looking for?
How much type 1 error willing to accept?
How much type 2 error (B) willing to effect
What sample size is needed to achieve this..
Calculation has degrees of freedom; control 3 things (alpha, B, effect size), 4th (sample size) determined by these; determine PROSPECTIVELY
Often work out for power (1-B) of 90% to account for attrition
Power will increase if incr sample size & incr If incr effect size

1385
Q

evidence-based minimum set of items for reporting in systematic reviews and meta-analyses.

A

PRISMA

1386
Q

EBM

A

Using best available research to guide clinical care
Identify clinical question
gather, appraise & analyse the evidence & relevance to practice
Mofify practice as a result

1387
Q

EBM

A

Using best available research to guide clinical care
Identify clinical question
gather, appraise & analyse the evidence & relevance to practice
Mofify practice as a result

1388
Q

Crossover trial design

A

Group A Rx then washout w no Rx, then group B no Rx & then washout Rx

1389
Q

3 key questions when appraising study

A
  1. Are the results valid (eg. were the groups treated equally apart from experimental therapy?) lack of blinding or lack allocation concealment generally overestimates Rx effects, ITT analysis preserves benefit of randomisation, loss to follow-up impacts study’s validity
    1. What are the results
      Will the results help me care for my pts?
1390
Q

What’s hazard ratio?

A

A weighted RR over entire duration of a study, derived from time-to-event or Kaplan-Meier curve. Takes into account timing of events which might not be evenly distributed throughout study period (cf overall RR)

1391
Q

What’s hazard ratio?

A

A weighted RR over entire duration of a study, derived from time-to-event or Kaplan-Meier curve. Takes into account timing of events which might not be evenly distributed throughout study period (cf overall RR)

1392
Q

As sensitivity reduces, what happens to FNs?
As spec reduces, FPs?

A

Increase
increase

1393
Q

What does RR 2 mean? 0.5?

A

RR 2=2 2x more likely
RR 0.5=0.5 half as likely

1394
Q

What type of study OR useful for?

A

Case control, where no way of knowing how many ppl had the exposure (ie. Don’t know the denominator); it’s the probability of the event occurring vs probability of the event not occurring, cf risk which is the chance of the outcome vs the chance of the outcome in whole population (hence better for prospective & cohort)
OR & RR are closer for rare outcomes, incr event rate, the diff more extreme (OR will overestimate RR if OR>1 & underestimate the true RR if OR <1 if common outcome.

1395
Q

What DOES attenuate OIH?

A

propofol, NMDA antag (ketamine, Mg++), pregabalin, N2O, gradual taper opioid dose, rotation
NOT clonidine

1396
Q

When, what dose & how aprepitant given?

A

Before surgery, 40mg PO
onset of action 1hr
peak 4hrs

1397
Q

what CAN spirometry measure

A

FVC
TV
IRV
ERV
NOT RV so not TLC

1398
Q

What does brachial plexus posterior cord stimulate?

A

Triceps twitch
Medial cord= wrist twitch (FCU)
Lateral cord elbow flexion (corocobrachialis)

1399
Q

amount of pleural fluid that causes appreciable blunting post costophrenic angle laterlal view

A

100mL

1400
Q

risk seroconversion to hep C following needlestick injury IF

A

2% if source known +ve, seroconversion rates are 0-10%

1401
Q

Causes macrocytic anaemia
micro
normo

A

Alcoholic (liver disease)
B12 def
Compensatory reticulocytosis
Drugs (cytotoxic, Azothioprine)/Dysplasia bone marrow
Endocrine (hypothyroidism)
Folate

Thalassaemia
Anaemia chronic disease
Iron def
Lead poisoning
Sideroblastic (body unable to utilise Fe, rbcs aneucleated, high ferritin but low transferrin & TIBC)

ACD, renal, marrow
haemoglobinopathies

1402
Q

Causes macrocytic anaemia
micro
normo

A

Alcoholic (liver disease)
B12 def
Compensatory reticulocytosis
Drugs (cytotoxic, Azothioprine)/Dysplasia bone marrow
Endocrine (hypothyroidism)
Folate

Thalassaemia
Anaemia chronic disease
Iron def
Lead poisoning
Sideroblastic

ACD, renal, marrow
haemoglobinopathies

1403
Q

what’s a caution with idarucizumab?

A

Hereditary fructose intolerance (it contains sorbitol which can cause adverse reaction w parenteral admin). Idarucizumab is 5g, lasts 24hrs, no dose adj for renal impairment.

1404
Q

What’s warm & cold ischaemia time?

A

Warm ischaemia (inevitable with DCD): donor is from asystole—> cold perfusion.
Recipient warm ischaemia= from removal from ice—> reperfusion.
Cold ischaemia= from end donor warm ischaemia to onset recipient warm ischaemia.

1405
Q

Absolute contrainidcations to donation of any organ

A
  1. Active invasive Ca in the last 3 yrs excl non-melanoma skin Ca & primary brain tumour
  2. Haematological malignancy
  3. Untreated systemic infection
  4. Variant CJD
  5. HIV disease (not infection)
1406
Q

DECRA trial reduced

A

ICP
duration MV
ICU LoS
BUT worse functional outcome @ 6 mths, no change hospital LoS & more meical/surg complicaitons

RESCUE ICP decomp crani pts w TBI had lower mort but higher rates poor functional outcome vs medical care

1407
Q

What element of diathermy enhances it’s safety?

A

High frequency current (1MHz), ass’d with higher threshold for adverse physiological effects

1408
Q

Order of frequency for the bleeding disorders that incr APTT

A

vWD, haemophilia A, haemophilia B

*haemophilia A X-linked recessive so can be passed from mother to son

1409
Q

What DOESN’T disrupt the endothelial glycocalyx?

A

Hypoglycaemia (hyperglyc, hyper/hypovol, trauma, hyper Na, surgery, HES, thrombin & TNF alfa do)

1410
Q

What’s fluid responsive?

A

SV incr by >=10% after a fluid challenge

1411
Q

Strategies to limit risk for chn w URTI needing to proceed:

A

Avoid BZD, if need premed use clonidine
Consider IV lignocaine to suppress laryngospasm but avoid topical lignocaine to VCs
Pre-op salbutamol neb 2.5mg for chi w current & recent (<2wks) URTI
Least invasive device (FM vs LMA vs ETT)
Deep removal of airway device is ass’d w incr airway obstruction despite reducing laryngospasm & PRAE
Experienced anaesthetist
Propofol blunts airway reflexes but limited BD
VA good bronchodilator BUT does’t suppress airway reflexes; use for intra-op bronchospasm but not induction/emergence (avoid des)
Don’t use VA for laryngospasm!
Better to do IV vs inhalation induction

1412
Q

Strategies to limit risk for chn w URTI needing to proceed:

A

Avoid BZD, if need premed use clonidine
Consider IV lignocaine to suppress laryngospasm but avoid topical lignocaine to VCs
Pre-op salbutamol neb 2.5mg for chi w current & recent (<2wks) URTI
Least invasive device (FM vs LMA vs ETT)
Deep removal of airway device is ass’d w incr airway obstruction despite reducing laryngospasm & PRAE
Experienced anaesthetist
Propofol blunts airway reflexes but limited BD
VA good bronchodilator BUT does’t suppress airway reflexes; use for intra-op bronchospasm but not induction/emergence (avoid des)
Don’t use VA for laryngospasm!
Better to do IV vs inhalation induction

1413
Q

Shelf life adult red cells
Paeds red cells
Washed red cells
Platelets
FFP/cryo
extended life plasma is thawed FFP that can be stored @ 2-6deg for max

A

42 days
35 days
28 days
5 days (agitator)
12 months if <-25deg (FFP can be used for up to 24hrs if kept 1-6degc). either can usewithin 4 hrs afer thawing if at room temp. don’t put cryo in fridge (may re-precipitate).Blood products through 170-200micron filter.
5 days

1414
Q

Placental abruption

A

After 2-4% of minor & up to 50% major obs trauma
Clinical findings: PV bleed, abdomen cramp, uterine tenderness, leakage amniotic fluid, maternal hypovolaemia, larger than normal uterus for GA, change FHR
After trauma, PV bleed ominous sign
Trans abode US < 50% accurate for placental abruption, CTG more sensitive (foetal distress)
Minimum 4hrs CTG, 24 if frequent uterine activity, abdominal tenderness, PV bleed, ROM, hypotension

1415
Q

Placental abruption

A

After 2-4% of minor & up to 50% major obs trauma
Clinical findings: PV bleed, abdomen cramp, uterine tenderness, leakage amniotic fluid, maternal hypovolaemia, larger than normal uterus for GA, change FHR
After trauma, PV bleed ominous sign
Trans abode US < 50% accurate for placental abruption, CTG more sensitive (foetal distress)
Minimum 4hrs CTG, 24 if frequent uterine activity, abdominal tenderness, PV bleed, ROM, hypotension

1416
Q

Highest bacterial kill rate @ 30 secs

A

alcohol (immediate antimicrobial stronger & quicker but no residual effect)

1417
Q

Rate of correction chronic hyponatraemia to limit risk cerebral demyelination?

A

4-6mmol/24hrs

1418
Q

BTF recommended CPP for survival & favourable outcome

A

60-70mmHg

(& also, jugular venous sats <50% may be threshold to avoid to reduce mort & improve outcomes)

1419
Q

PFT pattern CF

A

mixed obstructive/restrictive

1420
Q

difference paeds (<9yo) vs adult rule of 9s

A

Head 18% (adult 9%)
Each leg 14% (adult each leg 18%)

1421
Q

WOMAN trial:

A

TxA cf placebo reduced death due to bleeding but not death from all causes or hysterectomy
if gave witin 3hrs, reduced death
no diff thromboembolic events, complications (incl seizures), surgical interventions

issues: low fragility index, change power calc & hypothesis after commencement

1422
Q

Normal CI & level indicating cardiogenic shock?

A

2.6-4.2, <2.2mL/min2 suggests cariogenic shock

1423
Q

First sign total spinal child?

A

Unconsciousness

Less CV changes; sudden apnoea, LOC & dilated pupils

Factors for total spinal:
Drug: incr dose
Baricity
Prior drugs (eg. Epidural)
Pt:
Higher BMI or abdominal girth or pregnancy, compressed thecae sack (eg. Spinal canal anomaly), dwarfism
Technical: higher insertion, lying vs sit, cephalic direction

1424
Q

What’s the standard deviation
What’s the variance?

A

Square root of the variance

Sum of the squared deviation from the mean

1425
Q

Which measure of central tendency is used for nominal data?

A

Mode

1426
Q

Which measure of central tendency is used for nominal data?

A

Mode

1427
Q

Periop instructions irreversible MAO-Is? reversible?

A

Withold 2 wks preop
Reversible discontinue day of OT

1428
Q

Donepezil MOA, anaes implications

A

Anti cholinesterase inhibitor for mod-severe AZD (as is rivastigmine, galantamine)

Antagonise ND-NMBDs (may get ACh receptor down regulation) Require higher dose of ND-NMB, ND-NMB have decr duration of action.

Donepezil, galantamine & rivastigmine enhance effect of sux, risk phase=II interaction.

Neostigmine may be relatively ineffective, incr doses may be required.

Should cease pre-op: galantamine & rivastigimne have short half lives (7-8 & 3-4hrs, respectively), can discontinue the day before surgery. Donepezil has long half-life 70hrs, washout period 2-3wks. BUT risk decline cognitive function so multi-D consideration.

If continue, could use attach or cis (spontaneously inactivated but still need higher doses of these) OR could use sugammadex reversal if use lg doses roc or vec.

1429
Q

Donepezil MOA, anaes implications

A

Anti cholinesterase inhibitor for mod-severe AZD (as is rivastigmine, galantamine)

Antagonise ND-NMBDs (may get ACh receptor down regulation) Require higher dose of ND-NMB, ND-NMB have decr duration of action.

Donepezil, galantamine & rivastigmine enhance effect of sux, risk phase=II interaction.

Neostigmine may be relatively ineffective, incr doses may be required.

Should cease pre-op: galantamine & rivastigimne have short half lives (7-8 & 3-4hrs, respectively), can discontinue the day before surgery. Donepezil has long half-life 70hrs, washout period 2-3wks. BUT risk decline cognitive function so multi-D consideration.

If continue, could use attach or cis (spontaneously inactivated but still need higher doses of these) OR could use sugammadex reversal if use lg doses roc or vec.

1430
Q

How long keep giving dantrolene?

A

until end-tidal carbon dioxide is < 45 mmHg and core temperature is <38.5oC (cool until 38 deg C, prevent a fall to < 36deg C)’maximum 10mg/kg has been quoted but sometimes need to exceed this for crisis (latest guidelines no ceiling)
actual body weight, max 300mg initial dose.

1431
Q

How long keep giving dantrolene?

A

until end-tidal carbon dioxide is < 45 mmHg and core temperature is <38.5oC (cool until 38 deg C, prevent a fall to < 36deg C)

1432
Q

To what do charcoal filters reduce volatile []?

A

<5ppm
Before place the activated charcoal filters, flush machine w high-flow O2 or air (any mix) >10L/min for 90s. Place charcoal filters on insp & exp limbs, reduce FGF 1-3L/min w filters, change them after 12hrs.

1433
Q

Risks of MRI w pacemakers

A

Static field: movement of pulse generator’s internal components
Modulated RF field: induces voltage across pacemaker electrodes, heating cardiac tissue
Gradient magnetic field: over- and under-sensing
Risk/benefit decisions, reprogramming in liaison w cardiologist, ALS equip & personnel, continual monitoring, consent, interrogation post examination

Lithotripsy at least 15cm from pacemaker, use low shock waves.

1434
Q

Which medication attenuates analgesic effects of salmon calcitonin?

A

Ondansetron (5HT3 antagonists)
SC reduces pain & improves mobilisation after osteoporosis-related vertebral compression fractures. Reduces acute NOT chronic PLP.
Causes sedation, nausea, skin flushing, serotonertic activity.

1435
Q

widespread saddle ST segments suggests?

A

acute pericarditis

1436
Q

Differentiating dextrocardia & limb lead reversal

A

For limb lead reversal, normal R wave progression in precordial leads
BOTH limb lead reversal (LA/RA reversal), lead 1 complex totally inverted, aVR positive complexes, RAD

1437
Q

how much CRYO will increase fibrinogen by 0.5-1g/L?

A

1 unit per 5-10kg body weight, so for 70kg pt, 7-14 units

1438
Q

what’s the positive likelihood ratio?

A

Sensitivity/ (1-spec) how much more likely is it that a patient who tests positive has the disease compared with one who tests negative.

-ve likelihood ratio= 1-sens/spec

1439
Q

Adr dose moderate pads anaphylaxis?

A

2 mcg/kg (0.1mL.kg)

life-threatening 0.2-0.5mL/kg (4-10mcg/kg)

Draw up 1mg in 50mL= 20mcg/m

infusion 0.3-6mL/kg/hr (0.1-2mcg/kg/min)

1440
Q

Best Rx for prevention post-herpetic neuralgia?

A

Amitryptilline (low doses for 90 days from onset HZ rash). (?NNT 2), gabapentin NNT 3.2)
PHN= chronic pain 3/12 after HZ.
Incr risk older age, female, worse acute pain, severe rash, dermatomal pain before the rash.

Immunisations >60yo reduces both HZ & post-herp neuralgia

Start antivirals within 72hrs to accelerate acute pain but that doesn’t reduce incidence/severity/durn of neuralgia.

CONTINUOUS OR REPEAT PVB in acute phase HZ reduce incience of PHN at 3,6,12 months!

Continuous epidural in acute phaze of HZ reduces incidence of PHN at 312

Mx w gabapentanoids (pregabalin & 5% lignocaine plaster more effective than TCAs (APMSE), Gabapentin, TCAs (amitriptyline, nortriptyline may have better SE profile), consider lignocaitn (multimodal). Capsaicin cream or patch. Opioids if not responding.
UTD: start with low dose gabapentin or pregabalin, up-titrate to limit SEs
amitriptylline 2nd line (may not tolerate SEs, nortriptylline may be better (anticholinergic; sedation & dry mouth, care ++ in elderly

Early appropriate analgesia vital in management of HZ, may assist reduction incidence PHG.
Nerve blocks in acute phase reduce duration of HZ-associated pain. Topical aspirin, lignocaine, CR oxycodone help acute pain.

1441
Q

Electrolyte abnormality after LL crush injury?

A

hypocalcaeima

1442
Q

Parecoxib (–> valdecoxib) MOA

A

inhibit COX-2 mediated PG synthesis
Onset analgesia 7-14mins, peak within 2 hrs
DOA 6 to >24hrs (great for day stay)

1443
Q

Best indicator response to DKA resus

A

Improvement ketones

1444
Q

Delegation of staff during neonatal resus:

A

Aim: simultaneous Ax & management, goal effective spont or assisted respns within 1 minute.
Equipment: resuscitaire on, warmer 27 deg, towels to warm bub & for btwn scapulae, flowmeter 10L/min, blender 21% (term), Neopuff PEEP 5cmH2O, PIP 20-25cmH20 preterm, 30cmH2O term, PPV rates will be 40-60/min 10Fr suction, size 0.1 laryngoscope, size 3,3.5 tracheal tubes, T-piece, self- or flow-infl 240mL bag, have Adr, fluids (slaline 10mL/kg, blood 10mL/kg), 5Fr umbi catheter ready in case prolonged resus
Monitoring: pulse ox for R) side, pre-ductal, stethoscope
Senior anaes as TL, airway, CPR
2nd clinician for Ax Hr, ventilation
Post-resus care: handover to peds/neonatal team, debrief

1445
Q

Which infusates need central access?
Minium size PICC if need to sample?

A

Those with vesicant properties (blister potential & tissue injury eg. calcium, amphotericin B), hypertonic solutions (max osmolarity peripheral 500mOsmol/L, >10% DEXTROSE OR >5% PROTEIN), those with pH <5 or >9, vasopressors & inotropes

3Fr
basilic vein above elbow= preferred PICC insertion site(cephalic vein acute angle @ SC junction)
PICC should be in distal third SVCvc or cavo-atrial junction, if LL to IVC. confirm tip position, UL PICCs move avg 2.2 rib spaces w UL movement soinsert w line fixed so tip optimally placed when arm comfortable). If a CVC or for high flows (eg. HD), RA better.

consider if duration therapy 10 days-2 months. can be inserted & revoved without GA in SOME chn. Generally tunnelled if instionn >6wks

Generally use 10mL or larger syringes to flush as smaller higher pressures risk damaging lines. Care with volumes!

MIDLINES ARE 8-12 CM, FOR PERIPHERAL SOLUTIONS ONLY, STILL STERILE GOWN/MASK/1 MIN HAND SCRUB & STERILE GLOBES DUE TO DWELL TIME (usually 6-10 days, may be longer). FAR AWAY ENOUGHT FROM ELBOW THAT CAN FLEX, ENSURE THE 8-12CM DOESN’T CROSS THE AXILA.

1446
Q

When can umbilical vein catheters be used?

A

First 7-10 days of life. 2.5-8Fr (5Fr for newborn Resus). Ideally terminates in IVC above diaphragm but for Resus, can use once free flow of blood on aspiration achieved. Generally 4cm depth insertion (first mark 5cm).
Not if abdominal wall defects, peritonitis, nec enterocolitis.]

1447
Q

Central lines paeds:

A

generally 5cm lines for chn <15kg, 8cm 16-40kg & 13cm if >40kg. 4-5Fr if <6/12, 5Fr if 6/12-5yrs, 7Fr if >5yrs. Confirm tip position.
Confirm tip position by radiography.

1448
Q

art lines paeds

A

24-22g, 22-20g for neonates
if <22g, need 0.012 diameter wire. paeds, art line fluid continuously infused is 1-2mL/hr vs 3mL/hr w pressurised fluid bag. Paeds use syringe drive. Also have special needle-free access port (limit deadspace & conamination). Syringe driver rather than pressure bag (limits volume infused, prevent clotting of line). low volume tubing.

1449
Q

TCA MOA & toxicity

A

Block NAdr & serotonin reuptake.

block fast Na channels–> QRS prolongation, inhibit K channels, direct myocardial depression. also block muscuarinic histamine & alpha 1 adrenergic receptors.

So, sedation/coma, seizures, hypoT, tachy, broad complex dysrhythmias & anticholinergic syndroem in OD.
Rx w IV, high flow O2 (ABC)
sodibic 1mmol/kg until BP improves & QRS narrows.
hyperventilate to pH 7.5-7.55.
once airway secure, NGT & 1g/kg activated charcoal.
Rx seizures q IV benzo.
treat hypotension w crystalloid.
consider vasopressors.
ongoing arrhythmias, more sodibic, lignocaine is a 3rd line agent after bicarb & hyperventilation.
avoid 1a (procainamide) & 1c (flecainide) antiarrhythmics, B blockers & amiodoarone (may worsen hypoT & conduction abnormalities). ICU

1450
Q

TCA MOA & toxicity

A

Block NAdr & serotonin reuptake.

block fast Na channels–> QRS prolongation, inhibit K channels, direct myocardial depression. also block muscuarinic histamine & alpha 1 adrenergic receptors.

So, sedation/coma, seizures, hypoT, tachy, broad complex dysrhythmias & anticholinergic syndroem in OD.
Rx w IV, high flow O2 (ABC)
sodibic 1mmol/kg until BP improves & QRS narrows.
hyperventilate to pH 7.5-7.55.
once airway secure, NGT & 1g/kg activated charcoal.
Rx seizures q IV benzo.
treat hypotension w crystalloid.
consider vasopressors.
ongoing arrhythmias, more sodibic, lignocaine is a 3rd line agent after bicarb & hyperventilation.
avoid 1a (procainamide) & 1c (flecainide) antiarrhythmics, B blockers & amiodoarone (may worsen hypoT & conduction abnormalities). ICU

1451
Q

TCA MOA & toxicity

A

Block NAdr & serotonin reuptake.

block fast Na channels–> QRS prolongation, inhibit K channels, direct myocardial depression. also block muscuarinic histamine & alpha 1 adrenergic receptors.

So, sedation/coma, seizures, hypoT, tachy, broad complex dysrhythmias & anticholinergic syndroem in OD.
Rx w IV, high flow O2 (ABC)
sodibic 1mmol/kg until BP improves & QRS narrows.
hyperventilate to pH 7.5-7.55.
once airway secure, NGT & 1g/kg activated charcoal.
Rx seizures q IV benzo.
treat hypotension w crystalloid.
consider vasopressors.
ongoing arrhythmias, more sodibic, lignocaine is a 3rd line agent after bicarb & hyperventilation.
avoid 1a (procainamide) & 1c (flecainide) antiarrhythmics, B blockers & amiodoarone (may worsen hypoT & conduction abnormalities). ICU

1452
Q

Best way to monitor success of thyroid treatment

A

Free T4, sometimes T3, improvements may take 6-8 wks. TSH interpret with caution as may lag.

1453
Q

Best way to monitor success of thyroid treatment

A

Free T4, sometimes T3, improvements may take 6-8 wks. TSH interpret with caution as may lag.

1454
Q

what are vaptans & indications?

A

Vasopressin receptor antagonists, used for euvolaemic or hypoevolaemic hyponatremia, SEs thirst, incr urination, dry mouth.

Also fluid restrict <1.5L/day if the urine Na + K divided by serum Na is <1, if the ratio is >1, <1L/day
incr solute (sodium, proetin, urea)

Inc solute intake with frusemide incr UO but risk hypokalemia
Incr daily protein (urea therapy)
Tolvaptam causes thirst
Glucocorticoid deficiency or hypothyroidism: manage underlying cause (thyroxine or hydrocort)
No urea or vaptans if awaiting transplant

1455
Q

VCV & PCV

A

PCV:
Delivers constant square pressure waveform with Pins being the control variable, initial higher flow early in insp phase to achieve this, volume variable depending on lung compliance
Adv= good for hypoxia & poor lung compliance. Overall incr mean airway pressures, may improve oxygenation & be beneficial if hypoxaemia (in practice since PEEP is the dominant influence on mean airway pressure, need relatively high I:E ratios for this benefit), square waveform improves alveolar recruitment, early higher flow may reduce WoB, improve pt comfort & promote less ventilatory dys-synchrony, limited peak pressure helps limit barotrauma risk, pressure control variable allows for leak.Suggested teaching in ARDS is for PCV but this has weak evidence. PCF for OLV. Laparoscopy PCV.
-ves: variable volume so may be hard to control hypercapnia. Uncontrolled volume risk volutrauma. High early insp flow may breach pressure limit early if the pt has high airway resistance & may end up not having good volumes.

VCV: delivers set volume, fairly constant flow, pressure waveform parabolic slope as lungs distend.
Useful if wanting strict control PaCO2 (eg. Neurosurgery) & situations where stable MV important. potentially disadvantageous wrt ventilatory dyssynchrony (lower initial flow). Useful to have stable MV over range of changing pulmonary characteristics eg. Resistance fluctuations. Lower initial flow rate advantage if airway resistance high.
Disadvantage: lower Pawmean, theoretical disadvantage if severe hypoxia. Insp pause doesn’t significantly help & may incr WoB. May get poorer recruitment in lung areas w poorer compliance (long time constant). Volumes may be stable if there’s a leak. Insufficient flow may—> pt-vent dyssynchrony.
Pre-op:
To determine the severity & stability (deterioration/exacerbations) of relevant comorbidities & modifiable/optimisable risk factors
Hx/exam/Ix:
In addition to my standard anaesthetic assessment (incl thorough airway assessment), I’d specifically look for:
Functional assessment: ADLs
Multi-D planning/optimisation: consultations: who want & what questions
Targets aiming for with optimisation

1456
Q

CCS angina tool

A

1: symptoms only strenuous/prolonged exertion
2: ordinary activity slightly limited (eg. Walk/stairs rapid, after meals, cold/wind, emotional stress, >2 blocks on level ground or >1 flight stairs)
3: marked limitation ordinary activity (1 flight stairs, 1-2 blocks flat)
4: unable to undertake ordinary physical activity without discomfort, may have symptoms @ rest

1457
Q

time IM sux takes to break laryngospasm

A

45-60 secs (3-4mins to full paralysis).
if use intralingual, 2mgkg & full relaxation in 75 secs, so faster onset than IM

1458
Q

how far place defib pads from an implanted defib?

A

8cm

1459
Q

diameter of scavenger tubing

A

19 or 30mm

1460
Q

diameter of scavenger tubing

A

19 or 30mm

1461
Q

Which drug does not impact thrombin time?

A

Warfarin (& lmwh, fondaparinux or direct Xa inhibitors)- predominantly or exclusively inhibit Xa.
dabigatran, argatroban, bivalirudin & DO impact TT.

1462
Q

When to medically intervene with a seizure post ECT?

A

120secs, half induction dose of propofol or BZD

1463
Q

For pts @ high risk of vasoocclusive crises in sickle cell disease, what’s the target HbS level w transfusion?

A

<30%

1464
Q

What does overlapping confidence intervals between the means of 2 groups mean?

A

diff btwn the 2 groups not stat sig

1465
Q

Which thrombophilias hae highest relative risk of initial VTE?

A

ATIII deficiency (10-30% annual risk first VTE), also very high risk in preg

Anitphospholipid syndrome also very high RR initial VTE & the highest absolute risk fo VTE in preg

Compound FVL & Prothrombin 20210A gene mutation heterozygotes have elevated risk in pregnancy (as do either here or homo). APC resistance is common in pro, worsening risk of pts w FVL having thrombotic diathesis in pregnancy

1466
Q

Diff in sore throat with normal vs videolaryngoscope?

A

No difference.

VL may reduce number of failed intubations (esp if diff airway), may improve glottic view & reduce laryngeal/airway trauma but no evidence suggests they reduce the number of intubation attempts, hypoxia or resp complications or impacts time for intubation.

Intraosseous access:
Medullary cavity. Direct, non-collapsible access to central venous circulation. Venous sinusoids, emissary veins, systemic. Recommended if don’t get IV access within 90s in critically ill children.

1467
Q

Aprepitant Pk

A

POBA 60%, t1/2B 9 hrs, CYP3A4, take daily 1hr prior to chemo or pre=op, for highly emoetogenic, interfere w OCP. Causes GORD, tacky, abdominal discomfort, hiccups, tachypnoea.

1468
Q

L5 radiculopathy no improvement after 1 week NSAID, best option?

A

or acute radicular pain, lumbar epidural corticosteroid administration is effective for short-term relief (not long-term)

1469
Q

L5 radiculopathy no improvement after 1 week NSAID, best option?

A

or acute radicular pain, lumbar epidural corticosteroid administration is effective for short-term relief (not long-term)

1470
Q

Part of the stuomach most sensitive to US for residual gastric content?

A

he gastric antrum is of particular interest as it is easily identifiable on ultrasound in the epigastric region. It is also the more dependent area of the stomach, meaning any gastric content will gravitate towards this region. Th

1471
Q

how long run Mg++ in fusions post-delivery in PET?

A

24hrs

1472
Q

what’s impaired after lung transplant?

A

Vagus & sympathetic nerves, pulmonary & bronchial vessels & lymphatics are interrupted. Pulmonary art & vv are reanastamosed but not bronchial arteries.
Cough reflex is interrupted.
Vent response to CO2 initially may be impaired but resolves.

1473
Q

Agreement between VBG & ABG?
So, what expect PaO2 to be on abg if it’s 7.29 on VBG?

A

pH (good correlation, pooled mean difference 0.035pH units; vbg usually 0.03-0.04 pH units), pCO2 (if normal & pt not shocked however correlation poor with PaCO2 >45mmHg)), bicarb & BE from a VBG usually adequate for clinical decision making however lactate above 2, CO2 above 45 lack of correlation & PO2 values compare poorly
VBG used to guide DKA
ABG necessary for PaCO2 in severe shock, to accurately determine PaCO2 if hypercapnia, to accurately determine arterial lactate >2mM

7.32

1474
Q

Agreement between VBG & ABG?
So, what expect PaO2 to be on abg if it’s 7.29 on VBG?

A

pH (good correlation, pooled mean difference 0.035pH units; vbg usually 0.03-0.04 pH units), pCO2 (if normal & pt not shocked however correlation poor with PaCO2 >45mmHg)), bicarb & BE from a VBG usually adequate for clinical decision making however lactate above 2, CO2 above 45 lack of correlation & PO2 values compare poorly
VBG used to guide DKA
ABG necessary for PaCO2 in severe shock, to accurately determine PaCO2 if hypercapnia, to accurately determine arterial lactate >2mM

7.32

1475
Q

Tumour lysis syndrome:

A

Ass’d w acute leukaemia & high-grade lymphomas. May be sopnt, usually after chemo & occ w single-therapy dexamethasone. hyperK, renal failure, acidosis. Hyperphosphataemia, incr uric acid (serum & urine), hypocalcaemia.
Mx w aggressive fluid, hyperK Rx, consider RRT, consider rasburicase (urate oxidase).

1476
Q

Factors attenuating HPV

A

Alkalosis
ACE-I
Endothelia receptor antagonists
Prostacyclin analogues
Phosphodiesterase inhibitors
Vasopressin
Acetazolamide high doses
NO
Hypothermia
Volatiles

Factors promoting incr pulmonary pressures via VC (hence promote HPV):
Acidaemia
Hypercapnia
Hyperaemia
Catecholamines
Hyperthermia
Newborn/infants (HPV response reduces w time)
Fe deficiency

1477
Q

RIsks w ferric carboxymaltose

A

Nausea
Hypophosphataemia
Injection site reactions (incl skin discolouration)
Headache
HTN
dizziness

1478
Q

Hypercalcaemia management:

A

Incr Ca++ excretion (N/S IVF promotes renal elimination)
Loop diuretics: dear resorption of Ca++ in LoH
steroids: inhibit effects of vi D (which promotes GI & renal Ca++ reabsorption)

Reduce Ca++ release:
Calcitonin (inhibit osteoclasts)
Bisphosphonates (inhibit osteoclast)
Inorganic phosphate

Stop drugs that may incr Ca++: Ca, via D, thiazides)

1479
Q

How does ondansetron reduce tramadol efficacy?

A

CYP2D6 metabolism so less O-desmethyl tramadol production; O-desmethyl tramadol is 200x more potent @ MOP (tramadol is racemic, S=SSRI & mu, R=SNRI)

1480
Q

Clinical signs severe AS:

A

Palpable systolic carotid thrill or precordial esp leaning forward on expiration
Paradoxical splitting S2
Signs LV failure

1481
Q

Contents NaCl 0.9%
Hartmann’s
Plasmalyte

A

Na & Cl 154mmol/L. osm 300
Na+ 131, Cl- 111, Ca++ 2, K+ 5, lactate 28 (bicarb), 276mosmoL/L (248 effective, the lactate doesn’t count- otherwise 276)
Na+ 140, Cl- 98, K+ 5, Mg++ 1.5, acetate 27, gluconate 23, osm 294mOsmol/L (271 product info)

1482
Q

Block procedure: pop sciatic:

A

Indications: foot & ankle surgery, BKA, post compartment knee
Amenable to catheters

Consent
IV
Monitoring: as per PG03, BP, GCS, RR + have SpO2 (if sedation, on) & ecg avail
Assistant: trained/skilled/briefed
Position: prone, lat or supine knee flexed, ergonomically for clinicians & comfy ofr pt
Prep: chlorhex allow to completely dry
Local ready: 15-20mL 0.75% & 1% for skin, syringe 20mL for block & 5mL for skin
US, sterile sheath
Linear transducer, 100mm sonoplex or 8cm epidural needle if catheter
SB4UB:
Pt verify too, use consent form to cross check with nurse, pt ID, side, check site marking, allergies. Before needle, re-verify site (marking visible) & re-verify if change sides or pt position.

DOCUMENT block, side, dose LA (% & volume), any catheter instructions (these should be designated marked specific to regional eg. Yellow, protocol for their management), post monitoring instructions, any adverse events, f/up plan & contact if queries.

1483
Q

Block procedure: pop sciatic:

A

Indications: foot & ankle surgery, BKA, post compartment knee
Amenable to catheters

Consent
IV
Monitoring: as per PG03, BP, GCS, RR + have SpO2 (if sedation, on) & ecg avail
Assistant: trained/skilled/briefed
Position: prone, lat or supine knee flexed, ergonomically for clinicians & comfy ofr pt
Prep: chlorhex allow to completely dry
Local ready: 15-20mL 0.75% & 1% for skin, syringe 20mL for block & 5mL for skin
US, sterile sheath
Linear transducer, 100mm sonoplex or 8cm epidural needle if catheter
SB4UB:
Pt verify too, use consent form to cross check with nurse, pt ID, side, check site marking, allergies. Before needle, re-verify site (marking visible) & re-verify if change sides or pt position.

DOCUMENT block, side, dose LA (% & volume), any catheter instructions (these should be designated marked specific to regional eg. Yellow, protocol for their management), post monitoring instructions, any adverse events, f/up plan & contact if queries.

1484
Q

Goals for pyloric stenosis resus

A

Volume status corrected
UO >1mL/kg/hr
K+ >3.5
Bicarb <26
Cl- >100

1485
Q

Why helium preferred in IABP

A

ts low density facilitates rapid transfer of gas from console to the balloon. It is also easily absorbed into the blood stream in case of rupture of the balloon.

1486
Q

How to avoid gas trapping in COPD

A

Low RR, prolong exp time, reduce minute ventilation, rase insp flow rate (which shortens insp time & prolongs exp time), permissive hypercapnia gen ok depending on pt factors, pH >7.2, haemodynamics.

1487
Q

most periop strokes occur

A

postop, D2

1488
Q

Steps if bleed after PAC

A

Go back on bypass if catheters still in
bleeding lung side down, DLT for lung protection, PEEP bleeding lung, draw PAC back deflated (DON’T WEDGE), reverse coagulation, bronchoscopy to locate site bleeding& consider BB, PEEP bleeding lung
Confer with friends, consider surgical pulmonary artery repair, lung resection, or angiographic embolization

1489
Q

Which central lines most frequently complicated by thrombosis?

A

PICCS (virchow’s: trauma endothelial injury, hypercoaguable pts)
pt: prev Hx DVT, obesity, malign, critical illness, DM, COPD
Device: incr number of lumens, L)-sided, exchanging over guidewire, catheter relatively large cf vein into which inserted (C: vein ratio optimal <45%),
Cephalic higher rate than basilic, more proximal (eg. Axillary)—> lower rates DVT.
Provider: pt selection, proper placement & positioning, US-guidance, minimal eattempts, appropriate catheter selection, PICC tip @ CAJ or RA (greater blood flow, lower risk VTE; higher rate VTE if tip terminates in mid to pro third of SVC. Electrocardiographic guidance may be ass’d w lower rates DVT than radiographic.

1490
Q

How to tell that PICC in correct position via ecg guidance?

A

Maximum height of P wave, suggests tip at SA node at cavo/RA junction; biphasic when in RA, inverted suggests approaching RV

1491
Q

How to tell that PICC in correct position via ecg guidance?

A

Maximum height of P wave, suggests tip at SA node at cavo/RA junction; biphasic when in RA, inverted suggests approaching RV

1492
Q

NMBD w least cross-reactivity

A

Cis<atrac<panc<vec<roc<sux

1493
Q

2015-2017 mortality report ANZCA greatest cause of anaesthesia related death

A

Anaphylaxis (8 cases), aspiration 7

1494
Q

pancoast tumour

A

Tumour @ lung apex
NSCLC, may spread to ribs & vertebrae, may compress brachiocephalic vein, subclass artery, phrenic n, RLN, vagus, stellate ganglion— Horner’s syndrome
Brachial plexus involvement—> pan coast syndrome
Rx may involve radiation/chemo neoadjuvant, surgery may= r/o upper lobe & ass’d structures & mediastinal LN.

1495
Q

Absolute contraindications to systemic thrombolytic therapy in acute PE

A

Absolute contraindications to systemic thrombolytic therapy in acute PE include an intracranial neoplasm, aortic dissection, recent (ie, <2 months) intracranial or spinal surgery or trauma, history of a hemorrhagic stroke, active bleeding or bleeding diathesis (eg, severe thrombocytopenia), or nonhemorrhagic stroke within the previous three months. Relative contraindications include severe uncontrolled hypertension (ie, systolic blood pressure >200 mmHg or diastolic blood pressure >110 mmHg), non-hemorrhagic stroke more than three months prior, surgery within the previous 10 days, pregnancy

1496
Q

Absolute contraindications to systemic thrombolytic therapy in acute PE

A

Absolute contraindications to systemic thrombolytic therapy in acute PE include an intracranial neoplasm, aortic dissection, recent (ie, <2 months) intracranial or spinal surgery or trauma, history of a hemorrhagic stroke, active bleeding or bleeding diathesis (eg, severe thrombocytopenia), or nonhemorrhagic stroke within the previous three months. Relative contraindications include severe uncontrolled hypertension (ie, systolic blood pressure >200 mmHg or diastolic blood pressure >110 mmHg), non-hemorrhagic stroke more than three months prior, surgery within the previous 10 days, pregnancy

1497
Q

Most sensitive sign of PTx using US

A

Lung point, 100% specific, 100% PPV when seen but may not be seen in large PTx
presence of lung sliding 100% NPV for PTx but absence of lung sliding not specific enough to r/o PTx in criticallhy ill (DDx inflammatory adherences)
Blines help rule out PTx

1498
Q

Magnesium in eclampsia, loading/infusion/boluses if furtherseizures?

A

4g over 20 mins (preprepped 4g/100mL
0, 1g/hr, 2g bolus over 5 mins if further seizure (can repeat 5 mins later)

1499
Q

interventions to reduce SSI in known MRSA colonisation

A

Reasonable for pts known s aureus carriers with high risk SSI (cardiothoracic, ortho, hardware implantation, immunocompromised)

Mupirocin 2% to nares BD for 5 days
Chlorhex 2% or 1% triclosan washday 5 days
vancomycin 1g (1.5g if >80kg actual body weight), commence 120mins prior to incision (should be given at a rate of 1g over at least 60 minutes and 1.5g
over at least 90 minutes (in adult patients). Vancomycin should be timed to begin 15 to 120 minutes before skin incision.
teicoplanin 15mg/kg up to 800mg (400mg if <=80kg), 5mins before procedure (?15-120mins prior to skin incision)

1500
Q

Normal cerebral oximetry:

A

60-80 but lower values (eg. 55-60) may not be abnormal in some cardiac pts. Act if desaturated >20%

Depends on adequate CBF & O2 content; reduction in CBF (decreased CO, in/outflow obstruction (eg. Head position/ties, vasoconstriction eg. If hypocapnia) may reduce values, as may decrease O2 content: inadequate ventilation, low HCt or incr cerebral metabolism.

Course of actions:
Optimise DO2 (incr CO, incr MAP, FiO2, PaCO2 (optimise ventilation), consider transfusion (r/o anaemia)
optimise venous drainage (neutral, ET ties
Optimise O2 consumption: adequate depth & analgesia, temp, r/o seizures.

1501
Q

massive transfusion paeds

A

> 40mL/kg in 24h or >20mL/kg in 2hrs & anticipated ongoing loss

Key signs: narrow pulse pressure/lactic acidosis/reduced UL

est blood volumes preterm 90-100mL/kg, term-3mths 90mL/kg, >3 mths 80mL/kg

NO permissive hypoT

cryo , plt, FFP rbc 10mL/kg
fib conc 70mL/kg
TxA 15mg/kg
Ca 0.3mL/kg gluconate

1502
Q

Safest med for neuropathic pain in preg

A

Gabapentin or pregab, both B3
Other meds carbamazepine, lamotrigine, phenytoin, valproate are cat D

1503
Q

ISB, diaphragm twitch (phrenic stimulation)

A

Withdraw needle, insert 15deg post & lateral

1504
Q

main action of insulin in euglycaemia in preventing hyperglyc

A

Under euglycemic conditions, most insulin-mediated glucose uptake occurs in muscle

1505
Q

Prader Willi

A

hypothalamic-pituitary abnormalities & severe hypotonia in infancy, hyperphagia w risk morbid obesity, learning difficulties & behavioural problems or psychiatric problems.hypotonia at birth partially improves with age.

characteristic narrow forehead, almond-shaped eyes, thin upper lip, down-turned mouth, very small hands & feet. GH deficiency–> short stature.

decreased bond mineral density without altered Ca, phosphate, vit D or parathyroid hormone.
cognitive dysfunction- food-seeking & obsessive behaviours.

clinically & genetically heterogenous. involves chromosome 15. mostly sporadic.

early Dx, multi-D care & GH treatment.

Complications linked to obesity.

Anaes considerations:
-aggression/violence
-convulsions
-morbid obesity
-sleep apnoea
-sensitivity to sedative/hypnotic agents.
Incr risk post hyoxia & hypercapnia independent of obesity; altered CO2 & O2 responses

-potentially difficult airway; obesity, dentition, neck circumference, may have micrognathia/palatal abnormalities/limited neck mobility. Have difficult airway equipment. Aspiration risk (GI motility, obesity).Rumination, stealing Bx so risk of aspiration is extreme, treat as unfasted can get regard even >10 hrs after last ate
Abnormally thick & viscous saliva (glycol premed)

-risk periop Resp failure; kyphoscoliosis, obesity, hypotonia may predispose to restrictive lung disease. OSA risk obstruction. consider polysomnography.

-primary myocardial involvement (*predisposition to cardiovascular complications is independent of obesity); consider 12-lead & echo. Cor pulmonate. Arrhythmia.

-HTN

-Hypotonia (improves beyond 2yo); can still use NMBDs but monitor closely. Sux has been used safely.

-difficult IV access; US

-thermoregulation disturbance

-Glucose intolerance & DM

-prone to hypoglycaemia after long periods of fasting, disturbed thermoregulation (tendency towards poikilothermia), may get hyper or hypothermia, occ pemetabolic acidosis; 50% insulin dependent, tend to use circuling glucose for lipogeneiss vs basal metabolic need

1506
Q

TUrner
Noonan

A

Turner:
Short stature
DIFFICULT AIRWAY: May have micrognathia, short webbed neck
Ovarian dysgenesis
XO karyotype
CARDIAC: 30% bicuspid AV, 10% coarctation
50% renal abnormalities

Noonan:
Phenotypically similar Turner: short, webbed neck, may have micrognathia
CARDIAC: PV stenosis, HCM, ASD, ToF, aortic coarctation, MV anomalies, AV canal defects
Hydronephrosis
Put dysfunction

1507
Q

CHARGE
VACTERL

A

Ocular Coloboma
Heart defects eg. R)-sided aortic arch, VSD, ASD, PDA, ToF, double outlet RV
Atresia choanal (diff airway), growth
Retardation of development
Genital anomalies
Ear anomalies/deafness

Vertebral anomalies
Anal atresia
Cardiac defects eg. VSD
Tracheo-oesophageal fistulae
Renal abnormalities
Limb (radial) dysplasia

1508
Q

TxA dosing

A

15mg/kg then 1mg/kg/hr infusion
Novoseven IV bolus 50micro/kg if critical, ongoing bleeding (d/w haem).

1509
Q

Risk dental damage anaesthesia

A

1:4500, 5x incr risk if pre-existing pathology, incr if age 50-70yo, 3.4x incr risk if dental restorations, 20x incr risk difficult to intubate, NOT incr if emergency or inexperienced laryngoscopists. 20% w emergence.

1510
Q

Risk factors acute cerebro-[0pntine demyelinitis

A

4-6mEq/L in first 2hrs 1-2hrs, achieve with 100mL boluses over 10 mins; recheck Na+ after each.
Overall, if low risk no >10mEq/L in first 24 hrs & no >18mEq first 48hrs

High risk if <105Na+, hypoK, malnutrition, ETOH, liver disease

If Na+ overcorrected, may need to give desmopressin

Seek cause once pt stabilised

1511
Q

B, C cylinder D cylinder O2 capacity

A

B= 170
C=490
CD=630L

1512
Q

Congenital diaphragmatic hernia:

A

May be diagnosed in-utero
Presents with Resp distress, scaphoid abdomen; abdo organs protruding through breach in diaphragm—> pulmonary hypoplasia, pulmonary HTN, persistent R)—>L) shunting through DA. Most common on L) side.
I&V, avoid BMV as risk gastric insufflation.
Goals= avoid pulmonary HTN (blunt SNS, avoid hypercapnia) but gentle ventilation (risk barotrauma to hypo plastic lung, consider HFOV (controversial), NO, echo. Surgery within a few days when pulmonary hypoplasia stabilised.
Abdo/subcostal incision or thoracoscopic, patch to close diaphragm, don’t need OLV (lung gradually expands).
Avoid pulmonary HTN, avoid barotrauma.
Sig mortality risk.
Thoracoscopic “minimally invasive” but longer duration, higher PaCO2, risk acidosis. Also require higher ventilator pressure, high r recurrence rates after thoracoscopic & little evidence to suggest mortality benefit.

1513
Q

Congenital diaphragmatic hernia:

A

May be diagnosed in-utero
Presents with Resp distress, scaphoid abdomen; abdo organs protruding through breach in diaphragm—> pulmonary hypoplasia, pulmonary HTN, persistent R)—>L) shunting through DA. Most common on L) side.
I&V, avoid BMV as risk gastric insufflation.
Goals= avoid pulmonary HTN (blunt SNS, avoid hypercapnia) but gentle ventilation (risk barotrauma to hypo plastic lung, consider HFOV (controversial), NO, echo. Surgery within a few days when pulmonary hypoplasia stabilised.
Abdo/subcostal incision or thoracoscopic, patch to close diaphragm, don’t need OLV (lung gradually expands).
Avoid pulmonary HTN, avoid barotrauma.
Sig mortality risk.
Thoracoscopic “minimally invasive” but longer duration, higher PaCO2, risk acidosis. Also require higher ventilator pressure, high r recurrence rates after thoracoscopic & little evidence to suggest mortality benefit.

1514
Q

Minimum age paeds self report pain levels?

A

4yo w age-appropriate tool; 7-10yo skills of measurement, classification, seriation (asc/desc order)

1515
Q

SVT in pregnancy

A

Unstable: sync CV, pads trajectory away from uterus, foetal monitoring during & after
Vagal
Adenosine won’t enter foetal circle & maternal side effects short-lived
Metoprolol if adenosine ineffective, slow infusion less hypotension
Verapamil if adenosine & B blockers contraindicated (higher risk hypoT)
Amiodarone only if life-threatening & can’t use other therapies; preg class B, hypothyroidism, short-term only

1516
Q

Extinguisher types

A

red= water, for paper/wood/cloth & plastic
Foam (blue) can use for liquids, oil, carbonaceous
White (powder) can use on carbonaceous, liquids/oil, can use for electrical
CO2 (black) for oil/liquids & electrical

1517
Q

TB

A

Don’t need airborne precautions if extrapulmonary & latent TB
airborne transmissible: P2 or N95
filter on exp circuit
pt in -ve pressure room or surgical mask
can use PAPRs, N95,
Fit is the most important
N95 gets @! least 95% airborne particles & not oal resistant
R 95,99,100 oil resistant
P95,99,100 are oil proof

1518
Q

TB

A

Don’t need airborne precautions if extrapulmonary & latent TB
airborne transmissible: P2 or N95
filter on exp circuit
pt in -ve pressure room or surgical mask
can use PAPRs, N95,
Fit is the most important
N95 gets @! least 95% airborne particles & not oal resistant
R 95,99,100 oil resistant
P95,99,100 are oil proof

1519
Q

aprepitant works on receptors for

A

Substance P (NK-1 receptors)

1520
Q

emphysema ecg

A

R) axis or 90 deg axis
Reduced voltage QRS esp precordial
P waves prom II, III, aVF, flat 1, aVL
Clockwise rotation heart; delayed R/S transition precordial & may have no R wave V1-2, persistent S in V6
Sagging PR & ST segments

1521
Q

Most common cause POVL:

A

ION
PION>AION
1/60,000-1/120,000GAs, 3/10,000 spine & 8/10000 cardiac

1522
Q

Glossopharyngeal nerve block

A

Topical spray, mucosal contact w LA-soaked pledgets or direct LA injection at posterior tonsillar pillar or peristyloid
Sensory post 1/3 tongue, vallecula, anterior epiglottis, pharyngeal walls, tonsils

1523
Q

management carboxyHb poisoning

A

100% O2 via non-rebreather, cardiac monitoring, intubate if induicated (eg. GCS)
The half-life of carboxyhemoglobin (COHb) in a patient breathing room air is approximately 250 to 320 minutes; this decreases to 90 minutes with high-flow oxygen provided via a nonrebreathing mask. Thus, the most important interventions in the management of a CO-poisoned patient are prompt removal from the source of CO and institution of high-flow oxygen by face mask.”
Hyperbaric Oxygen only indicated if:
· CO level >25 percent
· CO level >20 percent in pregnant patient (see ‘HBO during pregnancy’ below)
· Loss of consciousness
· Severe metabolic acidosis (pH <7.1)
· Evidence of end-organ ischemia (eg, ECG changes, chest pain, or altered mental status)
Note – SpO2 is useless in CO poisoning

1524
Q

management carboxyHb poisoning

A

100% O2 via non-rebreather, cardiac monitoring, intubate if induicated (eg. GCS)
The half-life of carboxyhemoglobin (COHb) in a patient breathing room air is approximately 250 to 320 minutes; this decreases to 90 minutes with high-flow oxygen provided via a nonrebreathing mask. Thus, the most important interventions in the management of a CO-poisoned patient are prompt removal from the source of CO and institution of high-flow oxygen by face mask.”
Hyperbaric Oxygen only indicated if:
· CO level >25 percent
· CO level >20 percent in pregnant patient (see ‘HBO during pregnancy’ below)
· Loss of consciousness
· Severe metabolic acidosis (pH <7.1)
· Evidence of end-organ ischemia (eg, ECG changes, chest pain, or altered mental status)
Note – SpO2 is useless in CO poisoning

1525
Q

Why an increased rocuronium dose of 1.2 to 1.5 mg/kg for rapid sequence induction has been recommended in patients with major burn injury.64 It must be noted, however, that even with a dose of 1.5 mg/kg of rocuronium, the onset of time to effective paralysis approximates 90 seconds in burned patients compared with less than 60 s in nonburned patients with a dose of 0.9 mg/kg.

A

incr ACh receptors; require 2-3fold incr dose if 30% TBSA burns

1526
Q

Best evidence for smoking cessation:

A

Varenicline (RR 2.3)
Behavioural therapy 2.0 nortryptiline
NRT (1.9)
Bupropion 1.7, same as clonidine

		RCT & meta-analyses have shown that more intensive intervention= greater abstinence rate so follow-up beyond the lone pre-anaesthesia consultation is vital Combination of pharmacological therapy with multisession face-to-face counselling significantly improves changes of quitting (RR 10.8, cochrane review Thomsen 2014)
1527
Q

EST contraindications

A

Absolute contraindications:
acute myocardial infarction (MI) (within 2 days)
high-risk unstable angina
uncontrolled cardiac arrhythmias causing symptoms of hemodynamic compromise
active endocarditis
symptomatic severe aortic stenosis
decompensated symptomatic heart failure
acute pulmonary embolus
pulmonary infarction
acute noncardiac disorder that may affect exercise performance or be aggravated by exercise (eg, infection, renal failure, thyrotoxicosis)
acute myocarditis or pericarditis
physical disability that would preclude safe and adequate test performance
inability to obtain consent
acute aortic dissection

Relative Contraindications:
left main coronary stenosis or its equivalent
moderate stenotic valvular heart disease
electrolyte abnormalities
tachyarrhythmias or bradyarrhythmias
atrial fibrillation with uncontrolled ventricular rate
hypertrophic cardiomyopathy
mental impairment leading to inability to cooperate
high-degree AV block
severe arterial hypertension

1528
Q

Normal amplitude precordial ecg

A

3 milivolts ULN

1529
Q

Tapentadol:

A

Mu agonist & Norad reuptake inhibitor
18-fold lower affinity for mu than morphine but only 3x less potent as analgesic due to dual mechanism & synergy.
No active metabolites, no relevant functional serotonin re-uptake inhibition
Includes actions on descending pathways of pain inhibition.
Similar efficacy to conventional opioids w reduced GI adverse effects (N, V, constipation)
Lower rates abuse & doctor shopping vs oxycodone
No adverse HR or BP effects in clinically recommended (max 500mg/day) ranges, even in pts w hTN
Glucoronidation; may require dose adj in severe hepatic dysfunction. No dose adj required for mild-mod renal impairment, lack of studies on severe so not recommended.

1530
Q

Xenon:

A

our atm has 0.0875ppm

environmentally safe

no MH

$10/L expensive, most cost related to priming/flushing of the anaes circuit

Low BGPC 0.115 (rapid onset/offset)

minimal CV depression even in cardiac disease
neuroprotection
profound analgesia

doew have unreliagle bis like other non-GABA-ergic agents
MAC about 70%, extrapolated from studies using other agents since impractical/unsafe to run higher [] xenon in a closed circuit (risk hypoxia)

1531
Q

Xenon:

A

our atm has 0.0875ppm

environmentally safe

no MH

$10/L expensive, most cost related to priming/flushing of the anaes circuit

Low BGPC 0.115 (rapid onset/offset)

minimal CV depression even in cardiac disease
neuroprotection
profound analgesia

doew have unreliagle bis like other non-GABA-ergic agents
MAC about 70%, extrapolated from studies using other agents since impractical/unsafe to run higher [] xenon in a closed circuit (risk hypoxia)

1532
Q

What are bellwether procedures (WHO essential services criteria) that level 1 hospitals can & should be able to perform 24/7 to ensure delivery of emergency & essential surgical care?

A

Laparotomy
LSCS
Repair open fractures

1533
Q

Neuropathic & nociplastic pain

A

Neuropathic pain= pain caused by lesion or disorder of the somatosensory nervous system

Nociplastic pain= pain that arises from altered nociception despite no clear evidence of actual or threatened tissue damage causing activation of peripheral nociceptors or evidence of disease or lesion of the somatosensory system causing pain.

1534
Q

Which products have risk of CMV transmission? Incidence of CMV transmission due to blood transfusion?

A
  • Cellular components, so FFP, cry & other plasma-derived components don’t require CMV screening
    All cellular products in Oz are leucodepleted but neither leudodepletion nor CMV seronegativity completely eliminates CMV transmission risk.
    CMV seronegative products should be used for pregnant women during pregnancy (but not delivery)
    Intrauterine tennsfusions
    Neonates
    Granulocyte transfusions for CMV neg pts.
    Depending on policy, solid organ transplants, haematology/onc/i9mmunodeficient pts

0.9% risk seroconversion for CMV negative blood products (ie 9 per 1000)
Another study suggested (from a modelling approach), residual risk of leucodepleted-only products is 1 in 13million; neglibible).

1535
Q

CRASHIII

A

No sig reduction in 28d in-hospital mortality if gave <3hrs of injury. Was a sig reduction in mortality in pts w mild-mod (GCS 9-15), better result if gave early. No diff modified rankin scale.

1536
Q

What to give in an arrest for hyperkalaemia (eg using sux post intubation in a pt w critical illness neuropathy)

A

Calcium chloride

1537
Q

What to give in an arrest for hyperkalaemia (eg using sux post intubation in a pt w critical illness neuropathy)

A

Calcium chloride

1538
Q

Which product has the most useful levels of vWF?

A

Cryo (8U/L vs 1.5U/L in FFP)

Also, cry has 15g fibrinogen/L (fs 20g/L in fib conc or 2g/L in FFP)

1539
Q

Dorsal penile nerve block

A

Useful for circumcision
hypospadius repair
Paraphimosis reduction
Repair of penile lac
Release of trapped penile skin

From S2-4 via pudendal nerve, which runs in pudendal canal w pudendal artery.
Pudendal nerve divides in pudendal canal—> terminal branches to the dorsal penile nerves & the perineal nerve branch.

The subpubic space is roofed by scarpa’s fascia, deep membranous continuation of superficial abdo fascia.
Dorsal penile nerve on each side runs under inf pubic ramus, deep to suspensory ligament, each lying in its own space.
They continue directly within Buck’s fascia on the penis next to dorsal vessels.

Aim w penile block= inject LA into the bilateral spaces deep to scarpa’s & buck’s fascia on either side of suspensory ligament (which is continuous w Buck/s fascia)

0.1mL/kg/side (max 5mL)
CIMPLE
La WITHOUT epinephrine, avoid ropiv (intrinsic vasoconstrictor)
Bleb to skin @ dorsal of penis base. Gently retract penis down to make scarpa’s fascia taut.
Advance until touch pubic symphysis, withdraw slightly then redirect to pass below pubic symphysis, 5-10mm lat to midline, advance sl caudally & until “pop” as needle enters compartment just under scarpa’s fascia. Aspirate, infiltrate.
Repeat the other side (can do in 1 from the midline, my preference is 2 separate injections to avoid risk of midline structures.

Dorsal nerve block doesn’t achieve total anaesthesia; innervation of the frenulum is via perineal branch. Achieve this with further Anaes @ base of ventral penis or partial ventral ring block.

1540
Q

how long after removing epidural catheter can prophylactic clexane be given?

A

4hrs

1541
Q

Infant dose iM sux

A

5mg/kg (chn 4mg/kg), max block @ 3-4 mins, lasts 15-20mins

1542
Q

How does French relate to outer diameter of singl lumen tube?

A

Fr unit is equal to 0.33mm outer diameter.
So, a size 9ETT which has OD of 12.2mm, equivalent to 37Fr (12.2/0.33)=36.9 aka 37Fr

external diameters:
size 4=5.4
size 5=6.8
size 6=8.2
size 7=9.6
size 8=11
size 9=12.2

1543
Q

minimum PACU nurse:pt ratios

A

1 nurse for each patient that has not regained airway reflexes, 1 nurse for 3 awake patients who have regained airway reflexes

1544
Q

anaphylaxis adult refractory hypotension

A

Norad 3-40microg/min
vasopressin 1-2 units bolus then 2 units/hr
Glucagon 1-2mg IV every 5 mins until response

1545
Q

axillary nerve function

A

Supplies deltoid (abduction of shoulder beyond 90 degrees), tires minor (ER)
Sensation skin over shoulder “regimental badge” area

1546
Q

side effects SGLT2 inhibitors

A

Euglycaemic DKA, hyperglycaemic DKA
Glycosuria
UTIs
Hypotension

They have LOW risk of hypoglycaemia as their efficacy to incr urinary glucose excretion attenuates @ lower plasma glucose levels
may cause hyperglycaemic DKA

1547
Q

Aortic XC & unclamp phys/pathophysiology & amelioration strategies

A

INCREASE LV AFTERLOAD due to impedance to aortic flow:
—> may get increased contractility & cardiac output (anrep effect, incr inotropy)
—>increases myocardial wall stress (=(delta p x radius)/ 2w)
DEPENDING on myocardial supply:demand matching & cardiac function, result may be increase or decrease CO; if pre-existing incr LV pressure & poor coronary perfusion (inadequate supply to meet demand; CorPP = ADP - LVEDP), risk poor perfusion & ischaemia & reduced CO
—> risk hypertensive crisis & end-organ damage (particularly renal failure)
—> MAP increase much higher for supracoeliac (approx 50%) vs suprarenal (20-30%) or infrarenal (10-20%)
—> myocardial ischaemia, functional AR, incr CSF pressure & ICP

CATECHOLAMINES: promote active vasoconstriction (SVR)proximal & distal to clamp, further increase arterial resistance

INCREASE PRELOAD due to passive recoil of circulation distal to XC:
—> increase preload (higher degree for supracoeliac>suprarenal as higher volume of blood distal to XC, splanchnic circulation a significant reservoir volume has 25-30% of CO; supracoeliac much higher increase in PCWP, EDA, ESA & higher reduction in ejection fraction; higher rate of regional wall motion abnormalities supracoeliac>suprarenal>infrarenal)
—> increased contractility of LV DEPENDING on where it exists on the frank-starling curve (pre-existing LV failure or poor compliance more likely to precipitate acute LV failure & reduced CO, particularly for supra coeliac XC)

REDUCED TISSUE PERFUSION DISTAL TO XC:
—> all reduce perfusion distal to X-clamp, producing anaerobic metabolism, production of vasoactive metabolites (CO2, K+) in the tissues, including lower limbs for all levels. The amount of tissue/organ impacted increases with more proximal clamp level.
—> infrarenal does increase renovascular resistance, reduce GFR & reduce renal blood flow by 40% (RAAS alterations)—> risk Perioperative renal failure
—> suprarenal—> direct reduction in renal blood flow due to interruption perfusion to kidneys
—> supracoeliac—> splanchnic hypoperfusion, risk of ischaemia, large volume of blood passively recoils to increase preload
—> risk anterior spinal artery syndrome if interruption to artery of adamkiewicz & inadequate MAP to support collaterals (ScPP = MAP - CSFpressure)
Stasis distal to XC may increase risk atheroembolism on XC release

Management:
-aim to have SBP approx 80-100mmHg @ time of XC (pt-specific target); avoid excess fluid (restrictive pre-XC, aim to preserve vital organ perfusion)
-anticipating the haemodynamic response to XC (particularly HTN), medications prepared:
-adequate depth of anaesthesia
-adequate analgesia (rapid-acting agent eg fentanyl boluses)
-consider bolus epidural 5mL 0.2% ropivacaine depending on pts haemodynamics
-vasopressors off
-vasodilators prepared (rapid-acting, titratable best): GTN 50mcg/mL (50mg in 1L N/saline, useful as reduces LV preload & dilates coronaries but reduces perfusion pressure collateral circulation), hand bolus 25mcg in emergencies, could also use propofol 20-60mg IV boluses
-if adequate myocardial function, consider rapid-acting B blocker esmolol 10-20mg boluses +/- infusion
-could consider frusemide if not hypovolaemic
-while on XC, avoid warming LLs (reduce risk tissue burns)

*mesenteric artery aneurysms have supracoeliac XC but it’s not on for long (3-5mins) vs on for 1 hr w AAA

Un-clamping:
Magnitude of response depends on level of the clamp
Reduction in after load—> reduces ventricular wall stress & O2 demand
However risk reduced myocardial O2 supply as:
-reduced preload (blood redistribution back to limbs—> central hypovolaemia)
-distal vasodilation with release of vasoactive mediators from ischaemic tissues distal to clamp(reduce peripheral vascular resistance) & increased vessel permeability
—> risk reduced coronary perfusion pressure, supply < demand & myocardial ischaemia
—> reduce renal perfusion & risk renal injury
-potassium: risk cardiac arrhythmias/arrest
-CO2 incr SNS activation, increase PVR & risk R) heart failure
-acidosis risks myocardial depression & reduced CO
-risk pulmonary oedema (combination of vascular permeability, LV systolic dysfunction)

Manage by:
-anticipation of “decamping hypotension” by:
-load with fluid/blood products pre-XC removal (adequate “fill”), albumin (care w Cl load), guided by CVP (aim 5mmHg above baseline just prior to unclamping), ensure the fluids warm
-AIM= normotension w XC on, CVP 5mmHg > baseline by time of unclamping
-clear communication w surgeons; attenuate w gradual release of XC (eg. One leg at a time, can always re-clamp if haemodynamics unmanageable)
-discontinuation/reduction of vasodilators
-vasopressors (NAdr), positive inotropes
-CHECK CALCIUM & replace as needed
-correct acid base disturbance prior to unclamp
-increase minute ventilation to attenuate myocardial depression with lactic acidosis
-consider sodibic
-Anticipate & manage arrhythmias
-anticipate risk thromboembolism, incr RVSP & RHF
-if epidural, limit boluses until aorta closed (care w SNS blunting)

1548
Q

Cohen dimensions/use

A

9Fr
65cm
Min ETT 7

1549
Q

Arndt:

A

5Fr (paeds) 4.5mm ETT
7Fr 7mm ETT
9Fr 8mm ID but size 9 tube best
Fuji/ambu min size 8.5
EzBlocker: min size 9
BB cuff 6-9mL

1550
Q

prone benefits (16hrs/day in ards

A

Improves mortality in mod-severe ARDS
Physiology of prone positioning:
Alters gas exchange to consistently improve oxygenation
-ameliorates ventral-dorsal transpulmonary pressure difference; Ptp = (Paw - Ppl). When supine, higher dorsal Ppl & greater expansion of ventral vs dorsal alveoli. Effect exaggerated in ARDS (excess lung weight). Tends to overinflated ventral alveoli, collapse dorsal. Ventilation more homogenous in prone, less over distension of ventral alveoli (& cyclic atelectasis), improved distribution of ventilation, improves oxygenation
-reduces dorsal lung compression (by heart & diaphragm), there’s a smaller volume of “dependent lung” since heart is dependent
-improved cardiac output (may be due to increased lung recruitment & reduced HPV, incr RV preload & decr RV after load & decr PVR)
-improves lung perfusion
-may increase FRC & reduce shunt (*assuming abdomen unsupported)
-may alter distribution of extravascular lung water & secretions (secretion mobilisation, improved bronchial drainage & ventilation)
-improved delivery of aerosolised meds
-more homogenous perfusion

CONTRAINDICATIONS:
Facial or pelvic fractures
Burns or open wounds on ventral body surface
Conditions (RA, trauma) ass’d w spinal instability
Conditions ass’d w incr ICP (ICP >30 or CPP <60mmHg)
life-threatening arrhythmias
Acute bleeding eg. Shock
Trach surgery or sternotomy within 2 wks
*pts w ant chest tube & air leaks, severe burns or recent lung transplant or critical conditions limiting life expectancy excluded from prone positioning in severe ARDS trial

Consider implications of prone on chest tube drainage, consider tube position (2-4cm above carina), ensure all central & lg-bore access firmly secured
Consider how pt’s head/neck shs will be supported
Feeds need to be stopped, stomach empty
Consider process if copious secretions (suction ready)
Care re: pressure areas, brachial plexus injury
Reassess lines/tube position etc after turn.
Rev tren & periodic tilt, document skin checks.

Risks:
Pressure injuries (nerve, crush, retinal damage)
Venous stasis
Dislodge ett or other lines
Impaired volumes if diaphragm limitation
Vomit
Arrhythmias
desats

1551
Q

How does blom singer prosthesis work?

A

Voice is produced by temporarily blocking the stoma so that exhaled air from the lungs can be directed from the trachea through the prosthesis into the esophagus and then out through the mouth

1552
Q

Problem with MLT size 4

A

Can’t fit adult bougie, paeds bougie too short

Adult bougie 14Fr, 70cm length, ID3mm
paeds 8Fr, 35cm, ID 1.6mm

1553
Q

If group A Rh-ve cryoprecipitate is not available for use in an A Rh-ve patient, of the following your next best choice should be 

A

AB Rh+ (Universal Donor, Absence of Anti-A and Anti-B antibodies, Rhesus matching not required)

1554
Q

What’s induced hypotension

A

Deliberate hypotension is defined as the reduction of systolic blood pressure (BP) to 80–90 mmHg, and that of MAP to 50–65 mmHg or 30% lower than the baseline level

1555
Q

characteristics of drugs effectively cleared by haemodialysis

A

Low PB (eg. Dabigatran 30% vs other DOACs 90%) better dialysis potential
Large MW better trapping in dialysis membrane
Large VD poorly dialysable

Barbiturates
Lithium
Isoniazid
Salycilates
Theophylline
Metformin/methanol
Ethyene glycol
Dabigatran
Carbamazepine, uraemia:)

1556
Q

Compared to a normothermic patient, a patient with mild intraoperative hypothermia (35.0 degree C) will have

A

Increased blood loss, no effect on INR or APTT

1557
Q

To minimise the risk of developing propofol infusion syndrome, the maximum recommended propofol infusion rate averaged over a 48-hour period is

A

Recommendations in the literature include the avoidance of infusion rates of more than 5 mg kg−1 h−1 for more than 48 h5, 51 to always use propofol in combination with other sedative agents (such as opioids), and to monitor pH, lactate, and creatine kinase when infusions are prolonged, especially if high doses cannot be avoided.52 The AstraZeneca Summary of Product Characteristics for Diprivan 1% and the German Medical Association recommend a lower maximum infusion rate of 4 mg kg−1 h−1.53

Monitor daily pH, CK, lactate, Cr, K+

Use the minimum dose & use other sedative adjuncts

No established management, change to alternaive sedation & treatment should focus on the clinical features shown to be associated with mortality: ECG changes, hyperkalaemia, hypotension, and fever
could use hamofiltration (doesn’t eliminate props but does for it’s water-soluble metabolites) but care re: citrate levels (hypocalcaemia) in pt w PRIS as citrate metabolised in mitochondria, citrate tox –> hypocalcaemia

Risk factors: critical illness, higher mort in TBI, higher dose & duration (eg. >5mg/kg/hr doubles risk PRIS cf lower doses, >48hrs incr risk.

1558
Q

risks of infection with CVCs

A

Femoral > IJ > subclavian

1559
Q

Patients with obstructive sleep apnoea undergoing surgery, have been shown to have an increased incidence of

A

difficult intubation, obstructed breathing, OPIOID SENSITIVITY

Pulmonary complications
postop delirium

1560
Q

According to the ANZCA PS 50 “Recommendation on Practice Re-entry for a Specialist Anaesthetist” it is recommended that after an absence of more than 12 month from practicing clinical anaesthesia a re-entry program should be offered. The duration of the program for every year of absence would usually be at least

A

4 weeks per year off

1561
Q

The lowest pinprick ketone level that would support a diagnosis of euglycaemic ketoacidosis is

A

1.1
>0.6 suggests at risk of DKA ketosis. BE <-5 also suggests metabolic acidosis

1562
Q

A 65-year-old woman has presented with a grade 2 subarachnoid haemorrhage equally suitable for treatment with surgical clipping or endovascular coiling. The factor shown to most effectively reduce mortality in early subarachnoid haemorrhage treatment is

A

MOST IMPORTANT GOAL= PREVENT REBLEEDING. clip or coil as early as faesible (pref within 24hrs) the only effective treatent to prevent rebleeding. Short-term outcomes improved w endovascular coil vs surgical clip.

1563
Q

The transducer that provides the best resolution for an ultrasound guided median nerve block is  

A

Linear Transducer 8-14MHz.

1564
Q

A patient with persistent pain on oral hydromorphone 12mg per day is admitted to hospital unable to tolerate oral intake. The equivalent parenteral morphine dose per day is 

A

20mg IV morphine

1565
Q

Which pts w LAST should get intralipid?

A

patients with LAST who exhibit seizures or impending seizures, or signs of cardiovascular toxicity (ie, arrhythmias, severe hypotension, or cardiac arrest) along with ACL

1566
Q

what conditions incr X descent?

A

consrictive pericarditis (also incr y descent)

1567
Q

HELLP classifications

A

Mississippi divides into 3 classes based on plt (class 1 <50, 2 50-100, 3 100-150, AST, ALT & LDH (>600), relates to bleeding incidence, M&M
Tennessee complete/incomplete, depending on whether has all of: AST+/- ALD >40IU/L, put <100, LDH >600, AST >70

1568
Q

apnoeas hypopnoeas for paeds OSA

A

Reduction in airflow >=90% lasting >=90% of the duration of 2 normal breaths
Hypo is reduction in airflow >=30% lasting >=90% of 2 normal breaths + desaturated 3% or arousal

1569
Q

Best position for gastric US

A

R) side down, encourages drainage of gastric content to dependent antrum, incr send to detect smaller volumes

1570
Q

Best position for gastric US

A

R) side down, encourages drainage of gastric content to dependent antrum, incr send to detect smaller volumes

1571
Q

Do you need ABx prophylaxis for IUD insertion?

A

No

1572
Q

modified valsalva

A

LL elevation w supine @ end of strain

1573
Q

Ecg findings posterior MI:

A

In V1-3: Horizon ST dep, tall, broad R waves, upright T waves, dominant R wave (R/S >1) in V2
Posterior MI either RCA or L)Cx

1574
Q

Which type of pharmacotherapy NOT effective for smoking cessation periop

A

Fluoxetine (an SSRI)

1575
Q

how may low albumin impact the anion gap?

A

It may underestimate it; Alb decr by 1g/L—> anion gap decr by 0.25mmol. use corrected AG (AG + (0.25 x (40-alb))

1576
Q

what’s not a contraindication to HBOT?

A

Brain abscess
Congenital spherocytosis is a relative contraindication, asthma, Tx surgery, emphysema w CO2 retention, URTI, seizures, middle ear surgery/isusues, fevers, optic neuritis
ABSOLUTE= bleo, cisplatin, premature infants, untreated PTx, disulfiram

1577
Q

drugs that can give down ett

A

Adrenaline
Atropine
Lignocaine
naloxone
May need to give 3-10x dose

1578
Q

If intubating via tracheostomy stoma, what use?

A

Small tracheostomy tube/6 cuffed tracheal tube; consider aintree catheter & fibreoptoc scope/bougie/aec

1579
Q

are strained broth & jelly clear fluids?

A

no

1580
Q

What’s the COPS protocol for Rx of delayed neurological deficits w CSF drains?

A

plt should be >100, INR <1.3, normal APTT, anticoag & antiplt witheld.CSF drain status; normal or malfunction? replace if malfunctioning, lie flat & ICP <10mmHg, drain 10mL increments.
O2 delivery: Sats>95%, Hb >120, CI >2.5mL/kg/min
Pt status: MAP >90mmHg, SCPP >80mmHg
Neurological Ax & correct factors altering DO2 (sepsis, bleeding, compartment syndrome, tamponade, visceral ischaemia, AF)

(Goals spinal drain MAP>85, ICP<15, if ICP >15 incr MAP until ScPP >70, keep SpO2 >95%)

prior: consent, document
plt >100, INR <1.3, normal APTTm, anticoag & antiplt witheld, avoid placement if evidence raised ICP

best done 24hrs preop, if traumatic/bloody tap delay heparin @ least 60mins. If 2x bloody taps, d/w surgeons consider potponing insurgery.

sterile setup, L4/5, 16g.

ScPP >60, avoid hypoT or large incr CVP (ScPP=MAP-CSFp)
zero@ phlebostatic axis (4th ICS, mid A-P point chest wall; corresponds to RA)
CSF pressure <10mmhg (goal ScPP >60mmHg); intermittent (5-10mL if CSFP >10mmHg) vs continuous preferred to avoid large vol drainage. Too much drain risks SAH (10-20mL/hr intermittent best, aim CSF pressure <10)
<=15mL/hr if no SC ischaemia, <=20mL/hr if SC ischaemia.
Consider <72hrs.
Avoid hypo.
If bloody CSF drain may indicate ICH—> imaging
If new-onset LL neurological deficit (ONGOING MONITORING!), SC ischaemia vs neuraxial haematoma; incr ScPP, imaging.
Hold heparin 4-6 hrs (ASRA) before remove drain (& ensure plt >100 & INR <1.3 & APT N), hold heparin 1hr after removal

ASRA: restart Heparin 1hr after procedure, restart heparin 1hr after catheter removal

1581
Q

Why gauge pressure on N2O cylinder doesn’t nec represent the contents remaining?

A

Remains below CT (38deg) so exists as vapour in equilibrium w liquid phase. Has BP of -88

1582
Q

stress steroids for children adrenal insufficiency

A

Hydrocortisone 2mg/kg @ induction then wt-based infusion: 25mg/24hrs if up to 10kg, 50mg/24hrs if up to 20, over 20 100 for prepubertal, 150mg/24hr if pubertal (or post could be 2mg/kg 4hrly IV or IM

1583
Q

stress steroids for children adrenal insufficiency

A

Hydrocortisone 2mg/kg @ induction then wt-based infusion: 25mg/24hrs if up to 10kg, 50mg/24hrs if up to 20, over 20 100 for prepubertal, 150mg/24hr if pubertal (or post could be 2mg/kg 4hrly IV or IM

1584
Q

anaphylaxis <6 months

A

0.01mg/kg=0.01kmL/kg 1:1000

1585
Q

risks torsades

A

female
hypoMg++
hypoK
hypoCa
brady
long QT
TCA OD

1586
Q

risks torsades

A

female
hypoMg++
hypoK
hypoCa
brady
long QT
TCA OD

1587
Q

risks torsades

A

female
hypoMg++
hypoK
hypoCa
brady
long QT
TCA OD

1588
Q

Guedel stages anaesthesia

A
  1. Analgesia & amnesia (initiation—> LOC): normal m tone, voluntary eye movement
  2. Delirium & unconsciousness (from LOC to onset automatic breathing): Excitement: tone normal to incr, irreg shallow Br, mod eye movement, risk swallow/retch/vomit; corneal reflex disappears, secretion tears increases.
  3. Surgical anesthesia: br becomes regular, less volumes through the 4 planes, eye movt sl—> nil
  4. Resp paralysis: flaccid, no eye movement
1589
Q

OIH

A

Risk for chronic pain post
Incr post pain, opioid consumption, nociceptive threshold changes
Keep Remi TCI <0.1mcg/kg/min, <2.7ng/mL
Remi sparing: ketamine, prop, NO, COX2, alpha2, gabapentin
Dexmed or MR instead of Remi

1590
Q

L) heart cath for aortic stenosis shows

A

low aortic pressure

1591
Q

best RSI for pt with hyperkalaemia & poor renal function?

A

Rocuronium 1.2mg/kg (min impact w renal impairment)

1592
Q

best RSI for pt with hyperkalaemia & poor renal function?

A

Rocuronium 1.2mg/kg (min impact w renal impairment)

1593
Q

During spinal surgery, the anaesthetic agent that is least likely to decrease motor evoked potentials is

A

Ketamine: increase MEPs (opioids small dose-dep MEP depression)

1594
Q

During spinal surgery, the anaesthetic agent that is least likely to decrease motor evoked potentials is

A

Ketamine: increase MEPs (opioids small dose-dep MEP depression)

1595
Q

A four-year-old boy is in refractory ventricular fibrillation. The recommended dose of amiodarone is

A

5mg/lg, so if 16kg, 80mg

1596
Q

A four-year-old boy is in refractory ventricular fibrillation. The recommended dose of amiodarone is

A

5mg/lg, so if 16kg, 80mg

1597
Q

A four-year-old boy is in refractory ventricular fibrillation. The recommended dose of amiodarone is

A

5mg/lg, so if 16kg, 80mg

1598
Q

ropivacaine product info:

A

cumulative doses of up to 800mg for surgery & post analgesia administered over 24hrs well tolerated, then post continuous infusions at rates of UP TO 28mg/hr for 72hrs

1599
Q

Pacemaker interpretation:

A

Spike before p wave= atrial paced
Before qrs= ventricular paced

FTC= pacemaker stimulations don’t —> myocardial activation (see a pacing spike, no activity, genuine or during refractory period.

Failure to sense (under sensing) doesn’t sense true p or r waves, may —> overpacing eg. Pacing after intrinsic activity

Oversensing= senses signals that aren’t true p or r waves (which normally inhibit pacemaker), DDD pacemaker would lead to overpacing if over senses atrial activity, DDI would lead to underpacing.
Failure to pace= doesn’t pace as expected
Failure to rate adjust (only elevate to rate responsive PM)

1600
Q

Considerations for trialing opioids in chronic non-cancer pain:

A

-risk Ax (Hx substance abuse, Fix substance abuse, psych disorder, aberrant drug taking Bx).
-Multimodal: Self-management & non-opioid
-opioid trial- set goals based on function. Agreement, consent (incl no replacement of lost scripts). Dispensing according to risk Ax.
-trial 6-8 wks (long-acting oral or transdermal)
-regular re-Ax & documentation: 5A’s:
Analgesia
Activity
Adverse effects
Affect
Aberrent behaviour

Initially weekly.
Limited dose 100mg/day OME.
Involve other colleagues.
Wean within 3/12 after acute pain.
Slow: 10-25% of starting dose @ monthly intervals, fast 10-25% at weekly intervals.
Alert to dependence/addiction.

1601
Q

Converting opioids:

A

calculate 24hr OME, decrease dose new opioid by 25-50% due to incomplete cross-tolerance.

1602
Q

Rationale for optical stylet:

A

Combine fiberoptic scope & intubation stylets. Can use independently, w laryngoscopes, or with SGA devices.

1603
Q

Sedation levels:

A

minimal: drug-induced state where respond purposefully to verbal commands or light tactile stimulation, resp & CV unaffected.
moderate: drug-induced state of depressed consciousness during which pts retain the ability to respond purposefully to verbal commands & tactile stimulation
deep: depressed cons, not easily roused, may respond only to noxious. may need to have airway support, CV function generally maintained

1604
Q

Factors ass’d w incr risk AFE

A

CS>instrumental
Induction/augmentation labour (esp if hyper-stimulation)
Instrumental
CS
And age >35, male foetus, multiple pregnancy, eclampsia, uterine rupture, placenta praevia, placental abruption.
Overall rare so unpredictable.

1605
Q

Factors ass’d w incr risk AFE

A

CS>instrumental
Induction/augmentation labour (esp if hyper-stimulation)
Instrumental
CS
And age >35, male foetus, multiple pregnancy, eclampsia, uterine rupture, placenta praevia, placental abruption.
Overall rare so unpredictable.

1606
Q

Factors ass’d w incr risk AFE

A

CS>instrumental
Induction/augmentation labour (esp if hyper-stimulation)
Instrumental
CS
And age >35, male foetus, multiple pregnancy, eclampsia, uterine rupture, placenta praevia, placental abruption.
Overall rare so unpredictable.

1607
Q

Factors ass’d w incr risk AFE

A

CS>instrumental
Induction/augmentation labour (esp if hyper-stimulation)
Instrumental
CS
And age >35, male foetus, multiple pregnancy, eclampsia, uterine rupture, placenta praevia, placental abruption.
Overall rare so unpredictable.

1608
Q

Active cannabis use perioperatively is associated with

A

MI

tachycardia
5-fold incr MI risk in hr following cannabis, 24-fold incr risk MI in the hour following cocaine
Peripheral vasodilation (—> compensatory orthostatic hypoT & incr CO, DO2, cardiac work).
MI risk rapidly decreases after 1hr.
Physiologic effects blunted in chronic cannabis users.

1609
Q

THC

A

Rapid onset (rapid distribution to VRG, effects within seconds to mins inhaled, may be 1-2hrs w oral).
highly lipid soluble so redistrubtue ++ , can accumulate once ingested, plasma half-life 20hrs but tissue half life may be days (up to 30) depending on frequency/chronicity of use.
can’t predict particularly as THC concentration, route of delivery varies.
Most commonly smoked, ingestible products more risk of psychosis/violent behaviour.

Effects:
CV: acute: tachycardia, vasodilation, orthoasasis. Chronic: atheromatous disease
*incr risk angina & MI esp 1hr after injestion.
Marijuana may incr stimulatory effects of amphetamines & cocaine & the depressive effects of ETOH & BZD. Low doses THC–> SNS stimulation, tachy, HTN. High doses may inhibit SNS activity–> brady & hypoT.

Pulmonary: bronchodilator, Airway effects of smoking marijuana mild cf tobacco; short-term bronchodilation but chronic cough & mild airflow obstruction over the long term. May get upper airway oedema, rarely PTx with valsalva-like maneouvers.
airway hyperreactivity (may be more significant than tobacco as burns at higher temperature., airway oedema. chronic= bronchitis, emphysema
*hyperreactivity risks PRAEs, Observe for stridor if UA oedema. chronic cough & wound dehiscence
CNS: acute: anxiety or anxiolysis, paranoia, psychosis, dizzinesss, headache, analgesia. Chronic effects are similar but tolerance develops
GI: dry mouth, anti nausea & incr appetite, abdominal pain. Chronic: hyperemesis.
*may have electrolyte abnorms/dehydration with cannabinoid hyperemesis syndrome.
endocrine: long term gynaecomastia, anovulation, galactorrhea.

withdrawal: can develop within a day of cessation, may take weeks to resolve. anxiety, irritability, restlessness, anorexia, abdominal cramp, tremors, sweat, fever, chills, headache-> depressed mood & lack appetite.

Could use BZD, synthetic THC (used for chemo-induced nausea, off-label for cannabis withdrawal.)

Pre-op:
Hx: last use (look for signs intoxication, most concerning Anaes implications are w acute intox)
Ask re: freq, duration, route, mode.
Exercise tolerance, CVS/angina symptoms.
Other substance use.
*if evidence acute intox, consider delay for 1hr after use due to high risk of MI within 1 hr of use. Await resolution of tachy/postural hypotension.
CNS signs of intoxication: concern re: violent emergence.
Resp exam.
High-potency formulations may get HTN, tachy, fever (Ddx MH, thyrotoxicosis)
Informed consent unlikely possible with acute cannabis intoxication.

Intra-op:
Anticipated higher doses needed
Depth of Anaes monitoring but BIS may not be reliable.
Maintenance requirements not clear in literature, cross-tolerance w other anaesthetic agents unknown..
Cannabis users report higher pain scores & require more rescue analgesics.
Multimodal incl regional.

Post-op: consider withdrawal.
Analgesia ongoing multimodal
Complication surveillance

1610
Q

ESKD considerations:

A

Encephalopathy (uraemia)
Myopathies
Neuropathies (esp ANS)
Osteodystrophy (calcium homeostasis)

May get cardiac failure, myocarditis

pleural effusions

Impaired gastric emptying

Immunocompromised

Access sites, volume & electrolyte status related to dialysis & timing (know baseline wt & bloods, volume status up & down, know usual UO & fl restriction); HD improves water balance, acid-base, electrolytes, HTN, decr pericarditis, neuropathy & renal osteodystrophy. improves plt function but usually not coagulopathy.

Haem:
Chronic compensated normocytic normochromic anaemia common (intake, EPO, hepcidin (ACD), microangiopathy)
High vWF/VIII (coagulopathy), low factors II, V, VII, IX, X
Impaired plt dysfunction w uraemia

Water imbalance: RAAS/protein & chronic haemodialysis

Renal impairment: hyperK, hyperMg++, low Ca++ (related to lack of renal vit D activation), acidosis (which may incr Ca++ liberation from bone)

Pk implications (renal active metabolites, incr VD so may need higher props dosing, CNS sensitivity, atrac/cis organ independent)

Comorbid:
HTN
DM common

AV fistula: usually block, GA for revision (may be longer pro edure), consider pt comorbidities

1611
Q

ESKD considerations:

A

Encephalopathy (uraemia),. sensitivity to GA agents
Myopathies
Neuropathies (esp ANS- postural BP, sensorimotor peripheral)
Osteodystrophy (calcium homeostasis), osteoporosis

May get cardiac failure, haemorrhagic pericarditis (coagulopathy), pericarditis (?circulatory toxins), arrhythmias

pleural effusions, pulmonary oedema

Impaired gastric emptying

Immunocompromised

Access sites, volume & electrolyte status related to dialysis & timing (know baseline wt & bloods, volume status up & down, know usual UO & fl restriction); HD improves water balance, acid-base, electrolytes, HTN, decr pericarditis, neuropathy & renal osteodystrophy. improves plt function but usually not coagulopathy.

Haem:
Chronic compensated normocytic normochromic anaemia common (intake, EPO, hepcidin (ACD), microangiopathy); often they tolerate low Hb/Hct well, aim Hb at least 60, Hct0.3, use EXERCISE TOLERANCE or CV SYMPTOMS or CNS symptoms as indicator of pts ability to tolerate anaemia. if must use blood, LDF to reduce induction of HLA alloimunisation.
High but defective vWF, high VIII (coagulopathy), low factors II, V, VII, IX, X, incr bleeding time
Impaired plt dysfunction w uraemia

Water imbalance: RAAS/protein & chronic haemodialysis; “dialysis disequilibrium syndrome”

Renal impairment: hyperK (aim <5.5 preop, HD or resonium A, delay if pathology unknown), hyperMg++, low Ca++ (related to lack of renal vit D activation), acidosis (which may incr Ca++ liberation from bone), high phosphate, low albumin

Pk implications (renal active metabolites, incr VD so may need higher props dosing, CNS sensitivity, atrac/cis organ independent)

Comorbid:
HTN (aim <160/<90 preop
DM common

AV fistula: usually block, GA for revision (may be longer pro edure), consider pt comorbidities

1612
Q

Block levels required for urology surgery

A

T11-L2 & S2-4 for prostatectomy
Kidney T10-L1
Bladder needs T11-L2 for dome
Penis/scrotum S2-4

spinal level <T10 allows earlier recognition of bladder perf (back/abdo/sholulder pain if intraperitoneal collection, most are extraperitoneal w paraumbilical, inguinal or suprapubic pain (may be covered by spinal)

1613
Q

Sengstaken-Blakemore tube:

A

Sengstaken-Blakemore tube:For life threatening upper GI bleed from ruptured oesophageal/gastric varices
3 lumens (aspiration gastric contents, inflate yes balloon, gastric balloon.
Intubate pt before insert, generally inserted at time of endoscopy if vatceal bleeding not controlled
Confirm in stoach (aspiration gastric contents)
200mL n/saline slowly into gastric balloon
Proximal end of tube secure to pts face
Oesophageal balloon: air approx 40mL, 25-30cmH2O (manometer)
Usually deflate balloons regular 8-10hr intervals, max time tube in 24-48hrs

1614
Q

Repositioning w DLT:

A

FiO2 100%, bronchial cufff down prior to transfers, re-confirm position

1615
Q

Mediastinal mass:

A

Thyroid
Thymoma
Teratoma
Terrible lymphoma
“Safe” positioning
Symptoms or 50% tracheal compromise
Ax CT scan: trachea & great vessels
Airway strategy
Bleeding risk w mass manipulation
Discuss with surgeon
Mediastinoscopy: SVC obstruction a contraindication
Why not EBUS? Stent before?
Continuous BP measurement on R) UL (risk nominate), BP cuff on L)
IV access LL if concern SVC compression
Vagal response w any procedures near aortic arch
RLN, Tx duct injury
* Side:
1-2cm R) main bronchus (?1/20 it’s <1cm to R) UL bronchus), bronchus intermedius is 1.5-2cm
1/250 R) UL bronchus off the trachea

  • Side:
    Main bronchus 4-6cm

1-3mL in bronchial cuff (max 4mL)- don’t need much air if size cuff properly

1616
Q

Stance on “renal dopamine” & other diuretics?

A

Diuretics not recommended as prophylaxis for renal failure
No evidence frusemide or dopamine improve mortality
Low-dose dopamine may reduce renal blood flow in early ATN, may cause pre-capillary vasoconstriction & increase risk post-op AKI
Mannitol (0.25-0.5g/kg may incr UO & SCAVENGE FREE RADICALS (attenuate repercussion injury), may reverse cortical ischaemia but not conclusively shown to improve outcome
Fenoldopam dopamine agonist may incr RBF, natriuresis, dear splanchnic & SVR- some evidence re: reducing risk of IV contrast but less evidence for open surgery, studies underpowered.

1617
Q

specific antidote to glycine

A

Mg++ infusion. barbiturates, Mg++ or phenytoin for seizures w TURP syndrome (seizures may be BZD resistant)

1618
Q

Differentials & Mx for broad complex tacchy:

A

Regular:
VT unstable shock (or any unstable for that matter), amiodarone
Svt w BBB: adenosine
irreg:
AF w BBB manage as for narrow complex (B blocker or diltiaz consider amiodarone or dig if HF)
Polymorphic: Mg++

1619
Q

Other considerations TURP:

A

TURP syndrome; iso-osmolar (or sl hypo) hyponatremia
CV risk pt factors, large resections & blood loss, fluid absorption
Hypothermia w irrigant
Transient bacteremia- treat pre-op UTIs
DVT 5-10%
Significant bleeding (more if procedure >60mins & prostates >50g; difficult to estimate blood loss due to constant irrigation; 15mL/g resected tissue roughly. Blood transfusions in approx 10%. Tissue plasminogen activator released from prostate.
Optimise volume via PPV (fluid responsiveness)
If minimal UO & risk ARF, 500mL fluid challenge
If volume responsive & more fluid not detrimental, give more, otherwise PAC; insert CVP; raised? Consider mannitol/frusemide. If urea rise, fluid o/load, met acidosis, hyperK consider haemodialysis (diuretics only used for limited time (40-80mg IV frusemide; loop better natriuretic effect vs thiazides)

1620
Q

Other considerations TURP:

A

TURP syndrome; iso-osmolar (or sl hypo) hyponatremia
CV risk pt factors, large resections & blood loss, fluid absorption
Hypothermia w irrigant
Transient bacteremia- treat pre-op UTIs
DVT 5-10%
Significant bleeding (more if procedure >60mins & prostates >50g; difficult to estimate blood loss due to constant irrigation; 15mL/g resected tissue roughly. Blood transfusions in approx 10%. Tissue plasminogen activator released from prostate.
Optimise volume via PPV (fluid responsiveness)
If minimal UO & risk ARF, 500mL fluid challenge
If volume responsive & more fluid not detrimental, give more, otherwise PAC; insert CVP; raised? Consider mannitol/frusemide. If urea rise, fluid o/load, met acidosis, hyperK consider haemodialysis (diuretics only used for limited time (40-80mg IV frusemide; loop better natriuretic effect vs thiazides)

1621
Q

Benefits of Laser TURP

A

Almost no chance TURP syndrome (minimal fluid absorption)
Pt can be on anticoagulation, better haemostats, minimal blood loss, easier estimate than with standard TURP (irrigant ++)
Shorter IDC use & shorter LoS despite longer OT (less familiarity w technique?)
Lower risk LT issues (bl neck damage/incontinence, lower rates retrograde ejaculation)

MUST be under GA, pt can’t cough (so not spinal)
Complex equipment/laser safety issues

Vapourisation:
KTP (green light)= a modified Nd:YAG laser beam, incr the frequency of the Nd:YAG, absorbed by Hb not water, massive heating & instant vaporisation virtually no bleeding & not the “sloughing” risks of standard Nd:YAG (eg. Risk BOO or missed prostatic Ca) BUT the vaporisation means can’t get histology. Can use NaCl irrigation.
Thullium laser= another type of photovapourisation

Other types laser: HoloLap (holmium); weaker wavelength than green light, cuts vs vaporise, still good heamostatic control but slower/longer OT times. Able to preserve histology.

1622
Q

Cyanotic congenital heart diseases

A

5 T’s mnemonic:
ToF
Total anomalous PV drainage
Transposition of the great vessels
Trunks arteriosus
TCV abnormalities & hypo plastic R) heart syndrome

1623
Q

immunosuppressants for renal transplant
RA

A

Cyclosporine & tacrolimus both nephrotic, HTN, high K, low Mg++, cyclosporin neurological (paraesthesia), macro hyperglyc
Myocphenolate BM suppression, malignancy risk, viral reactivation
Azathioprine risk hepatoxocicity & BM suppression, N&V/diarrhoea

Steroids
Methotrexate: hepatotoxicicity, ILD
sulfasalazine: neutropenia, thrombocytopenia, pulmonary fibrosis
azathioprine: BM suppression, hepatitis

1624
Q

Complications after pneumonectomy

A

Cardiac arrhythmias: 40% get AF
Pulmonary oedema: 2-5%, mort >50%, more common after R) pneumonia
Bronchopleural fistula: more common w R) pneumonectomy, early= cough, air leak, falling fluid level. Later may be empyema.
Cardiac herniation: R)-sided if there was stripping of pericardial sac